You are on page 1of 378

Contents

PEDIATRICS................................................................................................................................................ 2
ALLERGY & IMMUNOLOGY TiKi TaKa ............................................................................................. 88
CARDIOLOGY TiKi TaKa ............................................................................................................................... 93
DERMATOLOGY TIKI TAKA .................................................................................................................... 138
ENT ............................................................................................................................................................ 144
GASTROENTEROLOGY TiKi TaKa................................................................................................................ 147
GENITOURINARY TIKI TAKA....................................................................................................................... 156
HEMATOLOGY & ONCOLOGY TiKi TaKa .................................................................................................... 170
HEPATOLOGY TiKi TaKa ............................................................................................................................. 194
INFECTIONS TiKi TaKa ............................................................................................................................... 210
NEUROLOGY TiKi TaKa .............................................................................................................................. 216
OPHTHALMOLOGY TiKi TaKa..................................................................................................................... 241
POISONING TiKi TaKa ................................................................................................................................ 245
PREVENTIVE MEDICINE TiKi TaKa ............................................................................................................. 250
PSYCHIATRY & ETHICS TiKi TaKa ............................................................................................................... 253
PULMONOLOGY TIKI TAKA........................................................................................................................ 285
RHEUMATOLOGY TiKi TaKa ....................................................................................................................... 314
SURGERY TIKI TAKA ................................................................................................................................... 330

PEDIATRICS
PEDIATRICs
______________
===================================================== Cardiology
============================================================================
=
. Myocarditis:
______________
. Is an inflammation of the myocardium, Caused by: infections, toxins, and autoimmune
diseases.
. Viral infection ( enteroviruses in particular) the most common cause of myocarditis in
children.
. Classically occurring in spring and summer, myocarditis develops more often in infants and
young children than in older children and adolescents.
. Leads to tissue necrosis, thereby worsening myocardial function, dilating the heart, and
increasing end diastolic volume with resultant pulmonary edema and CHF.
. Myocarditis typically presents as a flulike illness: fever, anorexia, lethargy, and irritability
followed by respiratory distress from acute heart failure.
. A holosystolic murmur may be identified secondary to dilated cardiomyopathy and the
resulting functional mitral regurgitation.
. Hepatomegaly can develop due to passive congestion Other features of heart failure include
fatigue, tachypnea, tachycardia, and decreased perfusion.
. Affected children should be monitored in the intensive care unit because of the risk of acute
decompensation and fatal arrhythmias.
. The workup for suspected myocarditis includes a (CBC) with differential, ESR and C-reactive
protein, cardiac enzymes, and blood and viral cultures.
. A chest radiograph demonstrates cardiomegaly and pulmonary edema.
. Electrocardiography most commonly shows low voltage QRS and sinus tachycardia.
. Echo.is the best means of evaluating myocardial function and often shows global hypokinesis,
left vent. hypertrophy, left vent. dysfunction & pericardial effusion.
. The gold standard for diagnosis is a myocardial biopsy, which also allows for disease stage
classification.
. Ventricular septal defect (VSD):
__________________________________
. The most common congenital heart malformation.
. VSDs range from small and asymptomatic to large with significant left-to-right shunting.
. Indications of significant VSD is: left-to-right shunt First, he is symptomatic Second, he has
the telltale murmur.
(Affected patients have pansystolic murmurs loudest at the left lower sternal border.
. There maybe also a diastolic murmurs at the apex because of increased flow across the mitral
valve ).
. This patient should have an echocardiogram with a bubble study to evaluate his congenital
heart defect.
. Most small ventricular septal defects close spontaneously and require no treatment, as long
as there are no signs of pulmonary vascular disease.
. Reassurance and surveillance via EKG and echocardiography are needed in most patients.
. Tetralogy of Fallot (TOF):
____________________________
. Is the most common cyanotic congenital disease in children less than 4 years.
. Patients often present with cyanotic spells.

. The classic tetrad includes:


(1) subpulmonary stenosis (single S2), (2) overriding aorta, (3) ventricularseptal defect (VSD)
(pansystolic murmur), and (4) right ventricular hypertrophy.
. Because a large unrestrictive VSD is always present, the right ventricular pressure is the same
as the left ventricular and aortic pressures.
. Pulmonary artery pressure and flow are inversely proportional to the degree of subpulmonary
obstruction.
. The degree of cyanosis correlates precisely with the degree of pulmonary stenosis.
. Untreated patients with TOF are at risk for "hypercyanotic" or "tet" spells.
. These spells are often precipitated by exertion (such as feeding) and are characterized by
irritability, cyanosis, and tachypnea.
. The exact etiology of these episodes is unclear, it may be predisposed by peripheral
vasodilation, hyperventilation, and/or contractility of the Rt. ventricular outflow tract.
. Regardless of the etiology, however, an increased right to left shunt across the ventricular
septal defect develops, causing a decrease in pulmonary blood flow and
increased systemic venous return of deoxygenated blood leading to worsening cyanosis.
. If untreated, "tet" spells can lead to syncope or death.
. The treatment of a "tet" spell is to place the child in a knee to chest position, which increases
systemic vascular resistance resulting in
increased blood flow from the right ventricle to the pulmonary circulation.
. Morphine and an intravenous fluid bolus can also be given to increase pulmonary blood flow.
. Oxygen is of little benefit because the abnormality is decreased pulmonary blood flow, not
insufficient oxygenation
. Atrial septal defects:
________________________
. Are relatively common among children and may cause symptoms.
. On exam, there is a widely split and fixed S2, systolic ejection murmur in the left upper
sternal border, and,
if there is a large shunt, a diastolic murmur at the left lower sternal border.
. Transposition of the great vessels:
_____________________________________
. A cyanotic heart disease which presents with cyanosis in the first 24 hours of life.
. It is commonly seen in infants of diabetic mothers and in males.
. The aorta arises from the right ventricle, and the pulmonary artery from the left ventricle.
. As a result, deoxygenated blood coming from the body goes to the right atrium and ventricle
and is cycled back to the body through the aorta.
. Oxygenated blood from the lungs is returned to the lungs by the left side of the heart through
the pulmonary artery.
. During intrauterine life, the fo ramen ovale and ductus arteriosus provide mixing of the
deoxygenated and oxygenated blood,
resulting in an almost normal oxygenation of the fetal circulation.
. A patent foramen ovale or ductus arteriosus can maintain life after birth, as seen in a patient
with normal Apgar scores.
. The foramen ovale and ductus arteriosus begin to close after birth, leading to decreased
mixing and poor oxygenation.
. As the aorta is located in front of the pulmonary artery, the S2 aortic component is better
heard than the S2 pulmonic valve component and is audible as a single S2.
. There is usually no murmur because the foramen ovale and ductus arteriosus have closed.
. The neonate then presents with cyanosis and tachypnea within the first few hours of life.
. Chest x-ray can be initially normal but show some evidence of increased pulmonary blood
flow after 1-2 weeks.
. Echocardiography confirms the diagnosis.

. Prostaglandin E1 can be intravenously given to stabilize the neonate by maintaining the


patency of the ductus arteriosus, which is important for survival.
. Surgical treatment is definitive.
.N.B:
- Prostaglandin E1 is a vasodilator used to prevent ductus arteriosus closure in such infants,
to increase or maintain pulmonary blood flow and improve oxygenation until definitive
surgery can be performed.
- Closure of the ductus arteriosus in such patients would cause progressive hypoxia and
metabolic acidosis.
. Coarctation of the aorta (COA):
_________________________________
. A congenital defect characterized by stenosis of the aorta, usually near the ductus arteriosus.
. present with hypertension in the upper part of the body (high BP in the arms) and relative
hypoperfusion in the lower part of the body (low BP in the legs).
. The prevalence of COA in patients with Turner's syndrome is approximately 7%.
. The clues to the diagnosis are occasional headaches and increased BP in both arms, leg
muscle fatigability while climbing stairs (due to lower body hypoperfusion).
. And a mild, continuous murmur heard all over the chest (due to the development of
collaterals between the hypertensive and hypoperfused vessels).
. Rib notching caused by the dilatation of the collateral chest wall vessels is specific for
coarctation.
. Mitral stenosis:
__________________
. also causes a diastolic rumble but does not typically cause a pansystolic murmur at the left
lower sternal border unless there is tricuspid regurgitation as well.
. Other features to suggest rheumatic fever such as fever, arthritis, or skin lesions.
. Tricuspid atresia:
____________________
. Suspect tricuspid atresia in a cyanotic infant with left axis deviation.
. This condition is characterized by an absent connection between the right heart cavities, and
a hypoplastic or absent right ventricle.
. Ventricular septal defect (holosystolic murmur on auscultation) occur in 90 % of cases.
. Transposition of the great arteries occur in 30% of cases (which is not likely to be in a case
where the chest x- ray does not demonstrate pulmonary overcirculation).
. The associated heart defects, such as atrial septal defect (ASD), ventricular septal defect
(VSD), or patent ductus arteriosus (PDA), are necessary for survival.
. Venous blood passes through the ASD to reach the left cavities, and then through the VSD
back to the outlet portion of the right ventricle to reach the pulmonary circulation.
. Sometimes, there is an associated pulmonary stenosis, and the blood passes through the PDA
to reach the pulmonary circulation.
. Patients with normal arteries (without TGA) present in the first two weeks of life with
progressive cyanosis, tachypnea, easy fatigability, and poor feeding.
. EKG reveals left ventricular hypertrophy, which occurs since the left ventricle handles both
pulmonary and systemic venous returns.
. Echocardiography shows a fibromuscular membrane in the place of the tricuspid valve, a
variably small right ventricle, ventricular septal defect, and a large left ventricle.
. The treatment generally includes PGE1 (to keep the ductus arteriosus open) and balloon atrial
septostomy
(if the ASD is not large enough to allow an adequate flow from the right to left atrium)

. postpericardiotomy syndrome:
______________________________
. It is reactive pericarditis with a pericardial effusion that present after surgery for congenital
heart disease.
. Symptoms usually occur 1-6 weeks after surgery.
. Although the cause is not known, it is thought to be an autoimmune response, possibly to a
viral infection.
. Most children develop mild symptoms which are self limited. In infants, pericardial effusions
can present with abdominal pain, vomiting, and decreased appetite.
. Older children may complain of chest pain Fever is often present On examination, findings
are consistent with pericardial and/or pleural inflammation,
including tachycardia, poor perfusion, pulses paradoxus, distant heart tones, and jugular
venous distention.
. If the effusion is large enough to affect the function of the heart , pericardial tamponade can
occur.
N.B:
- Most cardiac murmurs in childhood are benign (innocent) and require only observation. Know
the following features of a benign murmur:
systolic ejection quality.
varies with position;
Asymptomatic patient
Murmur intensity grade 2 or less
Normal S2
No audible clicks
Normal pulses
No other abnormalities.
- Pathological murmurs may indicate or lead to congenital heart disease, and are very likely
when the patient has any of the following:
symptomatic patient
Murmur intensity more than grade 2.
Pansystolic murmur
Murmur loudest at upper left sternal border
Absent or diminished femoral pulses
Murmur's quality is unchanged with position.
. Rheumatic fever:
__________________
. Suspect rheumatic fever in a child with sore throat, low-grade fever, pericarditis, arthritis,
chorea and subcutaneous nodules.
. Other features include erythema marginatum, elevated ESR and prolonged PR interval on EKG.
. Rheumatic fever is caused by is group A streptococcus.
. Diagnosis is made if two major or one major and two minor criteria (Jones criteria) are
satisfied, in addition to
evidence of recent streptococcal infection (via detection of antistreptolysin antibodies,
cultures and other various antigen detection kits).
. Jones criteria:
----------------. The five major criteria are:
- Polyarthritis.
- Carditis.

- Chorea.
- Subcutaneous nodules.
- Erythema marginatum.
. The three minor criteria are:
- Fever.
- Arthralgia.
- Previous rheumatic fever.
. The treatment is benzathine penicillin G.
. Patients with chorea, pericarditis and arthritis may require additional therapy with antiepileptics, salicylates, and codeine, respectively.
. Jervell-Lange-Nielson syndrome:
_________________________________
. Is an autosomal recessive disease characterized by congenital deafness and QT interval
prolongation on EKG.
. QT interval prolongation predisposes to a particular type of ventricular tachycardia called
torsades de pointes.
. Torsades de pointes cause syncopal episodes and sudden death.
. The clue to the diagnosis is a syncopal episode without following disorientation (post-episode
confusion is more characteristic for a seizure), hearing impairment & family history.
. If the patient is asymptomatic or does not have a clearly documented syncope, beta blocker
alone is sufficient.
. If the patient is symptomatic or has a documented episode of syncope, the treatment will be
beta-blocker plus a DDD pacemaker.
. Wolff-Parkinson-White (WPW) syndrome:
_______________________________________
. Characterized by an accessory pathway between the atrium and ventricle resulting in
preexcitation and an increased risk for tachyarrhythmias.
. WPW is usually seen in otherwise healthy children or adolescents.
. Affected patients can either be asymptomatic or present with chest pain, palpitations,
syncope, or cardiac arrest.
. In WPW, an accessory pathway conducts antegrade from the atria to the ventricles faster than
the conduction through the AV node, which
allows part of the ventricle to depolarize early.
. This results in the classic EKG findings of a shortened PR interval, a slurred initial portion of
the QRS (the delta wave), and a widened QRS complex.
. If there is retrograde conduction from the ventricles to the atria, supraventricular tachycardia
(SVT) can result.
. The EKG features of WPW will not be apparent during episodes of SVT, but the delta wave
should be visible once the rhythm is slowed.
. Sudden death can occur from dysrhythmias if WPW is untreated.
. A patent ductus arteriosus (PDA):
___________________________________
. Usually associated with a continuous flow murmur due to constant movement of blood from
the high-pressure aorta to the low-pressure pulmonary artery.
. Small PDAs are often asymptomatic and detected incidentally on routine cardiac auscultation.
. Physical examination is often unremarkable in small PDA, although mildly accentuated
peripheral pulses can occur.

=============================================================== ENT
============================================================================
=
. Epiglottitis:
_______________
. In unimmunized children H. influenza type B is the most likely cause.
. In Hib vaccinated patients:H. influenzae type A and nontypable strains, H. parainfluenzae,
Streptococcus pneumoniae
. Sudden onset of; fever, sore throat, dysphagia and drooling.
. Airway obstruction is the most concerning complication of epiglottitis.
. This is a pediatric emergency and should be managed emergently with endotracheal
intubation; however, since intubation of such patients is difficult,
preparation for possible tracheostomy is also standardly performed.
. Allergic Rhinitis:
____________________
. Allergic rhinitis is characterized by an immunoglobulin E (lgE) mediated hypersensitivity to fo
reign allergens.
. It may be acute or chronic, and can be confused with recurrent colds.
. It may present with itchy eyes and tearing, swollen mucosa, headaches and nasal
obstruction,if the age of onset is in the second decade of life.
. Diagnosis is by allergy testing and findings of eosinophilia in the nasal discharge.
.N.B:
- The most common predisposing factor for acute bacterial sinusitis is a viral upper respiratory
infection.
. Choanal atresia: Extremely high yield question for USMLE.
__________________
. Suspect choanal atresia in an infant who presents with cyanosis that is aggravated by feeding
and relieved by crying.
. Failure to pass a catheter through the nose 3-4 cm into the oropharynx is suggestive of the
diagnosis.
. The diagnosis is confirmed by CT scan with intranasal contrast.
. First step in management consists of placing an oral airway and lavage feeding.
. Definitive treatment involves repairing the obstruction with surgery or endoscopy.
. Brain abcess after otitis media:
__________________________________
. An acute onset of headaches and focal neurologic symptoms after an episode of acute otitis
media or sinusitis most likely has a brain abscess.
. Hypothermia, hypotension.
. Ring-enhanc ing lesions are usually seen on CT or MRI scan.
. Acute bacterial sinusitis:
____________________________
. Streptococcus pneumoniae is the most common cause of acute sinusitis in childhood,
followed by Haemophilus influenzae and Moraxella catarrhalis.
. Staphylococcus aureus and anaerobes are common etiologic agents of chronic sinusitis, and
are rarely seen in acute cases.
. Diagnosis of acute bacterial sinusitis for children <6 years of age should be based on clinical
rather than radiographic criteria.

. The treatment is antibiotics, in uncomplicated sinusitis, the treatment of choice is amoxicill in


45-50 mg/kg/day.
. Cholesteatomas:
_________________
. Cholesteatomas in children can either be congenital or acquired secondary to chronic middle
ear disease.
. Congenital cholesteatomas typically found in younger patients around the age of five.
. New-onset hearing loss or chronic ear drainage despite antibiotic therapy are typical
presenting symptoms of acquire cholesteatomas, and
granulation tissue and skin debris may be seen within retraction pockets of the tympanic
membrane on otoscopy.
. Complications of cholesteatomas include hearing loss, cranial nerve palsies, vertigo, and lifethreatening infections as brain abscesses o rmeningitis.
. Should be referred to an otolaryngologist for a dedicated otologic exam, possibly
accompanied by a CT and/or surgical visualization to confirm the diagnosis.
. Acute otitis media:
_____________________
. The most common causative organism is Streptococcus pneumoniae (40% of cases), followed
by non-typable Haemophilus influenzae & Moraxella catarrhalis.
. Acute otitis media should be considered in any patient with symptoms of ear drainage and
difficulty hearing.
. Ear pain is also common, but may be absent in young patients.
. Nonspecific systemic symptoms: fever, irritability, and diarrhea can sometimes be present.
. The first-line treatment is 10 day course of amoxicillin.
. juvenile angiofibroma (JNA):
______________________________
. Any adolescent with a nasal obstruction, visible nasal mass, and frequent epistaxis is
considered to have a juvenile angiofibroma (JNA) unless proven otherwise.
. This is typically found in the back of the nose or upper throat (nasopharynx) of adolescent
boys.
. It is a benign growth, but is capable of eroding and locally invading.
. Removal is often difficult because the tumor is unencapsulated and may be deeply invasive.
. Recurrence of the tumor after surgical resection is common.
. Retropharyngeal abscesses:
____________________________
. Present with: fever, irritability, and fatigue along with a sore throat, dysphagia, trismus, a
muffled or "hot potato" voice, and/or neck stiffness.
. On examination, unilateral cervical lymphadenopathy and decreased range of motion of the
neck are common.
. The oropharynx may be erythematous and a unilateral pharyngeal bulge may be present if the
child is able to open the mouth.
. Respiratory distress, if present, is concerning for airway obstruction.
. Diagnosis can be confirmed with either plain X-ray or a CT scan of the neck.
. Lateral neck X-ray often show widening of the soft tissue space between the trachea and the
spine.
. Due to false +ve & -ve results of X-ray, so the diagnosis should be confirmed by CT, which will
delineate the abscess and help distinguish between an abscess and cellulitis.
. N.B:
- Inflamed, fluctuant cervical lymph nodes are typically caused by streptococcal or
staphylococcal infection, The antibiotic of choice is dicloxacillin.

- dicloxacillin will treat infections caused by methicillin-sensitive Staphylococcus aureus (MSSA)


but not methicillin- resistant Staphylococcus aureus (MRSA).
. Croup:
________
. Also known as laryngotracheitis or laryngotracheobronchitis (LTB).
. Is characterized by laryngeal inflammation that results in hoarseness, a barking cough, and
varying degrees of respiratory distress over time.
. The typical patient is less than 3 years of age, and the most common causative agent is
Parainfluenza virus.
. Croup is usually a clinical diagnosis Lateral neck x- rays show subglottic narrowing.
. Give a trial of epinephrine in cases of croup before intubation.
. A trial of racemic epinephrine should precede any invasive procedure in cases of croup, as
this decreases the need for intubation.
. Epinephrine acts by its alpha-adrenergic that is beneficial by reducing bronchial secretions
and mucosal edema and
. Beta-adrenergic effects that help croup patients who are also wheezing by promoting smooth
muscle relaxation.
. Laryngomalacia or congenital flaccid larynx :
________________________________________________
. is the most common cause of chronic inspiratory noise in infants.
. Laryngoscopy shows flaccidity of the larynx, and collapses during inspiration.
. It is a self-limiting condition in most cases, and generally subsides by 18 months of age.
. The mother should be instructed to hold the child in an upright position for half an hour
after feeding, and to never feed the child when he is lying down.

=================================================================
Ophthalmology
==================================================================
. Neonatal Conjunctivitis:
__________________________
. Gonococcal conjunctivitis: A copious amount of purulent drainage in newborns who are 2-5
days old is most consistent with gonococcal conjunctivitis.
. Chlamydia trachomatis conjunctivitis: caused by Chlamydia trachomatis is usually milder and
presents > 5 days after birth.
. Chemical conjunctivitis can result from prophylactic silver nitrate eye drops, but it is usually
fairly mild.
. viral conjunctivitis:
_______________________
. The conjunctiva is red with copious, watery discharge.
. Children are more commonly infected through contaminated swimming pools.
. It is mostly caused by adenovirus type 3.
. Chemical conjunctivitis:
__________________________
. It is the most common cause of a red eye that presents within the first 24 hours of life.
. Approximately 80% of patients who receive silver nitrate prophylaxis (to prevent gonococcal
conjunctivitis)
experience mild conjunctival irritation and tearing that generally resolve within 24 hours.

. Gonococcal conjunctivitis:
___________________________
. is acquired through contact with infected genital secretions, and occurs 2-5 days after birth.
. It presents as a hyperacute and highly purulent conjunctivitis with copious, purulent eye
discharge with swollen eyelids and chemosis.
. It is the most destructive neonatal eye infection, since it may result in corneal perforation and
permanent blindness if left untreated.
. The treatment is ceftriaxone.
. Chlamydial conjunctivitis:
___________________________
. Develops a few days to several weeks after birth ( > 5 days after birth).
. Manifests with conjunctival congestion, edema, and mucoid or frank purulent discharge.
. Pneumonia usually appears 3 to 19 weeks after birth, and symptoms include cough,
tachypnea, and rales; there is no fever, and wheezing is rare.
. The latter two features permit to differentiate chlamydial pneumonia from RSV bronchiolitis.
. The treatment is oral erythromycin for 14 days, as topical agents are ineffective.
. Trachoma:
___________
. caused by Chlamydia trachomatis serotype A-C. It is a major cause of blindness worldwide.
. The active phase of the disease is characterized by follicular cionjunctivitis and pannus
(neovascularization) formation in the cornea.
. conjunctivitis usually presents on the 5th to 14th day of life as congestion and discharge that
may be scant, mucoid or frankly purulent.
. Concurrent infection occurs in the nasopharynx, leading to nasal discharge.
. The diagnosis can be made by Giemsa stain examination of conjunctiva! scrapings.
. Topical tetracycline or oral azithromycin should be started immediately.
. The treatment is erythromycin, which has to be administered systemically (oral) to decrease
the risk of chlamydial pneumonia.
. Repeated infections can lead to scarring of the cornea.
. Retinal hemorrhages:
______________________
. Common findings in child abuse. Multiple retinal hemorrhages that differ in size and color
support the suspicion of child abuse.
. A precipitating factor (stepfather/mother, financial issue) is usually present in the history In
addition, there may be
bruises that usually have associated swelling and tenderness.
. In this case, nothing in the history and physical findings is suggestive of child abuse;
therefore, reporting to the authorities is not necessary.
. Herpes simplex keratitis:
___________________________
. Presents as pain, photophobia, and decreased vision.
. Dendritic ulcer is the most common presentation.
. There may also be minute clear vesicles in the corneal epithelium.
. Orbital cellulitis:
_____________________
. Is an infection that is posterior to the orbital septum which is unilateral and more common in
children with paranasal sinus infection.

. Patients present with an abrupt onset of fever, proptosis, restriction of extraocular


movements and swollen, red eyelids.
. Differentiating preseptal cellulitis and orbital cellulitis is important in any patient who
presents with a swollen and painful eye accompanied by fever.
. Pain with eye movement is more suggestive of orbital cellulitis it can occur in preseptal
cellulitis as well.
. Both proptosis and decreased visual acuity are almost always seen in orbital cellulitis.
. CT can be used to confirm the extent of infection and to identify a potential abscess.
. Treatment requires intravenous broad-spectrum antibiotics.
. Preseptal cellulitis:
_______________________
. Is an infection of the soft tissue anterior to the orbital septum, that is most commonly caused
by contiguous spread of infection from local facial or eye lidtrauma.
. It is often present with: eyelid swelling, tenderness, erythema or discoloration.
. The presence of decreased eye movements, decreased visual acuity, proptosis or double
vision is more suggestive of orbital cellulitis.
. Case of leukocoria (white reflex):
____________________________________
. Every case of leukocoria is considered a retinoblastoma, until proven otherwise.
. So, this child needs prompt referral to an ophthalmologist.
. Retinoblastoma is the most common intraocular tumor of childhood.
. The underlying pathology involves inactivation of the Rb suppressor gene, which may be
familial or sporadic.
. Retinoblastoma is a highly malignant tumor, and failure to diagnose and treat the disease
early may lead to death from liver and brain metastases.
. The other manifestations of the disease may include strabismus, decreased vision, ocular
inflammation, eye pain, glaucoma, and orbital cellulitis.
. The diagnosis is highly suspected with US or CT scan findings of a mass with calcifications.
. strabismus:
_____________
. an esodeviation (medial deviation of the eye), which is the most common type of strabismus,
representing over 50% of all cases.
. Strabismus is the most common cause of amblyopia.
. The cover test: asking the child to fix his sight on a target as the examiner alternately covers
one eye while observing the movement of the other.
. A normal eye keeps the same position and does not move, whereas a misaligned eye shifts to
refixate the object when the normal eye is covered.
. The standard of treatment is occlusion of the normal eye.
. This procedure forces the affected eye to correct itself in order to be properly functional, thus
stimulating proper visual maturation.
. In case the amblyopia is secondary to abnormalities of refraction, appropriate lenses should
be prescribed so as to have a well-formed retinal image.
. The normal or less severely affected eye is then covered.
. If the amblyopia is caused by an opaque media, surgical removal of the media should be
performed .before occlusion therapy.

=========================================================================
Neurology ==========================================================

. Night Terrors:
________________
. Occurs in young children 2-12 years during Non-REM sleep.
. Characherized by fear, craying or screaming, decreased level of consciousness.
. and amnesia of the event next morning.
N.B.:
- Todd's palsy is characterized by transient hemiplegia that occurs after a seizure.
. Migraine headaches:
_____________________
. are the most common cause of headaches in the pediatric population, occurring in 50% of
cases before the age of 20 years.
. The aura can be visual (e g , blurred vision, scotoma, photophobia, distortion of objects) or
sensory (e g perioral paresthesia, numbness of extremities, shoulder pain, abdominal pain,
otalgia).
. Migraine headache is a clinical diagnosis, and further investigations are not usually indicated.
. Neuroimaging is required only when there are behavioral changes, decline in school
performance, decline in growth parameters,
early morning headaches, seizures, orabnormal neurologic findings.
. In complicated migraines, neurologic signs (e g , basilar signs, ophthalmoplegia, or
hemiparesis) may be present, although the work-up reveals nothing abnormal.
. These neurologic symptoms may last only during the headache or persist for several days or
weeks.
. Brain abscesses:
_________________
. can present with fever, headache, focal neurologic changes, seizure, spasticity, or signs of
increased intracranial pressure.
. Congenital heart disease (particularly right-to-left shunts), head trauma, infections of the jaw
or mouth, infections of the face or scalp, meningitis, and
cranial instrumentation increase the risk of developing a brain abscess.
. Absence (petit mal) seizures:
_______________________________
. Characterized by a sudden cessation, without warning, of ongoing mental activity.
. Each episode rarely lasts longer than 30 seconds.
. A typical absence seizure is characterized by brief (typically lasting for a few seconds) periods
of impaired consciousness.
. Patients may also present with automatisms, but do not have a post- ictal state.
. Hyperventilation during the EEG reveals a generalized 3Hz spike-and-wave pattern on a
normal background.
. Minor motor symptoms are common, but complex automations and clonic activities do not
occur.
. Best diagnosed with EEG studies.
. An EEG with activation procedures (hyperventilation, photic stimulation, sleep) helps in
further diagnosis and classification of seizures.
. Ethosuximide is used almost exclusively for childhood absence seizures & valproic acid is the
2nd choice.
.N.B:
- An atypical absence seizure lasts longer.
- The characteristic EEG pattern is slow spike-and-wave activity with a frequency less than
2.5Hz

. Seizures:
___________
. Seizures are classified as partial or generalized:
A) Generalized seizures involve the brain diffusely from the beginning of the event.
B) Partial seizures arise from a discrete focus in the brain.
. In Partial seizures there are initial symptoms related to a specific focus in the brain e.g. an
unusual smell could be classified as an olfactory hallucination,
referred to as an "aura" which is indicative of a partial seizure arising from a specific focus in
the brain.
. Partial seizures can be further classified into simple partial seizures, complex partial
seizures, and partial seizures with secondary generalization.
1. Simple partial seizures: No Loss of consciousness.
2. Complex partial seizure: Loss of consciosness (failure to respond to various stimuli during
the episode) + automatisms during their loss of consciousness
as chewing, picking movements of the hands, or lip smacking.
- The EEG pattern is usually normal or may show brief discharges.
. Partial seizure with secondary generalization: usually have tonic-clonic manifestations.
. A post-ictal state lasting several hours can be seen with both complex partial seizures and
partial seizures with secondary generalization so this is not
a helpful differentiating factor.
. The presence of blood-tinged sputum is suggestive that the patient bit his tongue and would
make a partial seizure with secondary generalization more likely.
. Loss of bladder or bowel control is also more likely in seizures with secondary generalization.
. Phenytoin & carbamazepine are used as first-line drugs for ttt of 1ry generalized tonic-clonic
seizures or partial seizures with or without 2ndry generalization.
. Todd's paralys:
_________________
. Represents a postictal condition that usually rapidly improves with restoration of motor
function within 24 hours.
. May follow generalized as well as focal seizures.
. The exact pathophysiology of this condition is not well understood, but most probably
involves alteration in neuronal electrical activity.
. It indicates that a structural abnormality underlying the seizure is present.
. Cephalohematoma:
__________________
. Is a subperiosteal hemorrhage; hence, it is always limited to the surface of one cranial bone.
. There is no discoloration of the overlying scalp, and swelling is usually not visible until
several hours after birth
because subperiosteal bleeding is a slow process.
. Most cases do not require any treatment and resorb spontaneously within 2 weeks to 3
months, depending on the size.
. Rarely, phototherapy may be necessary to improve the hyperbilirubinemia.
.N.B.: Caput succedaneum:
------------------------- is a diffuse, sometimes ecchymotic, swelling of the scalp.
- It usually involves the portion of the head presenting during vertex delivery.
- It may extend across the midline and across suture lines.

. Parinaud's syndrome:
______________________
. Consists of paralysis of vertical gaze that may be associated with pupillary disturbances and
eyelid retraction (Collier's sign).
. The lesion is on the rostral midbrain at the level of the superior coll iculus and CN III.
. It is most commonly caused by germinomas and pinealomas at this region.
. The endocrine syndrome results from interruption of hypothalamic inhibiting pathways, and
sometimes beta-hCG secretion and consequent Leydig's cell stimulation.
.N.B.:
- In 90% of cases, medulloblastoma arises from the cerebellar vermis, and is not associated
with Parinaud's syndrome.
- Neuroblastoma is the third most common malignancy in the pediatric population, It arises
mostly from the adrenal glands and sympathetic paraspinal ganglia.
- Although craniopharyngioma can also cause an endocrine syndrome, it results in visual field
defects owing to its sellar location.
. Myotonic muscular dystrophy (MMD):
____________________________________
. an autosomal dominant disease which is also known as Steinert disease.
. It is the second most common muscular dystrophy in the United States.
. The pathology is distinct in that all types of muscles (i.e. smooth, striated, cardiac) are
involved.
. The patient initially appears normal at birth, then slowly develops muscle weakness and
progressive muscle wasting;
especially in the distal muscles of the hands, posterior forearm muscles, and anterior
compartment of the lower legs.
. The characteristic facial appearance: temporal wasting, thin cheeks, and an upper lip in the
shape of an inverted V.
. Pertinent physical findings: emaciated extremities, atrophy of the thenar and hypothenar
eminences, proximal muscle weakness, positive Gowers sign,
winged scapula, and myotonia Myotonia is defined as delayed muscle relaxation, and the
classic example is the inability to release the hand after a handshake.
. In addition, abnormalities of the endocrine, immunologic, and nervous systems occur.
. Endocrine manifestations include diabetes mellitus, testicular atrophy, frontal baldness and
hypothyroidism
. Breath holding spells:
________________________
. are episodes of apnea that are sometimes associated with a loss of consciousness, and are
precipitated by frustration, anger, or pain.
. These episodes usually occur in children ages 6-18 months.
. Classically, the child gets frustrated and holds their breath, which may lead to loss of
consciousness followed by spontaneous resolution.
. Unlike a seizure, there is no incontinence or postictal phase associated with breath holding
spells.
. These spells can be simple, cyanotic, pallid, or complicated.
. In a simple breath holding spell, there is no change in circulation or oxygenation.
. Cyanotic spells are associated with cyanosis during the spell, and may be associated with
hypoxia and decreased blood flow to the brain.
. However, children with cyanotic breath holding spells usually have no long-term adverse
effects from the episode.

. Pallid spells usually occur after painful events or minor trauma, and the affected child has
pallor.
. Complicated breath holding spells are a more severe form of the cyanotic or pallid types with
associated seizure-like activity However, EEG is normal.
. Breath holding spells are diagnosed based on the clinical history.
. Since iron deficiency anemia may be associated with breath holding spells, patients are
generally tested to rule-out anemia as a predisposing cause.
. The most important step in managing affected patients is reassuring the parents that the
episodes are not harmful and will not have any long-term effects.
. No treatment is necessary for breath holding spells, and affected children will eventually stop
having episodes.
. Anemic patients, however, should be treated with iron supplementation.
. Brain tumors:
_______________
. In the pediatric population, CNS tumors are the most common solid tumors and the second
most common malignancies (after leukemias).
. Approximately 60% of these tumors are infratentorial, 25% are supratentorial, and 15% arise
in the midline.
. Astrocytomas are the most common histologic type for both supratentorial and infratentorial
groups.
. Medulloblastoma:
__________________
. Is the second most common posterior fossa tumor (after cerebellar astrocytoma) in children.
. It is highly radiosensitive and can metastasize through the CSF tract Over 90% of
medulloblastomas develop in the vermis.
. This patient has posterior vermis syndrome (truncal dystaxia & horizontal nystagmus).
. Craniopharyngioma:
_____________________
. A young boy with symptoms of increased intracranial pressure ( headaches, vomiting),
bitemporal hemianopia, & a calcified lesion above the sella has a
craniopharyngioma until proven otherwise.
. Craniopharyngiomas are derived from epithelial remnants of Rathke's pouch.
. It is not a true pituitary tumor, and is characterized by nests of squamous cells in a loose
stroma, resembling the appearance of embryonic tooth bud enamel.
. Its location is suprasellar and inferior to the optic chiasm, which is why it can cause bitemporal hemianopia (by pressing the optic chiasm).
. Associated endocrine symptoms such as diabetes insipidus, and growth failure associated
with either hypothyroidism or growth hormone deficiency may occur.
. The diagnosis is usually made by (CT) or (MR): Presence of a cystic calcified parasellar lesion
on MRI is almost diagnostic of craniopharyngioma.
. Surgical removal is the treatment of choice.
. pituitary adenoma may present with similar symptoms; however, the condition is more
frequent in women.
. Furthermore, prolactinoma is an important part of the syndrome, and calcification of the
gland does not occur.
. Neurocutaneous syndromes:
___________________________
A) Type 1 neurofibromatosis:
----------------------------

. Characterized by cafe-au-lait spots, axillary freckles, Lisch nodules of the iris, neurofibromas,
and bony lesions.
B) Type 2 neurofibromatosis:
---------------------------. Is associated with brain tumors, especially bilateral acoustic neuromas.
. Bilateral acoustic neuromas are diagnostic of neurofibromatosis.
. Patients with neurofibromatosis have a tendency to form tumors in the central nervous
system (CNS), peripheral nervous system, skin and viscera.
C) Sturge-Weber syndrome: A neurocutaneous syndrome
------------------------. Characterized by a congenital unilateral cavernous hemangioma along trigeminal nerve
distribution & intra-cranial calcifications that resemble a tramline in X-ray.
. Also present with: focal or generalized seizures, mental retardation, and a port wine stain or
nevus flammeus along the branches of trigeminal nerve.
. Other pertinent findings are hemianopia, hemiparesis, hemisensory disturbance, and
ipsilateral glaucoma.
. Treatment is aimed at controlling the seizures and reducing intraocular pressure Argon laser
therapy is successful in removing the skin lesions.
D) Tuberous sclerosis:
---------------------. Ash leaf hypopigmentation, cardiac rhabdomyomas, kidney angioleiomyomas, mental
retardation, seizures.
N.B.:
- The characteristic features of neurofibromatosis 1: cafe-au-la it spots, macrocephaly, feeding
problems, short stature, and learning disabilities.
. Patients may later develop fibromas, neurofibromas or different tumors.
- The characteristic features of neurofibromatosis 2: are bilateral acoustic neuromas and
cataracts.
. Strokes in children:
______________________
. are uncommon but When they occur, they are often the result of a congenital abnormality,
infection, or systemic illness.
. Many childhood strokes are caused by sickle cell anemia, and suspected by (patient's race,
history of dactylitis, and prior pain crisis are all consistent with sickle cell disease).
. The mechanism of strokes in children with sickle cell anemia is not fully understood, but
RBCs adherence to the endothelium, activation of van Willebrand's factor,
and hyperviscosity are all thought to contribute.
. Acute strocke syndrome:
_________________________
. Children who have trauma to the soft palate with a sharp object can either compress the
internal carotid artery (causing a thrombosis that embolizes
to the brain and causes stroke) or dissect the internal carotid artery (leading to ischemic
stroke).
. The onset of symptoms can be delayed up to 24 hours after the traumatic episode.
. The diagnosis should be suspected clinically and can be confirmed with MRl/MRA of the
brain.

. Treatment is somewhat controversial and ranges from close observation with supportive care
to aggressive treatment with anticoagulation, with similar outcomes.
. Epidural hematomas:
_____________________
. usually caused by injury to meningeal blood vessels & do not usually cross suture lines
because of the better dural attachment at the sutures.
. The classic history of an epidural hematoma consists of a direct head injury followed by a
lucid interval and then
. rapid neurologic deterioration with headache, vomiting, seizures, confusion, and lethargy.
. Affected patients can progress to coma and death due to herniation if not treated.
. Infants with open fontanelles may present with bulging fontanelles, irritability, seizures, and
hypotonia.
. Older children and adolescents usually present with more classic symptoms of headache,
vomiting, and deteriorating mental status.
. CT scan of the head is the diagnostic test of choice --> a biconvex mass.
. The key to evaluating a child with an epidural hematoma is to search for the presence of
clinical features that necessitate an emergent craniotomy.
. which include any of the following: GCS < 8, signs of increased ICP, pupillary abnormalities,
hemiparesis, or cerebellar signs.
. Febrile seizure:
___________________
. Diagnostic Criteria of Febrile seizures:
- Seizure associated with a temperature >38C (1004F).
- Age <6 years.
- No CNS infection, No acute systemic metabolic cause of seizure & No history of previous
afebrile seizures.
. classified as: simple or complex.
. A simple febrile seizure is a generalized tonic-clonic seizure that lasts <15 minutes and
usually occurs once in a 24-hour period.
. It does not cause permanent brain injury or significantly increase the child's future risk of
epilepsy.
. Although febrile seizures can recur, children who experience simple febrile seizures are less
likely to have a recurrence.
. A complex febrile seizure is diagnosed if the seizure is focal, lasts >15 minutes, or occurs
more than once with a total duration of seizure activity >30 minutes.
. Children with complex febrile seizures are at increased risk for future febrile or afebrile
seizures, especially if there is family history of epilepsy in a parent or sibling.
. Febrile seizures are associated with viral or bacterial infections (e.g. otitis media) However,
serious intracranial infections (e.g. meningitis) must be ruled out.
. Once an intracranial infection has been ruled out, the patient can be treated for the fever and
underlying infection.
. The patient can then be discharged home with parental education about the use of
antipyretics and seizure precautions.
. Cerebral palsy:
_________________
. Cerebral palsy is a non-progressive disorder characterized by impaired motor functioning.
. The most common cause is cerebral anoxia.
. Present as: hypotonia, hyperactive deep tendon reflexes, learning disabilities, along with an
intrapartum history of prolonged labor and low APGAR scores at birth.
. Hydrocephalus:
________________

. Symptoms: Poor feeding, irritability, decreased activity, Vomiting.


. Physical examination: Tense and bulging fontanelle, Prominent scalp veins, widely spaced
cranial sutures, rapidly increasing head circumference.
. The best diagnostic approach in symptomatic children with rapidly increasing head
circumference is to perform diagnostic imaging.
. A CT scan of the brain is the best initial choice in an infant who is acutely symptomatic & will
reveal ventricular dilation as well as the infant's anatomy.
. In otherwise stable and asymptomatic infant, a sedated MRI could be considered to spare the
child radiation exposure.
. Both CT and MRI provide greater detail than ultrasound, which requires a widely open
anterior fontanelle and is most useful in infants under 6 months.
. Treatment: a shunt that is placed from the ventricle to the peritoneum, pleura, or right
atrium; which allows the excess CSF to drain.
. Guillain-Barre syndrome:
___________________________
. Suspect Guillain-Barre syndrome in a child who presents with an ascending polyneuropathy
one week after an apparent viral infection (e.g. diarrhea).
. The underlying pathology involves mainly the peripheral motor nerves, although sensory and
autonomic nerves may also be affected.
. Friedreich Ataxia:
_____________________
. an autosomal recessive condition and genetic counsel ing is recommended for prenatal
diagnosis for parents with one affected child.
. FA is associated with necrosis and degeneration of cardiac muscle fibers leading to
myocarditis, myocardial fibros is and cardiomyopathy.
. Cardiac arrhythmia and congestive heart failure contribute to a significant number of deaths.
. Klumpke's paralysis:
_____________________
. is a brachial palsy that occurs in newborns following excessive traction on the arm.
. It consists of hand paralysis and ipsilateral Homer's syndrome (ptosis and miosis).
. It is secondary to injury to seventh and eighth cervical nerves and first thoracic nerve.
. Confirmation MRI, which demonstrates nerve root avulsion or rupture.
. Treatment depends on the severity of the injury and, generally, it consists of partial
immobilization and appropriate positioning to prevent contractures.
. Gentle massages and range-of-motion exercises can be started by 7-10 days of age.
. If by 3-6 months there is no improvement, neuroplasty, neurolysis, end-to-end anastomosis,
and nerve grafting may be attempted.
. Erb-Duchenne palsy:
_____________________
. is another type of brachial palsy in which the injury involves the fifth and sixth cervical
nerves.
. It presents with: absent Moro reflex and intact grasp reflex of the affected arm.
. Patients present with a characteristic position, which consist of adduction and internal
rotation of the arm with pronation of the forearm.
. Subarachnoid hemorrhage (SAH):
________________________________
. SAH can be caused by an intraventricular hemorrhage, which is common in premature infants.
. Accumulation of the blood in the subarachnoid space may lead to destruction of the
arachnoid villi and cisterns, thereby blocking the flow or decreasing the

absorption of CSF, and leading to communicating hydrocephalus.


. SAH is the most common cause of communicating hydrocephalus.
. CT scan: dilation of the entire ventricular system with distinct enlargement of the
subarachnoid space over the cerebral cortex is very suggestive of nonobstructive
or communicating hydrocephalus secondary to a subarachnoid hemorrhage (SAH).
.N.B:
- Dandy-Walker anomaly and Chiari malformation will both reveal CT findings consistent with
obstructive or noncommunicating hydrocephalus.
. Dandy-Walker Anomaly will demonstrate a cystic expansion of the fourth ventricle, and Chiari
malformation will reveal protrusion
of the structures of the posterior fossa through the foramen magnum.
============================================================================
=== Hematology ===========================================================
. Acquired aplastic anemia:
___________________________
. present by: Normocytic or macrocytic anemia, Leukopenia, reticulocytopenia and
thrombocytopenia.
. B.M. biopsy (essential for diagnosis) --> profound hypocellularity in all cells with fatty
infiltration.
. Result from B.M. supression due to acquired or congenital causes:
A) Acquired Causes of Aplastic Anemia:
- Idiopathic.
- Drugs (e.g, NSAIDs, sulfonamides, etc).
- Toxic chemicals (e.g. benzene, glue, etc ).
- Viral infections (e.g. HIV, EBV, etc ).
- Immune disorders.
- Thymoma.
B) Congenital causes of aplastic anemia --> Fanconi's anemia.
. Fanconi's anemia:
____________________
. are more common in children, and Fanconi anemia (FA) is the most common congenital cause.
. It is an autosomal recessive or X-linked disorder associated with the clinical manifestations
summarized below.
. Most patients with FA are diagnosed by the age of 16 years and have a predisposition for
developing cancer.
. Numerous genes, all believed to involve DNA repair, have been implicated.
. Bone marrow: Aplastic anemia and progressive bone marrow failure.
. Appearance: Short stature, microcephaly, abnormal thumbs, upper limb anomalies, renal
malformations and hypogonadism.
. Skin: Hypopigmented/hyperpigmented areas, cafe au lait spots, and large freckles.
. Eyes/ears: Strabismus, low-set ears, and middle ear abnormalities (e.g. hemorrhage,
incomplete development, chronic infections, deafness, etc ).
. Diagnosis of FA is made by chromosomal breaks on genetic analysis combined with the
clinical findings.
. The patient likely has bleeding secondary to thrombocytopenia, fatigue from macrocytic
anemia, and pounding in his ears
from possible conduction deifects or chronic hemorrhage.

. The definitive treatment for aplastic anemia is hematopoietic stem cell transplantation.
. Blood counts start to decrease at the age of 4-12 years.
. Intial manifestations: Thrombocytopenia followed by neutropenia then anemia.
. Diamond-Blackfan anemia (DBA): congenital hypoplastic anemia
______________________________
. Congenital pure red cell aplasia.
. Macrocytic anemia, low reticulocyte count, and congenital anomalies.
. The majority of cases are sporadic, although dominant and recessive inheritance is found in
15 % of cases.
. The primary pathology is an intrinsic defect of erythroid progenitor cells which results in
increased apoptosis.
. Present in the 1st 3 months with pallor and poor feeding.
. Congenital anomalies: Cleft palate, webbed neck, sheilded chest, triphalangeal thumbs and
cardiac anomalies.
. The macrocytic anemia of DBS is distinct from that of megaloblastic anemia as there is no
hypersegmentation of the nucleus in neutrophils & other blood cells in the former.
. Electrophoresis reveals elevated fetal Hb levels
. Chromosomal studies are normal.
. Therapy is mainly corticosteroids. For unresponsive patients, transfusion therapy is indicated.
. CBC --> Normocytic or macrocytic anemia with reticulocytopenia.
. WBCs and Platelet counts are normal.
. Transient Erythroblastopenia of childhood (TEC):
__________________________________________________
. Acquired red cell aplasia which occurs in healthy children between 6 months and 5 years.
. Gradual onset of symptoms as pallor and dcreased activity.
. Lab. --> Normocytic normochromic anemia with Hb level 3-8 mg/dl and extremely low
reticulocytic count.
. Glucose-6-phosphate dehydrogenase (G6PD) deficiency:
__________________________________________________
. The most common RBC enzyme deficiency causing anemia is (G6PD) deficiency.
. In affected patients, oxidant drugs (e g , antimalarials and sulfas) and infection can cause
episodic hemolysis.
. Thalassemias:
_______________
. Autosomal recessive disorders of hemoglobin seen most commonly in people of
Mediterranean descent.
. Clinical symptoms can range from minor to severe depending on the number of globin chains
that are affected.
. Individuals with thalassemia trait usually have no symptoms and are diagnosed when routine
laboratory studies reveal a microcytic anemia with a normal RDW.
. Clinically, thalassemia is confused with iron deficiency anemia; If microcytic anemia does not
respond to empiric oral iron therapy, iron studies should be done.
. If the patient has normal iron studies, which rules out iron deficiency and makes thalassemia
trait much more likely.
. Hemoglobin electrophoresis can be performed to confirm the diagnosis.
. For individuals with thalassemia trait, no treatment is usually necessary.
. Sickle cell disease:
______________________

. Characterized by chronic hemolysis of sickled cells, leading to a high RBC turnover and
anemia.
. Patients with sickle cell anemia will have sickle cells and reticulocytes on their peripheral
blood smear.
. Reticulocytes are present because the bone marrow is attempting to compensate for the
chronic hemolysis characteristic of sickle cell anemia.
. Painful crisis, the most common manifestation of sickle cell anemia.
. These episodes due to intermittent vasoocclusion in connective tissue & muscle, are
characterized by severe acute pain & tenderness, fever, tachycardia, and anxiety.
. Hemolysis is mainly extravascular and leads to reticulocytosis, hyperbilirubinemia and
elevated serum LDH with low serum haptoglobin.
. The hematocrit is generally 20-30 percent, owing to the decreased RBC volume.
. Iron deficiency anemia is not common in SCD as the hemolysis of RBC is extravascular, and
the iron is usually restored and reused;
. however, iron deficiency secondary to increased utilization or urinary iron loss can be present
in up to 20% of patients.
. Megaloblastic anemia occurs in vitamin B12 or folic acid deficiency, which is not usually seen
with SCD;
although folate deficiency secondary to increased folate utilization can be seen in some
individuals.
. Anemia of chronic disease may occur in SCD if iron reuptake and use is decreased due to
infection, inflammation or some chronic illness.
. Iron deficiency, folate deficiency, and anemia of chronic disease occur less frequently in SCD
patients.
. Sideroblastic anemia is seen in inherited or acquired defects affecting the biosynthesis of
heme within red cell precursors not related to SCD.
. Salmonella and Staph aureus are the most common cause of osteomyelitis in patients with
sickle cell disease.
. Pneumococcal vaccination plus penicillin prophylaxis can prevent almost all cases of
pneumococcal sepsis in patients with sickle cell anemia.
. Those with sickle cell trait have some protection against malaria.
. Complications of sickle cell disease:
-------------------------------------. Sickle cell disease is a chronic, well-compensated hemolytic anemia with appropriate
reticulocytosis.
. An acute drop in hemoglobin is can occur secondary to a hyperhemolytic crisis, splenic
sequestration, or an aplastic crisis.
1. An aplastic crisis: a transient arrest of erythropoiesis that results in a severe drop in
hemoglobin and virtual absence of reticulocytes on peripheral smear (reticulocytes < 1%).
. It may be caused by infections such as parvovirus 819.
. Treatment generally consists of a blood transfusion.
. It is important to note that an aplastic crisis in sickle cell disease differs from aplastic
anemia, which occurs in other patients & characterized by pancytopenia.
2. A hyperhemolytic crisis: is a rare complication of sickle cell disease that is characterized by
sudden, severe anemia accompanied by an appropriate reticulocytosis.
3. Splenic sequestration: occurs in sickle cell patients who have not yet developed autosplenectomy.
. This is caused by vasoocclusion and pooling of red cells within the spleen, which may lead to
severe hypotensive shock.

. It is characterized by a dramatic drop in hemoglobin concentration that is accompanied by


persistent reticulocytosis.
. The classic physical examination finding is a rapidly enlarging spleen.
. Splenic sequestration carries a mortality of 10-15% and a 50% chance of recurrence, so
splenectomy is usually recommended after the first episode.
4. Acute chest syndrome: is characterized by the presence of fever, chest pain, and an infiltrate
on chest X-ray.
. The etiology of acute chest syndrome is thought to be multifactorial and related to pulmonary
infarction and infection.
. While a drop in hemoglobin may be seen, severe anemia without associated reticulocytos is is
not seen in acute chest syndrome
5. A vasoocclusive (painful) crisis: is characterized by acute onset of pain and is caused by
vasoocclusion secondary to the sickling of red blood cells.
. These crises may be precipitated by changes in weather, dehydration, or infection.
. Treated by adequate hydration and opioid analgesics.
. Hydroxyurea is indicated in sickle cell patients with frequent, acute, painful episodes, as it
tends to increase the Hb F levels.
. Hb F: retard sickling, so increased HbF levels lead to reduced sickling of red blood cells and
consequently, decrease episodes of vaso-occlusion.
. Severe anemia and a low reticulocyte count are not seen in vasoocclusive crises
.N.B:
- patient has sickle cell trait, which is characterized by a Hb S concentration ranging from 3540% (sickle cell disease Hb S > 40%).
- Individuals with sickle cell trait have a heterozygous genotype (AS) and are generally
asymptomatic.
- Symptoms occur very rarely, and the most common of these is painless hematuria.s
- In a patient with a chronic hemolytic anemia and compensatory reticulocytosis, increased
demand for folic acid can lead to folic acid deficiency,
- For this reason, patients with sickle cell anemia should be on folic acid supplementation
- vitamin B12 is also required However, in contrast to folic acid, B12 stores can be adequately
maintained by most every patient via diet alone.
. Hereditary Spherocytosis:
___________________________
. Is a common cause of hemolysis and hemolytic anemia in the newborn.
. Indicators of hemolysis: reticulocytosis and hyperbilirubinemia.
. A positive Coombs' test points towards autoimmune hemolytic anemia, and a positive
osmotic fragility test indicates hemolysis confirms hereditary spherocytosis.
. The peripheral smear may reveal anisocytosis and polychromatophilic reticulocytes and
spherocytes.
. Severe anemia and hyperbilirubinemia may require phototherapy or exchange transfusions.
. HS is an autosomal dominant trait and is the most common hereditary hemolytic anemia in
white population.
. There is congenital RBC membrane defect in HS resulting in extravascular hemolysis
occurring only in the presence of spleen.
. The osmotic fragility test is the next best test for the diagnosis of the patient.
.N.B: Other types of anemia:
----------------------------

1- Iron deficiency anemia often presents with pallor, pica, and fatigue, although these
symptoms are often not present in children.
- Laboratory studies typically show a microcytic, hypochromic anemia with a low reticulocyte
count, high ROW, low serum iron, high TI BC, and low serum ferritin.
2- Anemia of chronic disease is unlikely in a child with no medical problems.
- This type of anemia can be normocytic or microcytic, and laboratory studies will show a low
serum iron and low TIBC.
3- Vitamin B12 deficiency presents as a macrocytic anemia with an elevated MCV.
4- Reticulocytosis is a physiologic response of the bone marrow to anemia provided adequate
iron stores are present.
5- In hemolytic anemia (spherocytosis), the reticulocyte counts are much higher (sometimes as
high as 9%) Spherocytosis is unlikely with just 3% reticulocyte count.
.N.B.:
- Approximately 5%- 15% of the patients with thymic tumors have pure red cell aplasia, This
finding is most common in older women.
- Sugar water test is done for the diagnosis of Paroxysmal Nocturnal Hemoglobinuria (PNH).
. Anemia of premature infant:
_____________________________
. Premature, 8 weeks of age, pallor is suggestive of mild anemia.
. This condition usually presents at the second or third month of life, and the common
symptoms and signs are pallor, tachycardia, and tachypnea.
. Although the infant may be getting iron via breast feeding, iron supplementation in the form
of oral ferrous salts or iron-fortified milk formula is still warranted.
. Anemia of prematurity:
_________________________
. is the most common anemia in premature and low birth weigh infants.
. The pathology involves a combination of diminished RBC production, shortened RBC life
span, and blood loss.
. Iron supplementation does not prevent falling hemoglobin levels, and iron deficiency is not
the cause of anemia of prematurity.
. The treatment involves iron supplementation, periodic hemoglobin checking and blood
transfusion, if needed. Erythropoietin is not routinely used.
. Iron deficiency anemia:
_________________________
. is common in infants and toddlers who drink excessive amounts of cow's milk.
. Empiric oral iron therapy is appropriate for children with microcytic, hypochromic anemia.
. Term infants usually have adequate iron stores to sustain them for the first six months of
life.
. After six months, dietary factors (insuffic ient intake, malabsorption) become the most
important cause of iron deficiency.
. Early introduction to or excessive intake of cow's milk is of particular concern because cow's
milk is low in iron and can cause occult intestinal blood loss in infants.
. Children should not be started on cow's milk until one year of age,
. Microcytic anemia:
____________________
. caused by iron deficiency can often be differentiated from thalassemia by an elevated RDW,
which is typically greater than 20% in iron deficiency.
. The reticulocyte count is low in patients with iron deficiency due to decreased erythropoiesis.

. N.B: Howell-Jolly bodies:


-----. These bodies are nuclear remnants within RBCs that are typically removed by the spleen.
. They are present on peripheral blood smear as single, round, blue inclusions on Wright stain.
. Their presence indicates physical absence of spleen or functional hyposplenism due to
splenic autoinfarction, infiltrative disorders of spleen, or splenic congestion.
. Leukemia:
___________
. Leukemic patients can present with numerous infections, diarrhea and failure to thrive.
. Although such patients present with more serious bacterial and fungal infections.
. Hepatomegaly and splenomegaly may be found.
. Laboratory studies reveal: leukocytosis, thrombocytopenia and anemia. Other findings may
include hyperuricemia,hyponatremia, elevated LDH and hypokalemia.
. Acute lymphoblastic leukemia:
______________________________
. Acute lymphoblastic leukemia is the most common leukemia in children.
. The first symptoms are nonspecific: anorexia, irritability and lethargy.
. Patients may have a history of viral respiratory infection or exanthem from which they do not
fully recover.
. On physical examination, there may be pallor, hepatosplenomegaly, petechiae, and/or
lymphadenopathy.
. Diagnosis of leukemia is suggested by presence of anemia, thrombocytopenia and blast cells
on peripheral blood smear, but is confirmed by B.M. examination.
. Lymphoblasts lack peroxidase positive granules but often contain cytoplasmic aggregates of
periodic acid Schiff (PAS) positive material.
. Immunostaining for terminal deoxynucleotidyltransferase (TdT) is positive in more than 95%
of patients TdT is expressed only by pre B and pre T lymphoblasts.
. Myeloblasts on the other hand contain peroxidase positive material.
. Acute myeloid leukemia (AML):
______________________________
. Seen more commonly in adults.
. It can present similarly to ALL with anemia, thrombocytopenia and high WBC count.
. The main differentiating and diagnostic factor is demonstration of greater than 25% of
myeloblasts in the bone marrow.
. Chronic lymphocytic leukemia (CLL):
__________________________________
. usually presents in elderly patients.
. with fatigue, lymphadenopathy and splenomegaly.
. Labs show lymphocyte count of greater than 5000/mm3, made of mature-appearing cells.
. Hodgkins' disease (HD):
_________________________
. The most common presenting sign is Painless, firm, cervical or supraclavicular adenopathy.
. Depending on the extent and location of nodal and extranodal disease, there maybe
symptoms and signs of airway obstruction, pleural or pericardial effusion,
hepatocellular dysfunction, or bone marrow infiltration.
. Although signs and symptoms are similar to ALL, presence of lymphoblasts makes ALL most
probable.

. Burkitt lymphoma:
___________________
. It is a neoplasm of mature B cells. It is associated with the Epstein-Barr virus infection.
. Most patients present with either a mass involving the mandible or abdominal viscera.
. High mitotic index is typical. Histological examination shows characteristic "starry sky
appearance".
. It is a very aggressive tumor but responds well to the high dose chemotherapy.
. Myelodysplastic syndromes (MOS):
__________________________________
. They are clonal stem cell disorders which may progress to acute leukemias.
. These are usually seen in elderly patients, and are characterized by pancytopenia.
. Henoch-Schonlein Purpura:
___________________________
. Is an lgA-mediated vasculitis of small vessels, affecting children more than adults.
. Many cases follow an upper respiratory tract infection.
. The clinical manifestations: a classic tetrad of rash, arthralgias, abdominal pain, and renal
disease.
. The rash is typically purpuric and distributed symmetrically over the lower legs, buttocks and
arms.
. Arthralgias most commonly affect the knees and ankles, these symptoms are always
transient, and there is no permanent damage to the joints.
. GIT symptoms most commonly colicky abdominal pain (mostly due to local vasculitis) that is
frequently associated with vomiting,
. Two common pathologies which should be ruled out emergently are GI bleeding and
intussusception ( a surgical emergency, and is treated with air/barium enema).
. Renal involvement relatively mild disease characterized by asymptomatic hematuria &
proteinuria with a normal or slightly elevated plasma creatinine; however,
. More marked findings may occur including the nephrotic syndrome, hypertension, & acute
renal failure (treatment includes steroids and monitoring of renal function).
. Other organs as the lung and central nervous system, may be involved.
. Confirmation of the diagnosis requires evidence of tissue deposition in the skin or kidney of
lgA by immunofluorescence microscopy.
N.B.:
1- Minimal change disease: electron microscopy uasually shows Podocyte fusion in the
glomerulous.
2- Membranous nephropathy: electron microscopy shows Glomerular basement thickening & is
considered the most common cause of adult nephrotic syndrome.
3- focal segmental glomerulosclerosis: electron microscopy shows Focal and segmental
sclerosis.
4- Goodpasture's syndrome: electron microscopy shows Linear deposition of lgG on the
basement membrane is seen.
. Lymphadenitis in children:
_____________________________
. Cervical lymphadenopathy is common in children.
. Lymphadenitis is diagnosed when the lymph node becomes tender and erythematous in
addition to being enlarged.
. Lymphadenopathy maybe acute or subacute/chronic, and unilateral or bilateral.
. Acute, unilateral lymphadenitis in children is usually caused by bacterial infection.
. Staphylococcus aureus is the most common followed by group A streptococcus.

. Patients with bacterial lymphadenitis are usually less than 5 years old and nontoxic
appearing.
. The affected lymph node is tender, warm, erythematous, and usually 3 to 6 cm in size.
. In some cases, the infection can progress to induration and fluctuance.
N.B.: Other causes of unilateral lymphadenitis:
1- Tularemia: caused by Francisella tularensis, can present with acute unilateral cervical
lymphadenopathy Affected children.
. usually have fever, chills, headache, and malaise in addition to the lymphadenopathy.
. In addition, tularemia is a zoonosis and presents after contact with an infected animal (e g
rabbits, hamsters, or blood-sucking arthropods).
2- Peptostreptococcus is an anaerobic bacteria that can cause acute, unilateral lymphadenitis
However,
it is usually seen in older children with a history of periodontal (gum) disease.
3- Nontuberculous mycobacteria (most commonly Mycobacterium avium-intracellulare)
. are one cause of unilateral subacute-chronic lymphadenopathy Affected children are usually
less than 5 years old and
. present with firm, nontender lymphadenopathy that is usually less than 4 cm in size.
. The skin over the lymph node often thins and develops a violaceous color.
. Fever and tenderness are unusual with this infection.
4- Epstein-Barr virus usually presents with bilateral subacute-chronic lymphadenopathy along
with
systemic symptoms such as fever, pharyngitis, hepatosplenomegaly and lymphocytosis.
. GeneticB-cell deficiencies:
_____________________________
. Patients with genetic B-cell deficiencies begin to develop recurrent infections after passing 6
months (when the newborn's levels of maternal antibodies drop).
. The deficient humoral immune response in these patients impairs the body's ability to
destroy encapsulated organisms.
. Hence, recurrent sinopulmonary infections with H. influenzae and S. pneumoniae are
common.
. Lack of lgA also predisposes to Giardia infection.
. Severe combined immune deficiency (SCID):
__________________________________________
. is a life-threatening syndrome due to adenosine deaminase deficiency & is an autosomal
recessive.
. Patients typically present with recurrent sinopulmonary infections, oral candidiasis,
persistent diarrhea, opportunistic infections and viral infections.
. The diagnosis is confirmed by the following clinical features absent lymph nodes and tonsils,
lymphopenia, absent thymic shadow on chest x-rays,
. Lymphopenia is a classic finding.
. And abnormal T, B, and natural killer cell enumeration by flow cytometric analysis.
. Bruton's agammaglobulinemia: X-linked agammaglobulinemia
______________________________
. Also known as X-linked agammaglobulinemia
. is an X-linked recessive disorder characterized by a defect in Bruton's tyrosine kinase, a
signaling molecule expressed in B cells.

. Patient's presentation is classic a male infant who is initially asymptomatic until 6-9 months
of age,
after which he begins to experience recurrent pyogenic (S. pneumoniae and H. influenzae)
infections.
. absent tonsils and other palpable lymphoid tissue
. There is normal number of T lymphocytes (CD3-positive) but a very low number of B
lymphocytes (CD19-positive).
. The low number of B lymphocytes leads to decreased immunoglobulin (Low Serum lgG, lgM
and lgA), causing decreased ability of immune system to attack bacteria.
. The maternal lgG antibodies are responsible for keeping the infant free from infections in the
first 6-9 months of life.
. The diagnosis is confirmed by decreased serum concentrations of lgG, lgA, lgM and lgE, along
with absent or markedly decreased B cells on the smear.
. Treatment intravenous immune globulin ( IVIG) infusion.
. Fortunately, patients receiving regular infusions of IVIG may have an almost normal life.
. Hyper-lgM syndrome (HIM):
____________________________
. Characterized by high levels of lgM with deficiency of lgG, lgA and poor specific antibody
responses to immunizations.
. HIM presents with recurrent sinopulmonary infections and Pneumocystis jiroveci pneumonia.
. The unique susceptibility to opportunistic infections and neutropenia, with high lgM levels,
distinguishes HIM from XLA or other hypogammaglobulinemias.
. Common variable immunodeficiency (CVID):
__________________________________________
. Also known as acquired hypogammaglobulinemia.
. Similar presentation to Bruton's agammaglobulinemia; however, patients with (CVID) usually
manifest with less severe symptoms at a later age (about 15-35-yrs old).
. The serum concentrations of lgG, lgA, lgM and lgE may also be decreased, but there is no
absence or decrease in the number of circulating B cells.
. Furthermore, there is no sexual predominance in CVID.
. Transient hypogammaglobulinemia (THI):
________________________________________
. Characterized by decreased lgG levels, normal lgA levels, and variable lgM levels.
. These immunoglobulin levels all normalize by 6-11 months of age.
. lgA deficiency:
_________________
. Selective lgA deficiency is characterized by decreased lgA levels (less than 50mcg/ml) with
normal serum concentrations of other immunoglobulins,
although lgG2 subclass deficiency may occur.
. There is no other evidence of any immune defects.
. Patients commonly present with respiratory, gastro intestinal, and genitourinary infections +
transfusion reaction.
. Recurrent sinopulmonary (Streptococcus & Hemophilus) and GI infections (Giardia), a
consequence of lgA's role in mucosal barrier protection.
. Patients with lgA deficiency may also form antibodies to lgA, predisposing them to
anaphylactic transfusion reactions.
. DiGeorge syndrome:
____________________
. DiGeorge syndrome is an example of a contiguous gene syndrome in which neighboring genes
on a chromosome are deleted.

. DiGeorge syndrome is specifically caused by the microdeletion of 22q11.


. The underlying pathology is a defect in the development of the 3rd and 4th pharyngeal
pouches, leading to: cyanotic heart disease,
. craniofacial skeleton anomalies (low-set ears and micrognathia), absent thymus, absent
parathyroid glands, recurrent infections, or cognitive impairment.
. Absence of the parathyroid glands result in hypocalcemia; therefore, the patient's calcium
levels must be monitored during surgery.
. Prolonged QT intervals may be caused by hypocalcemia.
. Severe calcium deficiency can lead to laryngeal spasm, convulsions (a neonate with
hypocalcemic seizures), and cramping.
. Thymic aplasia and consequent T cell deficiency, these children are susceptible to fungal
infections.
. Other congenital defects are esophageal atresia, anomalies of the great vessels, mandibular
hypoplasia.
. Chronic granulomatous disease (CGD):
_______________________________________
. Is a defect of phagocytic cells due to dysfunction of the NADPH oxidase enzyme complex
(impaired oxidative metabolism within the phagocytes) leading to:
. Recurrent and uncontrolled infections with catalase-positive organisms, as S. aureus, Serratia
marcescens, Burkholderia cepacia, Klebsiella, and Aspergillus sp.
. Patients are not susceptible to catalase-negative organisms (Streptococcus pyogenes,
Streptococcus pneumoniae, and Haemophilus influenzae).
. CGD should be suspected in all patients presenting with unexplained infections caused by
Aspergillus, Serratia marcescens, and Burkholderia cepacia.
. The most common findings are lymphadenopathy, hypergammaglobulinemia, hepatomegaly,
splenomegaly, anemia of chronic disease,
underweight, chronic diarrhea, short stature, gingivitis, and dermatitis.
. Neutrophilic functions such as chemotaxis, phagocytosis and degranulation are intact in these
patients; only intracellular killing is deficient.
. Thus neutrophils filled with bacteria are a classic finding on Gram stain.
. The diagnosis is made by nitroblue tetrazolium (NBT) slide test, flow cytometry, or
cytochrome C reduction.
. Treatment includes prevention of infection with daily trimethoprim-sulfamethoxazole and
gamma-interferon three times a week.
. Bone marrow transplantation is experimental but curative.
. Wiskott-Aldrich syndrome (WAS):
__________________________________
. An X-linked disorder characterized by the triad of thrombocytopenia, eczema, and recurrent
bacterial infections.
. Recurrent infections are usually due to Streptococcus pneumoniae, Neisseria meningitidis,
and Haemophilus influenzae.
. The thrombocytopenia is caused by decreased platelet production, and the few platelets that
exist are typically quite small.
. Platelet counts in patients with WAS are typically less than 50,000/mm3 and the mean
platelet volume is also usually reduced.
. Decreased platelet production is the primary pathophysiologic cause of thrombocytopenia in
patients with WAS.
. Idiopathic thrombocytopenic purpura ( ITP):
_____________________________________________
. An immune-mediated disorder caused by antibodies that bind to platelets and facilitate their
removal from the circulation.

. Affected children typically present with easy bruisability or bleeding.


. Intraventricular hemorrhage ( IVH):
_____________________________________
. IVH is most commonly seen in premature and LBW infants.
. Patients may present with pallor, cyanosis, hypotension, seizures, focal neurologic signs,
bulging or tense fontanel, apnea and bradycardia.
. However, many cases remain asymptomatic, thus mandating transfontanel ultrasound for all
newborns with predisposing risk factors.
. Studies have shown that the incidence of IVH is inversely proportional to birth weight (the
lower the birth weight, the greater the likelihood of IVH).
. Pelvic dystocia, prenatal infections, congenital anomalies, and macrosomia are not directly
incriminated in IVH.
. Hemorrhagic disease of the newborn (HDN):
___________________________________________
. Routine perinatal care, includes parenteral injection of vitamin K after delivery.
. This shot is given to prevent hemorrhagic disease of the newborn (HON), a potential
consequence of vitamin K deficiency.
. Humans obtain vitamin K from two sources diet and via the gut flora.
. Deficiency in newborn babies is the result of poor placental transfer, absent gut flora, and
inadequate levels in breast milk.
. HDN results from deficiency of vitamin K-dependent clotting factors (II, VII, IX, X).
. Signs include bruising, bloody stools, and (less commonly) intracranial hemorrhage.
. Prolonged PT and PTI support the diagnosis, but the diagnosis is confirmed by reversal of
symptoms with vitamin K administration.
. Mild vitamin K deficiency may present with an elevated PT level and a normal PTT level.
. Polycythemia:
_______________
. Defined as a central venous hematocrit level greater than 65%.
. The most common cause of polycythemia in term infants is delayed clamping of the umbilical
cord resulting in excess transfer of placental blood.
. As the hematocrit rises, the viscosity of the blood increases as well as the arterial oxygen
content, this change in blood flow may affect various organs.
. In newborns, the hematocrit level peaks within 12 hours of birth and then decreases over the
subsequent 12 hours.
. Not all infants with polycythemia become symptomatic However, symptomatic infants may
present with manifestations to any organ.
. The most common findings are plethora and CNS disturbances like lethargy, irritability,
jitteriness, and seizures.
. Other common symptoms are respiratory distress, tachypnea, and cyanosis, caused by the
decrease in pulmonary blood flow due to the hyperviscosity of blood.
. Poor feeding, hypoglycemia, and hypocalcemia are also commonly seen in polycythemic
infants.
. Treatment includes hydration and, if symptomatic, partial exchange transfusion.
. Purpura in children:
______________________
. The common causes of purpura in children: thrombocytopenia, clotting factor deficiencies,
and vasculitis.

. Nearly 2.5% of the normal population have a platelet count of< 150,000/?L, and clinical
manifestations of thrombocytopenia (e g petechiae, purpura) usually do
not occur unless the platelet count is < 100,000/?L.
. Common clotting factor deficiencies causing purpura in children: van Willebrand disease,
hemophilia A (factor VIII deficiency) & hemophilia B (factor IX deficiency).
. These conditions do not usually cause abdominal pain and other renal findings (proteinuria).
. If there is no thrombocytopenia (paltelets > 100.000), then vasculitis should be high in
suspecious, especially if there is no abdominal pain or renal affection.
. Henoch Schonlein purpura (HSP): a small vessel vasculitis that usually presents with the
classic tetrad
. lower extremity palpable purpura, arthritis/arthralgias, abdominal pain, and renal disease.
. HSP is the most common form of systemic vasculitis in children.
. 50% of patients develop HSP after upper respiratory infection & have other nonspecific
symptoms as subcutaneous edema, joint pains, bloody stools, and scrotal edema.
. Small bowel intussusception is a common complication of HSP.
. Laboratory testing is usually normal (including a normal platelet count), although hematuria
or proteinuria may be found.
. Diagnosis is usually based on clinical findings, and skin biopsy is rarely necessary.
. Treatment is usually supportive, but NSAIDS are sometimes given to abdominal pain.
.N.B: Purpura fulminans:
- seen with bacterial infections (e.g. N. meningitidis, S. pneumoniae) However, patients are
usually very ill with fever, hypotension, and evidence of DIC.
. Immune thrombocytopenia:
________________________
. usually occurs in children between 2 and 6 years of age.
. The pathogenesis involves antibodies that bind to platelets, and subsequent destruction of
these complexes in the spleen.
. The condition is usually preceded by a viral infection and presents with purpura, petechiae,
hematuria or GI bleeding & there is no adenopathy.
. Laboratory studies usually show no abnormalities, except thrombocytopenia.
. The course is self-l imited, and spontaneous recovery occurs in the majority of patients.
. There is controversy on treatment, but corticosteroids are drugs of choice in all age groups
for thrombocytopenia less than 30,000/mm3, and/or for severe symptoms.
. Patients with a platelet count of more than 30,000/mm3 usually have very few symptoms and
do not require treatment (Observation).
. Hemophilia:
_____________
. Presence of spontaneous hemarthrosis and soft tissue hematomas raise the suspicion for
hemophilia, for which factor VI 11 assay is diagnostic.
. Spontaneous hearthrosis is very unlikely with any other form of bleeding disorder, including
Von Willebrand's disease.
. The best next step is a complete set of coagulation studies followed by factor VIII and IX
levels.
. Prolonged PTI, normal prothrombin time, normal bleeding time, normal fibrinogen level and
low serum factor VIII activity are the typical lab findings.
. The standard treatment for hemophilia is to replace the factor VIII.
. However, mild hemophilia may be treated with desmopressin (DDAVP), which causes release
of factor VIII from the endothelial cells.
. Leukocyte adhesion defect:
____________________________

. An immune deffeciency disease present with: recurrent bacterial infections and necrotic
periodontal infection.
. Delayed separation of the umbilical cord (>3 weeks) is characteristic and presents an
important clue to the correct diagnosis.
. The leukocytes fail to express some adhesion molecules on their surface.
. The leukocyte number is increased, but the inter-leukocyte communication is defective;
therefore, chemotaxis and cytotoxicity are impaired.
. Normal lymphocyte count and gamma globulin concentration help to differentiate this
condition from a variety of cell and/or humoral immune defects.
. Hemolytic uremic syndrome: Extremely high-yield question for the USMLE!!!
____________________________
. It is usually preceded by an acute diarrheal illness due to the pathogens Escherichia coli
serotype 0157 H7, Shigella, Salmonella, Yersinia, and Campylobacter.
. It is less commonly preceded by an upper respiratory infection.
. GI bleeding is common, Physical examination frequently reveals purpura and hypertension.
. The hallmark finding is microangiopathic hemolytic anemia.
. Other typical features: acute renal failure, fever, oliguria (or anuria) and thrombocytopenia
. The peripheral smear reveal schistocytes (which represent fragmented RBCs) and giant
platelets.
. lntravascular hemolysis results in elevated levels of (LDH) and indirect bilirubin, as well as
reticulocyte count BUN and creatinine levels are markedly elevated.
. Moderate leukocytosis may be present.
. The urine contains hemoglobin, hemosiderin, albumin, RBCs, WBCs, and casts.
. Suspect HUS when: a diarrheal illness and presents with acute R.F., microangiopathic
hemolytic anemia, fever, thrombocytopenia and peripheral smear of schistocytes.
.N.B:
- Howell-Jolly bodies: are nuclear remnants of RBCs present in sickle cell atients & are generally
removed by a functioning spleen.
- Their presence in peripheral smear suggests functional asplenia in sickle cell patients (Sickle
cell patients usually have infarcted spleens by the first 18 to 36 months of life).
- Heinz bodies: are aggregates of denatured hemoglobin & are commonly seen in patients with
hemolysis due to G6PD deficiency and thalassemia
- Helmet cells are fragmented red blood cells, their presence is suggestive of traumatic
hemolytic conditions such as DIC, HUS and TIP.
- Basophilic stippling are ribosomal precipitates which appear as blue granules & seen with
thalassemias, as well as lead or heavy metal poisoning.

=========================================================== Pulmonary
============================================================================
===========
. Pseudomonas Aeruginosa :
________________________
. Is a common cause of severe pulmonary infection in patient with cystic fibrosis (CF).
. So the best treatment for a patient with severe exacerbation of cystic fibrosis is
combination of aminoglycosides (Tobramycine) and antipseudomonas penicillines
(piperacillin).

.N.B.: Staphylococcal aureus is a frequent cause of mild pulmonary infection in patient with
cystic fibrosis SO;
when pulmonary infection is mild (cough and mucous production only) oral antistaphylococcal antibiotic is preferred (Dicloxacillin).
.N.B.:
- Mothers with gestational diabetes mellitus (GDM) usually give birth to large babies with the
most common complications:
- Clavicular fracture which is usually greenstick fracture and heal rapidly without
complications.
. The characteristic laboratory findings of:
____________________________________________
1) Iron deficiency anemia are: decreased serum iron, decreased percent of saturation (serum
iron/ TIBC) & increased TIBC.
2) Sideroblastic anemia: is characterized by increased serum iron levels and normal TIBC.
3) In dimorphic anemia: two distinct forms of red cells are circulating
4) Megaloblastic anemia is characterized by an elevated MCV, elevated MCH, and normal MCHC.
5) Anemia of chronic disease is associated with decreased TIBC.
. Cystic fibrosis:
________________
. Is an autosomal recessive disorder that is more common in Caucasian people.
. The cause is a defect in the CFTR (Cystic fibrosis transmembrane conductance regulator)
protein, which leads to impaired chloride transport in tissues
. and consequent multi-system involvement, particularly:
(1) increased salt content in sweat gland secretions.
(2) thick, viscous secretions in the lungs, pancreas, reproductive tract, and liver.
. Young child presents with recurrent upper respiratory tract infection, persistent nasal
discharge and bilateral nasal polyps.
. Also can be present with recurrent upper respiratory tract infections, bronchiectasis and
failure to thrive.
. The patient may have pulmonary manifestations, which begins as a persistent, productive
cough, and progresses to obstructive airway disease.
. A deficiency of pancreatic enzymes leads to fat and protein malabsorption, thereby causing
bulky, greasy, foul-smelling stools and
subsequent deficiency of fat-soluble vitamins (A, D, E and K).
. Vitamin A deficiency presents as dry skin, and Vitamin K deficiency presents as epistaxis.
. Vitamin K is an important cofactor of several coagulation factors factors II , VII , IX, and X as
well as proteins C and S.
. Failure to thrive is the result of chronic malabsorption from pancreatic insufficiency.
. Other possible presentations are meconium ileus at birth and infertility.
. Meconium ileus is virtually diagnostic for cystic fibrosis, an autosomal recessive disorder.
. A mutation of a single gene localized on chromosome 7 is responsible for this disorder.
. patient is suffering from an acute exacerbation of lung disease, and his presentation
(shortness of breath, excessive use of accessory respiratory muscles, hypotension) suggestive of
severe attack.
. The most common etiologic agent is Pseudomonas aeruginosa.
. The three most common organisms that cause pneumonia in patients with cystic fibrosis are
Haemophilus, Pseudomonas, and Staphylococcus.
. The treatment is rapid administration of empiric antibiotics and the usual choice is a
combination of two agents with coverage against Pseudomonas aeruginosa,
(ceftazidime or a penicillin derivative such as ticarcillin + an aminoglycoside such as amikacin
or gentamicin).

.N.B: It is essential to memorize the following for the USMLE


- Gram-positive diplococci - Streptococcus pneumoniae.
- Gram-positive cocci in clusters - Staphylococcus.
- Gram-negative cocci - Neisseria.
- Gram-positive rods - Listeria and Bacillus.
- Gram-negative rods - Pseudomonas, Haemophilus, Klebsiella, Legionella.
. Pertussis:
____________
. Caused by Bordetella pertussis infection.
. It is a highly contagious infection, but its incidence has dramatically decreased today because
of immunization.
. It usually presents with severe bouts of coughing spells after an upper respiratory tract
infection.
. The coughing spells can be so severe that they can cause rectal prolapse, epistaxis and
pneumothoraces.
. These spells may last from two to ten weeks
. In patients with apparent subcutaneous emphysema secondary to severe coughing
paroxysms, chest x-rays must be obtained first to rule out pneumothorax(same mechanism).
. Presence of leukocytosis with predominant small and normal-appearing lymphocytes further
supports the diagnosis.
. The recommended first line treatment is a macrolide antibiotic (erythromycin, azithromycin,
or clarithromycin), regardless of the age or the immunization status.
. Treatment is given whenever pertussis is suspected or confirmed, and regardless of the stage
of the disease.
. For pertussis prevention, all close contacts should be given erythromycin for 14 days,
regardless of age, immunizations, or symptoms.
. Foreign body aspiration:
__________________________
. A sudden onset of respiratory distress may be caused by foreign body aspiration.
. Direct laryngoscopy & rigid bronchoscopy is the procedure of choice for both diagnostic and
therapeutic purposes.
. Flexible bronchoscopy is a useful diagnostic tool but not theraputic.
. Segmental lung resection is used in some cases where rigid bronchoscopy fails to remove the
foreign body.
Vascular rings can compress the trachea:
________________________________________
. Most often due to a double aortic arch, right-sided aorta, pulmonary sling, or anomalous
innominate or left carotid artery, causing tracheal compression.
. Patients present with signs of chronic upper airway obstruction including stridor, wheezing,
coughing, and shortness of breath.
. Symptoms are often worse while supine and relieved with neck extension.
. Patients are commonly misdiagnosed as having reactive airway disease but will not respond
to inhaled bronchodilators and corticosteroids.
. The treatment for severe disease is surgery.
. Asthma:
_________
. Characterized by airway hyperreactivity, bronchospasm and inflammation.

. It is a common illness in childhood, and about 10% of children present to the ED with
unremitting asthma (i.e. continuous wheezing despite adequate treatment).
. Of these patients, less than 10% require mechanical ventilation; but, hospitalization is
mandatory.
. If the child has a "silent chest" or absent air entry, and continues to desaturate despite
prednisone treatment; so, the best option is hospitalization and mechanical ventilation.
. Theophylline is an option when wheezing is severe;but it must be given I.V.and levels must be
monitored as the drug has a tendency to increase the heart rate.
. For acute status asthmaticus, theophylline does not help much.
.N.B.
- Tracheostomy is a surgical procedure when the upper airways are obstructed. It is nearly
always an elective procedure, and never done to relieve asthma.
- Racemic epinephrine may help a little in moderate asthma, but in severe cases, its
administration will just delay the initiation of mechanical ventilation.
- Racemic epinephrine has a better role in the management of croup.
- Antibiotics are not indicated if there is no evidence of infection.
. Kartagener's syndrome:
________________________
. This syndrome is an autosomal recessive disorder due to dysmotile cilia.
. Characterized by a classic triad of situs inversus, recurrent sinusitis, and bronchiectasis.
. The typical radiographic finding is dextrocardia (the apex of the heart is in the right chest).
. The aberrant production or attachment of dynein arms can frequently be verified on electron
microscopy,
and results in impaired cilial function, poor clearance of secretions, and consequent chronic
secondary infections.
. N.B:
. A large thymic shadow is a normal chest radiograph finding in children less than 2 years old.
. Because the thymus is a soft organ, it can appear as a scalloped border.
. It can also have a straight inferior border which usually appears on the right side and is called
a "sail" sign because it resembles the sail of a ship.
. Hyaline membrane disease (HMD): or Respiratory distress syndrome (RDS)
_________________________________
. Occurs primarily in premature infants who are less than 28 weeks gestational age (the
incidence of RDS is inversely proportional to the gestational age).
. Other risk factors: male sex, cesarean section without labor, perinatal asphyxia, and maternal
diabetes.
. Maternal diabetes increases the incidence of RDS in several ways Most importantly, maternal
diabetes delays the maturation of surfactant production in the lungs.
. The mechanism is that fetal hyperinsulinism antagonizes the actions of cortisol and may
delay the lung maturation process.
. The decreased production and secretion of surfactant is the 1ry cause.
. In the absence of surfactant, atelectasis ensues and results in perfused but non-ventilated
alveoli, and consequently hypoxia.
. HMD presents as: tachypnea, prominent grunting, intercostal and subcostal retractions, nasal
flaring, and duskiness within a few minutes after birth.
. Breath sounds may be normal or diminished with a harsh tubular quality, & on deep
inspiration, fine rales may be heard, especially over the lung bases posteriorly.
. The characteristic chest-x ray: fine reticular granularity of the lung parenchyma.
. Treatment includes early mechanical ventilation and surfactant administration.
. Transient tachypnea:
_______________________

. usually follows an uneventful normal term vaginal delivery or cesarean delivery.


. It is characterized by the early onset of tachypnea, sometimes with retractions or expiratory
grunting, and cyanosis that is relieved by minimal oxygen.
. The lungs are usually clear without rales or rhonchi & chest-x ray shows prominent
pulmonary vascular markings, fluid lines in the fissures, overaeration,
a flat diaphragm, and occasionally, pleural fluid. Hypoxemia, hypercapnia, and acidosis are
uncommon.
. The distinguishing feature from (HMD) is the x-ray finding
. Persistent pulmonary hypertension of the newborn (PPHN):
__________________________________________________
. Should be suspected in all term and post-term infants with cyanosis with or without fetal
distress.
. Persistence of fetal circulatory pattern of right-to-left shunting through the PDA and foramen
ovale after birth is due to very high pulmonary vascular resistance.
. The associated hypoxia is universal and unresponsive to 100% oxygen.
. The chest-x ray may be normal or may show parenchymal opacification in the chest,
depending on the etiology.
. Although the clinical presentation of PPHN is indistinguishable from HMD, the x-ray findings
can differentiate.
. Meconium aspiration syndrome:
_______________________________
. Usually occurs in term or post-term infants, either in utero or more often with the first
breath.
. Thick & particulate meconium is aspirated into lungs, resulting in small airway obstruction &
consequent respiratory distress,that present within the 1st hour of birth.
. Partial obstruction of some airways may lead to pneumothorax or pneumomediastinum.
. Patchy infiltrates, coarse streaking of both lung fields, increased anteroposterior diameter,
and flattening of the diaphragm characterize the typical chest-x ray
. Intracranial hemorrhage:
___________________________
. In neonates classically presents as periods of apnea, pallor or cyanosis, poor suckling,
abnormal eye signs, high-pitched, shrill cry, muscular twitching,
convulsions, decreased muscle tone or paralysis, metabolic acidosis, shock, and a decreased
hematocrit.
. The fontanel may be tense and bulging.
. The condition is rarely present at birth and is diagnosed on the basis of the history, clinical
manifestations, and trans-fontanel cranial (CT).
. Bronchiolitis:
________________
. is defined as the first episode of wheezing associated with an upper respiratory tract
infection.
. The infection is usually caused by respiratory syncytial virus.
. It affects 50% of children in the 1st 2 years of life, especially those who are prone to airway
reactivity, & there is increased incidence of asthma later in life.
. The WBC count is normal, and the chest x-ray may show air trapping or atelectasis.
. The treatment is supportive care with humidified oxygen and bronchodilators.
. Ribavirin may be administered in high- risk children.
==================================================================
Rheumatology

============================================================================
===
. Septic Arthritis:
___________________
. Septic arthritis can occur following skin or upper respiratory tract infections.
. Arthrocentesis is both diagnostic and therapeutic, and is mandatory in all suspected cases.
. Blood cultures and synovial fluid cultures must be obtained prior to administering the
antibiotics.
. Empiric antibiotic therapy ( IV nafcill in or IV cefazolin) should be started with pending
synovial fluid cultures.
. Clubfoot (talipes equinovarus):
______________________________
. Presents with equinus and varus of the calcaneum and talus, varus of the midfoot, and
adduction of the forefoot.
. This is a common foot deformity and may be congenital, teratologic, or positional (in utro).
. Initial treatment involves, stretching and manipulation of the foot, followed by serial plaster
casts, malleable splints, or taping.
. Because conservative treatment corrects the majority of cases, so Surgical treatment is
indicated if conservative management
gives unsatisfactory results, and is preferably performed between 3 and 6 months of age.
. Remember that the treatment of clubfoot should be started immediately.
. Spondylolisthesis:
____________________
. Is a developmental disorder characterized by a forward slip of vertebrae (usually L5 over S1 )
that usually manifests in preadolescent children.
. In the typical clinical scenario, back pain, neurologic dysfunction (e g , urinary incontinence),
and
a palpable "step-off" at the lumbosacral area are present if the disease is severe.
. Hemophilic arthropathy:
_________________________
. patients with coagulopathies (Hemophilia) --> Recurrent hemarthroses may then lead to a
joint injury called "hemophilic arthropathy".
. Although the exact mechanism underlying the joint damage is not known, iron deposition and
synovial thickening with fibrosis are characteristic.
. Iron deposition and cytokines released from the blood stimulate synovial proliferation,
fibrosis and cartilage injury.
. Growing pains:
________________
. Common in children between the ages of two and twelve.
. It is usually poorly localized and common in the legs below the knees and bilateral.
. It awakens the child at night and responds to reassurance, massages or analgesics.
. In all cases, it resolves in the morning.
. The diagnosis of growing pain is a diagnosis of exclusion, all the physical and laboratory
findings are negative.
. X- rays are not required if the physical exam is unremarkable.
. All such children are observed, and parents are assured that this condition will resolve with
time.
. Transient synovitis:
______________________

. Is a frequent cause of hip pain in children, typically occurring in male children ages 3-10
years old.
. The cause of transient synovitis is unknown, but the condition usually follows mild trauma or
viral infection.
. Transient inflammation of the joint synovium leads to pain and decreased range of motion.
. Patients usually have a history of a recent viral infection and present with a limp and/or
unilateral hip or knee pain A low grade fever may be present, but
patients are generally afebrile.
. On examination, the affected hip will be flexed, slightly abducted, and externally rotated,
There is mild limitation of motion.
. The patient may complain of pain with palpation or passive motion.
. Laboratory findings characteristic of transient synovitis include a slightly elevated white
blood cell count and erythrocyte sedimentation rate.
. Four clinical criteria have been shown useful in differentiating septic arthritis from transient
synovitis;
white blood cell count > 12,000/mm3; temperature > 39C (102F); erythrocyte sedimentation
rate > 40 mm/h; and refusal to bear weight.
. If at least three of these four criteria are met, further work-up is indicated to rule out septic
arthritis.
. Radiographs of the hip should be done to exclude bony lesions and Legg-Calve-Perthes
disease.
. Widening of the joint space or an enhanced pericapsular shadow may be seen with transient
synovitis.
. Additional work-up is not usually necessary, unless the diagnosis is uncertain.
. Treatment consists of rest and non-steroidal anti-inflammatory medications (NSAIDs).
. Most patients experience resolution of symptoms within several days, although transient
synovitis may contribute to the development of arthritis later in life.
. Compartment syndrome:
_______________________
. supracondylar humerus fractures may be complicated by neurovascular injury or
compartment syndrome.
. Compartment syndrome is characterized by severe pain, pallor, poikilothermia, paresthesias,
and the late findings of pulselessness and paralysis.
. Patients with predisposing injuries should be continually monitored for the development of
compartment syndrome, particularly as swelling from the injury increases.
. Initial treatment includes removal of any bandages, measurement of compartment pressures,
and emergent orthopedic evaluation for possible fasciotomy.
. N.B.:
1- The United States Preventive Services Task Force (USPSTF) recommends vision screening for
children aged 0-5 years,
primarily to identify those with strabismus, amblyopia, and refractive errors.
Early diagnosis and treatment are being emphasized, because poor visual acuity resulting
from the above conditions may impair
a child's future academic performance and selfimage,and may even lead to blindness.
2- Lead toxicity causes microcytic anemia and permanent neurologic damage, so identification
of affected patients is desirable at an early stage.
Risk factors for environmental lead exposure include minority race or ethnicity, low
socioeconomic status, pre-1950 housing,
occupational exposure from parents, and recent immigration.
- The USPSTF recommends no screening for asymptomatic children aged 1-5 years at average
(no) risk of lead toxicity.

3- The Centers for Disease Control (CDC) recommend beginning the meningococcal vaccination
series between 11-12 years.
Children who are at high risk of the disease (i.e , asplenic) may be vaccinated as early as 2
years of age.
4- The CDC recommends administering the rotavirus vaccine between 2 and 8 months.
first dose should be administered between 6 and 14 weeks of age Rotavirus.
vaccination should not be initiated for infants aged 15 weeks and older.
The final dose should not be given beyond 8 months of age.
. Hand-foot syndrome or dactylitis:
____________________________________
. Is the earliest manifestation of vaso-occlusion in sickle cell anemia, so needs a complete
workup for previously asymptomatic sickle cell patients.
. Patients usually present at the age of 6 months to 2 years with an acute onset of pain and
symmetric swelling of the hands and feet.
. Although only soft tissue swelling is initially seen, fever is sometimes present.
. The pathophysiology of dactylitis involves vascular necrosis of the metacarpals and
metatarsals, which may be seen on plain radiographs as osteolytic lesions.
. Ewing's sarcoma:
__________________
. A highly malignant tumor that is found in the lower extremity more than the upper extremity.
. The most common sites are the metaphysis and diaphysis of the femur, followed by the tibia
and humerus.
. It is very aggressive and metastasizes early to the lungs and lymph nodes.
. Patients are typically white males in their first or second decade of life.
. Presents with pain and swelling for weeks or months, Erythema and warmth of the local area
are sometimes seen.
. Patients may initially be diagnosed with osteomyelitis, due to the misleading presentation of
intermittent fevers, leukocytosis, anemia, and increased ESR.
. But, Ewing's sarcoma is characterized radiologically with its lamellated appearance or "onion
skin" periosteal reaction.
. This lesion is usually lytic, central, and accompanied by endosteal scalloping
. The "onion-skin" appearance is often followed with a "moth-eaten" or mottled appearance and
extension into soft tissue.
. The treatment includes surgery, radiation, and multi-drug chemotherapy.
. Osteogenesis imperfecta: Extremely high yield question for USMLE step-1 and step-2.
__________________________
. An autosomal dominant disease characterized by mutations in the gene coding for type I
collagen.
. There are four types of (OI). All types can have joint hyperlaxity, hypotonia, wormian bones,
and early hearing loss.
. Type II is the most severe form and is usually lethal within the first year of life.
. Types I, III , and IV can all be further categorized by the presence or absence of
dentinogenesis imperfecta.
. Dentinogenesis imperfecta is a disorder of tooth development The teeth are usually
discolored (bluish gray or yellow-brown), translucent, and weak.
. Both primary and permanent teeth are affected.
. Patients with the severe type II form of this disease typically expire in utero due to multiple
intrauterine and/or perinatal fractures.
. Since type I collagen is an important structural protein that is present in the skin, sclera,
bone, tendon and ligament,

. patients with this disorder present with multiple recurrent fractures, blue sclera, hearing loss,
joint laxity, short stature, and scoliosis.
. Clinical findings include limb deformities, growth retardation, multiple fractures and blue
sclerae.
. Cardiac anomalies such as aortic root dilatation are not usually seen in (OI). Aortic root
dilatation can be seen in disorders such as Marfan syndrome.
. Horseshoe kidneys are not a feature of (OI). Patients with Turner syndrome can have
horseshoe kidneys
. Patients with (OI) have normal intelligence not mental retardation.
. Ash leaf macules are hypopigmented macules seen in patients with tuberous sclerosis.
. A lytic bone lesion in a child:
_________________________________
. Differential diagnosis for a lytic bone lesion in a child:
- infectious (Brodie abscess from osteomyelitis), - endocrine (hyperparathyroid osteitis fibrosa
cystica),
- neoplastic (Ewing sarcoma, Langerhans cell histiocytosis, metastases) and - idiopathic (benign
bone cyst, aneurysmal bone cyst).
. If there is concomitant hypercalcemia, it narrows the differential diagnosis to
hyperparathyroid state versus lytic bone neoplasm.
. Parathyroid adenoma is the most common cause of primary hyperparathyroidism, and
typically occurs in patients over 50 years of age.
. A lytic bone lesion is most likely occurs in children.
. Langerhans cell histiocytosis (LCH, Langerhans cell granulomatosis, histiocytosis X) is known
to causes solitary, lytic, long bone lesions.
. Eosinophilic granuloma, the least severe form of histiocytosis X, generally presents in
children and young adults as a solitary bone lesion.
. This lesion may be painful, have overlying tender swelling, and cause pathological fractures.
. Though these tumors can be locally destructive, they typically resolve spontaneously and are
therefore regarded as benign and treated conservatively.
. Aseptic necrosis of the femoral head: Extremely high yield question for USMLE!!!
_______________________________________
. Osteonecrosis = A vascular necrosis.
. is a common complication of sickle cell disease.
. It involves occlusion of end arteries supplying the femoral head, bone necrosis, and eventual
collapse of the periarticular bone and cartilage.
. The femoral head has two main sources of blood supply - ascending arteries and the foveal
artery, which lie within the ligamentum teres.
. The foveal artery is patent early in life, but may become obliterated in older patients.
. This explains why aseptic necrosis of the femoral head is uncommon in children.
. Traction apophysitis (Osgood-Schlatter disease):
__________________________________________________
. is a common cause of knee pain, particularly in adolescent male athletes.
. During early adolescence (ages 13-14 for males, and ages 10-11 for females), there are periods
of rapid growth in which
the quadriceps tendon puts traction on the apophysis of the tibial tubercle where the patellar
tendon inserts.
. This traction apophysitis is worsened by sports that involve repetitive running, jumping, or
kneeling, and it improves with rest.
. Approximately 1/4 of affected individuals have bilateral disease.

. Physical examination, there is edema and tenderness over the tibial tubercle,a firm mass can
sometimes be felt due to heterotopic bone formation.
. Pain can be reproduced by extending the knee against resistance.
. Radiographic findings are nonspecific: anterior soft tissue swelling, lifting of tubercle from
the shaft, and irregularity or fragmentation of the tubercle.
. Treatment consists of activity restriction, stretching exercises, and non-steroidal antiinflammatory medications.
. Clavicular fracture in neonat:
_________________________________
. Clavicular fracture presents with irregularity, crepitus, and fullness over the fracture site, and
decreased movements of the arm.
. Predisposing factors are shoulder dystocia, traumatic delivery, and large size of the infant.
. Generally, no specific treatment is required (reassurance).
. Acquired torticollis (wryneck):
_________________________________
. Refers to neck twisting usually due to asymmetric muscle activity.
. The most common causes of acquired torticollis are upper respiratory infections, minor
trauma, and cervical lymphadenitis.
. More serious causes: retropharyngeal abscess and atlantoaxial subluxation.
. Cervical spine radiographs is needed in children with torticollis to ensure there is no cervical
spine fracture or dislocation, which requires extreme caution.
. Metatarsus adductus:
______________________
. A congenital foot deformity, which is most frequent in first-born infants & is attributed to the
molding effect of the primigravid uterus.
. Approximately 10% of patients also have an associated acetabular dysplasia; hence, careful
hip examination is required.
. Metatarsus adductus is subdivided into three types:
1- Type I metatarsus adductus:
. characterized by deformity of the feet, specifically adduction of the anterior aspect of the
foot with a convex lateral border and concave medial border.
. The ankle movements are normal, and passive and active movement of the foot overcorrects
the deformity into abduction.
. AP radiographs reveal mild adduction of the metatarsals at the tarsometatarsal articulation,
and an increased angle between the 1st and 2nd metatarsals.
. These cases tend to spontaneously correct by themselves; therefore, treatment is not
necessary (reassurance).
2- Type II metatarsus adductus:
. characterized by feet that correct to the neutral position with passive and active movements;
. this is managed with orthosis or corrective shoes, and sometimes plaster casts if initial
treatment gives no results.
3- Type III metatarsus adductus is characterized by rigid feet and do not correct; these are
managed with serial casts.
. Surgical treatment may be required if there is significant residual metatarsus adductus in
children 4 years of age.
. Osteomyelitis:
________________
. Staphylococcus aureus is the most common cause of osteomyel itis in both infants and
children.

. 2nd common: Group B streptococcus and Escherichia coli in infants, and Streptococcus
pyogenes in children.
. Salmonella and Staph aureus are the most common cause of osteomyelitis in patients with
sickle cell disease.
. Staphylococcus epidermidis is a frequent cause of osteomyelitis associated with prosthetic
devices.
. Septic arthritis:
___________________
. Like osteomyelitis in a child, a septic joint is usually hematogenous in origin and frequently
follows an upper respiratory tract infection.
. The most common organisms causing septic arthritis in children are Staphylococcus aureus
and streptococcus.
. It is characterized by severe pain, leukocytosis and an elevated ESR.
. The typical patient generally refuses to walk, and keeps the affected limb externally rotated
(the external rotation position allows for maximal pus accumulation).
. Ultrasound guided aspiration of the synovial fluid should be performed immediately.
. The synovial fluid white cell count of more than 100,000/ul indicates a definite septic
arthritis.
. An x-ray is often normal.
. Empiric antibiotics: in infants a combination of antistaphylococcal agent (nafcillin or
vancomycin) and third generation cephalosporin and
for children over age 5 years antistaphylococcal agent (nafcillin or vancomycin) alone is a
reasonable empiric antibiotic therapy.
. Septic joint in a child is a true surgical emergency and needs immediate drainage.
. A delay of even 4-6 hours can lead to avascular necrosis of the femoral head.
. Lyme arthritis:
_________________
. occurs as a result of untreated Lyme infection. It is caused by Borrelia burgdorferi, a
spirochete carried by the deer ticks.
. It spreads via blood to various areas of the body, including the joints.
. The early symptoms are mild and are easily overlooked, arthritis is the hallmark of late
disease.
. It tends to involve the knee in most patients Warmth, swelling from effusion and restriction
of movements differentiate it from arthralgias, which occur early in the disease.
. The first episode occurs within six months of having erythema migrans, the episodes
gradually disappear in about ten years, if left untreated.
. In United States, Lyme disease is prevalent in: Connecticut, Rhode Island, New York,
Pennsylvania, New Jersey, Delaware, Maryland and Wisconsin.
. Slipped capital femoral epiphysis (SCFE):
___________________________________________
. is a common hip disorder seen in adolescents.
. SCFE occurs when the capital femoral epiphysis displaces from the femoral neck.
. Boys > girls, and the usual age is 10 to 16 years old.
. Obesity is a significant risk factor, additional risk factors: hypothyroidism,
panhypopituitarism, and growth hormone deficiency or supplementation.
. Children with endocrine issues are more likely to have bilateral disease and to present at an
earlier age.
. Commonly present with an insidious onset of knee or hip pain and a limp, usually without a
history of trauma.
. Sometimes, minor trauma can increase the pain and bring the patient to medical attention.

. Patients tend to hold the hip in passive external rotation and exhibit decreased internal
rotation, abduction, and flexion.
. Diagnosis is made with plain radiographs of the hip (AP and frog leg lateral views), which
show the posteriorly and inferiorly displaced femoral head.
. Treatment: emergent orthopedic consultation and surgical fixation at the current degree of
slippage to avoid the risk of avascular necrosis.
. Supracondylar fractures:
__________________________
. are the most common fractures in the pediatric population.
. The most common complication is entrapment of the brachial artery, resulting in loss of the
radial artery pulse.
. therefore, all patients with a supracondylar fracture must have their radial pulse checked.
. After reduction of the fracture, the radial pulse must be reassessed.
. Duchenne muscular dystrophy:
______________________________
. should be suspected in a child under the age of 5 who presents with proximal muscle
weakness (difficulty climbing stairs),
Gower's sign (use his hands to "walk up" his legs in order to assume an upright position),
diminished reflexes, and pseudohypertrophy of the calf muscles.
. Mental retardation.
. Cardiomyopathy.
. Serum CK (and aldolase)levels are used for screening the muscular dystrophies.
. Muscle biopsy can confirm the diagnosis in most cases.
. The gold standard is genetic studies, which is required in atypical cases.
. Prognosis: most of patients are wheelchair bound by age of 12 years.
. Becker muscular dystrophy:
____________________________
. Similar to Duchenne muscular dystrophy but:
. After age of 5 years (usually around 12 y).
. Milder than Duchenne muscular dystrophy.
. Caulf pain with exercise.
. Normal or near normal mentality.
. Prognosis: patients are able to walk untile early childhood.
. Nursemaid's elbow:
____________________
. occurs when infants or children are lifted or pulled by the hand or arm.
. This pulling injury occurs when the radial head slips through parts of the annular ligamen.
. The child typically keeps the hand in a pronated position, and refuses (cries out in pain)
attempted fo rearm supination
. Rotating the hand and forearm to a supinated position with pressure over the radial head
usually reduces the annular ligament with an audible click, and
restores full, normal use of the extremity.
. The response to reduction of the displaced ligament is diagnostic.

============================================================ GIT
============================================================================
======
. Congenital abdominal wall defects:
____________________________________

. Diagnosis Congenital abdominal wall defects can be made antenatally with maternal serumAFP measurements and ultrasonography.
. The two types of abdominal wall defects are described in the table below:
* Gastroschisis:
---------------. Bowel protrudes through a defect on the right side of the umbilical cord.
. Bowel is not covered by a protective membrane.
. Bowel looks "angry" and matted.
. Not typically associated with any other abnormalities outside the GI tract.
. If the diagnosis was made antenatally, management would begin with planned spontaneous
vaginal delive ry at a tertiary neonatal surgical unit,
. Caesarean section is performed only in the presence of the usual obstetric indications.
. After delivery, the next step in management is to immediately wrap the exposed bowel with
sterile saline dressings and
. Cover with plastic wrap in order to prevent insensible heat and large fluid losses.
. The neonate is then maintained in a thermo-neutral environment.
. Further steps in the management of gastroschisis include inserting an orogastric tube (to
decompress the stomach), establishing peripheral intravenous access (to
provide fluids), and administration of broad-spectrum antibiotics.
. Definitive treatment is surgical; however, surgery should be preceded by the above measures.
. Primary closure is successful in 70% of neonates, but if it is unsuccessful, staged closure with
Silastic silo can be used.
* Omphalocele:
-------------. Intra-abdominal contents protrude through the umbilical ring The abdominal herniat ion
usually includes small bowel and may include large bowel and liver.
. Bowel is covered by an amnioperitoneal membrane.
. Can be associated with other congenital abnormalities ( e g , heart, kidney).
. The management of omphalocele is similar to that of gastroschisis:
. Delivery is also planned at a tertiary care center, and caesarean section is reserved for the
usual obstetric indications,
(This is based on the lack of evidence that caesarean delivery improves the outcome of
patients with uncomplicated omphalocele ).
. Immediate care of the newborn with omphalocele also involves, sterile wrapping of the
bowel,insertion of an orogastric tube,
stabilizing the airway (to ensure adequate ventilation), and establishing peripheral
intravenous access.
. Small defects (< 2 cm) can be repaired with primary closure, but most defects will require a
staged procedure with Silastic silo.
. Umbilical hernia:
___________________
. Is due to an imperfect closure or weakness of the umbilical ring, it is especially common in
low birth weight, female, and black infants.
. It appears as a soft swelling covered by skin that protrudes during crying, coughing or
straining, and can be reduced easily through the fibrous ring at the umbilicus.
. The hernia consists of omentum or portions of the small intestine.
. Most umbilical hernias disappear spontaneously by 1 year of age.
. Surgery is advised if the hernia persists to the age of 3-4 years, exceeds 2 cm in diameter,
causes symptoms, becomes strangulated,
or progressively enlarges after the age of 1 to 2 years.
. Necrotizing enterocolitis:

____________________________
. Suspect necrotizing enterocolitis in any preterm or low birth weight infant with fever,
vomiting, abdominal distension, bloody diarrhea, leukocytosis & pneumatosis intestinalis.
. These symptoms usually occur after the introduction of formula feeding.
. The radiologic finding of pneumatosis intestinalis (i.e. intramural air) in infants is diagnostic.
. Increased gastric residual volume in a preterm neonate is highly suspicious for necrotizing
enterocolitis.
. The condition usually presents at 3-10 days after birth, and is due to bowel wall injury
resulting from perinatal asphyxia.
. Preterm neonates are at high risk for other complications such as hyaline membrane disease,
which presents with respiratory distress and ground glass appearance of the lungs on chest xrays.
N.B:
____
. Trachea-esophageal fistula: presents with excessive drooling, choking, coughing, and cyanosis
due to aspiration of feedings.
. Duodenal atresia: Bilious vomiting without abdominal distention is the cardinal sign for
duodenal atresia.
. Meconium aspiration syndrome: Clinical features of meconium aspiration syndrome include
thick, greenish amniotic fl uid
in the mouth and throat at birth, accompanied by cyanosis and respiratory distress.
. Pyloric stenosis is characterized by projectile nonbilious vomiting and palpation of an oliveshaped mass immediately after the infant has vomited.
. Celiac disease:
_________________
. The presence of dermatitis herpetiformis and chronic non-bloody diarrhea in a child of 12-15
months is suggestive of celiac disease.
. The child has Kwashiorkor-like clinical features associated with dermatitis herpetiformis
(erythematous vesicles symmetrically,
distributed over the extensor surfaces of elbows and knees).
. This scenario is suggestive of celiac disease, which is also called gluten-sensitive enteropathy.
. It is due to abnormal hypersensitivity to gluten and presents characteristically at 12-15
months of age, when gluten-containing foods
such as wheat, rye or barley are introduced into the child's diet.
. Pyloric stenos:
_________________
. Is a common congenital anomally typical presentation is non-bilious projectile vomiting which
gradually becomes more frequent and
forceful in a 4-8 week old infant Pertinent physical findings are a palpable abdominal mass
and visible peristaltic waves in the upper abdomen.
. Hypokalemic alkalosis occurs due to the frequent episodes of vomiting.
. Abdominal ultrasound is the most useful tool for confirming the diagnosis, and shows a thick
hypoechogenic ring in the pyloric region.
. Additionally, the loss of large volumes of fluid via vomiting leads to contraction alkalosis due
to the action of aldosterone.
. In order to compensate for the metabolic alkalosis, the respiratory system responds via
hypoventilation to create a secondary respiratory acidosis.
. i.e. have a primary metabolic alkalosis with respiratory compensation.
. This is characterized by a plasma pH greater than 7.45, PaC02 greater than 40 mm Hg and
HC03- greater than 24 mEq/L.

. Pyloric stenosis is treated su rgically; however, surgery should be undertaken only after the
infant's hydration status and electrolyte levels have been stabilized.
. Meckel's diverticulum:
________________________
. Typically presents with painless melena in 2 to 3-year-old children.
. It is present in 2-3% of the population, and is the most common anomaly of the GIT.
. It results from the failure of the vitelline duct to obliterate during the fetal development.
. Heterotropic gastric tissue may be present in the diverticulum, which results in ulcerations
and bleeding.
. The diagnosis of a Meckel's diverticulum is best made with technetium-99m pertechnetate
scanning (uptake by heterotopic gastric mucosa).
.N.B.:
1- Peptic ulcer disease may present in preschool children with epigastric pain related to meals
with a positive family history.
2- Anal fissure is a painful condition associated with constipation it may occur in infancy, and
presents with blood streaked stools.
3- lntussusception may present in an infant or a young child with irritability, vomiting, red
currant-jelly stools and abdominal mass.
4- Inflammatory bowel disease may present in preschool children with diarrhea, abdominal
pain, low-grade fever and malaise.
. A choledochal cyst:
_____________________
. a congenital abnormality of the biliary ducts characterized by the dilatation of intra or extrahepatic biliary ducts or both.
. It has a multifactorial origin, but most of the cases are related to an anomalous
pancreaticobiliary junction, which
. leads to weakness and dilatation of the biliary wall due to the reflux of alkaline pancreatic
secretions into the biliary tree.
. Various types of choledochal cysts are:
- Type 1: Most common type, features the dilatation of the entire common hepatic and
common bile ducts or segments of each.
- Type 2: Relative isolated protrusions or diverticulae from the common bile duct wall.
- Type 3: Cyst found in the intraduodenal part of common bile duct
- Type 4: Multiple dilatations in the intra and extra hepatic biliary tree.
- Type 5: Isolated dilatation of intrahepatic bile ducts.
.N.B.:
- Caroli's syndrome is a congenital disorder characterized by intrahepatic dilatation of bile
ducts.
. Pseudo-pancreatic cyst:
________________________
. is a complication of acute or chronic pancreatitis characterized by retroperitoneal/epigastric
mass and high amylase.
. However, it will rarely cause a right upper quadrant mass and very mild jaundice by
compressing the common bile duct.
. It is easily diagnosed by ultrasonogram.
. Intestinal atresia:
_____________________
. It can occur anywhere from the duodenum to the colon.

. Jejunal atresia is thought to occur due to an intrauterine vascular accident that causes
necrosis and resorption of a segment of bowel.
. The severity of the obstruction can vary from a membranous web to full atresia and loss of
bowel length.
. Prenatal ultrasounds can diagnose intestinal atresia.
. If not discovered prenatally, the infant will develop vomiting and abdominal distention along
with feeding intolerance shortly after birth.
. Duodenal atresia appears as a "double bubble" on radiography.
. Jejunal atresia presents as a "triple bubble" on radiographs, indicating dilation of a larger
amount of small bowel than that seen in duodenal atresia.
. Treatment should initially be focused on resuscitation and stabilization of the patient,
followed by surgical correction.
. Duodenal atresia usually presents with bilious vomiting a few hours after the first feeding.
. It is usually associated with other congenital anomalies, and can be a feature of Down's
syndrome.
. Midgut volvulus:
___________________
. usually presents in a child less than one month old with bilious vomiting, abdominal
distension and passage of bloodstained stools.
. Volvulus is associated with malrotation of the gut, and can be complicated by perforation and
peritonitis.
. lntussusception:
__________________
. is the most common cause of intestinal obstruction in the first two years of life.
. The classic presentation: is an infant with colicky abdominal pain, vomiting, and red currant
jelly stools.
. The currant jelly stools are caused by bowel ischemia, which leads to bleeding and mucus
production.
. Between episodes of abdominal pain, the child appears well.
. On physical examination, the abdomen is usually soft, nontender, and nondistended, A
sausage-shaped mass is often palpated.
. An abdominal radiograph may show air fluid levels due to obstruction.
. An ultrasound commonly shows a targetoid lesion in which one bowel segment is nested
inside another bowel segment.
. An air contrast enema is both diagnostic and therapeutic in the majority of cases.
. Emergent laparotomy is indicated for the treatment of intussusception when air contrast
enema is unsuccessful.
. However, laparotomy becomes the treatment of choice if the child presents with peritoneal
signs.
. The most common type of intussusception is ileocolic.
. Hirschsprung's disease:
________________________
. usually presents with failure to pass meconium within the first 24 hours.
. In this disease, an aganglionic segment of the colon causes the intestinal obstruction.
. Cyclical vomiting:
__________________
. Recurrent self-limiting episodes of vomiting and nausea in children in the absence of any
apparent cause.
. The etiology of this condition is unclear; however, its incidence is high in children whose
parents have a history of migraine headaches.
. Complications that may arise from the condition are anemia and dehydration.

. Treatment consists of anti-emetics and reassurance of the parents.


. Infantile gastroesophageal reflux disease (GERD):
___________________________________________________
. The child typically has frequent vomiting, but maintains adequate weight gain.
. Symptoms usually begin in the first few months of life, peak around 4 months, and generally
resolve by 1 year
. Conservative therapy is first-line management in these infants, and make sure that the
feedings are the appropriate volume and are given at regular intervals.
. Breast milk or formula can be thickened with rice cereal, which can decrease the frequency of
vomiting.
. The infant should also be positioned to help decrease intraabdominal pressure by keeping the
child's head elevated or laying the child prone when awake.
. Parents should be reassured that most children will have resolution of their symptoms by 12
months.
. If infants develop respiratory symptoms due to GERO such as stridor, apnea, or failure to
thrive, a more detailed evaluation is necessary including
. an upper gastrointestinal series, pH probe monitioring, or endoscopy.
. These children may require pharmacotherapy if conservativetreatments do not improve their
symptoms.
. N.B:
- Batteries lodged in the esophagus on x-ray should be removed immediately under endoscopic
guidance to prevent mucosal damage and esophageal ulceration.
- Batteries located distal to the esophagus pass uneventfully in most cases and need only to be
observed with stool examination and/or follow up x-rays to confirm excretion.
. Hemolytic-uremic syndrome (HUS):
__________________________________
. Caused by a toxin released from Escherichia coli.
. The toxin enters the systemic circulation and injures the endothelial cells in the kidney.
. When there is progressive elevation of BUN and creatinine, the chances for improvement of
the kidneys' function is poor.
. When the kidneys are damaged, the mortality is 5-10%.
. Turcot's syndrome:
____________________
. It is autosomal recessive and mainly occurs in teens.
. An association between brain tumors (primarily medulloblastomas and gliomas) & FAP
(Familial Adenomatous polyposis) or HNPCC (Hereditary nonpolyposis colorectal cancer).
. Gardner's syndrome:
_____________________
. is autosomal dominant where colonic polyps are seen with prominent extraintestinal lesions.
. Includes: desmoid tumors, sebaceous or epidermoid cysts, lipomas, osteomas (especially
mandible), supernumerary teeth & juvenile nasopharyngeal angiofibroma.
. Peutz-Jeghers syndrome:
_________________________
. An autosomal dominant inherited disorder characterized by intestinal hamartomatous polyps
in association with mucocutaneous melanocytic macules.
. Esophageal atresia:
_____________________

. The most common esophageal anomaly is esophageal atresia with a tracheoesophageal


fistula.
. This is characterized by an atretic esophageal pouch that communicates distally with the
trachea just above the carina.
. This anatomic arrangement allows air to enter the stomach, and this leads to gastric
distention.
. The discontinuous esophagus prevents the infant from completely swallowing, thereby
resulting in drooling or regurgitation during feeding.
. In addition, gastric fluid ascends into the distal esophagus through the fistula, into the
trachea and lungs, thereby
producing aspiration pneumonia Pneumonitis and atelectasis occur frequently, and rattles are
heard during breathing.
. The inability to pass a feeding tube into the stomach is suggestive of esophageal atresia with
or without tracheoesophageal fistula.
. Infantile colic:
___________________
. is defined as excessive crying in an otherwise healthy infant for more than 3 hours a day,
more than 3 days a week, and more than 3 weeks in a month.
. Colic usually presents within the 1st 3 weeks of life and resolves spontaneously by four
months of age.
. Most parents report that the crying occurs at the same time of day, typically in the early
evening.
. The crying begins suddenly, and the child generally cannot be comforted.
. The cause of colic is not known, it maybe due to a digestive tract origin (eg, trapped gas)
leading to abdominal pain.
. Because the etiology is not known, there is no accepted treatment (resolves spontaneously by
four months of age).
. Proposed treatments: soothing measures, simethicone (which decreases the surface tension of
gas bubbles), or probiotics.
. Congenital diaphragmatic hernia:
__________________________________
. The first step is immediate placement of an orogastric tube & connecting it to a continuous
suction, to prevent bowel distension and further lung compression.
. Endotracheal intubation and mechanical ventilation are also priorities for all infants with
severe CDH who present in the first hours of life.

=============================================================== Neonatology
=====================================================================
. The Apgar score is measured in newborns at 0 and 5 minutes of life. It has five components,
and is scored as follows:
A) Color of the newborn
----------------------0 body and extremities are blue/pale
1 body is pink and extremities are blue
2 body and extremities are pink.
B) Heart rate
------------0 heart shows no activity

1 HR <100 beats/min
2 HR > 100 beats/min
C) Reaction to nasal stimulation:
--------------------------------0 no response to stimulation
1 grimace
2 active cough.
D) Tone/ Activity:
-----------------O limp
1 some flexion of extremities
2 active flexion of extremities.
E) Respirations:
---------------0 completely absent
1 slow and irregular
2 good respiratory effort.
. Breast milk:
______________
. The protein in human milk is 70% whey and 30% casein, and the protein content is highest at
birth and decreases over the first month of life.
. Whey is more easily digested than casein and helps to (improve gastric emptying).
. Human milk also contains lactoferrin, lysozyme, and secretory immunoglobulin A proteins
that confer improved immunity to the infant.
. The main carbohydrate in both human milk and standard infant formulas is lactose.
. Although calcium and phosphorus content is significantly lower in human milk when
compared to formula, these minerals are better absorbed from human milk.
. Breast milk has an inadequate supply of vitamin D and exclusively breast-fed infants must
receive supplemented vitamin D.
. Breast feeding:
_________________
A) Advantages of breast feeding:
--------------------------------. It confers protection against infections such as meningitis, bacteremia, necrotizing
enterocolitis, diarrheal illnesses, otitis media, and respiratory infections.
. Mothers who breastfeed also benefit by having decreased postpartum bleeding, more rapid
uterine involution, decreased menstrual blood loss, increased child spacing,
earlier return to pre-pregnancy weight, and decreased risk of breast and ovarian cancer.
B) contraindications to breastfeeding:
--------------------------------------. Infants with certain inborn errors of metabolism such as galactosemia, phenylketonuria, and
urea cycle defects.
. Other contraindications: maternal use of certain medications such as radioactive isotopes,
antimetabolites, and chemotherapeutic agents.
. Mothers using drugs of abuse should be counseled to quit using these drugs and should not
breastfeed if they are continuing to abuse drugs.
. Breastfeeding is also contraindicated with some maternal infections including herpes simplex
(if there are lesions on the breast), active tuberculosis, and HIV.
. Tobacco smoking, hepatitis C infection, mastitis nor maternal alcohol use.

. Neonatal sepsis:
__________________
. Group B streptococcus is the most common cause of neonatal sepsis, it is part of the normal
vaginal flora of women and
is transmitted to infants during passage th rough the birth canal.
. In neonates, infection presents either as early-onset sepsis or late-onset sepsis.
. Early sepsis presents within the first week of life and usually involves several organs
Symptoms include:
. Respiratory failure, meningitis, DIC, acute tubular necrosis and peripheral gangrene.
. It is usually refractory to therapy and quickly evolves to shock, coma and death.
. Late-onset GBS infection is usually a focal infection, most frequently meningitis (75%),
although arthritis, osteomyelitis, cellulitis and urinary tract infection may also occur.
. Meningitis presents with fever, lethargy, poor feeding, hypotonia, seizures and a bulging
fontanel.
. The diagnosis is confirmed by positive blood or cerebrospinal fluid cultures,
although all newborns with fever and sepsis should have full workup for sepsis, including
urine cultures.
. If the mother has already received antibiotics, culture results may be negative and a latex
agglutination test may be more helpful.
N.B.
1- Escherichia coli meningitis is less frequent than GBS meningitis It is the second most
common cause of neonatal meningitis.
2- Listeria is the third most common cause of neonatal meningitis It also tends to result in
multiple abscesses and pneumonia, in addition to meningitis.
3- Congenital toxoplasmosis classically presents with microcephaly, microphthalmia,
hepatosplenomegaly and chorioretinitis.
4- Herpes simplex virus encephalitis usually presents with focal neurologic signs. Herpes
involves temporal lobes, so patients may present with seizures.
. Neonatal sepsis:
__________________
. Sepsis in the neonate often presents with fever or hypothermia, jaundice, lethargy, and poor
feeding.
. Infants with a serious bacterial infection such as meningitis rarely present with classic
findings such as neck stiffness or Kernig's or Brudzinski's signs.
. As a result, all infants with a suspected diagnosis of sepsis should be evaluated with blood
cultures and a lumbar puncture
. Jaundice in the newborn has a broad list of differential diagnoses including sepsis, breast
milk jaundice, and breastfeeding jaundice.
.N.B: Breast milk jaundice:
. Temporary cessation of breastfeeding is the most appropriate next step in an infant with
suspected breast milk jaundice.
. Breast milk jaundice, which is a caused by a factor in human breast milk, results from
increased intestinal absorption of bilirubin and generally peaks
around two weeks of age.
. Liver function tests (Albumin, AST, ALT, alklaline phopshatase) should be obtained in infants
with a direct or conjugated hyperbilirubinemia,
to assess for conditions such as biliary atresia.
. Milk protein intolerance:

___________________________
. Suspected in infant with vomiting, bloody diarrhea, RBCs and eosinophils in stool, and there
may be a family history of an atopic disorder.
. It is a hypersensitivity reaction to cow's milk proteins, and therefore usually occurs in babies
who are fed with cow's milk;
however, it may occur in breastfed babies, as the mother's milk may contain proteins from
ingested cow's milk.
.N.B.:
- viral gastroenteritis: the associated diarrhea is usually non-bloody.
- Meckel's diverticulum typically presents as painless melena in a child who is approximately 2
years of age.
- Hirschsprung's disease may present with failure to pass meconium within the first 24 hours
of life and/or failure to thrive.
.N.B.: HIV testing in pregnant females:
--------------------------------------- It should be done to all pregnant women in the first trimester to prevent possible
transmission to the infant.
- Persistent oral thrush, lymphadenopathy and hepatosplenomegaly & maybe intractable
diarrhea, refractory infections and fa ilure to thrive the presenting symptoms.
- In neonates, serologic testing is not useful for HIV diagnosis because of the passage of
maternal antibodies to the fetal circulation through the placenta.
- PCR, viral culture and p24 antigen testing are more appropriate; HIV infection is confi rmed if
at least two of these tests are positive.
. Contraindications to breastfeeding:
_____________________________________
1. Certain active maternal infections (HIV, tuberculosis, herpes simplex lesions on or near the
nipple, malaria, sepsis, typhoid fever),
2. Eclampsia, nephritis, substance abuse, and breast cancer.
3. Transmission of HIV by breastfeeding is well documented; therefore, the presence of
maternal HIV infection is a contraindication to breastfeeding.
N.B.: Situations that are not a contraindication to breastfeeding:
1- Mastitis --> we encourage breastfeeding since this prevents engorgement, along with other
subsequent symptoms such as fissures and cracking of the nipple.
2- Maternal rubella infection --> there is currently insufficient evidence regarding the
associated risks.
3- Breast milk jaundice --> Mothers of such infants are instructed to switch their babies to
formula-feeding for 1-2 days.
This allows the increased indirect bilirubin levels of the infant to return to normal values.
After 2 days, the mother resumes breastfeeding,
and further hyperbilirubinemia usually does not recur.
4- Hemolytic disease of the newborn (erythroblastosis fetalis) --> breastfeeding because
antibodies in the mother's milk are inactivated
in the intestinal tract and do not contribute to further hemolysis of the infant's RBCs.
. Physiological jaundice:
_________________________
. usually presents after 24- 36 hours of birth.
. The pathophysiology involves a combination of increased bilirubin production, decreased
bilirubin clearance, and increased enterohepatic circulation.

. Breastfeeding jaundice:
_________________________
. Breastfeeding jaundice is an exaggeration of physiologic jaundice caused by the relative
dehydration that can occur in breastfed infants in the first week of life.
. Breastfed infants usually breastfeed for about 15-20 minutes on each breast, 8 times/day,
make about 4-6 wet diapers/day mixed with 8-12 stools/day,
& lose 10% of their birth weight in the first week.
. Ineffective breastfeeding in the first few days of life causes inadequate enteral intake,
prolongs the intestinal transit time, and leads to increased absorption of
unconjugated bilirubin through enterohepatic circulation.
. The infant can also become relatively dehydrated from the limited fluid intake and lose more
than the expected 10% of the birth weight in the first week of life (further increases the
bilirubin level).
. In otherwise healthy infants, the best treatment for breastfeeding jaundice is to increase the
frequency and duration of feedings to stimulate more milk production.
. As the milk production increases, the infant will become more hydrated, and the bilirubin
level will decrease.
. Sphingolipidosis:
___________________
1- a sphingolipidosis due to a deficiency in sphingomyelinase (Niemann-Pick's disease);
characterized by cherry red macula, protruding abdomen,
- hepatosplenomegaly, lymphadenopathy, and regression of developmental milestones.
- Sphingomyelin accumulates in the reticuloendothelial cells of liver, spleen, bone marrow and
brain.
2- Sphingolipidosis due to a deficiency in hexosaminidase A is known as Tay-Sachs'disease.
- It is characterized by hyperacusis, mental retardation, seizures, cherry red macula, but not
hepatosplenomegaly or cervical lymphadenopathy.
3- Sphingolipidosis due to a deficiency in glucocerebrosidase is known as Gaucher's disease.
- It is characterized by hepatosplenomegaly, anemia, leucopoenia and thrombocytopenia, but
not cherry red macula.
4- Sphingolipidosis due to a deficiency in galactocerebrosidase is known as Krabbe's disease.
- It is characterized by hyperacusis, irritability and seizures.
5- Mucopolysaccharidoses are characterized by coarse facial features, hydrocephalus and
umbilical hernia.
. Neonatal tetanus:
___________________
. Is generally seen in infants born to unimmunized mothers, frequently following umbilical
stump infection.
. Affected infants initially present in the first two weeks of life with poor suckling and fatigue,
followed by rigidity, spasms and opisthotonus.
. Mongolian spot:
_________________
. The classic description (ie , well-demarcated, flat blue/gray lesion on the sacral or presacral
area) of a Mongolian spot.
. This lesion is seen more commonly in dark skinned children.

. It is caused by entrapment of melanin-containing melanocytes during their migration from


the neural crest into the epidermis in fetal development.
. and usually disappears in the first few years of life, most cases do not require any treatment.
. Mostly located at the base of the spine, lower back, and buttocks, it can also be seen in other
parts such as the shoulders, arms, wrists, legs, ankles or abdomen.
. The face, palms and soles are usually spared
N.B.:
- Cafe-au-lait spots are well-demarcated, tan or light brown flat lesions that can vary in
number, size, and distribution.
. This finding may indicate neurofibromatosis if the diameter is larger than 0.5 mm and if more
than 5 lesions are found.
- Cutis Marmorata appears as a lace-like pattern on the skin in response to cold or stress.
. It can persist in some diseases such as Down's syndrome and trisomy 18.
- A salmon patch is a flat salmon-colored lesion commonly seen over the glabella, eyelids, and
neck.
. It is a vascular lesion that usually disappears in early childhood.
. Indications of the evaluation of neonatal jaundice include:
_____________________________________________________________
1. Conjugated hyperbilirubinemia (> 2 mg/dL)
2. Jaundice that appears in the first 24 - 36 hours of life.
2. Serum bilirubin rising at a rate faster than 5 mg/dl/24 hours.
3. Serum bilirubin greater than 12 mg/dl in full-term (especially in the absence of risk factors)
or 10 - 14 mg/dl in preterm infants.
4. Jaundice persists after 10 -14 days of life.
5. The presence of signs or symptoms.
N.B.: Conjugated hyperbilirubinemia (> 2 mg/dL):
- Association with light colored stools, hepatomegaly, and direct (conjugated)
hyperbilirubinemia.
- Is indicative of neonatal cholestasis and impaired hepatic excretion of bilirubin, either by
extrahepatic obstruction or liver cell injury.
- With conjugated neonatal hyperbilirubinemia, priority should be given to conditions that
require prompt diagnosis and treatment, such as
sepsis, endocrinopathy (hypothyroidism). and nutritional hepatotoxicity caused by metabolic
diseases (galactosemia, tyrosinemia).
- After these diagnoses are ruled out, the final step is to differentiate biliary atresia from
neonatal hepatitis.
. Breast-milk jaundice:
_______________________
. Appears in the second week of life; however, the hyperbilirubinemia is indirect
(unconjugated).
. The unconjugated bilirubin levels may rise as high as 10-30 mg/dl Levels drop rapidly as soon
as breastfeeding is stopped,
and the infant is given milk formula for 1-2 days.
. After 2-3 days the breastfeeding can be resumed, and usually there is no return of the
hyperbilirubinemia.
. Although it is a benign condition in some instances, phototherapy may be indicated.

. Physiologic jaundice:
_______________________
. Consists of moderate unconjugated hyperbilirubinemia (< 12-14 mg/dL) appears after the
first 24 hours of life and resolves before the end of the first week.
. It is a benign condition and is more common in preterm infants, infants of diabetic mothers,
and infants of Asian or Native American descent.
. Crigler-Najjar and Gilbert's syndrome:
________________________________________
. Are inherited deficiencies of UDP-glucuronyl transferase that result in unconjugated
hyperbilirubinemia.
. In Gilbert's syndrome, the deficiency is mild, and patients are asymptomatic due to the mild
indirect hyperbilirubinemia.
. In Crigler-Najjar syndrome, the enzyme is absent; patients present early in life, and die within
their first few years.
. Erythroblastosis fetalis:
___________________________
. is characterized by unconjugated hyperbilirubinemia and anemia.
. The diagnosis is suggested by a positive Coombs' test.
. Care of a neonate:
____________________
. Early neonatal care in an uncomplicated pregnancy include:
. initial physical assessment, removal of airway secretions, drying the infant & keeping him
warm, & early preventive measures (gonococcal ophthalmia prevention, vitamin K
supplementation).

===========================================================================
Endocrinology & Genetics
===========================================================
. Precocious pubarche (Puberty):
_________________________________
. It is very important to differentiate between precocious puberty that is caused by premature
activation of the hypothalamus-pituitary-gonad (HPG) axis,
and precocious pseudo-puberty that is caused by a gonadotropin-independent process,
typically an excess of sex steroids.
. In case of precocious pseudo-puberty: there is signs of severe androgen excess (i. e. severe
cystic acne, significant growth acceleration) which suggests precocious
pseudo-puberty It can be caused by late-onset congenital adrenal hyperplasia (21-hydroxylase
defficiency).
. Hypothalamic dysfunction leading to precocious puberty is usually less dramatic in
presentation Sequential development of testicular enlargement, penis enlargement, pubic hair
growth, and then growth spurt is typically present.
. Severe hypothyroidism is a rare cause of precocious puberty, and is characterized by slowing
(not acceleration) of growth.

. Klinefelter's syndrome may present with a height that is higher than normal, but signs of
androgen excess are not typical.
. 47, XYY karyotype may manifest as severe acne, but precocious puberty is not characteristic.
. Premature adrenarche: Extremely high yield question for USMLE
_______________________
. characterized by the isolated appearance of axillary hair before the age of six years.
. This change results from premature androgen secretion of the adrenal glands.
. The condition is generally benign and has no clinical significance.
. On the other hand, premature pubarche (pubic hair growth before the age of 8 years) is more
alarming, as it is associated in 50% of cases with a CNS disorder.
. Congenital adrenal hyperplasia (CAH):
_______________________________________
. Congenital adrenal hyperplasia (CAH): is a group of syndromes characterized by a deficiency
in one of the enzymes responsible for steroid synthesis.
. The most common syndrome is 21-hydroxylase deficiency, which results in excess 17-alphahydroxyprogesterone metabolite that is shunted to androgen,
thereby resulting in hirsutism or virilism.
. This deficiency also leads to a decreased production of mineralocorticoids and
glucocorticoids, and consequent hyponatremia and hyperkalemia.
. Complete deficiencies can present at birth with ambiguous genitalia, salt wasting and
dehydration.
. In partial deficiencies, patients usually present during puberty or adulthood with virilism,
which may or may not be associated with salt wasting.
. The diagnosis is suggested by increased levels of 17-alpha-hydroxyprogesterone and
confirmed with ACTH stimulation test.
. Adolescent onset of hirsutism and virilism, with normal menstruations and elevated 17hydroxyprogesterone, are diagnostic of congenital adrenal hyperplasia.
N.B.:
1- Thyroid dysgenesis is the most common cause of congenital hypothyroidism in United
States.
2- Iodine deficiency or endemic goiter is the most common cause of congenital hypothyroidism
worldwide, but is essentially not seen in the United States.
.N.B:
- Mammary gland enlargement and non-purulent vaginal discharge are common findings in
newborn infants.
. These are transitory physiologic events due to infant's physiologic target-organ response to
transplacentally acquired maternal hormones (estrogens).
. Therefore, such infants only require observation and routine care.
. Primary amenorrhea:
_____________________
. The most common cause of primary amenorrhea is Turner syndrome, a 45,XO karyotype
chromosomal disorder.
. It may also result from anatomic abnormalities (imperforate hymen, absent uterus),
hypothalamic/pituitary dysfunction, hyperandrogenism or pseudohermaphroditism.
. Turner syndrome is characterized by short stature, webbed neck, hypogonadism (streak
ovaries), lymphedema, high-arched palate,
congenitally bicuspid aortic valves, and coarctation of the aorta.
. Most commonly, ovarian failure in Turner syndrome manifests with delayed puberty, though
at least 1/5 of patients will have a normal puberty followed by early menopause.

. The decreased femoral pulses may indicate aortic coarctation, which occurs in approximately
10% of patients with Turner syndrome.
. This clinical diagnosis should be confirmed by karyotype analysis.
.N.B.:
- A progesterone challenge: is used in the diagnostic evaluation of amenorrhea in patients with
both a uterus and normally developed breasts.
- Hyperprolactinemia can cause secondary amenorrhea as prolactin decreases GnRH
production and release.
- 17-hydroxyprogesterone measurement is indicated when congenital adrenal hyperplasia is
suspected as a possible cause of primary amenorrhea.
. Serum 17-0H progesterone is elevated in 21- and 11-hydroxylase deficiencies and is decreased
in 17-hydroxylase deficiency
. McCune-Albright syndrome:
___________________________
. is a rare condition characterized by precocious puberty, cafe au lait spots and multiple bone
defects (polyostotic fibrous dysplasia).
. It is responsible for 5% of the cases of female precocious puberty, and may be associated with
other endocrine disorders,
such as hyperthyroidism, prolactin- or GH-secreting pituitary adenomas, and adrenal
hypercortisolism.
. McCune-Albright syndrome is sporadic and has been recently attributed to a defect in the Gprotein cAMP-kinase function in the affected tissue, thereby resulting
in autonomous activity of that tissue.
. Remember the 3 P's of McCune-Albright syndrome precocious puberty, pigmentation (cafe au
lait spots) and polyostotic fibrous dysplasia.
.N.B.: Adrenal tumors result most commonly in heterosexual precocious puberty (i.e , premature
development of male secondary sexual characteristics in a female).
. Cri-du-chat syndrome:
_______________________
. Is due to 5p deletion and presents as a cat-like cry.
. Also hypotonia, short stature, microcephaly with protruding metopic suture, moonlike face,
hypertelorism, bilateral epicanthal folds,
high arched palate, wide and flat nasal bridge, and mental retardation.
. Gaucher's disease:
____________________
. Is due to the deficient activity of the lysosomal enzyme, acid beta-glucosidase.
. The typical patient is an Ashkenazi Jewish adolescent with chronic fatigue,easy bruisability,
bone pain, and pathological fractures.
. Diagnosis is confirmed with radiologic (Erlenmeyer flask deformity of the distal femur) and
bone marrow studies (Gaucher cells with wrinkled paper appearance).
. Niemann-pick disease:
_______________________
. Types A and B result from the deficient activity of sphingomyelinase.
. this is a fatal disorder of infancy Clinical manifestations include failure to thrive,
hepatosplenomegaly, and

. a rapidly progressive neurodegenerative course that eventually leads to death by age 2-3
years.
. Beckwith-Wiedemann syndrome:
______________________________
. The exact cause is unknown, but sometimes associated with duplication of chromosome 11 p.
. This region contains the gene encoding for IGF-2, which may explain the macrosomia.
. Present with macrosomia, macroglossia, visceromegaly, omphalocele, hypoglycemia,
hyperinsulinemia, prominent eyes, prominent occiput, ear creases & pancreatic hyperplasia.
. It is usually sporadic, but occasional cases have familial inheritance.
. Sometimes, the hypoglycemia may be severe and intractable, and subtotal pancreatectomy
may be needed.
. Patients have an increased risk of neoplasms such as Wilms' tumor, hepatoblastoma, and
gonadoblastoma
. Lesch-Nyhan syndrome:
_______________________
. Is secondary to a deficiency in hypoxanthine-guanine phosphoribosyl transferase (HPRT).
. Symptoms: self-mutilation (self-injury, especially biting of the upper extremities), neurologic
features (mental retardation, dystonia, choreoathetosis, spasticity).
. Gouty arthritis, and tophus formation.
. Gout is usually seen in patients above 50 years of age; therefore, suspect Lesch-Nyhan
syndrome if you see a boy with gout.
. All victims of Lesch-Nyhan syndrome are male.
. Prader-Willi syndrome (PWS):
______________________________
. Is a sporadic disorder, in 50-70 % of cases, there is a deletion in the long arm of chromosome
15.
. PWS displays a particular type of genetic transmission called genomic imprinting, wherein the
phenotype expression depends on whether
the genetic defect is inherited from the mother or the father.
. presentation: severe hypotonia at birth or infancy, hyperphagia, obesity, short stature and
mental retardation.
. The typical craniofacial features are narrow bifrontal diameter, diamond-shaped eyes and a
small, down-turned mouth.
. Hypothalamic dysfunctions (GH deficiency and hypogonadotropic hypogonadism) may be
present.
. The associated short stature, obesity and hypotonia usually respond to GH administration;
however,
the implication of GH deficiency accompanying PWS in the development of these features is
still controversial.
.N.B:
- If you see a vignette with a child that is >2-years-old, it is unlikely that the diagnosis is Patau's
or Edward's syndrome.
. Macrosomia secondary to maternal diabetes:
________________________________________
. Important differ. diag. of Beckwith-Wiedemann syndrome; but here infants do not present
with dysmorphic features as omphalocele, prominent occiput & macroglossia.
. Moreover, the prenatal and birth histories of the patient maternat D.M.
. The common congenital problems that in an 'infant of diabetic mother:
1. Caudal regression syndrome

2. Transposition of great vessels.


3. Duodenal atresia and small left colon.
4. Anencephaly and neural tube defects.
. WAGR syndrome:
________________
. Characterized by Wilms' tumor, aniridia, genitourinary anomaly and mental retardation.
. It is related to a deletion in chromosome 11 involving the gene WT1. (Wilms' Tumor 1 gene)
and aniridia gene PAX6.
. Galactosemia:
_______________
. Is a metabolic disorder caused by galactose-1-phosphate uridyl transferase deficiency, eading
to elevated blood levels of galactose.
. Manifest a few days or weeks after the infant starts taking formula or breast milk.
. These include liver failure (hepatomegaly, direct hyperbilirubinemia, disorders of
coagulation), abnormal renal function, emesis, anorexia, acidosis & glycosuria.
. Present with: infant or newborn with failure to thrive, bilateral cataracts, jaundice and
hypoglycemia.
. Other common manifestations: aminoaciduria, hepatic cirrhosis, hypoglycemia, and mental
retardation.
. Such patients are at increased risk for E.coli neonatal sepsis.
. Galactosemia is NOT a self-limiting condition.
. Early diagnosis and treatment by el imination of galactose from the diet are mandatory.
. phenylketonuria:
__________________
. an autosomal recessive disorder.
. It occurs in approximately 1/10,000 births, and is more frequent in the Caucasian and Asian
populations.
. Pathology: a deficiency in phenylalanine hydroxylase, that breaks down phenylalanine into
tyrosine.
. This deficiency leads to the accumulation of phenylalanine and its metabolic products in the
blood and body tissues (particularly in the brain).
. If untreated, the defect can produce mental retardation and seizures.
. The most commonly used screening test is the determination of blood phenylalanine levels.
. Guthrie test, which is a qualitative (coloration) test, can also be used as it detects the presence
of metabolic products of phenylalanine in the urine.
. Treatment: a low-phenylalanine diet, since little amounts of phenylalanine are still necessary
for growth and development.
. Cereals, starches, fruits, vegetables, and phenylalanine-free milk formulas are recommended.
. High-protein foods should be avoided.
. Early diagnosis and treatment can improve the prognosis, with most (treated) patients having
normal mental development and a normal life span.
. Congenital hypothyroidism:
____________________________
. The most common cause is thyroid dysgenesis (ie , aplasia, hypoplasia, or ectopic gland).
. Other causes: inborn errors of thyroxin synthesis, and transplacental maternal thyrotropinreceptor blocking antibodies.
. Infants initially appear normal at birth, but gradually develop apathy, weakness, hypotonia,
large tongue, sluggish movement, abdominal bloating & umbilical hernia.
. Other signs: pathologic jaundice, difficult breathing, noisy respiration, hypothermia, and
refractory macrocytic anemia.

. Infants initially appear normal due to the presence of moderate amounts of maternal
hormones in the infant's circulation.
. So, screening is mandated in all states at birth to allow for the early detection, treatment, and
consequent improvement of the prognosis.
. Screening is done by measuring serum T4 and TSH levels.
. The treatment is levothyroxine.
. Fragile X syndrome:
_____________________
. Male infant with a characteristic appearance (la rge head, long face, prominent forehead and
chin, protruding ears), joint laxity, and large testes.
. Behavioral abnormalities: hyperactivity, short attention span, and autism, are common.
. It results from a full mutation in the FMR1 gene caused by an increased number of CGG
trinucleotide repeats accompanied by aberrant methylation of the FMR1 gene.
. Turner syndrome:
__________________
. Presentation: short stature, short webbed neck and broad chest with widely spaced nipples
and 45 XO karyotype.
. Other features: a low posterior hairline, gonadal dysgenesis, infertility and primary
amenorrhea, which may occur as the child grows.
. A higher incidence of cardiac anomaly: coarctation of the aorta (COA) and a bicuspid aortic
valve.
. Physical findings of COA include upper extremity hypertension and a delay in the radialfemoral pulse.
. Congestive heart failure may be seen in infants with severe cases.
. The diagnosis is made with an echocardiogram and the treatment is surgery.
. There is higher risk of renal abnormalities, particularly horseshoe kidney, and should have a
screening ultrasound after the diagnosis is made.
. Swollen hands and feet due to congenital lymphedema, which is common in patients with
Turner syndrome due to abnormal development of the lymphatic network.
. Edema that is due to lymphedema is generally nonpitting on physical exam.
. Patients are prone to osteoporosis which increases the risk of bone fracture, this is due to low
estrogen levels from gonadal dysgenesis.
. Edward's syndrome:
____________________
. trisomy 18.
. The features: micrognathia, microcephaly, rocker bottom feet, overlapping fingers and absent
palmar creases.
. Closed fists with index finger overlapping the 3rd digit and the 5th digit overlapping the 4th,
and rocker bottom feet.
. Congenital heart disease occurs in greater than 50 % of affected patients; ventricular septal
defect is the most common one.
. 80% of affected children die in the first month, 90% die by 1 year and remaining 5-10% are
mentally retarded.
. Osteomalacia caused by vitamin D deficiency:
_______________________________________________
. Vitamin D deficiency leads to decreased intestinal calcium absorption and hypocalcemia.
. Hypocalcemia stimulates parathyroid glands & secondary hyperparathyroidism results, which
brings serum calcium to normal or near normal, especially in early stages.
. Secondary hyperparathyroidism causes hypophosphatemia by increasing its urinary excretion.

. Therefore, vitamin D deficiency causes more marked hypophosphatemia than hypocalcemia,


especially in early stages.
. Labs show low or low-normal serum calcium, low serum phosphate, increased serum pTH,
low plasma 25-OH vitamin D & normal, low or elevated 1,25 dihydroxy vitamin D (calcitriol).
. In patients with Pseudohypoparathyroidism, labs show low serum calcium, high serum
phosphate and high serum PTH.
. Serum alkaline phosphatase and vitamin D levels are within normal limit.
. Patients with renal failure will have low serum calcium, high serum phosphate, high serum
PTH (due to secondary hyperparathyroidism) and low serum 1,25 dihydroxy vitamin D.
. Patients with X-linked hypophosphatemic rickets have low serum phosphate due to renal
phosphate wasting.
. They have normal serum calcium, normal serum PTH and normal serum alkaline phosphatase
and normal levels of serum 25-hydroxy vitamin D.
. There may also be a functional defect in the activity of 1-hydroxylase and as a result serum
levels of calcitriol may below.
. In type II vitamin D dependent rickets, there is mutation of vitamin D receptor.
. Therefore, these patients have normal serum levels of calcitriol but it is ineffective and as a
result osteomalacia occurs.
. Pubertal gynecomastia: Extremely high yield question for USMLE!!!
________________________
. is seen in approximately one-half of adolescent boys, at an average age of 14 years.
. It is often asymmetric or transiently unilateral, and frequently tender.
. In prepubertal males the testicular size is normally 2 cm in length and 3 ml in volume.
. The initial management involves reassurance and watchful waiting/observation.
. Down syndrome:
________________
. Patients with Down syndrome may have a wide range in severity of manifestations.
. The most common congenital heart defect is an endocardial cushion defect of the
atrioventricular (AV) canal.
. Other congenital heart defects: PDA & ventricular septal defect (condenital heart defects are
the most common cause of death in childhood).
. AV canal defects are diagnosed with ECho.
. The left to right shunt due to the (AV) defect can develop into pulmonary hypertension,that
manifested as a loud P2 on auscultation & has a high morbidity.
. AV canal defects may be partial or complete, but both need early surgical correction to
prevent pulmonary hypertension.
. Patients with Down's syndrome have a high predilection for duodenal atresia.
. Other GIT anomalies: Hirschsprung's disease, Esophageal atresia, Pylo ric stenosis,
Malrotation of the bowel.
. Atlantoaxial instability is commonly seen in Down syndrome, and most commonly occurs due
to excessive laxity in the posterior transverse ligament, which causes
increased mobility between the atlas (C1) and the axis (C2).
. Symptoms of atlantoaxial instability: behavioral changes, torticollis, urinary incontinence, and
vertebrobasilar symptoms as dizziness, vertigo, and diplopia.
. Also upper motor neuron symptoms such as leg spasticity, hyperreflexia, a positive Babinski
sign, and clonus.
. Patients with Down syndrome are normally hypotonic, and they may remain hypotonic or
have increased tone due to atlantoaxial instability.

. It is diagnosed by lateral radiographs of cervical spine in flexion, extension & in a neutral


position Open mouth radiographs can also visualize the odontoid.
. Treatment: surgical fusion of the first cervical vertebrae (C1) to the second (C2).
. Marfan syndrome:
__________________
. MSF is an autosomal dominant disorder that results from the mutations of the fibrill in-1
(FBN1) gene.
. It presents with tall stature, long and emaciated extremities, arachnodactyly, hypermobility of
the joints, upward lens dislocation, and aortic root dilation.
. Homocystinuria:
_________________
. An autosomal recessive disease caused by cystathionine synthase deficiency.
. Her features are characteristic of Marfan's syndrome (tall stature, long emaciated extremities,
arachnodactyly, hyperlaxity of the skin and joints).
. But her clinical presentation of a stroke is highly indicative of homocystinuria.
. Remember: Marfan's features + thromboembolic events = classic homocystinuria.
. Thromboembolic events at any age, due to the pathologic changes in the vessel walls and
increased adhesiveness of the platelets (commonly cerebral vessels).
. Lens dislocation (ectopia lentis): In Marfan's syndrome, the lens is dislocated upward, and in
homocystinuria, it is dislocated downward.
. Elevation of both homocysteine and methionine in body fluids confirm the diagnosis.
. The initial treatment: high doses of Vitamin B6.
. Restriction of methionine along with supplementation of cysteine is used for patients not
responsive to Vitamin B6 therapy.
. Glucose-6-phosphatase deficiency:
___________________________________
. is also known as type I glycogen storage disease and Von-Gierkes' disease.
. As its name suggests, this condition is caused by deficient glucose-6-phosphatase in the liver,
kidneys, and intestinal mucosa.
. The typical patient, is 3-4 months of age with hypoglycemia, lactic acidosis, hyperuricemia,
and hyperlipidemia.
. Hypoglycemic seizures may occur.
. The characteristic features: a doll-like face (fat cheeks), thin extremities, short stature, and a
protuberant abdomen (due to the enlarged liver and kidneys).
. The spleen and heart are normal.
===========================================================================
Nephrology =================================================================
. Systemic Lupus erythematosus:
_________________________________
. Positive anti-Smith antibodies and/or anti-double stranded DNA antibodies is specific and
confirmatory for the diagnosis of systemic lupus erythematosus
. Transient proteinuria:
________________________

. A urine dipstick can be positive in up to 10% of school-aged children. Proteinuria in children


can be transient (intermittent), orthostatic, or persistent.
. Transient proteinuria is the most common cause of proteinuria and can be caused by fever,
exercise, seizures, stress, or volume depletion.
. Orthostatic proteinuria is very common in adolescent boys and is defined as increased
protein when the patient is in an upright position
that returns to normal when the patient is recumbent.
. If the urinalysis shows no hematuria and is otherwise normal, the urine dipstick should be
repeated on at least two additional specimens.
. If these subsequent tests are negative for protein, the diagnosis is transient proteinuria that
are usually benign conditions that require no further evaluation.
. Posterior urethral valves:
____________________________
. are the most common congenital obstructive urethral lesion in males.
. There is midline lower abdominal mass probably represents a distended bladder.
. The finding of a distended bladder indicates that the obstructive lesion is distal to the
bladder neck.
. Posterior urethral valves are abnormal folds in the posterior urethral wall (distal prostatic
urethra) that obstruct urine flow out of the bladder.
. Affected infants may develop hydronephrosis, azotemia, and failure to thrive.
. A voiding cystourethrogram (VCUG) is the diagnostic test of choice.
. Bladder exstrophy:
____________________
. Results from a ventral defect of the urogenital sinus, which causes separation of the pubic
rami, external rotation of the hips,
and separation of the rectus abdominis muscles.
. The resulting ventral midline defect allows protrusion of a malformed bladder.
. This condition is associated with epispadias, recurrent urinary tract infections, and urinary
incontinence.
. Hypospadias:
______________
. is a congenital abnormality where the penile urethra opens on the ventral surface of the penis
rather than at the tip.
. It results from incomplete fusion of the urethral folds on the ventral penis.
. urachus:
__________
. is a tubular extension of the allantois that extends from the bladder to the umbilicus.
. It is normally obliterated during fetal development A patent urachus can result in an urachal
fistula, cyst or sinus.
. None of these conditions would interfere with urine drainage from the bladder.
. An omphalomesenteric duct cyst:
_________________________________
. is a focal failure of vitelline duct obliteration.
. Such a vitelline cyst may cause a small midline mass deep to the umbilicus.
. The cyst could be attached to the umbilicus and the wall of the ileum by vitelline remnants
and may cause small intestinal volvulus.
. Cryptorchidism:
_________________

. is a fa ilure of one or both testes to descend from the abdomen through the inguinal canal(s)
into the scrotum.
. Wilms tumor (nephroblastoma):
_______________________________
. Wilms tumor is usually diagnosed between the ages of two and five years.
. It is the most common primary renal neoplasm of childhood.
. 80% of the cases have an asymptomatic abdominal mass that is usually detected by the
mother/caretaker while bathing the child.
. The mass may be bilateral, symptoms may include hypertension, hematuria, abdominal pain,
and vomiting.
. In a few cases, it may present with lung metastases.
. It is associated with Beckwith-Wiedemann and Denys-Drash syndromes.
. The treatment is nephrectomy.
. If treated in the early stage, majority of the patients have a long-term survival.
. It arises from metanephros, which is the embryologic precursor of the renal parenchyma.
. Neuroblastoma (NBL):
______________________
. Neuroblastoma, which is the most common extracranial solid tumor of childhood.
. The median age at diagnosis is 2 years.
. The tumor arises from neural crest cells, which are also the precursor cells of the sympathetic
chains and adrenal medulla.
. For this reason, NBL may arise from the adrenal gland or any location along the paravertebral
sympathetic chains.
. The most common site involved is the abdomen, either from the adrenals or retroperitoneal
ganglia.
. The mass is usually firm and nodular in consistency Calcifications and hemorrhages are seen
on plain x- ray and CT scan.
. Up to 70 % of patients have metastatic disease at time of presentation, and the most common
metastatic sites are: long bones, skull, bone marrow, liver, L.N. & skin.
. The levels of serum and urine catecholamines and their metabolites (i.e. HVA and VMA) are
usually elevated;
. however, patients do not present with fainting spells, sweating, palpitations and
hypertension, as in pheochromocytoma.
. Renal tubular acidosis (RTA):
_______________________________
. Is a normal anion gap metabolic acidosis caused by a defect in the ability of the renal tubules
to reabsorb bicarbonate or excrete hydrogen.
. There are three types of RTA:
1. Type 1 or distal RTA occurs due to a defect in hydrogen ion secretion.
These patients are acidotic, hypokalemic, and have an elevated urinary pH In children, type 1
RTA is often a genetic disorder.
Patients commonly develop nephrolithiasis.
2. Type 2 RTA is caused by decreased bicarbonate reabsorption in the proximal tubule. Fanconi
syndrome is a common cause in children.
3. Type 4 RTA is caused by a defect in the sodium/potassium exchange in the distal tubule
which results in hyperkalemic, hyperchloremic acidosis.
In children, obstructive uropathy, renal disease, or multicystic dysplastic kidneys are
common causes.

. Renal tubular acidosis can present as growth failure and should be considered in the
differential diagnosis for failure to thrive.
. Screening labs will show a low bicarbonate level with an increase in chloride, producing a
normal anion gap metabolic acidosis.
. Minimal change disease:
_________________________
. The most common cause of nephrotic syndrome in patients younger than 16 years old.
. Light microscopy and immunofluorescence does not usually reveal any change in kidney
architecture.
. Electron microscopy shows diffuse effacement of foot processes of podocytes.
. Minimal change disease is a highly steroid-sensitive condition and is the most common cause
of nephrotic syndrome in children.
. For these reasons, empiric steroid therapy is indicated in any child with a clinical presentation
suggestive of nephrotic syndrome
. Steroids are the treatment of choice.
N.B.:
- Renal biopsy is commonly used in adult patients presenting with nephrotic syndrome.
- Diffuse thickening of glomerular basement membrane & subepithelial spikes are
characteristic for membranous glomerulonephritis, which is the most common
cause of nephrotic syndrome in adults, it is relatively uncommon in younger patients.
- Crescent formation is a typical finding for rapidly progressive glomerulonephritis.
. Vaginal foreign bodies:
_________________________
. Common in pre-pubertal children, Present with acute or chronic vaginal discharge, foulsmelling odor, and vaginal bleeding.
. If the object is large or has sharp edges, the patient may also complain of pain.
. The vaginitis produced by a foreign body can cause the child to have urinary complaints as
well.
. The most common vaginal foreign body found in children is toilet paper.
. Other items: toys or coins that the child has placed into her vagina.
. Any child with vaginal discharge should have a vaginal examination, which is usually
performed by placing the child in a knee to chest or frog-legged position.
. The external genitalia should be examined as well as the vaginal introitus.
. Foreign bodies can often be seen if the child is asked to valsalva during the examination.
. If needed, a nasal speculum may be used.
. If a foreign body is seen, irrigation with warmed fluid should be performed in an attempt to
flush out the foreign body.
. If irrigation is unsuccessful, examination and foreign body removal should be done with
sedation or general anesthesia.
. Nocturnal enuresis:
_____________________
. Due to a developmental disorder or maturational lag in bladder control while asleep.
. The condition is more common in boys and in those with a positive family history.
. More than 20% of the cases spontaneously resolve before the patients are school age.
. Reassurance of the parents is usually the first step in management, although some physicians
recommend bladder exercises and scheduled toileting.
. Conditioning with "wet" alarms and waking the child in the middle of the night have also
helped a moderate number of children.
. For patients with persistent nocturnal enuresis, the drug of choice is DDAVP (desmopressin) .
. The second line medication is imipramine.

N.B.:
- Bedwetting is considered normal before the age of 5 years and no investigations or medical
treatment should be prescribed for it.
. Fibromuscular dysplasia:
_________________________
. The most common cause of secondary hypertension in children is fibromuscular dysplasia.
. It is responsible for approximately 20% of all cases of renal hypertension.
. Aside from children, fibromuscular dysplasia is also generally seen in premenopausal women
(or women less than 50 years old).
. Physical examination reveals a hum or bruit in the costovertebral angle due to well-developed
collaterals.
. The right renal artery is more affected than the left.
. Angiography typically shows a "string of beads" pattern to the renal artery.
. Vesicoureteral reflux (VUR):
______________________________
. Is the retrograde flow of urine from the bladder to the ureter and renal pelvis.
. Reflux is a risk factor for UTI & Repeated attacks of UTI can lead to progressive renal scarring,
which is the major cause of end stage renal disease and hypertension in children.
. It is a risk factor for UTI as it facilitates the transport of bacteria from bladder to the upper
urinary tract,
. this explains why VUR is the most likely cause of acute UTI in children.
. VUR is present in 35-40 o/o of children with UTI.
. The diagnosis is made with voiding cystourethrogram (VCUG) followed by renal imaging.
. Diagnosis of VUR is best made with a voiding cystourethrogram or a radionuclide cystogram
(RNC).
. Renal ultrasonography is less sensitive in detecting reflux nephropathy.
. The American Academy of Pediatrics recommends that all children aged 2 to 24 months with
a first UTI should undergo a VCUG or RNC to detect the presence VUR.
. N.B:
- Chronic pyelonephritis is characterized by focal parenchymal scarring and blunting of calices
on IVP.
- Hydronephrosis is seen on IVP as dilation of the collecting system including the calyces,
pelvis and ureter, depending on the level of obstruction.
- Ureteropelvic obstruction would also lead to hydronephrotic changes in the kidney with IVP
showing the level of obstruction as a constriction in the flow of dye.
. Acute pyelonephritis:
_______________________
. usually present with fever, chills, nausea, vomiting and flank or suprapubic pain.
. Physical examination shows costovertebral angle tenderness.
. Urinalysis shows bacteriuria and pyuria.
. Therapy should be started with empiric antibiotics after withdrawal blood & urine for culture
and sensitivity and before its results are available.
. Severe disease with systemic manifestations warrants intravenous (IV) antibiotics.
. If patient is vomiting and hypotensive; therefore, IV antibiotics should be administered.
=========================================================================
Infection =====================================================================

. Measles (rubeola):
____________________
. It is caused by Paramyxovirus.
. Characterized by a prodrome of non-productive cough, coryza, non-purulent conjunctivitis,
followed by
. Koplik's spots (pathognomonic): red spots with bluish specks over the buccal mucosa,
opposite the premolar tooth & sometimes on the inner conjunctivae & vaginal mucosa
. And maculopapular rash initially appearing on the face then spread to involve the entire
body.
. Lab. findings: leukopenia, lymphopenia and Proteinuria may be seen.
. Diagnosis is mostly clinical and can be supported by a fourfold rise in hemagglutination
inhibition antibody titer.
. Leukopenia (T-cell cytopenia) and thrombocytopenia can be seen with measles infection.
. Vitamin A has been shown to reduce the morbidity and mortality rates of patients with
measles through immune enhancement.
. It also helps the gastrointestinal and respiratory epithelium to regenerate.
. N.B.: - Atypical measles:
- Occurs in persons who have previously received inactivated measles vaccine (was available in
the 1960's), (the measles vaccine available now is a live one).
- This form is potentially life-threatening, and characterized by atypical rash (not
maculopapular), the absence of Koplik spots, arthritis,
hepatitis and lung involvement Edema of the hands and feet may occur.
. Rubella:
__________
. characterized by low-grade fever, lymphadenopathy (sub-occipital and posterior auricular)
and rash.
. The rash is erythematous, maculopapular and classically begins on the face, spreading
subsequently down the body.
. congenital rubella infection:
_______________________________
. Transmitted via the placenta, the causative organism is Toga virus.
. Maternal infection manifests after an incubation period of 14 to 21 days with mild symptoms
such as rash, arthralgias and generalized lymphadenopathy.
. Since rubella infection confers permanent immunity, only primary infection in the pregnant
woman carries the risk of fetal disease.
. The severity of fetal disease depends on the time of pregnancy when the transmission took
place.
. Infections of early pregnancy are the most severe, and the associated adverse outcomes
include spontaneous abortion and congenital rubella syndrome (CRS).
. If transmission occurs in the first 4 weeks of pregnancy, the risk of developing CRS is 50%,
drops to 1 % if transmission occurs in the third trimester.
. Features of CRS: IUGR, deafness, cardiac malformations (e g , patent ductus arteriosus, atrial
septal defects), microphthalmia,
cataract, retinopathy, hepatosplenomegaly, thrombocytopenia, and CNS involvement.
. Chronic infection may result in growth retardation, radiolucent bone disease, jaundice,
hepatosplenomegaly, thrombocytopenia and purple skin lesions
("blueberry muffin spots").
. Maternal infection is confirmed by the presence of lgM to rubella virus, or a fourfold increase
in lgG on two serum samples obtained two weeks apart.

. In the infant, the diagnosis is confirmed by the presence of lgM or persistence of lgG beyond
the age of 6 months.
. Prevention of congenital rubella is primarily achieved by administration of rubella vaccine to
all females of childbearing age.
. If the immunologic status of a pregnant woman is unknown, rubella titers should be obtained
in the first trimester.
. Immunization should not be performed in pregnancy because of a theoretical risk for the
fetus, and such women should be advised to avoid anyone with possible
rubella infection.
. classic triad of congenital rubella syndrome (CRS) - sensorineural deafness, cardiac
malformations (e g , PDA and ASD), and cataracts.
. Erythema infectiosum or Fifth disease:
________________________________________
. caused by human parvovirus 819.
. Children have different presentations and develop a rash with a "slapped cheeks" appearance.
. Fever is not present or very mild in this disease.
. Roseola infantum:
___________________
. Caused by herpes virus 6.
. Characterized by the abrupt onset of high-grade fever, which is then followed by a
maculopapular rash, appearing on the trunk and then spreading peripherally.
. The patient is no longer febrile when the rash develops.
. There are no positive physical signs such as sore throat or lymphadenopathy during the
febrile stage.
. chickenpox (Varicella virus):
_______________________________
. The clinical manifestations of chickenpox in healthy children generally develop within fifteen
days after the exposure.
. Include a prodrome of fever, malaise, or pharyngitis, followed by the development of a
generalized vesicular rash, usually within 24 hours.
. The lesions are commonly pruritic and appear as successive crops of vesicles over a three to
four day period.
. Patients typically have lesions in different stages of development on the face, trunk and
extremities.
. New lesion formation generally stops within four days, and most lesions are fully crusted by
the 6th day in normal hosts.
. Impetigo :
____________
. Is a superficial skin infection with multiple vesiculopustules on the exposed areas of the face
and extremities.
. Is a contagious disease caused by Staphylococcus aureus, Streptococcus, or both.
. It presents as an erythematous macule, which rapidly evolves into vesicles and pustules.
. The pustules later rupture and leave honey-colored, crusted exudates.
. There are 2 forms of impetigo vesiculo-pustular and bullous types Bullous type is caused by
Staphylococcus.
. These vesiculopustules eventually rupture, and then appear encrusted with a characteristic
golden-yellow color.
. A history of skin trauma or insect bite is common, and local lymphadenopathy can be
present.
. Impetigo is most commonly seen in children, and is associated with post-streptococcal
glomerulonephritis.

. The etiologic agent is either group-A beta-hemolytic streptococci (GABS) or S. aureus, the
clinical presentations of these organisms are almost similar.
. Factors predispose to impetigo are warm and humid climate, poverty, crowding, poor
personal hygiene, and carriage of GABS or S . aureus.
. Nasal carriage of Staphylococci can cause recurrent impetigo.
. Topical mupirocin is the treatment of choice or oral erythromycin.
. Kawasaki disease or mucocutaneous lymph node syndrome:
________________________________________________________
. Criteria of Kawasaki's disease:
1. Fever for > 5 days, and
2. Four of the following symptoms:
Bulbar conjunctiva! injection.
Desquamation of the finger and toe tips, indurative edema.
Erythema, fissuring, and crusting of the lips, strawberry tongue, and diffuse mucosal
injection of the oropharynx.
Morbill ifo rm truncal exanthem.
Cervical lymphadenopathy.
. Kawasaki disease is one of the most common causes of generalized vasculitis in children.
. It is usually self-limited, although it can be fatal because of giant aneurysm formation,
thrombosis or rupture of the coronary arteries, leading to MI.
. 2D echocardiography is used to assess cardiac function and coronary vasculature (Coronary
artery aneurysms are the most serious complication of Kawasaki disease).
. A baseline echocardiography is to be performed within 7 days of the disease onset, then
repeated 6 to 8 weeks later.
. The laboratory of Kawasaki's disease: increased (ESR) and C-reactive protein, leukocytosis,
normochromic-normocytic anemia, and thrombocytopenia.
. Thrombocytopenia is a non-specific finding and does not characterize this illness.
. Fever in Kawasaki's disease poorly responds to acetaminophen.
. All patients with suspected Kawasaki disease should be hospitalized and treated with
intravenous immune globulin ( IVIG) and high-dose aspirin.
. The mainstays of systemic treatment of Kawasaki's disease are intravenous immunoglobulin
(IVIG) and aspirin.
. Aspirin is useful for fever and arthritis.
. This is one of the conditions in which aspirin is recommended in children despite concerns
about Reye's syndrome.
. Influenza vaccination is recommended in patients with Kawasaki's disease taking life long
aspirin therapy.
. IVIG has been shown to reduce the incidence of coronary artery aneurysms and other
complications.
. If untreated, up to 25 % of these patients may develop coronary artery aneurysms.
. Scarlet fever:
________________
. Caused by strains of Group A streptococcus that produce erythrogenic exotoxins.
. Has the same mode of transmission and age of distribution as streptococcal pharyngitis.
. The illness may follow a streptococcal pharyngitis, wound infections, burns, or streptococcal
skin infection.
. It begins acutely after an incubation period of 1 to 7 days.
. Initial symptoms include fever, chills, toxicity, abdominal pain, and pharyngitis.
. The rash initially appears on the neck, axillae, and groin within 12 to 48 hours, and
subsequently generalizes within 24 hours.

. The rash characteristically has a punctate or finely papular texture which is sometimes
readily palpable; hence, the "sandpaper-like" description.
. The pharynx is typically erythematous, swollen and possibly covered with gray-white
exudates, strawberry tongue may be present as in cases of Kawasaki disease.
. The area around the mouth appears pale in comparison with the extremely red cheeks, giving
the appearance of "circumoral pallor.
. Towards the end of the first week, desquamation begins in the face, progresses down the
trunk, and finally extends to the hands and feet.
. The most common condition with which Kawasaki's disease is confused is scarlet fever;
however, scarlet fever will have
a positive streptococcal throat test, and normal-appearing lips, as opposed to negative
streptococcal throat test and inflamed lips in Kawasaki's disease.
. Latex agglutination test is more rapid but less accurate in the diagnosis of scarlet fever.
. The treatment of choice for scarlet fever is a ten-day course oif penicillin V; however, in
penicill in-allergic patients, erythromycin or clindamycin can be used.
. Treatment of scarlet fever with penicillin doesn't prevent post-streptococcal
glomerulonephritis.
. Staphylococcal scalded skin:
______________________________
. Caused by exfoliative strains of S. aureus, and is characterized by the development of
superficial flaccid bullae followed by an extensive exfoliation of the skin.
. It is most common in infancy, and rarely occurs beyond five years of age.
. Herpangina:
_____________
. is a throat infection caused by enteroviruses, especially Coxsackie A.
. It is characterized by a high fever and a severe sore throat that may result in a complete
inability to swallow, sometimes necessitating IV hydration.
. Ulcerative lesions are found on the palate, tonsils, and pharynx Sometimes,
. these lesions appear on the palms and soles & is called (hand-foot-mouth disease).
. It generally does not produce a rash after taking an antibiotic.
. Infectious mononucleosis:
____________________________
. caused by the Epstein-Barr virus.
. Sometimes detected only when the patient develops a characteristic polymorphous rash after
taking ampicillin or amoxicillin for an apparent URT infection.
. A prodrome of malaise, fever and adenopathy, the virus is spread via the saliva, and the
condition usually resolves after 2-3 weeks.
. can also present with exudative pharyngitis and lymphadenopathy.
.N.B.:
- Group A beta-hemolytic streptococci (not Group B) is responsible for 20-25% cases of
pharyngitis in children.
. Group B streptococcus infections in the adult are serious. these are common in diabetics and
those with peripheral vascular disease.
. Complications include endocarditis, arthritis, pneumonia, empyema and meningitis.
. It does not present with a rash after taking amoxicillin.
. Toxic epidermal necrolysis:

_____________________________
. is another form of cutaneous hypersensitivity that is sometimes considered to be a variant of
Stevens-Johnson syndrome.
. Most cases are secondary to medications, such as sulfa drugs, anticonvulsants, and NSAIDs.
. Similar to staphylococcal-scalded syndrome, it presents with extensive erythema, tenderness,
and blister formation, followed by denudation of the epidermis.
. Mucous membranes are severely affected, and shedding of the nails may occur.
. Stevens-Johnson syndrome:
__________________________
. A severe variant of erythema multiforme.
. Characterized by skin lesions of erythema multiforme (target lesions), followed by
inflammatory bullae of two or more mucous membranes.
. Sometimes, the GI, respiratory, or GU tracts may also be involved.
. Rabies:
_________
. Fatal disease that is transmitted to humans by contact with saliva or nervous tissue from an
infected animal through a bite, open wound, or mucus membrane contact.
. Bats are a major reservoir for the rabies virus, Bat bites are often so quick and minor that the
victim is not even aware.
. For this reason, any direct contact between bats and humans requires post-exposure rabies
prophylaxis.
. People bitten by domestic animals suspected of being rabid or not available for observation,
or
by wild carnivores like raccoons, skunks, and foxes should also receive prophylaxis.
. Rabbits and small rodents like squirrels, chipmunks, and rats are ra rely infected with
rabies,so Post-exposure prophylaxis following bites is not routine.
. Viral meningitis:
___________________
. usually self-limited inflammation of the leptomeninges caused by a viral infection.
. 90% of cases are caused by non-polio enteroviruses, such as echovirus and coxsackievirus.
. The incidence of viral meningitis decreases with increasing age Infants are most commonly
affected, and disease morbidity and mortality is highest in this group.
. Present with a viral prodrome of constitutional and upper respiratory symptoms with lowgrade fever.
. Over the next 36-48 hours, the patient develops a high fever, headache, irritability, and nuchal
rigidity.
. Focal neurologic signs are absent.
. The patient may also present with seizures. Other symptoms of enteroviral infection may also
be seen such as pharyngitis, rash, or herpangina.
. In viral meningitis, (CSF) will show pleocytosis with lymphocytic predominance, although
neutrophils may predominate early in the course.
. The protein level is normal to slightly elevated and the glucose level is normal, CSF gram stain
will not show any organisms.
. Treatment is supportive; in most patients, symptoms resolve within 7-10 days.
. Viral meningitis can be caused by Epstein-Barr virus, but this is a much less common cause
than the enteroviruses.
. Bacterial meningitis:

_______________________
. caused by Streptococcus pneumoniae or Neisseria meningitidis.
. will have a similar presentation However,
. CSF examination will show an increased cell count with a predominance of neutrophils,
increased protein, and decreased glucose Gram stain will show bacteria.
. Suspect meningococcemia in a neonate with signs of meningitis and a petechial rash.
. 75% of patients with Meningococcus meningitis present with a petechial rash that is
prominent on the axilla, wrists, flanks and ankles.
. It appears within 24 hours of the infection, and the patient generally appears sick.
. Even though pneumococcus is the most common organism responsible for meningitis, it does
not cause a rash.
. In an infant with meningococcemia, watch out for Waterhouse-Friderichsen syndrome, which
is
. characterized by a sudden vasomotor collapse and skin rash (large purpuric lesions on the
flanks) due to adrenal hemorrhage Fulminant meningococcemia can occur
after a meningococcus infection, and approximately 10-20% of infants present with
vasomotor collapse, large petechiae and purpuric lesions.
. The condition carries an almost 100% mortality.
. Lumbar puncture (LP) is the key to establishing the diagnosis of meningitis and differentiating
between viral and bacterial causes.
. The first steps in management: supportive care with oxygen and fluids, a CBC, and blood
cultures.
. In contrast to suspected meningitis in adults, in whom antibiotics are followed by (CT) scan
and LP, children should first have an LP and then antibiotics.
. An LP is quick and easy to obtain in most children, even if they are agitated.
. Herniation is less likely in infants aged <1 year because the fontanelles have not fully closed.
. This patient should have an LP followed by administration of vancomycin (given the high
incidence of resistant S. pneumoniae) and a third-generation cephalosporin
. If the LP is unsuccessful or cannot be performed, then antibiotics should be given and the LP
obtained as soon as possible.
. Although antibiotics given before LP can affect the yield of (CSF) Gram stain and culture,
pathogens other than meningococcus can usually be
identified in the CSF up to several hours after the administration of antibiotics.
. Blood cultures must be obtained prior to the antibiotics.
. Many experts recommend dexamethasone for all children aged >6 weeks with pneumococcal
meningitis and for those with Haemophilus influenza type b meningitis
(if given within 1 hour of initial antibiotic therapy) to reduce the incidence of hearing loss with
bacterial meningitis.
. N.B.:
- CSF in tuberculous meningitis also shows a mildly elevated white cell count with lymphocyte
predominance; the protein will be very high and glucose will be low.
However, the presentation tends to be subacute rather than acute.
- The long-term neurologic sequelae associated with bacterial meningitis are:
1. hearing loss
2. loss of cognitive functions (due to the neuronal loss in the dentate gyrus of the
hippocampus)
3. seizures
4. mental retardation
5. spasticity or paresis

. TORCHs infections:
___________________
. Toxoplasmosis, rubella, CMV, HSV and syphilis.
. cause a syndrome characterized by microcephaly, hepatosplenomegaly, deafness,
chorioretinitis, and thrombocytopenia.
. Standard prenatal care for high risk women (immegrants) in USA includes:
screening for infection with syphilis, chlamydia, gonorrhea, and HIV, as well as rubella and
hepatitis B immunity.
. N.B.:
1- Folic acid supplementation prior to conception is important to prevent neural tube defects
such as spina bifida and anencephaly.
2- Zidovudine treatment early in pregnancy in women with confirmed HIV infection can reduce
the risk of vertical transmission of HIV to the fetus.
Also Caesarian delivery also reduces vertical transmission.
3- Malaria during pregnancy, causeS maternal anemia, also increases the risks of fetal loss and
IUGR. Microcephaly is not associated with malaria during pregnancy.
4- Smoking during pregnancy carries a risk of intrauterine growth retardation, as well as
neurologic and developmental defects. Microcephaly is not a feature.
. Congenital syphilis:
______________________
. Presents early on with hepatosplenomegaly, cutaneous lesions, jaundice, anemia, and
rhinorrhea.
. Metaphyseal dystrophy and periostitis may be seen on radiography.
. Late manifestations: frontal bossing, high arched palate, Hutchinson teeth, interstitial
keratitis, saddle nose, and perioral fissures.
. These late manifestations can be prevented with early treatment of the infant
. Serologic testing: initial screening (VDRL), rapid plasma reagin (RPR), (EIA); confirmatory
testing is with treponemal tests as the (FTA-ABS) or (TPPA).
. Parenteral penicillin G is the treatment of choice for syphilis.
. Congenital toxoplasmosis:
___________________________
. Remember the classic triad of congenital toxoplasmosis: chorioretinitis, hydrocephalus, and
intracranial calcifications.
. Microphthalmia, microcephaly, hepatomegaly, diffuse lymphadenopathy, jaundice and diffuse
petechiae may be seen.
. Mumps:
________
. Parotitis secondary to mumps.
. Orchitis is one of the most frequent complications of mumps, developing in about 20% of
cases.
. It is most common in postpubertal young men, ages 15-29. Because less than 15% of orchitis
cases of are bilateral, infertility is a rare complication.
. Treatment for mumps is supportive, with application of cold compresses to the parotid area
or testes.
. Other common complications of mumps are aseptic meningitis and encephalitis.
. Eczema herpeticum:
____________________
. is a form of primary herpes simplex virus infection that is usually superimposed on healing
atopic dermatitis lesions after exposure to herpes simplex virus.
. Numerous umbilicated vesicles over the area of healing atopic dermatitis are typical.

. It is frequently accompanied with fever and adenopathy.


. In infants, the infection may be life-threatening, and acyclovir treatment should be initiated as
soon as possible.
. Erythema toxicum:
___________________
. The healthy appearance of the neonate, the evanescent nature of the rash, and the distinctive
red halo surrounding the lesions support the diagnosis of erythema toxicum.
. The presence of numerous eosinophils in the pustules is diagnostic.
. Erythema toxicum is a benign, self-limited condition usually found in newborns after the first
2 days of birth.
. Treatment is not necessary.
.N.B.:
- Milia are small pearly white cysts, and are distinct from the rash of erythema toxicum.
- Sebaceous hyperplasia presents as little yellowish papules and are commonly found on the
face.
- Vaccination against hepatitis B decreases the incidence of hepatocellular carcinoma,
especially,
in regions with high levels of hepatitis B infection such as Asia (china) and Africa.
- The vaccination schedule for preterm infants should be conformed to the child's chronologic
age, not the gestational age.
- Because prematurity does not markedly change the immune response to vaccines, and the
risk of infection and complications in preterm infants is greater than term.
- The exception is that children should be 2 kg prior to receiving the first hepatitis B vaccine
(which is at birth).
- Small for gestational age infants have a weight under the 10th percentile for gestational age
at birth
- May have complications such as hypoxia, polycythemia, hypoglycemia, hypothermia, and
hypocalcemia.
.N.B.:
- Albendazole or mebendazole is the first-line treatment for Enterobius vermicuLaris infection,
Pyrantel pamoate is an alternative.
- Urinalysis is a preliminary investigation that should be performed first in all patients with
suspected renal disease.
- All children with recurrent episodes of nocturnal vulvar itching should be examined for
pinworms and treated empirically with mebendazole.
- Female infants less than 3 months of age sometimes develop vaginal spotting, discharge or
bleeding.
Due to maternal estrogens crossing the placenta to the fetal blood before birth, causing a
pubertal effect in the newborn,
which disappears as soon as the hormone is cleared from the infant's circulation, so
treatment is reassurance of the mother.
. Lyme disease:
_______________
. The causative agent is the spirochete Borrelia burgdorferi, which is transmitted to humans by
the bite of an infected tick of the Ixodes species.
. The first clinical manifestation is the typical annular rash, named erythema migrans, at the
site of the bite.

. The rash may be uniformly erythematous, or it may appear as a target lesion with central
clearing.
. Erythema migrans may be associated with systemic features: fever, myalgia, headache or
malaise.
. Individuals who are exposed to tick-infested areas should wear light-colored clothing so that
ticks can be spotted more easily and removed before getting attached.
. Wearing long-sleeved shirts and tucking pants into socks or boot tops can help keep ticks
from reaching the skin.
. Applying insect repellents containing DEET to clothes and exposed skin, and applying
permethrin to clothes, which kills ticks on contact, also reduce the risk of tick attachment
. Tinea corporis:
_________________
. Is a superficial fungal infection mostly seen in hot, humid climates.
. The lesions are pruritic, erythematous, scaly, and have a red ring with central clearing.
. Exposure to infected animals, people, and public places are major risk factors.
. Tinea corporis is most common in preadolescents.
. It is diagnosed clinically or with a skin scraping and potassium hydroxide examination.
. Topical antifungals ( Terbinafine) are the preferred therapy.
. Reye syndrome:
________________
. also known as fatty liver with encephalopathy.
. A rare illness seen exclusively in children less than 15 years old who were treated with
salicylates for a viral infection.
. Pathophysiology Reye syndrome is characterized by diffuse mitochondrial injury that leads
to;
. Present with: vomiting, agitation, and irrational behavior, progressing to lethargy, stupor, and
restlessness, convulsions may occur.
. The liver is enlarged but there is no icterus.
. Laboratory findings: hyperammonemia, normal or slightly elevated bilirubin and alkaline
phosphatase, prolonged prothrombin time, hypoglycemia,
and moderate to severe elevations in AST, ALT, and lactate dehydrogenase levels.
. Histologically, extensive fatty vacuolization of the liver without inflammation is presen
. Biopsy of the liver, kidneys and brain reveals microvesicular steatosis.
. Treatment is supportive
. Aspirin is therefore generally contraindicated in children, except in the treatment of Kawasaki
disease.
.N.B:
- Newborns of mothers with active hepatitis B infection should be passively immunized at
birth with hepatitis B immune globulin (HBIG) followed by
active immunization with recombinant HBV vaccine.
. Infant botulism:
__________________
. C. botulinum is the most common etiologic agent and is usually transmitted through food,
especially honey, which is the carrier in 1/3 of cases.
. In infantile form of botulism, the causative organism gains entry through the food and
produces toxin in the GIT, which subsequently triggers the symptoms.
. It is a protease that blocks acetylcholine release.
. Infants typically present between 2 weeks and 9 months with constipation and poor feeding.
. This is followed by progressive hypotonia, weakness, loss of deep tendon reflexes, cranial
nerve abnormalities (impaired gag reflex), and respiratory difficulties.

. Signs of autonomic dysfunction such as hypotension and neurogenic bladder can occur early
in the course of the disease.
. Staphylococcal scalded skin syndrome (SSSS):
______________________________________________
. is caused by exfoliative toxin-producing strains of S. aureus.
. It starts with a prodrome of fever, irritability, and skin tenderness, which is followed by
generalized erythema and superficial flaccid blisters.
. A positive Nikolsky sign (gentle lateral pressure on the skin surface adjacent to a blister
causes slipping and detachment of a superficial layer of skin).
. Scaling and desquamation follow, before resolution of the disease process SSSS usually
affects children below age 10.
. but adults with kidney disease or immune compromise may also be affected.
. Cultures from intact bullae are usually sterile, because this is a toxin-mediated process.
. The goal of treatment is to eliminate any focus of infection with appropriate antistaphylococcal antibiotics & provide supportive wound care of all denuded areas.
================================================================ Growth and
development===========================================================
. Rehydration after diarrhia:
_____________________________
. Adequate hydration is very important in the management of diarrhea, replacement of
electrolytes is also essential to prevent complications
such as water intoxication and hyponatremia.
. Water intoxication, hyponatremia, and seizures may result if diarrhea is treated with large
amounts of hypotonic or low-solute fluids (e g , water).
. Mild to moderated dehydration secondary to acute diarrhea --> Oral rehydration therapy with
Oral Rehydration Solution (ORS).
. Severe dehydration requires hospitalization and IV therapy.
. A healthy full term baby weight:
__________________________________
. Healthy, full term, and appropriate for gestational age infants may lose up to 10% of their
birth weight in the first week of life.
. The fetus exists in a state of relative total body water and extracellular fluid excess.
. After birth, excretion of this excess water accounts for the majority of the weight loss.
. The birth weight should be regained by 10 days of life.
. Signs of Neonate/Infant Dehydration:
--------------------------------------- Dry oral mucosa, lips, and tongue.
- Decreased number of wet diapers.
- No tears present when crying.
- Delayed capillary refill time (>2 seconds).
- Decreased skin turgor.
Managment of dehydration:
------------------------. The initial step in managing children with dehydration is to determine its severity.
. The ideal method of assessing dehydration is to determine the measured change in weight
because 1 kg of acute weight loss equals 1 L of fluid loss.

. A child's weight, however, changes constantly, making it difficult to obtain an accurate recent
"well" weight.
. So, the degree of dehydration often has to be determined by the clinical history and physical
examination & can be divided into:
1. Mild dehydration: presents with a history of decreased intake or increased fluid loss with
minimal or no clinical symptoms.
2. Moderate dehydration; decreased skin turgor, dry mucus membranes, tachycardia,
irritability, a delayed capillary refill (2-3 seconds),& decreased urine output.
3. Severe dehydration: cool, clammy skin, a delayed capillary refill (>3 seconds), cracked lips,
dry mucous membranes, sunken eyes, sunken fontanelle (if still present),
tachycardia, lethargy, and minimal or no urine output, Patients can present with hypotension
and signs of shock when severely dehydrated.
- Oral rehydration therapy should be the initial treatment in children with mild to moderate
dehydration.
- Children with moderate to severe dehydration should be immediately resuscitated with
intravenous fluids to restore perfusion and prevent end organ damage.
- Isotonic crystalloid is the only crystalloid solution recommended for intravenous fluid
resuscitation in children.
. Rehydration:
______________
. The signs and symptoms of hypernatremia are mainly neurologic and include lethargy,
altered mental status, irritability, and seizures.
. Hypernatremia can also cause muscle cramps, muscle weakness, and decreased deep tendon
reflexes.
. Hypovolemic hypernatremia develops secondary to renal losses (eg, diuretic use, glycosuria)
or extrarenal losses (eg, gastrointestinal upset, excessive sweating).
. Hypervolemic hypernatremia occurs due to exogenous sodium intake or mineralocorticoid
excess (eg, hyperaldosteronism).
. When treating a patient with hypernatremia, the sodium must be slowly returned to normal.
. In caseof hypernatremia and dehydration, the initial goal is to stabilize the infant with fluid
resuscitation as needed.
. When giving intravenous fluid boluses, only isotonic solutions such as normal saline or
lactated Ringer's should be used.
.N.B:
- Half normal saline (45%) and 5% dextrose are hypotonic solutions, they should never be used
for initial resuscitation.
. because they quickly exit the intravascular system and lower the sodium too rapidly
Precipitating drop in sodium levels can cause cerebral edema.
- Multiple studies have demonstrated that the expensive colloid solutions are no better than
crystalloids at fluid resuscitation.
. Developemental processes:
___________________________
- Age: 12 months:
---------------. Fine Motor: Tow finger pincer grasp & turns several pages of a book at a time.
. Gross Motor: Walks without assistance & - Waves bye & Climbs up on furniture.

. Language: Says 2-3 words & - Says "mama" and "dada" games (can identify each parent).
. Social: Imitates actions & - Plays reciprocal games (peek-a-boo) & indicates wants.
- Age: 2 year:
-------------. Fine Motor: Builds a twer of 6 cubes.
. Gross Motor: Walks up & down stairs
. Language: 200 word vocabulary & uses 2-word phrases
. Social: Follow 2 step commands.
- Age: 3 years:
---------------. Fine Motor: Copies a circle uses utenisles to feed self & stacks 9 blocks.
. Gross Motor: climbs stairs with alternating feet, rides a tricycle & kicks a ball.
. Language: uses 3 word sentences, stats first name & 3/4 (75%) of speech is intelligible.
. Social: wash/dry hands, helps with simple household tasks & group play.
- Age: 4 years:
---------------. Fine Motor: Copies a cross, draw a person, begins to use scissors & holds a crayon with a
tripod grasp.
. Gross Motor: Hops on one foot with out losing balance & jumps over objects.
. Language: Counts to 10, tells stories & use lurals and prepositions.
. Social: Cooperative play, has imaginary friends & imitate adults role.
--------------------------------. Language development:
----------------------- Social smile 2 months
- Babbles 6 months
- 2 words, obeys 1-step command 1 year
- 2-3 word phrases, obeys 2-step command 2 years
. Gross motor development:
-------------------------- Holds head 3 months
- Rolls back to front and front to back 4 months
- Sits well unsupported 6 months
- Walks alone 12 months
- Walks up and down stairs without help 24 months
. Fine motor development:

------------------------- Raking grasp 6 months


- Throws object 12 months
- Builds tower of 2 blocks 15 months
- Builds tower of 6 blocks/turn pages of books 24 months
. Social development:
--------------------- Recognizes parents 2 months
- Recognizes strangers (stranger anxiety) 6 months
- Imitates action/comes when called 12 months
- Plays with other children 18 months
- Parallel play 24 months
N.B:
- Developmentally, a 12 month old should be able to walk holding onto a hand or object. He
should have a neat pincer grasp that allows him to grab small objects.
- Language development at 1 year old consists of mama and dada said specifically to his
parents, and at least one other word.
- Socially, a 12 month old can roll a ball and makes postural adjustments when being dressed
. Toilet training:
__________________
. usually occurs between the ages of 2-4 years old.
. Parents should be counseled that several milestones must be met by the child to become
potty trained.
. Children must be aware of bladder filling, be able to consciously tighten the external
sphincter, have normal bladder growth, and be motivated to stay dry.
. They must also have met motor milestones to be able to walk to the toilet, sit upright on the
toilet, and the language to indicate the need to use the bathroom.
. The child will demonstrate readiness to begin toilet training when they communicate the need
to urinate or defecate prior to passing urine or stool,
and when they can withhold the urine or stool for a brief period of time.
. When a child is ready, toilet training consists of placing the child on the toilet at regular times
and using positive reinfo rcement as a reward.
. Girls are usually toilet trained faster than boys, but 90-95% of children will achieve daytime
continence by 5 years old.
. When a child is not ready for toilet training, forcing him to sit on the toilet will make him
more reluctant and can make potty training very difficult.
. The best step in the management of a reluctant child is to stop toilet training for several
months and allow the child to become interested on his own.
. Vitamin D deficiency rickets:
_______________________________
. This condition usually presents after several months of being Vitamin D-deficient.
. The typical pathology is - defective mineralization of growing bone or osteoid tissue
. The typical patients are low-birth weight infants, unsupplemented dark-skinned infants,
infants with inadequate sun exposure, and solely breastfed infants.
. Early manifestations: craniotabes (ping-pong ball sensation over the occiput or posterior
parietal bones), rachitic rosary, and thickening of the wrists and ankles.
. Other findings: Harrison groove and a large anterior fontanelle.
. Diagnosis is confirmed by obtaining the serum calcifediol level (decreased), alkaline
phosphatase level (increased), and

observing the characteristic radiologic changes(cupping and fraying of the distal ends of long
bones, and double contour along the lateral outline of the radius).
. Treatment includes oral Vitamin D administration and adequate sunlight (or artificial light)
exposure.
. The current recommendation to prevent rickets is oral Vitamin D supplementation, beginning
during the first 2 months of life and continuing until early adolescence.
. Sexual developement:
______________________
. Normal sexual behaviors for young children (ages 2-5) include:
. Talking about genitals or reproduction, occasional masturbation, enjoying being naked, and
curiosity about their own and other people's body parts.
. They may play "doctor" with each other or compare their bodies with other children's bodies.
. In addition, transient cross-dressing is often common in preschool boys.
. Extensive sexual knowledge as well as simulating foreplay or intercourse are concerning
behaviors in a young child and could be indicators of sexual abuse.
. Conduct disorder:
___________________
. Characterized by disruptive behavioral patterns that violate basic social norms for at least
one year in patients less than 18 years old.
. It includes aggressive behavior towards others or animals, behaviors resulting in property
damage, theft, or violations of societal rules.
. Constitutional growth delay:
_______________________________
. Constitutional growth delay is the most common cause of short stature and pubertal delay in
adolescents.
. Affected individuals have a normal birth weight and height, but between 6 months to 3 years
of age, the height growth velocity slows, and they drops percentiles on their growth curve.
. Around 3 years of age, the child regains a normal growth velocity and follows the growth
curve at the 5th to 10th percentile.
. Puberty and the adolescent growth spurt are delayed, but eventually occur.
. The child will have a normal growth spurt and reach a normal adult height.
. Bone age radiographs show a bone age that is delayed compared to the chronological age.
. Separation anxiety:
_____________________
. Separation anxiety is a normal behavior in young children between 9-18 months.
. usually manifests when parents leave the child or at night when the child is put to bed.
. The child responds by crying, clinging to the parent, and becoming upset Separation anxiety
usually lasts 2-4 months and gradually resolves as the child begins to
understand that his parents will return.
.The initial step in the management of children with speech delay is an audiology evaluation
especially when there is history of recurrent otitis media.

=====================================================================
Miscellaneous ==============================================
. Autism:
_________

. patient is usually less than 3 years old and presents with impairment in social interactions
and communication, delayed language development,
repetitive activities, and stereotypical behaviors.
. Treatment include special education and behavioral modification techniques, No
pharmacological agent has been found to be useful in autism.
. Asperger syndrome:
____________________
. These kids are usually more socially aware and communicative. Although patients may have
features of stereotypical movements and
self-injurious behavior, their language development is normal.
. Attention-deficit hyperactivity disorder (ADHD):
__________________________________________________
. Children with ADHD have poor impulse control, inattentiveness, and motor overactivity.
. Multiple factors contribute to development of ADHD: genetics, pregnancy or birth
complications, maternal drug use, abnormal brain development or traumatic brain injury,
and psychological stressors.
. There are three subtypes of ADHD - predominantly inattentive, predominantly hyperactiveimpulsive, or combined.
. To meet the diagnostic criteria for ADHD, a child must exhibit six or more symptoms of
inattention or hyperactivity- impulsivity before age 7 years, with the symptoms present for at
least 6 months.
. The symptoms must cause significant impairment in functioning and must occur in at least
two different settings
. Many children with ADHD have a family history of ADHD. However, this is not necessary for
the diagnosis.
. Educational testing may be appropriate, as many children with ADHD have coexistent learning
disabilities however.
. Teacher evaluations must be obtained first to establish the diagnosis of ADHD.
. Children with ADHD can have sleep problems, which should be addressed if the diagnosis of
ADHD is confirmed by teacher evaluations.
Inattention Symptoms
lmpulsivity Symptoms
No attention to detail;makes careless mistakes.
Interrupts others when they are speaking.

Hyperactivity

Cannot pay attention while completing tasks or while


Has a
problem waiting their turn.
playing (e.g while playing a board game).
Answers
question before the person finishes the entire question.
Does not appear to listen when spoken to.
Talks
constantly.
Does not follow instructions with regard to school work chores etc.
Physically
active all the time.
Has a hard time organizing tasks ,including fun tasks.
Cannot
perform activities quietly.
Tries to avoid situations where concentration is required (e g , school,homework).
Runs
or climbs inappropriately.
Easily distracted.
Cannot stay
in seat.

Loses objects needed to perform tasks (e.g., books, writing utensils, etc ).
Fidgets.
Forgetful.

. Undetected hearing impairment: can easily be confused with


________________________________
1- Attention deficit hyperactivity disorder (ADHD): However,
. poor language development and social isolation are not features of ADHD, and a hearing
impairment should be ruled out before a diagnosis of ADHD is considered.
2- Autism: However,
. Repetitive behaviors, poor eye contact, and impaired social interactions are not seen in
hearing impairment.
. In addition, features of autism generally appear before age three and social isolation is more
prominent than in hearing impairment.
. Management of a child who has accidentally taken liquid alkali:
---------------------------------------------------------------. The initial management is directed towards maintaining airway patency, The absence of any
added breath sounds and
presence of air movement bilaterally in this patient ensures patent airways.
. Next step, upper GI endoscopy is recommended in the first 24hrs to assess the extent of the
injury and to dictate further management.
. Barium swallow is not recommended due to the associated difficulty in determining the
extent of the injury with this procedure.
. Steroids are not recommended because these may increase the chance of perforation
Furthermore, giving steroids have not been proven to provide ,any benefit
in preventing strictures in such cases.
. Nasogastric lavage and oral antidotes such as vinegar are not recommended since these may
increase the extent of injury.
. Remember attempting to neutralize the alkali with vinegar or lavage is dangerous!
. Photo-protection in high risk group: (high yield)
______________________________________
. Sun exposure is associated with skin photo-aging and several types of skin cancer, including
melanoma.
. Encouraging photo-protection is the leading preventive measure in skin care, and sun
avoidance is the best method of photo-protection.
. Patients are instructed to avoid outdoor activities in the middle of the day (between 10 am
and 4 pm), which is the period of peak sun radiation intensity.
. Sunscreens are useful adjuncts to photo-protection, but offer insufficient protection from
ultraviolet radiation (UVR) when used alone.
. Overdependence on sunscreens may sometimes even increase or encourage outdoor exposure
Sunscreens should be applied 15-60 min
prior to sun exposure to allow enough time for protective film development.
. Currently, the available data shows little to no protection against melanoma with the use of
sunscreen lotions with SPF 15 - 30;
however, these can protect from non-melanoma skin cancers such as squamous cell
carcinoma.
. Resting under umbrellas decreases UVR exposure by 70%, but does not affect reflected
radiation.
. Shade from trees provides insufficient UVR protection .

. Clothing is considered to be an effective tool for sun protection; however, it does not
supercede sun avoidance.
. Iron poisoning:
________________
. The patient's symptoms and history of spilled multivitamin pills are highly suggestive of iron
poisoning.
. This is one of the most common causes of death by poisoning Elemental iron, when ingested
in large amounts, has a corrosive effect on the GI mucosa,
leading to abdominal pain, nausea, vomiting, diarrhea and hematemesis.
. Iron also accumulates in the mitochondria and tissue, thus impairing energy production and
resulting in cellular damage and systemic toxicity.
. Hypotension then occurs due to the increased capillary permeability and venodilation.
. The resulting peripheral hypoperfusion and mitochondrial damage lead to accumulation of
lactic and citric acid, and consequent metabolic acidosis.
. Other complications include liver necrosis, coagulopathy, drowsiness, seizures and coma.
Long-term sequelae include gastric scarring and pyloric stenosis.
. Since iron is radio-opaque, the tablets can be seen in the stomach on abdominal x-ray.
. The diagnosis is confirmed by measuring serum iron levels.
. The treatment depends on the severity of the poisoning: Ipecac syrup to induce emesis is
used in asymptomatic patients with an intact gag reflex.
. Intravenous deferoxamine, an iron chelator, is used in moderate to severe cases of
intoxication.
. Activated charcoal is not effective, and gastric lavage is not recommended in young children.
. Acute iron intoxicTION:
_________________________
. Is usually the result of a suicide attempt especially if symptoms of depression (isolative
behavior, decreased appetite, poor sleep) are present.
. Acute iron intoxication has five phases:
1- GIT phase --> 30 minutes to six hours after ingestion & is due to direct mucosal damage,
patient experience nausea, vomiting, hematemesis, melena, and abd. pain.
2- Latent phase --> 6 to 24 hours after ingestion, and is largely asymptomatic.
3- Shock and metabolic --> acidosis may occur 6 to 72 hours following the ingestion.
4- Hepatotoxicity --> 12 to 96 hours.
5- Bowel obstruction --> secondary to mucosal scarring can develop several weeks postingestion
. The diagnosis of acute iron intoxication is made by checking the serum iron concentration.
. Levels > 350 mcg/dL are consistent with toxicity.
. Treatment of iron poisoning involves deferoxamine, which binds ferric iron, allowing urinary
excretion.
.N.B: Common poisonings:
-----------------------1- There are no known cases of riboflavin (vitamin 82) poisoning.
2- Acute ingestion of vitamin A causes anorexia, bulging fontanelre (pseudotumor cerebri),
hyperirritability and vomiting.
3- Acute vitamin D ingestion causes anorexia, nausea, vomiting, diarrhea, headache, polyuria
and polydipsia.
4- Vitamin K may cause hyperbilirubinemia in premature infants.
5- Acute lead poisoning presents as vomiting, ataxia, colicky abdominal pain, irritability,
seizures, encephalopathy and cerebral edema.
6- Dimercaprol is the antidote for lead poisoning, as well as poisoning from other heavy metals
(e g , gold, arsenic, mercury).

7- Aspirin poisoning is characterized by lethargy, fever, hyperpnea, vomiting, tinnitus and


metabolic acidosis, abdominal radiographs are usually unremarkable.
. Lead poisoning:
_________________
. Risk for lead poisoning: live in an old house, especially if there is peeling paint or during
renovation projects.
. Other risk factors: lead pipes, living near a battery recycling plant, having a parent who works
with batteries or pottery, or having a playmate or sibling
with a history of lead poisoning.
. Capillary blood specimens (using a fingerstick) are widely used in childhood screening for
lead poisoning.
. Since false positive results are common, the first step after an abnormal fingerstick lead level
is to confirm the diagnosis with a serum (venous blood) lead level.
. If the serum lead level is >10 ?g/dL, then intervention is needed.
. Removing the child from the house would be the most important step in reducing exposure if
the diagnosis of lead poisoning is confirmed.
. Chelation therapy with either dimercaprol or dimercaptosuccinic acid (DMSA, succimer) is
typically used when lead levels are extremely elevated (>45 ?g/dL).
. Rechecking the lead level in one month is appropriate for children with mildly elevated lead
levels (<20 ?g/dL) after
a thorough history is taken and counseling is done about removing the child from any
potential lead exposure.
. Vaccination adverse reactions: Extremely high yield question for the USMLE!!!
_______________________________
. The adverse reactions to the DTaP vaccine are usually attributed to the pertussis component
of the vaccine.
. Mild reactions consist of local redness and swelling, irritability , and fever < 105F (40 6C).
. Less common reactions include inconsolable crying for > 3 hours and a temperature of > 105F
(40 6C).
. An immediate anaphylactic reaction, an encephalopathy, or any CNS complication within 7
days of administration of the vaccine is a contraindication for further
administration of DTaP.
. In these instances, DT should be substituted for DTaP since the adverse reactions are usually
attributed to the pertussis component of the vaccine.
. varicella vaccine:
____________________
. Guidelines recommend active immunization with varicella vaccine for healthy adults &
children exposed to varicella, ideally within the first 3- 5 days of exposure.
. because efficacy is decreased beyond this period.
. Because this is a live virus, the vaccine is only recommended for immunocompetent
individuals.
. Postexposure prophylaxis with varicella-zoster immune globulin product is indicated in
susceptible high-risk persons (lmmunocompromised patients, pregnant women)
exposed to varicella within 96 hours (preferably 72 hours) of exposure, it does not prevent
infection but effectively reduces disease severity.
. administration of any of these vaccines after these periods it will ineffective.
. Cat bites:
_____________

. Are of significant concern because these often result in deep puncture wounds, also infection
of such wounds with Pasteurella muftocida tends to develop quickly.
. And is associated with considerable pain, erythema, and swelling Localized cellulitis can
develop subacutely.
. In some cases, systemic effects (e g , fever and lymphadenopathy) may arise.
. Prophylactic treatment is thus recommended for such cases. For treatment of minor cat bite
wounds, amoxicillin/clavulanate for five days is recommended.
. Anabolic steroids:
____________________
. Are used to improve physique and athletic performance.
. Adverse effects, including acne, baldness, gynecomastia, hepatic dysfunction, altered lipid
profiles, virilization, testicular failure, and mood and behavior changes.
. Informed consent:
___________________
. When planning to perform a procedure on a patient, informed consent must be obtained.
. It should include an explanation of the following: the suspected diagnosis, the type of
procedure or treatment planned,
the risks and possible complications of the procedure, and the alternative treatments
available.
. When a procedure needs to be performed on a minor, the minor's parent must provide
informed consent.
. If the parent is a minor as well, which causes a dilemma because minors (<18 years old) are
generally considered to be incapable of providing consent However,
most states have exceptions to this rule that allow the following groups of minors to provide
consent:
military personnel, those who are married, pregnant minors, or those who are a parent,
among others.
. In these instances, minors are considered free from their parents.
. Fetal alcohol syndrome:
_________________________
. which is caused by moderate or excessive maternal alcohol intake during pregnancy.
. It affects fetal growth and morphogenesis, resulting in midfacial abnormalities (short
palpebral fissures, epicanthal folds, long philtrum, thin upper lip border),
. cardiac defects (atrial or ventricular septal defect, persistence of arterial canal), multiple joint
anomalies, prematurity, growth retardation & mental retardation.
. It is sometimes clinically difficult to differentiate it from Down's syndrome (trisomy 21 ) since
similarities include facial dysmorphism and heart malformations;
however, the normal karyotype in this case rules out the latter.
. Fetal alcohol syndrome is the most common cause of mental retardation in children.
. Withdrawal symptoms are usually not seen unless the mother was drinking just prior to
delivery.
. Neonatal abstinence syndrome (NAS):
____________________________________
. The opiates that infants are most commonly exposed to are heroin and methadone.
. Methadone is prescribed for pregnant women who are addicted to heroin to prevent
uncontrolled withdrawal in the fetus.

. Heroin and methadone are not associated with dysmorphic features or congenital anomalies
in the fetus, but
prenatal exposure can lead to increased risk of intrauterine growth retardation,
macrocephaly, sudden infant death syndrome, and neonatal abstinence syndrome (NAS).
. NAS: presents in the 1st few days of life & characterized by irritability, a high-pitched cry,
poor sleeping, tremors, seizures,
sweating, sneezing, tachypnea, poor feeding, vomiting,and diarrhea.
. Withdrawal usually presents within 48 hours after birth for heroin withdrawal and between
48 and 72 hours for methadone.
. The treatment for NAS: symptomatic care to calm the infant & help the infant sleep, such as
swaddling, providing small frequent feeds, &
keeping the infant in a low stimulation environment.
. Pharmacologic treatment used when supportive treatment does not control it; Morphine can
be administered & systematically weaned to help control opiate withdrawal.
.N.B.:
- Prenatal exposure to cocaine can result in jitteriness, excessive sucking, and a hyperactive
Moro reflex.
- Withdrawal symptoms are usually not as severe as with opiates; Long-term effects on
behavior, attention level, and intelligence may be seen.
. Serum sickness-like reaction:
_______________________________
. It most commonly occurs in young children following the treatment of viral infections with
antibiotic.
. May occur 1-2 weeks after administration of certain drugs, such as penicillin, amoxicillin or
cefaclor in the setting of a viral illness.
. Prominent symptoms: fever, urticaria! rash, polyarthralgia and lymphadenopathy.
. It derives its name from its similarities to true serum sickness, an immune-complex mediated
hypersensitivity reaction to non-human proteins.
. Serum-sickness like reaction is a clinical diagnosis, and should resolve with withdrawal of the
offending agent; it does not represent a true drug allergy.
. Deficiency of niacin (vitamin B3):
____________________________________
. leads to pellagra, which is characterized by diarrhea, dermatitis, dementia, and if severe,
death.
. Pellagra is common in 3rd world countries where the main diet consists of cereal or corn, but
can be seen in people with bowel disease that interferes with vitamin absorption.
. Present with GIT complaints (nausea, abdominal pain, or epigastric discomfort) along with
glossitis and watery diarrhea.
. Dermatitis seen in pellagra occurs in sun exposed areas and resembles a sunburn; it is also
typically bilateral and symmetric.
. As the rash progresses, the skin becomes hyperpigmented and thickened.
. Mental status changes can range from poor concentration to irritability, aggressiveness, and
dementia.
. Death can occur in severe niacin deficiency if untreated.
. Treatment of pellagra is niacin replacement.
.N.B:
----1- Deficiency of thiamine (vitamin B1 ) causes beriberi or Wernicke-Korsakoff syndrome
- These conditions are characterized by neurologic and psychiatric symptoms, and are often
seen in alcoholics or patients who have had weight loss surgery.

2- Deficiency of riboflavin (vitamin B2) can lead to cheilosis, glossitis, seborrheic dermatitis
(often affecting the genital areas), pharyngitis, and
edema and/or erythema of the mouth (Pure riboflavin deficiency is has been documented in
regions of the world with severe food shortages).
3- Pyridoxine (vitamin B6) deficiency causes irritability, depression, dermatitis, and stomatitis.
- It can also cause an elevated serum homocysteine concentration, which is a known risk factor
for venous thromboembolic disease and atherosclerosis.
4- Deficiency of cyanocobalamin (vitamin 812) causes macrocytic anemia and peripheral
neuropathy.
. Vitamin A deficiency:
________________________________
. Suspect vitamin A deficiency in a 2 or 3-year-old child with impaired adaptation to darkness,
photophobia, dry scaly skin, xerosis conjunctiva,
xerosis cornea, keratomalacia, Bitot spots and follicular hyperkeratosis of the shoulders,
buttocks, and extensor surfaces
. Sudden infant death syndrome (SIDS):
______________________________________
. is the leading cause of mortality in infants aged between 1 month and 1 year in the United
States.
. It is defined as the sudden, unexpected death of an infant that cannot be explained by the
history and a thorough post-mortem examination.
. It is recommended that infants be placed in a supine position while sleeping to reduce the
risk of SIDS.
. Anaphylaxis:
______________
. is an acute, life-threatening, lgE-mediated type I hypersensitivity reaction.
. The most common inciting agents for anaphylaxis are insect envenomation, drugs and foods.
. Food allergies are the major cause of pediatric outpatient anaphylaxis, and peanut allergies
account for the majority of fatal and near fatal reactions.
. Signs include bronchoconstriction, hypotension and urticaria.
. Intramuscular injection of epinephrine is the first-line treatment for anaphylaxis in a patient
with a patent airway.
. The most effective and potentially life-saving intervention in a patient with known
anaphylaxis is
prescription of self-administrable epinephrine for use upon reexposure to the inciting
allergen.
. Risperidone:
______________
. Is an atypical antipsychotic commonly used to treat schizophrenia and bipolar disorder.
. It is a dopamine & serotonin antagonist.
. It inhibits dopamine, which leads to elevated serum prolactin levels.
. The hyperprolactinemia causes: oligomenorrhea, amenorrhea, and galactorrhea, in
premenopausal females.
. The resultant side effects can include the breast tenderness, amenorrhea, and galactorrhea.
. Risperidone has been found to increase prolactin levels to a greater extent than do many of
the other antipsychotics.
. Cradle cap, or seborrheic dermatitis:

________________________________________
. is a common pediatric skin condition.
. This papular, scaly rash tends to affect the eyebrows, nasolabial folds, and scalp.
. Treatment: moisturizers, antifungals, and topical steroids.
. Atopic dermatitis:
____________________
. In infancy presents with pruritus and skin lesions typically distributed symmetrically over the
face, scalp, chest and extensor surfaces of the extremities.
. The diaper region is typically spared.
. Patients with exfoliative dermatitis (erythroderma):
_______________________________________________________
. usually have a prior dermatological condition such as psoriasis, atopic dermatitis or mycosis
fungoides (Sezary syndrome).
. Intraosseous access:
______________________
. Whenever intravenous access cannot be obtained in emergent pediatric cases, intraosseous
access should be attempted next.
. Intraosseous cannulation can be performed quickly in the ED, this technique does not require
the precision required for cannulation of small vessels& provides
a cannula large enough to deliver adequate fl uids.
. This route can be used in emergencies for 24 to 48 hours, at which point another intravenous
route should have been obtained.
. Associated complications such as osteomyel itis are rare.

============================================================================
=============================================

Dr. Hisham Elkilany.


_________________________

ALLERGY & IMMUNOLOGY TiKi TaKa

________________________________
ALLERGY & IMMUNOLOGY TiKi TaKa
________________________________
. HYPER-SENSITIVITY REACTIONS:
______________________________
______________________________
{1} Type "1" (IMMEDIATE):
__________________________
. Ex. Acute atopic dermatitis.
. Highly pruritic papules, vesicles & plaques.
. Light microscopy -> Spongiosis (edema of the epidermis).
{2} Type "2" (ANTIBODY MEDIATED):
__________________________________
. IgM or IgG + ANTIGEN.
. Ex. Immune hemolytic anemia & Rh hemolytic disease of the newborn.
{3} Type "3" (IMMUNE COMPLEX MEDIATED):
________________________________________
. Ag + Ab + COMPLEMENT.
. Ex. Serum sickness.
{4} Type "4" (CELL MEDIATED):
______________________________
. Dermal inflammation after direct contact with allergen.
. Ex. Tuberculin skin test & Allergic contact dermatitis.

. TRANSFUSION REACTIONS:
________________________
________________________
. 1 . ABO INCOMPATIBILITY:
___________________________
. Acute symptoms of hemolysis WHILE the transfusion is occuring.
. Ex -> DURING a transfusion, the pt becomes hypotensive & tachycardic.
. Back & chest pain & dark urine.
. ++ LDH & bilirubin.
. -- Haptoglobin.
. 2 . TRANSFUSION RELATED ACUTE LUNG INJURY (TRALI) = LEUKO-AGGLUTINATION
REACTION:
____________________________________________________________________________________
. Acute Shortness of breath from antibodies in the donor blood against the repient WBCs.
. Ex -> 20 mins after a pt. receives a blood transfusion, the pt becomes short of breath.
. Transient infiltrates on CXR.
. All symptoms resolve spontaneously.
. 3 . IgA DEFECIENCY:

______________________
. presents with anaphylaxis !
. In the future, use blood donations from an IgA defecient donor or washed RBCs.
. Ex -> As soon as the pt. received transfus., he becomes SOB, hypotensive & tachycardic.
. NORMAL LDH & BILIRUBIN.
. 4 . MINOR BLOOD GROUP INCOMPATIBILITY:
_________________________________________
. To kell, Duffy, Lewis or Kidd antigens or Rh incompatibility !
. Delayed jaundice.
. No specific therapy.
. Ex -> A few days after transfusion, the pt becomes jaundiced.
. The hematocrit doesn't rise with transfusion & he is generally without symptoms.
. 5 . FEBRILE NON-HEMOLYTIC REACTION:
______________________________________
. Small rise in temperature.
. No ttt required.
. Reaction against donor WBCs antigens.
. prevented by using filtered blood transfusions in the future to remove WBCs antigens.
. Ex -> A few hours after transfusion, the pt becomes febrile with rise 1 degree in temp.
. No evidence of hemolysis.

. RHINITIS:
____________
____________
{A} ALLERGIC RHINITIS:
_______________________
. Watery rhinorrhea & sneezing with more prominent eye symptoms.
. Early age of onset.
. Identifiable trigger (animals - environmental exposure).
. Usually seasonal symptoms but can be persistent throughout year.
. Nasal mucosa can be normal, pale blue or pale on exam.
. Associated with allergic disorders e.g. eczema & asthma.
. Tx -> Allergen avoidance.
. Tx -> Topical intra-nasal glucocorticoids.
{B} NON-ALLERGIC RHINITIS = VASOMOTOR RHINITIS:
________________________________________________
. Nasal congestion - Rhinorrhea - Postnasal discharge (postnasal drip = dry cough).
. Late age of onset > 20 ys.
. Can't identify clear trigger !
. Symptoms throughout the year but sometimes worse with seasons change.
. Nasal mucosa may be normal or erythematous.
. Less commonly associated with allergic disorders e.g. asthma or eczema.
. Routine allergy testing isn't necessary prior to initiating empiric ttt.
. May respond to 1st generation oral H1 antihistaminics (Chloramphenicol),
. Never ever responds to antihistaminics without anticholinergic properties (Loratidine)!
. Tx -> TOPICAL INTRANASAL GLUCOCORTICOIDS.
. The 3 most common causes of CHRONIC COUGH (> 8 weeks):
________________________________________________________

. UPPER AIRWAY COUGH $YNDROME (Post-nasal drip).


. BRONCHIAL ASTHMA.
. GERD.
. UPPER AIRWAY COUGH $YNDROME = POST-NASAL DRIP:
_________________________________________________
. NON-smoker.
. Caused by rhino-sinusitis conditions.
. Dry cough is most likely due to post-nasal drip associated with allergic rhinitis.
. Dx -> Confirmed by improvement of the nasal discharge & cough with H1 Anti-histaminics.
. Chlorpheniramine is an H1 receptor blocker that decreases the allergic response.
. Decrease in NASAL SECRETIONS is most likely to significally improve symptoms.
. ALLERGIC REACTIONS:
_____________________
_____________________
{1} . ANAPHYLAXIS = ANAPHYLACTIC SHOCK:
________________________________________
. Type 1 hypersensitivity reaction.
. Pts usually have prior exposure to the offending substance.
. Pts have preformed Ig E -> Histamine mediated peripheral vasodilatation.
. Bee stings - food & medications are the most common allergens.
. Acute onset of hypotension & tachycardia.
. Dangerous allergic reaction may progress to respiratory failure & circulatory collapse.
. Allergen exposure -> Sudden onset of symptoms in more than one system,
. Cutaneous (hives - flushing - pruritis).
. GIT ( Lip / tongue swelling - vomiting).
. Respiratory (Dyspnea - wheezing - stridor - hypoxia).
. Cardiovascular (Hypotension).
. It is a medical emergency.
. Tx -> INTRA-MUSCULAR EPINEPHRINE into the THIGH.
{2} . ANGIO-EDEMA:
___________________
. H/O of ICU pt on ACEIs e.g ENALAPRIL.
. Edema in the face, mouth, lips.
. Absence of pruritis & urticaria.
. Laryngeal edema may occur causing airway obstruction.
. occurs due to BRADYKININ release.
. it may occur at any time not just at the start of drug intake.
. Dx----> Low levels of C2 & C4.
. Tx----> STOP ACEIs + FRESH FROZEN PLASMA + Secure the airway.
. HERIDITARY angioedema:
________________________
. C1 esterase inhibitor defeciency.
. usually follows an infection, dental procedure or minor trauma.
. N.B. The most common cause of acquired isolated angioedema is ACE inhibitors use.
. N.B. C1q is NORMAL in heriditary angioedema.
. N.B. C1q is DEPRESSED in aquired angioedema.
. C4 levels are depressed in all forms !

{3} . URTICARIA:
_________________
. Sudden swellings of the superficial layers of the skin.
. Can be caused by insects or medications.
. May be caused by pressure, cold or vibration !
. Tx -> ANTI-HISTAMINICs (Diphenhydramine & koratidine).
. GRAFT VERSUS HOST DISEASE (GVHD):
___________________________________
___________________________________
. in pts with bone marrow transplantation.
. due to activation of the DONOR "T" lymphocytes.
. Skin ---> Maculopapular rash.
. Intestine ---> Bloody diarrhea.
. Liver ---> Abnormal LFTs & jaundice.
. PRIMARY IMMUNO-DEFECIENCY DISORDERS:
_______________________________________
_______________________________________
{1} COMMON VARIABLE IMMUNODEFECIENCY (CVID):
_____________________________________________
. ADULT with recurrent sino-pulmonary infections.
. NORMAL NUMBER OF "B" cells but don't make effective amounts of immunoglobulins.
. DECREASE in ALL subtypes of immunoglobulins (IgG, IgM & IgA).
. Frequent episodes of bronchitis, pneumonia, sinusitis & otitis media.
. CVID increases the risk of lymphoma.
. Dx -> Decreased immunoglobulins level & -- response to Ag stimulation of B-cells.
. Tx -> Antibiotics for infections.
. Tx -> Chronic maintainance with regular infusions of I.V. immunoglobulins.
. The clue to CVID is DECREASE in the OUTPUT of B-LYMPHOCYTES with,
. NORMAL NUMBER of B-cells & NORMAL amount of LYMPHOID TISSUE (LNs, adenoids &
tonsils).
{2} X-LINKED (BRUTON) A-GAMMA-GLOBULINEMIA:
____________________________________________
. MALE CHILDREN with recurrent sino-pulmonary infections.
. DIMINISHED B-cells & LYMPHOID TISSUES.
. ABSENCE of tonsils, adenoids & lymph nodes & spleen.
. NORMAL T-cells.
. Tx -> Antibiotics for infections.
. Tx -> Chronic maintainance with regular infusions of I.V. immunoglobulins.
{3} SEVERE COMBINED IMMUNODEFECIENCY:
______________________________________
. Combined = Defeciency in BOTH B & T cells.
. -- B-cells -> -- immunoglobulin production -> Recuurent sinopulmonary infections at 6ms
. -- T-cells -> AIDS related infections e.g. PCP, varicella & candida.
. Treat the infections as they rise.
. BM transpalntation is curative.
{4} Ig"A" DEFECIENCY:

______________________
. Recurrent sinopulmonary infections + ATOPIC DISEASE + ANAPHYLAXIS to blood
transfusions
. Anaphylaxis from blood transfusions from pts with "NORMAL" levels of IgA !
. Treat infections as they arise.
. ONLY use blood that is from Ig-A DEFECIENT donors or that has been WASHED !
. IVIG will NOT work as the amount of IgA in the product is too small to be therapeutic !
. The trace amounts of IgA in IVIG may provoke anaphylaxis !
{5} HYPER Ig"E" $YNDROME:
__________________________
. Recurrent SKIN infections with STAPHYLOCOCCI.
. Treat infections as they arise.
. Consider prophylactic antibiotics e.g Dicloxacillin & cephalexin.
{6} WISKOTT - ALDRICH $YNDROME:
________________________________
. IMMUNODEFECIENCY + THROMBOCYTOPENIA + ECZEMA.
. MARKED DECREASED T-LYMPHOCYTES.
. BM TRANSPLANTATION is the ONLY curative ttt.
{7} CHRONIC GRANULOMATOUS DISEASE (CGD):
_________________________________________
. Genetic disease results in extensive inflammatory reactions.
. Lymph nodes with purulent material leaking out !
. Aphthous ulcers & inflammation of the nares.
. Obstructive granulomas in the GIT or UT.
. Infections with odd combinations (Staphylococci, Bulkhorderia, Nocardia & Aspergillus).
. Dx -> ABNORMAL TETRAZOLIUM TEST !
. -- in respiratory burst that produces hydrogen peroxide.
. -- in NADPH oxidase that generates superoxide.

Dr. Wael Tawfic Mohamed


_________________________

CARDIOLOGY TiKi TaKa

______________________
CARDIOLOGY TiKi TaKa
______________________
. ISCHEMIC HEART DISEASE "IHD" = CORONARY ARTERY DISEASE "CAD":
_______________________________________________________________
_______________________________________________________________
. Risk factors:
________________
. DM (THE SINGLE WORST OR MOST DANGEROUS RISK FACTOR OF CAD is DIABETES
MELLITUS).
. HTN. (Most COMMON risk factor is hypertension).
. Smoking.
. Hyperlipidemia.
. Peripheral artery disease.
. Obesity.
. Family H/O (Family member must be young,female relative < 65ys & male relative < 55ys).
. IHD CHEST PAIN NOT CHANGING WITH BODY POSITION or RESPIRATION !
__________________________________________________________________
. Dull pain.
. Located sub-sternally.
. Lasts 15-30 minutes.
. Occurs on exertion.
. Radiates to the lower jaw or left arm.
. Changes with respiration = Pleuritic pain (Pneumonia - pleuritis - PE - Pneumothorax).
. Changes with position = Pericarditis.
. Tenderness on palpation = Costochondritis.
. The most common cause of chest pain that is NOT CARDIAC in origin is GERD ACID REFLUX !
__________________________________________________________________________________________
. Ex: pt comes to the ER with chest pain in the epigastrium & associated e' sore throat,
. A bad metallic taste in th mouth & cough is present.
. A proton pump inhibitor (Omeprazole) trial sh'd be done.
. D.D. of CHEST PAIN:
_____________________
_____________________
. 1 . Costochondritis:
_______________________
-> Chest wall tenderness.
-> Do physical examination.
. 2 . Aortic dissection:
_________________________
-> Radiation to the back.
-> Un-equal B.P. between both arms.
-> CXR: Widened mediastinum.

-> Confirm with Chest CT or TEE (Trans-esophageal echocardiography).


. 3 . Pericarditis:
____________________
-> Pain worse with lying flat, better when sitting up.
-> ECG -> ST elevation in all leads with PR depression.
. 4 . Duodenal ulcer disease:
______________________________
. Epigastric discomfort - pian is better on eating.
. Endoscopy is confirmatory.
. 5 . Gastro-esophageal reflux disease "GERD":
_______________________________________________
. Bad taste, cough & hoarsness.
. Response to PPIs "Omeprazole".
. 6 . Pneumonia:
_________________
. Cough, sputum & hemoptysis.
. Dx -> CXR.
. 7 . Pulmonary embolism:
__________________________
. Sudden onset SOB, hypoxia, tachycardia.
. CxR -> Clear lungs.
. Spiral CT - V/Q scan - D-Dimer.
. Most accurate: Pumonary angiography.
. 8 . Pneumothorax:
____________________
. Sharp, pleuritic pain with tracheal deviation.
. CXR is diagnostic.
. 9 . Pancreatitis:
____________________
. Alcoholic pt with chest pain radiating to the back.
. Nausea & vomiting.
. Dx -> Check amylase & lipase levels.
. 10 . Cholecystitis:
______________________
. Right upper quadrant tenderness & mild fever.
. Dx -> Abdominal U/$ for gall stones.
. EVALUATION OF CHEST PAIN IN THE EMERGENCY DEPARTMENT:
_______________________________________________________
Focused H/O & P/E
Assess vital signs
Obtain venous access
|
_______ STABLE ______________UN-STABLE_______
|
|

. Obtain ECG & CXR


. Stabilize hemodynamics.
. Administer aspirin if
. Check for underlying causes
low risk of aortic dissection
|
ECG consistent with AC$ ?
|
_______________________________________________
|
|
YES
NO
|
|
_______________________
CXR diagnostic ??
|
|
|
STEMI
NSTEMI
__________YES_________NO___
|
|
|
|
Treat with emergency
Treat with Treat cause
Assess for Pulmonary embolism
catheterization or
appropriate
Assess for pericarditis
thrombolytics
anticoagulation
Assess for aortic dissection

.
.
.
.
.
.

N.B.
Further testing for coronary artery disease (CAD):
sh'd NOT be done routinely in low risk pts as they frequently can've false +ve results.
Exercise EKG or pharmacological stress testing is most useful in intermediate risk pts.
High risk pts sh'd start pharmacological therapy & undergo coronary angiography,
if they have stable angina.

. Physical examination findings:


_________________________________
. Distressed pt with SOB & clutching chest.
. S3 gallop = Lt ventricular dilatation.
. S4 gallop = Lt ventricular hypertrophy.
. Jugulo-venous distension.
. Bilateral basal lung rales.
. LL edema.
. Diagnosis:
____________
. Best initial test -> EKG.
. N.B. If the case is very clear with diagnosis of ischemic pain & the given choices are:
. EKG & (Aspirin - Nitrates - Oxygen - Morphine) ..
. CHOOSE TTT 1st !
. N.B. CK-MB is the best to detect RE-infarction a few days after the initial infarction:
__________________________________________________________________________________________
. Both CK-MB & troponin levels rise 3-6 hs after the start of chest pain.
. The main difference is that CK-MB only stays elevated 1-2 days.
. While troponin stays elevated 1-2 weeks.
. so, CK-MB is the best to detect RE-infarction a few days after the initial infarction.
. N.B. MYOGLOBIN is the 1st cardiac marker to rise after chest pain:
_____________________________________________________________________
. Myoglobin elevates 1-4 hours after start of chest pain.

. When do you order a STRESS TEST ?


____________________________________
. When the case is NOT ACUTE.
. When the initial EKG & cardiac enzymes can't establish diagnosis.
. Angiography is the next best step in case of an abnormal stress test.
.
.
.
.
.
.

N.B.
Ischemic cardiac pain can sometimes be mistaken for epigastric pain,
but should remain high on differential.
especially in the setting of symptoms worsened with exertion.
An exercise stress test e'out imaging (Exercise EKG) is the most reasonable 1st step,
if the baseline resting EKG is normal.

. When do you order DIPYRIDAMOLE or ADENOSINE THALLIUM STRESS TEST or DOBUTAMINE


ECHO ?
________________________________________________________________________________________
. In pts who can't exercise to a target heart rate of > 85% of maximum.
. COPD.
. Amputation.
. Weakness due to a prevous stroke.
. Morbid obesity.
.
.
.
.
.
.

N.B.
Dipyridamole & coronary steal phenomenon:
Dipyridamole can be used during myocardial perfusion scanning,
to reveal the areas of restricted myocardial perfusion.
The redistribution of coronary blood flow to "Non-diseased" segments by Dipyridamole,
is called "Coronary steal phenomenon".

. When do you order EXERCISE THALLIUM TEST or STRESS ECHOCARDIOGRAPHY ?


________________________________________________________________________
. When EKG is UN-READABLE for ischemia !
. Lt BBB.
. Digitalis use.
. Pace maker is present.
. When do you order a SESTAMIBI NUCLEAR STRESS TEST ?
______________________________________________________
. Obese pts with large breasts.
. ANGIOGRAPHY is the MOST ACCURATE TEST to detect coronary artery disease.
. ACUTE CORONARY $YNDROME (AC$) MANAGEMENT:
___________________________________________
___________________________________________
. UN-STABLE ANGINA (EKG: ST segement depression & -ve CK-MB).
. NON-ST SEGMENT ELEVATION MI "NON-STEMI" (EKG: No ST segment elevation & +ve CK-MB).
. ST SEGMENT ELEVATION MI "STEMI" (EKG: ST sement elevation & +ve CK-MB).
. N.B. ST segment elevation in leads 2,3,aVF -> Inferior wall MI.
. N.B. ST segment depression in leads V1,V2 -> Posterior wall MI.
. The occluded culprit artery -> RIGHT CORONARY ARTERY.

. Inferior MI is associated weith hypotension, bradycardia & AV block.


. MI LOCATION & corresponding culprit occluded CORONARY VESSEL:
_______________________________________________________________
. Involved myocardium ___________ Blocked vessel ________ ECG leads involved
. Anterior MI
. Inferior MI
. Posterior MI

. Lateral MI

___________ LAD
___________ RCA or LCX

________ ST elevation V1-V6


________ ST elevation 2,3,aVF

___________ RCA or LCX


________ ST depression V1-V3
ST elevation 1,aVL (LCX)
ST depression 1,aVL (RCA)
___________ LCX, diagonal _________ ST elevation 1,aVL,V5,V6
ST depression 2,3,aVF

{1} ASPIRIN:
_____________
. The best initial therapy for all AC$.
. ANTI-PLATELET drug.
. LOWERS MORTALITY.
. Given in addition to Nitrates, oxygen & Morphine.
. CLopidogreal or Prasugrel are given in case of containdication to Aspirin e.g. Allergy.
. They are also given in pts under-going angioplasty or there is acute MI.
{2} THROMBOLYTICS & ANGIOPLASTY (PCI):
_______________________________________
. Both lower mortality.
. Angioplasty must be performed within 90 minutes of arrival at the emergeny department.
. If it can't be performed within 90 mins -> The pt sh'd receive THROMBOLYTICS.
. Contraindications to thrombolytics (Major bleeding - Recent surgery - Severe HTN).
. Complications of angioplasty (Rupture of coronary artery - Restenosis - Hematoma).
{3} BB, ACEIs & ARBs:
______________________
. BB ALWAYS LOWER MOTALITY.
. ACEIs & ARBs lower mortality ONLY IF there is left ventricular dysfunction!
. ACEIs lessens ventricular remodelling following MI.
{4} STATINS:
_____________
. Given to all pts with AC$.
. Side effect -> Liver toxicity.
. Statins inhibit intracellular HMG-CoA reductase enzyme,
. prevent conversion of HMG coA to mevalonic acid & ++ NO of cell membrane LDL receptors
. Statins also -- coenzyme Q10 synthesis involved in muscle cell energy production,
. so .. It contributes to statin-induced myopathy.
. CPK levels sh'd be checked in any pt on a statin who presents with myalgias.
. If highly elevated, the 1st step is to discontinue the statin.
. N.B. High dose Niacin therapy may cause cutaneous flushing & pruritis.
. This side effect is due to PROSTAGLANDIN INDUCED PERIPHERAL VASODILATATION.
. Can be reduced by low dose aspirin.

. DIFFERENT THERAPIES & THEIR MORTALITY BENEFIT:


________________________________________________
* ALWAYS LOWER MORTALITY:
__________________________
. Aspirin.
. Thrombolytics.
. Primary angioplasty.
. BB "Metoprolol".
. Statins.
. Clopidogrel or prasugrel.
* LOWER MORTALITY IN CERTAIN CONDITIONS:
_________________________________________
. ACEIs & ARBs if there is low ejection fraction (Lt ventricular dysfunction).
* DO NOT LOWER MORTALITY:
__________________________
. Oxygen.
. Morphine.
. Nitrates.
. CCBs.
. Lidocaine.
. Amiodarone.
. When do you give Calcium channel blockers "CCBs" ?
_____________________________________________________
. Intolerance to BBs (Severe asthmatic pt).
. Cocaine induced chest pain.
. Coronary vasospasm (Prinzmetal's angina).
. When do you place a pacemaker ?
__________________________________
. 3rd degree complete AV block.
. Mobitz 2 2nd degree AV block.
. New LBBB.
. SYMPTOMATIC bradycardia.
.
.
.
.

N.B. The strongest influence on long term prognosis following an STEMI is:
The duration of time that passes before restoring coronary blood flow.
PTCA -> Door to balloon time -> 90 minutes.
Fibrinolytics -> Door to needle time -> 30 minutes.

. N.B. Nitrates cause venodilatation which improves cardiac chest pain,


. Nitrates reduce cardiac preload thus decreasing myocardial oxygen demand.
. N.B. The main mechanism for pain relief in pts with anginal pain ttt with NITROGLYCERIN
. VENO-DILATATION (DILATATION of CAPACITANCE VESSELS) & -- in ventricular pre-load.
.
.
.
.
.

N.B. Following MI, ventricular emodelling occurs.


Remodelling leads to gradual dilatation of the left ventricle with thinning of walls.
This can results in CHF.
ACE inhibitors are given to lessens ventricular remodelling.
ACEIs sh'd be initiated within 24 hours of MIin all pts without contraindications.

.
.
.
.

N.B.
GERD is characterized by a retrosternal burning sensation after eating & lying down.
It may be accompanied by hoarseness & chronic cough especially while recumbant.
Initial ttt is an H2-receptor blocker or Proton pump inhibitor.

. N.B. Dihydropyridine CCB (Nifedipine) can worsen cardiac ischemia,


. Bec. they cause peripheral vasodilatation & reflex tachycardia.
. so they are contraindicated in pts with acute coronary $.
. N.B. NON-Dihydropyridine CCB (Diltiazem & Verapamil) can be used in STEMI after BB,
. BUT .. They do NOT improve mortality.
. N.B. Ischemic damage in MI may lead to diastolic dysfunction & stiff left ventricle,
. resulting in an atrial gallop (S4 4th heart sound).
. N.B. Pts presenting to the emergency department with chest pain & suspected AC$,
. should be administered ASPIRIN ASAP.
. Early anti-platelet therapy with aspirin reduces the rate of MI & overall mortality.
. COMPLICATIONS of Acute MI:
____________________________
{1} CARDIOGENIC SHOCK:
_______________________
. Dx -> Echo & SWAN-GANZ Rt heart catheterization.
. Tx -> ACEIs & urgent re-vascularization.
{2} VALVE RUPTURE:
___________________
. Dx -> ECHO.
. Tx -> ACEIs, Nitroprusside & intra-aortic balloon pump as a bridge to surgery.
{3} MYOCARDIAL FREE WALL RUPTURE:
__________________________________
. Mechanical complication of transmural ANTERIOR wall MI.
. Presents with profound chest pain & profound shock.
. Rupture of the Lt ventricle -> Hemopericardium & tamponade.
. Pericardial effusion & tamponade signs (Dyspnea, hypotension, JVD, Pulsus paradoxus).
. Tamponade compresses the Lt wall ventricle& decreases stroke volume -> -- BP & ++ HR.
. EKG -> Low voltage QRS complexes & Electrical alternans.
. Dx -> ECHO.
. Tx -> Pericardiocentesis & Supportive care & urgent surgical repair.
{4} INTERVENTRICULAR SEPTAL RUPTURE:
_____________________________________
. Around 5 days after infarction.
. Presents with profound chest pain & profound shock.
. Interventricular wall rupture causes a ventricular septal defect.
. Sudden onset of hypotension congestive biventricular failure (Right predominant).
. Loud holosystolic murmur heard best at the lower left sternal border.
. Dx -> Echo & SWAN-GANZ Rt heart catheterization.
. Step-up in oxygen saturation from Rt atrium to Rt ventricle.
. Tx -> ACEIs, Nitroprusside & urgent surgery.

{5} PAPILLARY MUSCLE RUPTURE:


______________________________
. 3-5 days post MI.
. Mechanical complication of MI.
. Life threatening due to necrosis of the surrounding tissues.
. Acute severe mitral regurge (pan-systolic murmur heard at the apex & axillary radation)
. Acute severe MR causes hypotension & pulmonary edema.
. No ST segment elevation.
{6} SINUS BRADYCARDIA:
_______________________
. Dx -> EKG.
. Tx -> Atropine, followed by pacemaker if symptoms are still present.
{7} COMPLETE 3rd DEGREE AV BLOCK:
__________________________________
. Dx -> EKG & Cannon "a" waves.
. Tx -> Atropine & PACEMAKER even if symptoms resolve.
{8} RIGHT VENTRICULAR INFARCTION:
__________________________________
. Associated with NEW INFERIOR WALL MI.
. due to occlusion of the Rt coronary artery.
. HYPOTENSION & Shock & JVD & Clear lung fields.
. EKG -> ST elevation in RV4-RV6 (Rt venticular leads).
. Tx -> High volume fluid replacement.
. Avoid nitroglycerin (will worsen hypotension).
{9} VENTRICULAR ANEURYSM (VA):
_______________________________
. Late complication occuring weeks to months post MI.
. Scarred or fibrotic myocardial wall due to healed transmural MI.
. may present with heart failure, refractory angina, ventricular arrhythmia or embolism.
. HALLMARK EKG -> PERSISTENT ST segement elevation after a recent MI + DEEEEP Q waves !
. Large VA can lead to progressive LV enlargement, arrhythmia, mural thrombus.
. Dx -> ECHOCARDIOGRAPHY -> Dyskinetic wall motion of a portion of the left ventricle.
{10} ACUTE PERICARDITIS:
_________________________
. Within 1-3 days post MI.
. Sharp pleuritic or positional chest pain within the 1st several days post MI.
. It is worse in supine position & improved by sitting up & leaning forwards.
. A pericardial friction rub may be heard.
. Diffuse ST elevations especially with PR depressions are typical ECG manifestations.
. Dressler's $ is post MI pericarditis (immune-mediated) occuring weeks to months post MI
. Fever, leukocytosis & pericardial friction rub.
. Tx -> NSAIDs.
{11} POST-INFARCTION ANGINA (Recurrent cardiac ischemia):
__________________________________________________________
. Recurrent chest pain.
. New or worsening ECG changes of ischemia e.g. ST elevation.
. It may cause ventricular tachycardia or fibrillation which may degenerate into asystole
. N.B. IWMI with Right Ventricular Infarction management:

_________________________________________________________
. Acute management of a STEMI generally includes reperfusion (Thrombolysis or PCI),
. antiplatelet therapy, morphine, heparin, nitrates & BB.
. Nitrates & BB are contraindicated in certain circumstances.
. e.g. AS, recent phosphodiesterase inhibitor use or Right Ventricular Infarction.
. RV MI with IWMI are associated with hypotension & JVD.
. When the Rt venticular stroke volume decreases, the COP is impaired.
. So .. Any medication that reduces preload (e.g. Nitrates or diuretics) will WORSEN it.
. so .. The 1st step in RV MI is generally IV FLUID RESUSCITATION to ++ RV stroke volume.
. Nitrates should be avoided.
.
.
.
.

N.B.
Acute MR can occur due to papillary muscle dysfunction in pts with acute MI.
Acute MR characteristically causes a rise in LEFT ATRIAL PRESSURE.
NO CHANGE in (Lt atrial size - Lt ventricular size - Lt ventricular EF).

. POST-MI DISCHARGE INSTRUCTIONS:


_________________________________
. Aspirin, clopidogrel, BB, Statin & ACEIs.
. Wait 2-6 weeks before resuming sexual relation.
. Anxiety is the most common cause of erectile dysfunction post-MI.
. Nitrates are containdicated in case of Slidenafil usage (Both -> severe hypotension).
. NON-STEMI MANAGEMENT:
_______________________
. No thrombolytic use.
. HEPARIN is uded routinely.
. LMW heparin is preferred to the IV form.
. Glycoprotein 2b/3a inhibitors lower mortality in those undergoing angioplasty & stent.
. Thrombolytics are used only if there is ST segment elevation or new LBBB.
. STABLE ANGINA MANAGEMENT:
___________________________
. Beta blockers are the 1st line therapy.
. ASPIRIN & METOPROLOL are the main stay of ttt. (Both lower mortality).
. Nitrates sh'd be used in case of anginal pain but they do NOT lower mortality.
. ACEIs & ARBs are used in case of congestive failure, systolic dysfunction or low EF.
. Coronary angiography is used routinely to determine who is candidate for CABG.
. CABG = Coronary artery bypass graft.
. You don't need angiography to diagnose CAD (A stress test can show reversible ischemia)
. N.B. You don't need angiography to initiate the following:
-> Aspirin + Metoprolol "Mortality benefit".
-> Nitrates "pain".
-> ACEIs/ARBs "Low EF".
-> Clopidogrel or prasugrel "Acute MI or can't tolerate aspirin".
-> Statins "LDL > 70 - 100".
. BB are the 1st line therapy for anginal syms, improves exercise tolerance.
. BB relieve angina by -- myocardial contractility & heart rate.

. BB improve survival rate in those with MI.


. CCB can combine BB if angina persists or there is contraindication to BB.
. CCB improve angina by causing peripheral & coronary vasodilatation.
. Nitrates short acting form ttt are used in acute setting.
. Nitrates long acting form are used in persistent angina.
. Preventive ttt for stable angina:
-> Aspirin.
-> Statin.
-> Smoking cessation.
-> Regular exercise & weight loss.
-> Control of blood pressure & diabetes.
. N.B. The main difference between saphenous vein grafts & internal mammary artery grafts
. is that veins grafts start to become occluded after 5 years,
. but internal mammary artery grafts are often patent at 10 ys.
. INDICATIONS for CABG:
________________________
. 3 coronary vessels with > 70 % stenosis.
. Left main coronary artery stenosis.
. CORONARY ARTERY DISEASE EQUIVALENTS:
_______________________________________
. DM - PAD - AAA - CAD.
. LIPID MANAGEMENT:
____________________
. The single strongest indication for a "statin" in a pt with AC$ & an LDL > 130mg/dl.
. The goal of therapy in this pt will be < 100 mg/dl.
. If there is associated DM, the goal will be < 70 mg/dl.
. RISK FACTORS IN LIPID MANAGEMENT:
____________________________________
. Cigarette smoking.
. Hypertension > 140/90 mmHg on antihypertensives.
. Low LDL < 40 mg/dl.
. Family H/O of early coronary artery dis. (Female relatives < 65, Male relatives < 55ys)
. Age (Male > 45 ys, Females > 55 ys).
. N.B. STATINS have the greatest efficacy in lowering mortality.
. The most common adverse effect of statins is LIVER TOXICITY.
. N.B. High dose Niacin therapy may cause cutaneous flushing & pruritis.
. This side effect is due to PROSTAGLANDIN INDUCED PERIPHERAL VASODILATATION.
. Can be reduced by low dose aspirin.
. SUMMARY OF MECHANICAL COMPLICATIONS OF MI:
____________________________________________

* RIGHT VENTRICULAR FAILURE:


_____________________________
. Time course -> Acute.
. Culprit coronary artery -> RCA.
. Exam -> Hypotension - Clear lungs - Kussmaul's sign (++JVD on inhalation).
. Echo -> Hypokinetic RV.
* PAPILLARY MUSCLE RUPTURE:
____________________________
. Time course -> Acute & within 3-5 days.
. Culprit coronary artery -> RCA.
. Exam -> Acute severe pulmonary edema + New holosystolic murmur.
. Echo -> Severe mitral regurgitation with flail leaflet.
* INTERVENTRICULAR SEPTUM RUPTURE/DEFECT:
__________________________________________
. Time course -> Acute & within 3-5 days.
. Culprit coronary artery -> LAD (Apical septal rupture) or RCA (Basal septal rupture).
. Exam -> Shock & chest pain + New holosystolic murmur + Biventricular failure.
. Echo -> Lt to Rt shunt at ventricular level & step up of oxygen level bet Rt AT & VT.
* VENTRICULAR FREE WALL RUPTURE:
_________________________________
. Time course -> Within 1st 5 days - 2 weeks.
. Culprit coronary artery -> LAD.
. Exam -> Shock & chest pain + JVD + Distant heart sounds (Tamponade signs).
. Echo -> Pericardial effusion with tamponade.

. N.B. VARIANT ANGINA = PRINZMETAL's ANGINA:


____________________________________________
. Caused by temporary spasm of coronary arteries.
. Young women are classically affected.
. The greatest risk factor is smoking.
. Aside from smoking, there is absence of cardiovascular risk factors.
. Associated e' other vasospastic disorders (Raynaud's phenomenon & Migraine headache).
. Episodes occur at the middle of the night to 8 a.m.
. Triggers -> exercise, hyperventillation, emotional stress, cold exposure & cocaine use.
. Anginal episode is accompanied by transient ST elevations with return to baseline.
. Tx -> CCBs "Diltiazem" or Nitrates.
. Non selective BB & Aspirin should be AVOIDED because they can promote vasoconstriction.
. N.B. COCAINE ABUSE INDUCING VASOSPASM:
________________________________________
. Dilated pupils & blood-crusted nose suggest cocaine abuse.
. Cocaine inhibits catecholamines reuptake from neuronal synapses -> Sympathomimetic.
. Cocaine induced vasospasm -> myocardial ischemia & or infarction.
. Pt presents with chest pain & ST elevations.
. Cocaine induced STEMI is ttt sameway as classic STEMI with PCI & thrombolysis.
. Aspirin & nitrates are also appropriate.
. BB SHOULD BE AVOIDED -> WORSEN THE SPASM.
. Tx -> CCBs.
. CONGESTIVE HEART FAILURE:

___________________________
___________________________
. DYSPNEA (SOB) is the essential feature.
. Orthopnea (Worse when lying flat, relieved when sitting or standing).
. Pripheral edema.
. Bilateral lung rales.
. Jugular venous distension (JVD).
. Paroxysmal nocturnal dyspnea (PND) "Sudden worsening at night during sleep".
. S3 GALLOP (Diastolic sound just after S2 = Left ventricular failure)
. Volume overload - Cardiac dysfunction - Pressure overload.
. Systolic dysfunction -> LOW EJECTION FRACTION + HEART DILATATION.
. Diastolic dysfunction -> PRESERVED EJECTION FRACTION.
. HYPERTENSION causing cardiomyopathy is the most common cause of systolic dysfunction.
. MYOCARDIAL INFARCTION causes dilated cardiomyopathy & -- EF.
. INFARCTION -> DILATATION -> REGURGITATION -> CHF.
. Dx -> Every CHF pt must undergo ECHOCARDIOGRAPHY to evaluate ejection fraction.
. Other tests used to determine the etiology of CHF:
. EKG -> MI or heart block.
. CXR -> Dilated cardiomyopathy.
. Holter monitor -> Paroxysmal arrhythmia.
. Cardiac catheterization -> To detect valve diameters & septal defects.
. Thyroid functions -> Both hypo & hyper thyroid states cause CHF.
. CBC -> Anemia.
. Tx -> SYSTOLIC DYSFUNCTION (LOW EF):
_______________________________________
. ACE Is & ARBS (if associated dry Cough), Both ACEIs & ARBs cause HYPERKALEMIA !
. BBs (Metoprolol & Carvedilol).
. Spironolactone -> Anti-aldosterone -> Side effects (Gynecomastia - Hyperkalemia).
. Diuretics (Loop diuretics e.g. Furosemide).
. Digoxin (inhibits Na-K ATPase).
. N.B. 1. Give ARBs instead of ACEIs in associated dry cough.
. N.B. 2. BB are never given in ttt of acute heart failure !
. BB are given as a chronic ttt of CHF.
. BB have anti-arrhythmic & anti-ischemic action ao they prevent sudden death.
. N.B. 3. Most common cause of death in CHF is ARRHYTHMIA -> SUDDEN DEATH.
. N.B. 4. Eplerenone is like spironolactone but e'out antiandrogenic SE (NO Gynecomastia)
. N.B. 5. Digoxin (cardiac glycoside) is a medication with narrow therapeutic window.
. Some drugs eg. VERAPAMIL ++ its concentration & -- it's clearance -> Digitalis toxicity
. Toxicity -> (Anorexia, Nausea & vomiting) & Biventricular tachycardia & yellow vision.
. Most common arrhythmia due to Digitalis is ATRIAL TACHYCARDIA WITH VARIABLE AV
BLOCK !
. Tx of Digoxin toxicity -> Na-K ATPASE, Lidocaine & Digibind (Abs against Digitalis).
. N.B. 6. DRUGS LOWERING MORTALITY IN CHF -> ACEIs - ARBs - BB - SPIRONOLACTONE &
ASPIRIN
. Hydralazine & nitrates have been proven to lower mortality in CHF.

. Implantable defibrillator lowers mortality & prevents sudden death due to arrhythmia.
. N.B. 7. Digoxin & loop diuretics "Furosemide" DO NOTTTT lower mortality.
. N.B. 8. EJECTION FRACTION (EF):
. Best initial test -> TRANS-THORACIC ECHOCARDIOGRAPHY (TTE).
. Most accurate test -> MUGA scan or nuclear ventriculography.
. N.B. 9. A NORMAL BNP excludes CHF !
. N.B. 10. If their is hyperkalemia 2ry to ACEIs, don't give ARBs (Also cause ++ K)!
. Give HYDRALAZINE (DIRECT ACTING ARTERIOLAR VASODILATOR) instead.
.
.
.
.
.
.
.

N.B. 11. A H/O of upper RTI followed by suuden onset cardiac failure in a healthy pt,
is suggestive of dilated cardiomyopathy.
It is the end result of myocardial damage due to toxic, metabolic or infectious agents.
Viral myocarditis is most commonly seen following Coxsackie virus B infection.
Viral myocarditis may cause dilated cardiomyopathy via direct viral damage.
Dx -> Echocardiogram -> Dilated ventricles with diffuse hypokinesia.
Low EF (Systolic dysfunction).

.
.
.
.
.
.

N.B. 12. Mechanism of edema in congestive heart failure:


In heart failure, there is defective circulating blood volume -> -- COP.
-- COP -> Renal hypoperfusion -> Activation of Renin Angiotensin Aldosterone system.
++ in concentration of both Angiotensin 2 & Aldosterone.
Angiotensin 2 -> -- Renal blood flow by constricting the efferent renal arteriole.
Aldosterone -> ++ Na Reabsorption -> ++ water retention -> Edema.

.
.
.
.

N.B. 13. HYPO-NATREMIA is a bad prognostic factor in pts with heart failure.
It indicates the severity of heart failure i.e. high level of neurohumoral activation.
-- Na is associated e' high levels of renin, aldosterone, vasopressin & norepinephrine.
-- water intake (Not ++ Na intake) may correct the electrolyte imbalance.

.
.
.
.
.

N.B. 14. HYPO & HYPER-KALEMIA are an imporatant electrolyte imbalance in HF.
It reflects the activity of RENIN-ANGIOTENSIN-ALDOSTERONE SYSTEM.
It may be due to drug interactions.
Hyperkalemia due to ACEIs (Enalapril), Digoxin & K sparing diuretics (Spironolactone).
Hypokalemia due to Thiazide diuretics (Furosemide).

. N.B. 15. AMYLOIDOSIS may cause congestive heart failure.


. It is extracellular deposition of excees proteins leading to organ dysfunction.
. Extra-cardiac manifestations (proteinuria & easy bruisability) are the key words!
. Common causes of amyloidosis include multiple myeloma (AL amyloidosis).
. Chronic inflammatory diseases e.g. rheumatoid arthritis (AA amyloidosis).
. Deposition of amyloid fibrils in the kidney -> proteinuria.
. Amyloid fibrils binding to liver-> -- coagulation factors synthesis-> Easy bruisability
. Amyloid fibrils deposition in the heart -> Restrictive cardiomyopathy.
. Restrictive cardiomyopathy is due to thickened ventricular wall & diastolic dysfunction .
Systolic function is relatively well preserved.
. Ventricular dimensions remain unchanged.
. N.B. 16. Medications to withhold prior to cardiac stress testing: BB - CCB - Nitrates.
. N.B. 17. Medications to continue (ACEIs - ARBs - Digoxin - Statins - Diuretics).
. N.B. 18. Pt e' recurrent pulmonary infections, chronic cough & significant smoking H/O.

.
.
.
.
.
.

++ antero-posterior diameter & wheezing on P/E.


This is consistent with COPD.
The associated chronic hypoxemia -> constriction of the pulmonary arterial system,
Constriction -> ++ PULMONARY ARTERY SYSTOLIC PRESSURE -> PULMONARY HTN.
Pulmonary hypertension -> Rt ventricular hypertrophy & Rt ventricular failure.
RVF -> ++ JVP, congestive HSM, Hepatojugular reflux & LL edema.

. N.B. 19. MASSIVE PULMONARY EMBOLISM


. is likely in post-operative pt with jugular venous distension & new onset Rt BBB.
.
.
.
.
.

N.B. 20. Diastolic dysfunction = impaired ventricular filling with preserved EF,
Due to poor myocardial relaxation or diminished ventricular compliance.
Usually due to systemic hypertension.
Tx -> Diuretics & anti-hypertensives.
In severe cases, diastolic dysfunction can cause decompensated heart failure.

. N.B. 21. Measurement of serum BNP can help distinguish bet. CHF & other dyspnea causes.
. A value > 100 pg/ml diagnoses CHF with,
. a sensitivity, specificity & predictive accuracy of 90, 76 & 83 % respectively !!!
. N.B. 22. In otherwise healthy pts who develop CHF,
. Myocarditis should be considered high on D.D.
. Viral infection, especially with Coxsackie B virus is the most common cause.
. N.B. 23. SYSTOLIC HEART FAILURE:
-> Depressed cardiac index (CI).
-> ICREASED TOTAL PERIPHERAL RESISTANCE (TPR).
-> LEFT VENTRICULAR END DIASTOLIC VOLUME (LVEDV).
. DIFFERENTIAL DIAGNOSIS OF DYSPNEA:
____________________________________
. Pulmonary embolism -> Sudden onset & Clear lungs.
. Bronchial asthma -> Sudden onset, wheezing & ++ expiratory phase.
. Pneumonia -> Slower, fever, sputum, UNIlateral rales or ronchi.
. Pneumothorax -> Unilateral -- in breath sounds& tracheal deviation.
. Panic attack -> Circumoral numbness, caffeine use & H/O of anxiety.
. Anemia -> Pallor, gradual over days to weeks.
. Tamponade -> Pulsus paradoxus, faint heart sounds & JVD.
. Arrhythmia -> Palpitations & syncope.
. Pleural effusion -> Dullness to percussion at lung bases.
. COPD -> Long smoking H/O & barrel chest.
. Methemoglobinemia -> Recent anesthetic use, BROWN blood not improved e' O2 & cyanosis.
. CO poisoning -> Burning building or car, Wood burning stove in winter, suicide attempt.
. ALL of the previous items will lack orthopnea - PND - S3 Gallop !
. ACUTE CARDIOGENIC PULMONARY EDEMA:
____________________________________
. Due to SEVERE LEFT VENTRICULAR FAILURE.
. ACUTE ONSET SHORTNESS OF BREATH.
. Rales - JVD - S3 gallop - Edema - Orthopnea.
. May be ascites & HSM in case of associated RVF.

.
.
.
.
.
.
.

Dx -> Brain natriuretic peptide BNP.


BNP is used if the etiology of SOB is not clear !
A normal BNP excludes pulmonary edema (BNP > 100 is diagnostic).
CXR -> Vascular congestion - cardiomegaly - pleural effusion & Kerley B lines.
ABG -> Hypoxia & Respiratory alkalosis (Hyperventillation with excess CO2 release).
EKG -> Af, AF or V.tachycardia -> Perform synchronized cardioversion.
Echo -> To detect if there is systolic or diastolic dysfunction.

. The BEST INITIAL TTT is LOOP DIURETIC (FUROSEMIDE).


. Furosemide removes a large volume of fluid from the vascular space.
. It causes VENODILATATION so it decreases the preload.
. PRELOAD REDUCTION -> LOOP DIURETIC "Furosemide" + MORPHINE + NITRATES + OXYGEN.
. POSITIVE INOTROPE -> DOBUTAMINE.
. AFTERLOAD REDUCTION -> ACEIs & ARBs.
. N.B. NITRATES "NITROGLYCERIN" (THE MOST RAPIDLY ACTIVE DRUG TO RELIEVE
SYMPTOMS).
. N.B. Dobutamine "+ve inotrope" is used if there is no response to preload reduction.
. N.B. Amrinone & Milrinone "Phosphodiesterase inhibitors" are also +ve inotropes.
. N.B. Nitroprusside & Hydralazine can -- afterload in acute cases.
. N.B. BB are -ve inotropes so they are contraindicated in acute pulmonary edema.
. N.B. BB are given only in ttt of chronic heart failure.
. VALVULAR HEART DISEASES:
__________________________
__________________________
. IMPORTANT CLUES:
___________________
. Young female -> MVP.
. Healthy young athlete -> HOCM.
. Immigrant, pregnant -> MS.
. Turner's $, Aortic coarctation -> Bicuspid Aortic valve (AS).
. Palpitations, Atypical chest pain not with exertion -> MVP.
. MURMURS:
___________
. SYSTOLIC -> AS - MR - MVP - HOCM.
. DIASTOLIC -> AR - MS.
. EFFECT OF CHANGE IN VENOUS RETURN:
_____________________________________
. Valvular lesion _____ ++ VR (Squatting & Raising legs) ____ -- VR (Standing & Valsalva)
__________________________________________________________________________________________
. AS,AR,MS,MR, VSD ____
(+)
____
(-) Give diuretics !
. HOCM & MVP __________
(-)
____
(+)
. EFFECT OF CHANGE IN AFTERLOAD:
_________________________________
. Valvular lesion _____ ++ AFTERLOAD (HAND GRIP)

____ -- AFTERLOAD (AMYL NITRATE)

__________________________________________________________________________________________
. AS __________________
(-)
____
(+)
. AR __________________
(+)
____
(-) Give ACEIs !
. MR __________________
(+)
____
(-) Give ACEIs !
. VSD _________________
(+)
____
(-) Give ACEIs !
. HOCM ________________
(-)
____
(+)
. MVP _________________
(-)
____
(+)
. LOCATION & RADIATION OF MURMURS:
___________________________________
. AS -> Heard best at 2nd Rt intercostal space & radiates to the carotid arteries.
. AR, TA, TR, VSD -> Heard best at Lt lower sternal border.
. MR -> Heard best at apex & radiates into the axilla.
. DIANOSIS:
____________
. Best initial test -> ECHOCARDIOGRAPHY (Trans-Thoracic TTE > Trans-Esophageal TEE).
. Most accurate test -> Left heart catheterization.
{1} AORTIC STENOSIS (AS):
__________________________
. Chest pain.
. Old pt with H/O of HTN.
. Syncope & HF are less common presentations.
. Syncope due to left ventricular out flow obstruction.
. Associated CAD.
. AS becomes sympomatic when it is severe (Valve areas < 1 cm2).
. CRESCENDO DECRESCENDO SYSTOLIC MURMUR.
. Heard best at 2nd Rt intercostal space & radiates to the carotid arteries.
. PULSUS PARVUS ET TARDUS (Week & delayed pulse stroke).
. EKG & CXR -> Lt ventricular hypertrophy.
. Tx -> Best initial ttt -> DIURETICS.
. Tx -> Failed -> AORTIC VALVE REPLACEMENT. (Give Warfarin till INR 2-3).
.
.
.
.
.

N.B.
Aortic stenosis in young individual mostly due to CONGENITAL BICUSPID AORTIC VALVE.
Pts with severe aortic stenosis often have large left ventricular mass, so..
requiring additional oxygen.
++ myocardial oxygen demand -> Anginal pain.

. N.B.
. Aortic stenosis in old individual is mostly due to AGE DEPENDENT SCLEROCALCIFIC
CHANGES
. They may present with exertional syncope due to restricted COP due to stenotic aorta.
. N.B.
. Indications for aortic valve replacement:
-> All symptomatic pts with AS (Syncope - Angina - Dyspnea).
-> Pts with severe AS undergoing CABG or other valvular surgery.
-> Asymptomatic pt with e' AS & either poor LV systolic function, LV hypertrophy > 15mm.
.
.
.
.

N.B.
The 3 most common causes of aortic stenosis in the general population are:
senile calcific aortic stenosis - bicuspid aortic valve - rheumatic heart disease.
A bicuspid aortic valve is the cause of aortic stenosis in the majority of pts < 70 ys.

{2} AORTIC REGURGITATION (AR):


_______________________________
. SOB & fatigue are the most common presentations.
. H/O of HTN, Rheumatic heart disease, endocarditis.
. Marfan's $, $yphilis & reactive arthritis are rare causes.
. DIASTOLIC DECRESCENDO MURMUR heard best at left sternal border.
. Quincke pulse -> Arterial or capillary pulsations in the finger nails.
. Corrigan's pulse -> High bounding pulses "Water-hammer pulse".
. Musset's sign -> Head bobbing up & down with each pulse.
. Duroziez's sign -> Murmur heard over the femoral artery.
. Hill sign -> Blood pressure gradient much higher in the lower limbs.
. EKG & CXR -> Lt ventricular hypertrophy.
. Tx -> Best initial therapy -> ACEIs, ARBs & Nifedipine.
. Tx -> Add loop diuretic (Furosemide).
. Tx -> Surgery (If EF < 55% or LVESD > 55mm).
.
.
.
.

N.B.
AR murmur is best heard along the left sternal border at the 3rd & 4th interspaces.
It may be heard in some pts only by applying firm pressure e' the stethoscope diaphragm
while the pt is sitting up, leaning forward & holding his breath in full expiration.

. N.B.
. In DEVELOPED countries,
. Congenital bicuspid aortic valve is the most common cause of AR in young adults.
. N.B.
. In DEVELOPING countries,
. Rheumatic heart disease is the most common cause of AR.
.
.
.
.

N.B.
AR causes widening of the pulse pressure, which can be felt as "water-hammer" pulse.
Lying down & turning to the left brings the heart closer to the chest wall,
making the pt more aware of the forceful heart beat.

{3} MITRAL STENOSIS (MS):


__________________________
. Dyspnea on exertion, HEMOPTYSIS, nocturnal cough.
. Rheumatic fever is the most common cause of MS.
. Immigrants from developing countries (Cambodia) due to RF or pragnants are more common.
. Dysphagia (Large atrium compressing the esophagus).
. Hoarseness (Large atrium compressing the recurrent laryngeal nerve).
. MS -> Lt ATRIAL DILATATION & congestion -> Cardiac emboli -> Af -> stroke.
. LOUD S1.
. DIASTOLIC RUMBLE AFTER AN OPENING SNAP "Extra-sound in diastole".
. EKG & CXR -> LT atrial hypertrophy.
. CXR -> Lt border Straightening & Elevation of the left main bronchus.
. Tx -> Best initial therapy -> Diuretics.
. Tx -> Most effective therapy -> Balloon valvuloplasty.

. Pregnancy is not a contraindication to valvuloplasty.


.
.
.
.
.
.

N.B. MS may be caused by INTRACARDIAC ATRIAL TUMOR (ATRIAL MYXOMA).


Atrial myxoma may obstruct the mitral valve.
Constitutional manifestations of the tumor (Low grade fever & weight loss).
Presents as a mass on echocardiography.
May be assocaiated e' neurological symptoms "side weakness" due to tumor embolization.
Atrial myxoma is NOT equal to Myxomatous valve degeneration (COMMON MISTAKE) !!!

{4} MITRAL REGURGITATION (MR):


_______________________________
. Dyspnea on exertion is the most common complaint.
. Any disease leads to heart dilatation e.g. Hypertension, ischemic heart disease.
. HOLOSYSTOLIC MURMUR OBSCURING BOTH S2 & S2.
. S3 GALLOP due to volume overload.
. Tx -> Best initial therapy -> ACEIs, ARBs & Nifedipine.
. Tx -> Add loop diuretic (Furosemide).
. Tx -> Surgery (If EF < 60% or LVESD > 40mm).
. N.B.
. DIASTOLIC & continous murmur as well as loud systolic murmurs on auscultation,
. should always be investigated using TTE (Trans-thoracic Doppler Echocardiography).
. Mid-systolic soft murmurs in an asymptomatic young pt are usually benign,
. No further investigations are needed.
. N.B.
. AORTIC STENOSIS -> Trans-Thoracic TTE.
. AORTIC DISSECTION -> Trans-Esophageal TEE.
.
.
.
.

N.B.
Acute MR can occur due to papillary muscle dysfunction in pts with acute MI.
Acute MR characteristically causes a rise in LEFT ATRIAL PRESSURE.
NO CHANGE in (Lt atrial size - Lt ventricular size - Lt ventricular EF).

. MITRAL VALVE PROLAPSE (MVP):


______________________________
. Mostly a young female.
. Mostly asymptomatic.
. Absent symptoms of CHF.
. Mid-systolic click over the cardiac apex.
. An accompanying short systolic murmur if mitral regurgitation is present.
. Squatting -> ++ VR -> ++ cardiac preload -> -- or eliminates the prolapse.
. Atypical chest pain (Lasting 5 - 10 seconds).
. Anxiety, palpitations & hyperventilation may be present.
. Dx -> Echocardiography is the best choice.
. Tx -> BB when the pt is symptomatic.
. Valve repair in refractory cases to BB.
. A few stitches into the valve can markedly tighten up the leaflets.
. Endocarditis prophylaxis is NOT recommended even in the presence of a murmur of MR !
. N.B. MVP is due to MYXOMATOUS DEGENERATION OF THE MITRAL VALVE LEAFLETS !

. INFECTIVE ENDOCARDITIS:
_________________________
_________________________
. Intermittent fever.
. New murmur.
. +ve blood culture.
. Tx -> Empiric vancomycin (Covering staph, strept & enterococci).
. VIRIDANS group streptococci (Strep. Mutans) are highly susceptible to penicillin.
. Tx Strept viridans & mutans with INTRA-VENOUS AQUEOUS PENICILLIN G or IV
CEFTRIAXONE.
. IV Ceftriaxone is easier to administer at home due to once daily dosing.
. Oral antibiotics are NOT recommended.
. INFECTIVE ENDOCARDITIS IN INTRAVENOUS DRUG USERS:
___________________________________________________
. HIV infection ++ infective endocarditis risk in IV drug abusers.
. STAPHYLOCOCCUS AUREUS is the most common causing organism.
. TRICUSPID valve involvement (RIGHT-sided) is more common than aortic valve.
. HOLO-SYSTOLIC MURMUR ++ with INSPIRATION indicates tricuspid involvement.
. AUGMENTATION OF INTENSITY WITH INSPIRATION IS VERY SENSITIVE SIGN.
. Septic pulmonary emboli are common (Cough, chest pain & hemoptysis).
. Splinter hemorrhages & Janeway lesions are common.
. Heart failure is more common in aortic valve dis. but rare in tricuspid valve dis.
. Vascular manifestations in infective endocarditis:
____________________________________________________
. Systemic arterial emboli -> Focal neurologic deficits, renal or splenic infarcts.
. Septic pulmonary infarcts.
. Mycotic aneurysm.
. Conjunctival hemorrhages.
. Janeway lesions -> Macular erythematous non tender lesions on the palms & soles.
. Immunologic manifestations in infective endocarditis:
_______________________________________________________
. Osler's nodes -> Painful violaceous nodules seen on finger tips & toes.
. Roth spots -> Edematous & hemorrhagic lesions of the retina.
. Glomerulonephritis.
. Arthritis or positive rheumatoid factor.
. Microbiology of infective endocarditis:
_________________________________________
* STAPHYLOCOCCUS AUREUS:
_________________________
. Prosthetic valves.
. Intravascular catheters.
. Implanted devices (pacemakers - Defibrillators).
. Injection drug users.
* STREPTOCOCCUS VIRIDANS:
__________________________
. Dental procedures.
. Incision & biopsy or respiratory tract.

* COAGULASE NEGATIVE STREPTOCOCCI:


___________________________________
. Intravascular catheters.
. Prosthetic valves.
. pacemakers - Defibrillators.
* ENTEROCOCCI:
_______________
. Nosocomial urinary tract infections.
* STREPTOCOCCUS BOVIS:
_______________________
. Colon carcinoma.
. Inflammatory bowel disease.
* FUNGI:
_________
. Immunocompromized host.
. Chronic indwelling catheters.
. Prolonged antibiotic therapy.
. SEPTAL DEFECTS:
_________________
_________________
.1. VENTRICULAR SEPTAL DEFECT (VSD):
_____________________________________
. HOLOSYSTOLIC MURMUR at the lower left sternal border.
. Mostly asymptomatic.
. Large defects may cause SOB.
. Dx -> ECHOCHARDIOGRAPHY.
. Dx -> Catheterization is used to determine the degree of Lt to Rt shunt.
. Tx -> Mild defects can be left without mechanical closure.
.2. ATRIAL SEPTAL DEFECT (ASD):
________________________________
. Mostly asymptomatic.
. Large defects may cause signs of Rt venticular failure (SOB - parasternal heave).
. FIXED SPLITTING of S2.
. Dx -> ECHOCARDIOGRAPHY.
. Tx -> Best initial -> Percutaneous or catheter devices.
. Tx -> Repair if the shunt ratio exceeds 1.5:1.

. ARTERIO-VENOUS FISTULA (AVF):


_______________________________
. Congenital -> Patent ductus arteriosus.
. Acquired -> Femoral trauma - Iatrogenic "femoral catheterization".
. Abnormal connection between the arterial & venous systems bypassing capillary beds.
. Shunting of large amounts of blood through the fistula -- SVR, ++ prelad & ++ COP.
. A state of high out put cardiac failure.

.
.
.
.
.

Widened pulse pressure.


Strong peripheral arterial pulsation (Brisk carotid upstroke).
Systolic flow murmur heard over cardiac apex not affected by Valsalva maneuver.
Tachycardia & flushed extremities.
Hypertrophied left ventricle -> Lt displacement of the point of maximal impulse.

. Dx -> Doppler ultrasonography.


. Tx -> Surgery if large symptomatic AVF.
. Other causes of high output cardiac failure:
. Thyrotoxicosis - Paget disease - Anemia - Thiamine defeciency.
. SPLITTING OF S2:
__________________
__________________
. WIDE (Delayed P2) --------> RBBB - PS - RVH - PHTN.
. PARADOXICAL (Delayed A2) -> LBBB - AS - LVH - SHTN.
. FIXED --------------------> ASD.

. CARDIOMYOPATHY:
_________________
_________________
{1} DILATED CARDIOMYOPATHY:
____________________________
. presents & managed in the same way as CHF.
. Most common causes (Ischemia - Viral - Alcohol - Adriamycin - Radiation - Chaga's dis).
. Dx -> Best initial test -> ECHOCARDIOGRAPHY to detect EF.
. Dx -> Most accurate test -> MUGA or nuclear ventriculography.
. Tx -> ACEIs, ARBs & spironolactone.
. Digoxin -- symptoms but doesn't prolong survival.
{2} HYPERTROPHIC CARDIOMYOPATHY:
_________________________________
. SOB on exertion.
. S4 Gallop.
. Dx -> ECHOCARDIOGRAPHY -> NORMAL EF.
. Tx -> BB & Diuretics.
. ACEIs may be used.
. Digoxin & spironolactone do NOT benifit !
{3} RESTRICTIVE CARDIOMYOPATHY:
________________________________
. H/O of sarcoidosis, amyloidosis, hemochromatosis, cancer, myocardial fibrosis.
. SOB is the main presenting complaint.
. KUSSMAUL's sign -> ++ JVP on inhalation.
. Cardiac catheterization -> Rapid x & y descent.
. EKG -> Low voltage.
. Dx -> Best initial test -> ECHOCARDIOGRAPHY is the main stay of diagnosis.
. Dx -> Most accurate test -> Endomyocardial biopsy.
. Tx -> Diuresis & correcting the underlying cause.

{4} HYPERTROPHIC OBSTRUCTIVE CARDIOMYOPATHY (HOCM):


____________________________________________________
. AUTOSOMAL DOMINANT inheritance.
. African American male chid.
. H/O of sudden death in the family.
. Dyspnea & chest pain.
. Syncope & light headedness.
. Sudden death, particularly in health athlets.
. Symptoms worsened by anything that ++ HR e.g. exercise, dehydration & diuretics.
. Worsened by anything that -- Lt ventricular chamber size e.g. ACEIs.
. Dx -> ECHOCARDIOGRAPHY (The septum is 1.5 times the thickness of the posterior wall).
. Tx -> BETA BLOCKERS ARE THE BEST INITIAL THERAPY for both HOCM & HCM.
. DIURETICS ARE CONTRAINDICATED in HOCM but they are useful in ordinary HCM.
. N.B. Systolic anterior motion (SAM) of the mitral valve is classic for HOCM.
. N.B. Catheterization is the most accurate test to detect precise gradients of pressure.
. N.B. EKG -> Non-specific ST & T wave changes are common.
. N.B. Left ventricular hypertrophy is common.
. N.B. Septal Q waves in the inferior & lateral leads are common in HOCM (Absent in MI).
. N.B. Digoxin & spironolactone are definitely always wrong in HOCM.
. N.B. Surgical myomectomy is the therapy only if all medical ttt fails.
.
.
.
.
.
.
.

N.B. A H/O of upper RTI followed by suuden onset cardiac failure in a healthy pt,
is suggestive of dilated cardiomyopathy.
It is the end result of myocardial damage due to toxic, metabolic or infectious agents.
Viral myocarditis is most commonly seen following Coxsackie virus B infection.
Viral myocarditis may cause dilated cardiomyopathy via direct viral damage.
Dx -> Echocardiogram -> Dilated ventricles with diffuse hypokinesia.
Low EF (Systolic dysfunction).

. N.B. Excessive alcohol consumption leads to DILATED cardiomyopathy.


. Preserved ventricular dimensions RULES OUT dilated cardiomyopathy !
. N.B. Hemochromatosis causes RESTRICTIVE cardiomyopathy.
. Classic findings (Pancreatic dysfunction - Bronzed skin - Hepatomegaly).
. N.B. Sarcoidosis causes RESTRICTIVE cardiomyopathy.
. More common in African Americans bet 20 - 3- ys.
. Classic findings (Bilateral hilar adenopathy & erythema nodosum).
. N.B. Constrictive pericarditis is difficult to distinguished from restrictive myopathy
. BUT .. INCREASE WALL THICKNESS rules out constrictive pericarditis.
. N.B. Hemochromatosis leads to restrictive cardiomyopathy & cardiac conduction problems.
. It does NOT cause endocardial fibroelastosis !!! (Common mistake).

. N.B. Dilated cardiomyopathy 2ry to Alcoholism:


. Thrombocytopenia - Macrocytosis - ++ Transaminases are suggestive of alcoholism.
. Total abstinence from alcohol is the mainstay of alcoholic cardiomyopathy management.
. Differences in therapy between hypertrophic cardiomyopathy & dilated cardiomyopathy:
_______________________________________________________________________________________
. BB, diuretics, ACEIs & ARBS, Spironolactone & Digoxin are all used in DILATED type.
. ONLY BBs & diuretics only are used in hypertrophic type.

. PERICARDIAL DISEASES:
_______________________
_______________________
{1} PERICARDITIS:
__________________
. Sharp & brief pleuritic chest pain (changes with respiration).
. Positional (Relieved by sitting up & leaning forwards).
. Viral in origin in most cases.
. Friction rub.
. Dx -> EKG -> ST segment elevation in ALL leads.
. Dx -> EKG -> PR segment is pathognomonic but isn't always present.
. Tx -> Best initial therapy -> NSAIDs (Indomethacin).
. Tx -> Add oral steroids (prednisone) in refractory cases.
.
.
.
.
.

N.B.
UREMIC PERICARDITIS (UP) occurs in 10 % of renal failure pts.
Typically those with BUN > 60 mg/dl.
Most pts do NOT present with the classic ECG changes of pericarditis (Diffuse ST elev.)
Tx -> HEMODIALYSIS -> Rapid resolution of chest pain & -- size of effusion.

. N.B. Pericardial effusions appear as an enlarged, "water bottle" shaped on CXR.


. P/E of pericardial effusion without a tamponade -> Diminished heart sounds &
. Difficulty to palpate the point of maximal impulse.
{2} PERICARDIAL TAMPONADE:
___________________________
. SOB + Beck's triad (Hypotension + JVD + Muffled heart sounds).
. May follow viral pericarditis after an URT infection.
. Tamponade is due to fluid accumulation in the pericardial cavity leading to,
. restricting venous return to heart -> Impaired ventricular filling -> -- LV preload.***
. & due to exaggerated shift of interventricular septum into the left ventricular cavity
. Clear lungs.
. PULSUS PARADOXUS -> -- in BP < 10 mmHg on inhalation.
. Dx: ECHOCARDIOGRAPHY (MOST ACCURATE)-> Diastolic collapse of the Rt atrium &
ventricle.
. Dx -> EKG -> Low voltage & ELECTRICAL ALTERNANS (Variation in the QRS complexes height)
. Rt heart catheterization Equalization of all the pressures in heart during diastole.
. Tx -> best initial therapy -> PERICARDIOCENTESIS.
. Most effective long term therapy -> Pericardial window placement.
. Most dangerous therapy -> Diuretics !

. N.B. Pulsus paradoxus -> Exaggerated fall in blood pressure > 10 mmHg on INSPIRATION.
. It is a frequent finding in cardiac tamponade.
. Other causes severe asthma & COPD.
{3} CONSTRICTIVE PERICARDITIS:
_______________________________
. The pericardium is thickened or scarred -> Restriction of diastolic filling.
. The heart is unable to fill properly during diastole -> Compromized COP
. Venous pressure becomes chronically elevated -> JVD, ascites & liver congestion.
. USA most common causes (Post-cardiac surgery - viral pericarditis - Radiation therapy).
. Out-side USA -> Tuberculous pericarditis is common in endemic areas (China - Africa).
. Fatigue or dyspnea on exertion.
. Signs of chronic RIGHT heart failure (Edema - JVD - HSM - Ascites).
. KUSSMAUL's sign -> ++ Jugular venous pressure on inhalation.
. Pericardial knock-> Extradiastolic sound: Heart hitting calcified thickened pericardium
. Dx -> CXR -> Calcification.
. Dx -> EKG -> Low voltage.
. Dx -> JVP tracing -> Prominent x & y descents.
. Dx -> CT & MRI -> Thickening of pericardium.
. Tx -> Best initial ttt -> Diuretics.
. Tx -> Most effective ttt -> Surgical removal of the pericardium "pericardiectomy".

. AORTIC DISEASES:
__________________
__________________
. AORTIC DISSECTION:
_____________________
. Dissection of the THORACIC aorta.
. Most common cause is SYSTEMIC HYPETENSION.
. Marfan's & Ehlers-Danlos $yndromes may cause dissection in YOUNG population.
. Tearing chest pain radiating to the back between the scapulae.
. Very severe "Ripping".
.
.
.
.
.
.
.
.
.

Dissection may extend to:


carotid arteries -> stroke & AMI.
renal arteries -> Acute renal failure.
aortic valve -> Aortic regurgitation: Early diastolic decrescendo murmur at the sternum
pericardial cavity -> Tamponade (Hypotension - ++JVP - pulsus paradoxus).
pleural cavity -> Hemothorax.
superior cervical sympathetic ganglion -> Horner's $.
spinal or common iliac arteries -> lower extremity weakness or ischemia.
abdominal pain -> mesenteric artery.

. > 20 mmHg difference bet. blood pressures of the Rt & Lt arms.


. Dx -> Best initial test -> CXR -> Widened mediastinum.
. Dx -> Most accurate test -> CT ANGIOGRAPHY = Trans-Esophageal ECHO TEE = MRA.
. Dx -> TEE NOT TTE (COMMON MISTAKE).
. Tx -> BB & Nitroprusside.
. Tx -> ICU with surgical correction.

. N.B. Aortic dissection is the most dangerous complication of Marfan's $.


. Tall pt with long extremities - high extensibility of hand joints - pectus carinatum.
. N.B. BB is the most important initial intervention for acute aortic dissection.
. Type "A" dissections involve the ascending aorta -> Tx: Medical therapy & surgery.
. Type " B" dissections involve only the descending aorta -> Tx: Medical therapy alone.
. AORTIC COARCTATION:
_____________________
. Narrowing of DESCENDING aorta distal to Lt subclavian artery at ligamentum arteriosum.
. Pts present with hypertension & may be asymptomatic.
. Headaches, epistaxis, blurred vision or heart failure due to severe hypertension.
.
.
.
.

BRACHIAL / FEMORAL DELAY.


Differential blood pressure in the upper & lower extremeties.
Well developed upper body compared to lower body.
Continous cardiac murmur.

. ECG -> Lt ventricular hypertrophy.


. High voltage QRS complexes, ST depression & T wave inversion in leads V5 & V6.
. CXR -> Notching of the 3rd - 8th ribs due to erosion by enlarged intercostal arteries.
. Classic 3 sign caused by indentation of the aorta at the site of coarctation.
. with pre & post stenotic dilatation.
. Dx -> ECHOCARDIOGRAPHY, CT & MRI.
. Associated conditions -> Bicuspid aortic valve, PDA, VSD & Turner's $.
. ABDOMINAL AORTIC ANEURYSM (AAA):
___________________________________
. Portion of the abdominal aorta grows to 1.5 times its normal size,
. or exceeds the normal diameter by more than 50 % through dilatation.
. It is a true anurysm since it involves all layers of the arterial wall.
. Dx -> CT or MRI to detect relation with surroundings.
. Dx -> U/$ must be done to monitor its size.
. Screening with U/$ for all smokers > 65ys.
. Small AAA < 5 cm -> Observation & monitoring.
. Large AAA > 5 cm -> Surical repaired.
. The strongest predictors of AAA expansion & rupture are:
. large aneurysm diameter, rapid rate of expansion & current cigarette smoking.
. Operative or endovascular repair indications:
-> Aneurysm size > 5 cm.
-> Rapid rate of expansion > 0.5 cm in 6 months or > 1cm per year.
-> Presence of syms (Abdominal, back or flank pain & limb ischemia) regardless its size.
.
.
.
.
.

N.B.
ASCENDING aortic aneurysms are due to cystic medial necrosis or connective tissue dis.
DESCENDING aortic aneurysms are due to atherosclerosis.
CXR can suggest thoracic (descending) aortic aneurysm by showing:
widened mediastinal silhouette, increased aortic knob & tracheal deviation.

. PERIPHERAL ARTERY DISEASE (PAD):

__________________________________
. CLAUDICATION (Pain in the calfs on exertion).
. Smooth, shiny skin with loss of hair & sweat glands & loss of pulses in the feet.
. PAIN + PALLOR + PULSELESSNESS = ARTERIAL OCCLUSION.
.
.
.
.

Dx -> Best initial test -> ANKLE - BRACHIAL INDEX (ABI) "Normal ABI > 0.9".
LL BP should be EQUAL to UL BP (Difference > 10 % = Obstruction).
Dx -> Most accurate test -> ANGIOGRAPHY.
Tx -> Best initial ttt (Aspirin - ACEIs for BP control - Exercise - Cilostazole).

.
.
.
.

N.B. SPINAL STENOSIS is an imp. D.D. for PAD.


Spinal stenosis will give pain that is worse with walking downhill &
less with walking uphill, or while cycling or sitting.
Pulses & skin exam will be NORMAL with spinal stenosis.

. Acute arterial embolus is very sudden in onset with loss of pulse & a cold extremity.
. It is also quite painful.
. H/O of AS or Af is present.
. BB are not contraindicated with PAD.
. If the pt needs them for ischemic disease, they sh'd be used.

. ARRHYTHMIAS:
_______________
_______________
. INITIAL MANAGEMENT OF CARDIAC ARREST:
________________________________________
. The 1st step is to make sure that the pt is truely unresponsive.
. Exclude sleeping or syncopal episode !!
. Open the airway -> Head tilt, chin lift & jaw thrust.
. Five rescue breaths if not breathing.
. Check pulse & start chest compressions if pulseless.
. CPR doesn't restart the heart; it keeps the pt alive until cardioversion is performed.
. The most iportant factors in improving survival in pts with sudden cardiac arrest:
-> ADEQUATE BYSTANDER CPR.
-> PROMPT RHYTHM ANALYSIS.
-> DEFIBRILLATION in pts found to be in ventricular fibrillation.
. i.e. The most imp. item is TIME TO RHYTHM ANALYSIS & DEFIBRILLATION.
. PULSELESSNESS -> Sudden loss of a pulse can be caused by asystole, VF, VT & PEA:
___________________________________________________________________________________
___________________________________________________________________________________
. (1) ASYSTOLE:
________________
. Besides CPR, therapy for asystole is with epinephrine & atropine.
. Vasopressin is an alternative to epinephrine (Both constrict bl. vs. in tissues).
. This shunts blood into critical central circulation e.g. heart & brain.

. Managaed with un-interrupted cardiopulmonary resuscitation CPR & vasopressor therapy,


. in order to maintain adequate cerebral & coronary perfusion.
. Defibrillation or synchronized cardioversion has no role in management !
. (2) VENTRICULAR FIBRILLATION (VF):
_____________________________________
. One of the most dangerous MI complications.
. Best initial ttt -> Immediate un-synchronized cardioversion = Defibrillation.
. Perform CPR if cardioversion is not effective.
. The most important next step after cardioversion is epinephrine or vasopressin.
. They don't restart the heart. They make the next attempt of defibrillation succeed.
. Amiodarone or lidocaine is given next to try to get subsequent shocks successful.
. MAGNESIUM is mandatory.
. VF is managed by shock, drugs, shock, drugs, shock, drug & CPR at all times inbetween.
. (3) VENTRICULAR TACHYCARDIA (VT):
____________________________________
. Wide complex tachycardia with a regular rate.
. Management is based on hemodynamic stability:
. Pulseless VT -> Same as VF.
. Hemodynamically STABLE -> Amiodarone then lidocaine then procainamide.
. Cardiovert the pt. if medications failed.
. Hemodynamically UN-stable -> Electrical cardioversion several times followed by drugs.
. N.B. Hemodynamic instability = Chest pain - Dyspnea - CHF - Hypotension - Confusion.
. N.B. Direct intracardiac medication is always a wrong answer.
. (4) PULSELESS ELECTRICAL ACTIVITY (PEA):
___________________________________________
. PEA = Electrical mechanical dissociation (EMD).
. The heart is electrically NORMAL but there is NO motor contraction.
. Sometimes, the heart may still be contracting but without blood inside .. No COP !
. Dx -> NORMAL EKG & NO PULSE !!!!
. Tx -> Correct the underlying cause (Tamponade, pneumothorax, Massive PE, K disorders).
. Managaed with un-interrupted cardiopulmonary resuscitation CPR & vasopressor therapy,
. in order to maintain adequate cerebral & coronary perfusion.
. Defibrillation or synchronized cardioversion has no role in management !
. ATRIAL ARRHYTHMIAS:
______________________
______________________
. Rarely associated with hemodynamic instability bec. COP is dependent upon ventricles.
. H/O of palpitations, dizziness or lightheadedness.
. H/O of exercise intolerance or dyspnea.
. H/O of embolic stroke.
. N.B. An irregularly irregular rhythm suggests Af even before an EKG is done !
. Af is the most common arrhythmia in the US.
{1} & {2} . ATRIAL FIBRILLATION (Af) & ATRIAL FLUTTER (AF):
____________________________________________________________
. Both have identical management .. Only 2 differences:
. 1 -> Flutter is a regular rhythm while fibrillation is irregular.
. 2 -> Flutter usually goes back into sinus rhytm or deteriorates into fibrillation.

{1} . Atrial fibrillation (Af):


________________________________
. IRREGULAR pulse (R-R interval) with ABSENT "P" waves & narrow QRS complexes.
. Af is a common complication after CABG.
. The most common cause of Af is HYPERTHYROIDISM.
. Tx -> Hemodynamically UN-stable -> Synchronized DC cardioversion.
. Synchronization helps to prevent deterioration into VT or VF.
. CHRONIC Af:
______________
. Af lasting more than 2 days.
. It takes several days to be a risk of clot formation.
. The majority of those who are converted to sinus rhythm will not stay in sinus.
. Af & AF are mostly caused by anatomic abnormalities of the atria from HTN or valve dis.
. Shocking the pt. into sinus rhythm doesn't correct a dilated left atrium.
. Over 90% will revert to fibrillation.
. SLOW THE RATE & COAGULATE "WARFARIN" are the STANDARD CARE for chronic Af.
.
.
.
.

The best initial therapy is to control the rate with BB, CCB or Digoxin.
Once the rate is under 100/min -> Give WARFARIN until the INR is between 2-3.
CCBs used (Diltiazem & verapamil) -> Block the AV node.
You need to use heparin only if there is a current clot in the atrium.

. N.B. Rate control drugs don't convert the pt. into sinus rhythm.
. N.B. Heparin is not necessary before starting a pt on warfarin.
. N.B. Controlling the rhythm or rate in pts with prolonged tachysystolic Af,
. usually improves the LV function significantly, sometimes even dramatically.
. Af CLASSIFICATION:
_____________________
. FIRST DETECTED -> Initial diagnosis, independent of duration.
. PAROXYSMAL -> Recurrent > 2 episodes that terminate spontaneously in < 7days.
. PERSISTENT -> Episodes lasting > 7days.
. LONGSTANDING PERSISTENT -> Persistent > 1 year duration.
. PERMANENT -> Persistent with no further plans for rhythm control.
. LONE Af:
___________
. LONE = paroxysmal, persistent or permanent Af with no structural heart disease.
. Pts e' low risk of stroke can safely prevent it e' using ASPIRIN alone e'out warfarin.
. Criteria of low risk of stroke from Af:
-> No cardiomyopathy, CHF or atherosclerosis.
-> No hypertension.
-> Age 75 or younger.
-> No DM.
-> No past stroke.
. When all the previous risk factors are absent, this is called "LONE Af".
. Tx of lone Af -> ASPIRIN & rate control with BB.
. CHADS2 SCORING:
__________________

.
.
.
.
.

C -> Congestive heart failure.


H -> Hypertension (BP > 140/90 mmHg).
A -> Age > 75 ys.
D -> DM.
S -> Previous stroke or TIA.

. ANTICAGULATION ttt in Af according to CHADS2 score:


______________________________________________________
. CHADS2 score -> Stroke risk -> Anti-thrombotic ttt.
.
0
->
Low
-> No anticoagulation (preferred) or Aspirin.
.
1
-> Intermediate -> Anticoagulation (preferred) or Aspirin.
.
2
-> High
-> Anticagulation.
.
.
.
.

N.B.
Antiarrhythmic drugs are reserved for pts with recurrent symptomatic Af episodes,
or those with Lt ventricular systolic dysfunction thought to be 2ry to uncontolled Af.
Long term use of antiarrhythmic drugs has significant side effects.

.
.
.
.

N.B.
Amiodarone cause pulmonary toxicity & sh'd be avoided in pts with preexisting lung dis.
So .. RESTRICTIVE lung disease pts can NOT be given Amiodarone.
Amiodarone is ABSOLUTELY CONTRAINDICATED.

.
.
.
.
.

N.B.
BBs causes bronchoconstriction.
So .. OBSTRUCTIVE lung disease (Asthma - COPD) pts can NOT be given BBs.
BBs are RELATIVELY CONTRAINDICATED in obstructive lung diseases.
BBs can be used SAFELY in RESTRICTIVE lung diseases.

.
.
.
.
.

N.B.
Af pts with past H/O of Wolf-Parkinson-White $yndrome,
Should be treated with cardioversion or anti-arrhythmics like procainamide.
AV nodal blockers (BB - CCB - Digoxin - Adenosine) should be AVOIDED,
because they ++ conductance through the accessory pathway after AVN blockage.

. N.B.
. Pts e' new-onset Af sh'd've TSH & free T4 levels measured to rule out hyperthyroidism.
{3} . SUPRA-VENTRICULAR TACHYCARDIA (SVT):
___________________________________________
. Palpitations in a pt who is usually hemodynamically stable.
. No structural heart disease.
. Heart rate 160-220/min.
. Mechanism of SVT -> "RE-ENTRY into the AV NODE."
. Narrow QRS complex tachycardia.
. No regular P waves as they are burried within the QRS complexes.
. PSVT most commonly results from accessory conduction pathways through the AV node.
. Vagal maneuvers & medications that -- conduction through the AV node reslove the attack
. Cold water therapy affects the AV conductivity. (VVVVVVV. imp.).
. The best initial step is vagal maneuvers (carotid message, valsalva & ice immersion).
. ADENOSINE if vagal maneuvers don't work.
. BB (Metoprolol), CCB (Diltiazem) or Digoxin if adenosine is not effective.
. If hemodynamically UNSTABLE -> DC CARDIOVERSION.
. N.B. you sh'd differentiate bet SVT & sinus tachycardia due to panic attacks,

. Sinus tachycardia has normal P wave morphology & relationship with QRS complex.
. Tx of sinus tachycardia due to anxiet -> Alprazolam.
.
.
.
.

N.B. You sh'd differentiate bet SVT & ventricular tachycardia (VT),
Ventricular tachycardia has WIDE QRS COMPLEXES,
while SVT have NARROW QRS complexes.
Tx of VT if hemodynamically stable -> Amiodarone.

.
.
.
.
.
.
.
.

N.B. You sh'd differentiate bet SVT & Af with rapid ventricular response (AF with RVR),
SVT presents as sudden onset, REGULAR, narrow complex tachycardias.
SVT HR 160 - 220 beats/min.
Can be managed by vagal maneuvers as carotid massage or Adenosine.
But Af with RVR has IRREGULARLY IRREGULAR RHYTHM with narrow complex tachycardia,
in addition to absence of P waves.
Af with RVR is managed by rate control (BB metoprolo or CCB Diltiazem),
if hemodynamically stable (No hypotension).

{4} . WOLFF - PARKINSON - WHITE $YNDROME (WPW):


________________________________________________
. Anatomic abnormaily in the conduction pathway.
. Supraventricular tachycardia alternating with ventricular tachycardia.
. Supraventricular tachycardia that gets worse after diltiazem or digoxin.
. DELTA waves on EKG.
. Dx -> Most accurate test -> Cardiac electrophysiology (EP) studies.
. Tx -> Acute therapy -> Procainamide or Amiodarone. (Only if there is current WPW).
. Tx -> Chronic therapy -> Radiofrequency catheter ablation is curative.
. Digoxin & CCB are DANGEROUS "they block the normal pathway forcing abnormal
conduction"
.
.
.
.

N.B. Pts with Wolf-Parkinson-White $yndrome who develop Atrial fibrillation with RVR,
Should be treated with cardioversion or anti-arrhythmics like procainamide.
AV nodal blockers (BB - CCB - Digoxin - Adenosine) should be AVOIDED,
because they ++ conductance through the accessory pathway after AVN blockage.

{5} MULTI-FOCAL ATRIAL TACHYCARDIA (MAT):


__________________________________________
. Associated with chronic lung disease e.g. COPD.
. Treat the underlying lung disease.
. Treat MAT as Af (But avoid BB because of the lung disease).
. N.B.
. AMIODARONE SIDE EFFECTS -> pulmonary fibrosis - Hepatotoxicity - Hypothyroidism.
. Always do pulmonary, liver & thyroid function tests.
.
.
.
.

N.B.
Amiodarone-induced hypothyroidism:
progressively worsening fatigue - difficulty concentrating & ++ forgetfulness.
Unintensional weight gain & dry skin.

.
.
.
.

N.B.
Pre-mature atrial beats are benign.
Neither require follow up nor treatment.
May be due to anxiety, CHF, hypoxia, Caffeine or electrolyte imbalance.

. N.B.

. Don't ONLY reassure the pt without advising him !!


. Tobacco & alcohol are reversible risk factors for PACs.
. BBs are helpful in those who are symptomatic.
.
.
.
.
.
.

N.B.
Heriditary hemochromatosis > Abnormal iron deposition -> Multisystem end-organ damage.
Iron deposition within the myocardium can lead to Dilated cardiomyopathy, heart failure
May lead to conduction abnormalities e.g. Sick sinus $yndrome.
Manifestations of hemochromatosis (DM, -- libido, hepatomegaly & testicular atrophy).
Sick sinus $ = Tachycardia-Bradycardia $ = Bursts of atrial tachy then bradyarrhythmias

.
.
.
.
.
.
.
.
.

N.B.
Multiple premature ventricular complexes (PVCs)
Identified by a wide QRS > 120 msec.
Bizarre morphology.
Compensatory pause.
PVCs can be seen in normal individuals.
They may follow a myocardial infections.
No ttt indicated if pt is asymptomatic.
BB are the 1st line of ttt in symptomatic pts.

.
.
.
.
.

N.B.
Grave's disease (insomnia - fatigability - weight loss - lid lag - tremor).
Atrial fibrillation is a common complication of hyperthyroidism.
Best initial ttt is BB "propranolol".
BB not only control Af, but alsodiminishes hyperthyroism symptoms.

.
.
.
.
.
.
.
.

N.B.
Ventricular premature beats (VPBs)
If associated with Acute coronary $,
You sh'd n't give LIDOCAINE (Class 1B anti-arrhythmic)!!
Although Lidocaine use deceases the risk of ventricualar fibrillation,
It may ++ risk of asystole !!
Loidocaine is not used prophylactically in pts with AC$.
Overall prognosis is not affected.

. N.B.
. Absence of P waves is characteristic for Af.
. PULMONARY VEINS are the most common frequent origin of ectopic foci that cause Af.
. N.B.
. DELTA waves are characteristic for WPW $.
. N.B.
. FLUTTER waves are characteristic for AF.
. TRICUSPID ANNULUS is the most common frequent of ectopic foci causing AF.
. N.B.
. HIGH RATE REGULAR RHYTHM NARROW QRS COMPLEXES TACHYCARDIA = PSVT.
. It mostly involves formation of a re-entry circuit within AV node or accessory pathway.
. BRADYCARDIA & AV BLOCK:
__________________________
__________________________

. HR < 60/min.
. EKG is mandatory to detect the cause of bradycardia.
. {1} SINUS BRADYCARDIA:
_________________________
. HR < 60/min with REGULAR rhythm & constant PR interval.
. May be associated with exaggerated vagal activity, hypoglycemia & sick sinus $yndrome.
. May be induced by some medications (Digitalis, BB & CCB).
. Mostly asymptomatic.
. May be dizziness - lightheadedness - syncope - fatigue & worsened angina.
. No ttt if asymptomatic no matter how low the heart rate is.
. If symptomatic -> Atropine is best initial ttt & PACE MAKER is the most effective ttt.
. Atropine ++ HR by -- vagal input.
. {2} FIRST DEGREE AV BLOCK:
_____________________________
. Delayed impulse transmission from atria to ventricles.
. Prolonged PR interval > 0.20 seconds i.e. > 5 small squares i.e. > 1 big square.
. The PR interval remains constant.
. There is a QRS complex present for every P wave.
. Same management.
. {3} SECOND DEGREE AV BLOCK:
______________________________
. {a} MOBITZ "1" = WENCKEBACH BLOCK:
_____________________________________
. Intermittent AV nodal block.
. Progressively lenghtening PR interval that results in dropped beat.
. PR interval longer than previously conducted beat until dropped QRS.
. Grouped QRS complexes followed by dropped complex.
. Low risk for complete heart block.
. Normal sign in aging population.
. No ttt either symptomatic or asymptomatic !!
. {b} MOBITZ "2":
__________________
. Bundle block below the AV node.
. No PR interval prolongation.
. Drops a beat with-OUT the progressive lenghthening of the PR interval.
. i.e. PR interval is constant until dropped QRS complex.
. P waves are episodic & unpredictable dropped QRS complex.
. High risk of progression to 3rd degree AV block.
. Tx -> Pacemaker even if asymptomatic.
. {4} THIRD DEGREE AV BLOCK:
_____________________________
. The atria & ventricles beat separately.
. Complete failure of impulse conduction from atria to ventricles.
. P-P waves intervals are constant (Some P waves may be burried inside QRS complexes).
. R-R waves are constant.
. PACEMAKER is mandatory.
. N.B.
. The most common cause of death in the 72 hours post AMI is VENTRICULAR ARRHYTHMIA.

. Manage arrhythmia from ischemia by correcting the ischemia.


. Don't put in an implantable defibrillator for an arrhythmia u can fix the its cause !
. N.B.
. To determine the risk of recurrence of arrhythmia, do ECHOCARDIOGRAPHY.
. If the echo shows a normal EF, the risk of recurrnce of arrhythmia is small.
. N.B.
. In pts with recurrent syncopes due to arrhythmia, Put an implantable defibrillator.
. An implantable defibrillator will prevent the next episode of syncope or sudden death.
.
.
.
.

N.B.
Torsades de pointes = polymorphic ventricular tachycardia + prolonged QT interval.
H/O of alcoholism + Recent fluconazole or Moxifloxacin ttt.
Tx -> Mg So4 (Alcohol is associated with -- Mg).

.
.
.
.

N.B.
Loop diuretics "Furosemide" cause hypokalemia & hypomagnesemia.
These electrolyte abnormalities can cause ventricular taachycardia.
so .. You should measure serum electrolytes before treating ventricular arrhythmia.

.
.
.
.
.
.

N.B.
Anti-arrhythmic medications with the property of use-dependence,
are more effective at higher rates because,
there is no time between heart beats for the medication to dissociate from its receptor
This phenomenon is seen with class 1C anti-arrhythmics e.g "FLECAINIDE" & class 4.
Class 1C prolong the QRS complex whereas class 4 don't.

. N.B. APPROACH TO WIDE-COMPLEX TACHYCARDIA:


_____________________________________________
AV DISSOCIATION ?
___________________
FUSION/CAPTURE BEATS?
_______________________
|
______YES_____________NO________
|
|
VENTRICULAR TACHYCARDIA
SUPRAVENTRICULAR TACHYCARDIA With
ABERRANCY
_________________________ _____________________________________________
|
|
_________________________
_____________________________
|
|
|
|
STABLE
UN-STABLE
STABLE
________
___________
_________
|
|
|
. IV AMIODARONE
. HYPOTENSION
. VAGAL MAEUVERS (Carotid massage)
. Altered mentation . Adenosin - BB - CCB - Digoxin.
. Respiratory distress
|
. SYNCHRONIZED CARDIOVERSION
. SYNCOPE MANAGEMENT:

______________________
______________________
. Management of syncope is based on 3 criteria:
________________________________________________
. Was the loss of consciousness sudden or gradual ?
. Was the regaining of consciousness sudden or gradual ?
. Cardiac examination normal or abnormal ?
{1} Was the loss of consciousness sudden or gradual ?
______________________________________________________
. Sudden -> Cardiac or neurologic etiology "seizures".
. Gradual -> Toxic metabolic problem - Drug toxicity - Hypoxia - Anemia - Hypoglycemia.
{2} Was the regaining of consciousness sudden or gradual ?
___________________________________________________________
. Sudden -> Cardiac etiology (Rhyhtm disorder or structural heart disease).
. Gradual -> Neuorologic "Siezures".
{3} Cardiac examination normal or abnormal ?
_____________________________________________
. Normal -> Ventricular arrhythmia.
. Abnormal -> Structural heart disease (AS - HOCM - MS - MVP).
. DIFFERENTIAL DIAGNOSIS OF SYNCOPE:
_____________________________________
.1. VASO-VAGAL = NEURALLY MEDIATED SYNCOPE = NEUROCARDIOGENIC SYNCOPE:
_______________________________________________________________________
. Most common type of syncope.
. Due to excessive vagal stimulation -> Profound hypotension & bradycardia.
. Triggers -> Prolonged standing - emotional stress - painful stimuli.
. Situations like medical needles & urination can precipiatate it.
. Prodromal symptoms -> Nausea - warmth - Diaphoresis.
. symptoms improve with supine position.
. Dx -> UPRIGHT TILT TABLE TESTING (in uncertain cases).
.2. ORTHOSTATIC HYPOTENSION:
_____________________________
. Postural changes in heart rate or blood pressure after standing suddenly.
. > 20 mmHg difference in BP bet standing & supine positions.
. DUE TO DECREASED BARORECEPTORS RESPONSIVENESS !
.3. AORTIC STENOSIS - HYPERTROPHIC CARDIOMYOPATHY - ANOMALOUS CORONARIES:
__________________________________________________________________________
. Syncope with exertion or during exercise.
.4. VENTRICULAR ARRHYTHMIAS:
_____________________________
. Prior H/O of coronary artery disease, MI, cardiomyopathy or reduced EF.
.5. SICK SINUS $YNDROME - BRADYARRHYTHMIAS - ATRIOVENTRICULAR BLOCK:
_____________________________________________________________________
. Sinus pauses on monitor, prolonged PR interval or QRS duration. (vvv. imp.).

.6. TORSADES DE POINTES = ACQUIRED LONG QT $YNDROME:


_____________________________________________________
. Hypokalemia, hypomagnesemia, medications causing prolonged QT interval.
.7. CONGENITAL LONG QT $YNDROME:
_________________________________
. Family H/O of sudden death, prolonged QT interval on ECG.
. Syncope with triggers (exercise, swimming, sudden noise, during sleep).
.8. SITUATIONAL SYNCOPE:
_________________________
. Middle aged or older male with prostatic hypertrophy,
. who lost his consciousness after awakening & voiding at night.
. Pathophysiology -> Autonomic dysregulation.
.
.
.
.
.

Treatment of syncope is based on etiology.


Majority of cases never get a specific diagnosis.
The most important is to EXCLUDE cardiac etiology e.g. arrhythmia.
The majority (> 80%) of mortality from syncope involves a cardiac etiology.
If a ventricular dysrrhthmia is diagnosed -> Implantable cardioverter/defibrillator.

.
.
.
.
.
.
.

N.B. Elderly pts are very sensitive to fluid loss.


Even mild hypovolemia may predispose them to orthostatic hypotension & syncope.
A recent episode of diarrhea with -- fluid intake due to -- appetite is a risk factor.
Patients on diuretics therapy are liable to syncope due to hypovolemia.
Mucosal dryness is a sign of hypovolemia.
Mostly noticed upon getting up in the morning.
BUN/Creatinine ratio is the best indicator of dehydration.

.
.
.
.
.

N.B. Syncope may be due to pericadial tamponade.


Tamponade is due to viral pericarditis following an URT viral infection.
Hypotension, jugular venous distension & distant heart sounds.
Electrical alternans (Varying QRS amplitudes on EKG).
CXR -> Cardiac silhouette enlargement.

. COMPARISON BETWEEN SEIZURES & SYNCOPE:


_________________________________________
. P.O.C. _______

SEIZURES

* Circumstances: . Sleep loss.


. Emotions.
. Alcohol withdrawal.
. Flashing light.
* Clues:
.
.
.
.
.
* Sequelae

_______

SYNCOPE

. Upright position.
. Emotions.
. Heat.
. Crowded places.

. Aura (Olfactory hallucinations).


. Pre-syncope lightheadedness.
occur with sleeping or sitting position.. NO
Head deviations.
. Pallor & diaphoresis.
Tongue biting & laceration.
. Weak & slow pulse.
Rapid strong pulse.
Unusual body posturing.

. Delayed return to baseline.


. Post ictal state (sleepy confused).

. IMMEDIATE SPONTANEOUS RETURN.

.
.
.
.

N.B.
Clonic jerks may occur during any syncope if it is prolonged due to brain hypoxia.
Absence of previous H/O of seizure & presence of structural heart disease,
excludes seizure & makes a diagnosis of syncope due to arrhythmia more reliable.

. SYSTEMIC HYPERTENSION:
________________________
________________________
. Systolic blood pressure > 140 mmHg.
. Diastolic blood pressure > 90 mmHg.
. A diabetic pt or chronic renal disease pt > 130/80 mmHg.
. Hypertension is the most common risk factor for MI (Death most common risk factor).
. 95% of cases of HTN has no clear etiology "Essential hypertension".
. Commonest cause of essential HTN is RIGIDITY OF ARTERIAL WALL "Atherosclerosis".
. 5 % of cases has definite etiology (Mostly YOUNG population) -> "Secondary HTN".
. Causes of SECONDARY HYPERTENSION:
-> Renal artery stenosis "systolic diastolic abdominal periumbilical bruit".
-> Glomerulonephritis.
-> Coarctation of the aorta "Upper extremity BP > Lower extremity BP".
-> Acromegaly.
-> Pheochromocytoma "Episodic HTN - flushing - Palpitations - Headache - Part of MEN 2".
-> Hyperaldosteronism "++Na,--K, Weakness, Metabolic alkalosis,-- Renin,++ Aldosterone".
-> Cushing $ (Central obesity - Abdominal striae - Facial plethora).
-> Exogenous therapeutic glucocorticoids.
-> Congenital adrenal hyperplasia.
-> Hyperparathyroidism (HTN - Hypercalcemia - renal stones - Neuropsychiatric symptoms).
-> Oral contraceptive pills "Stoppage of OCPs intake may correct the problem".
. Dx -> ROUTINE OFFICE TESTS -> Urinalysis - Lipid profile - Glucose level - EKG.
. Tx -> Best initial therapy -> LIFE STYLE MODIFICATION:
_________________________________________________________
-> Obese pts -> Weight loss (Most effective).
-> Thin pts -> ++ fruit & vegetables = DASH DIET.
-> Sodium restriction.
-> Dietary modification (Less fat & red meat, More fish & vegetables).
-> Exercise.
-> Tobacco cessation does NOT stop HTN but important to prevent cardiovascular diseases.
-> Councel for reduction of alcohol intake.
. Tx -> DRUG THERAPY:
______________________
. The best initial drug therapy is THIAZIDE DIURETICS.
. If BP is very high on presentation (above 160/100), 2 medications should be given.
. If diuretics don't control BP, the most important next step in management is:
. ACEI - ARB - BB - CCB.

. Medications that are not considered as 1st or 2nd line therapy are:
-> Central acting alpha agonists (Alpha methyl dopa - Clonidine).
-> Peripheral acting alpha antagonists (Prazosin - terazosin - doxazosin).
-> Direct acting vasodilator (Hydralazine - Minoxidil).
. Compelling indications for specific drugs:
_____________________________________________
-> Coronary artery disease -> BB - ACEI - ARB.
-> Diabetes mellitus -> ACE - ARB.
-> Benign prostatic hyperplasia -> Alpha blockers (Prazocin).
-> Depression & Asthma -> Avoid BB.
-> Hyperthyroidism -> BB 1st.
-> Osteoporosis -> Thiazides.
-> Essential tremors -> BB.
.
.
.
.
.
.

N.B.
Benign essential tremor are tremors occuring with posture "movement".
They are unlike Parkinson's disease which is characterized by resting tremors.
It usually disturbs the fine motor tasks e.g. handling a newspaper or pouring tea.
Inhibition of the tremor by a small amount of alcohol is typical.
Tx -> Propranolol.

.
.
.
.
.

N.B.
Isolated systolic hypertension (ISH) is an important cause of hypertension in elderly,
it is due to DECREASED ELASTICITY OF THE ARTERIAL WALL.
It sh'd be ttt due to strong association with ++ risk of cardiovascular events.
Initial ttt -> Monotherapy with a low dose thiazide, an ACEI or long acting CCB.

.
.
.
.
.
.
.
.
.
.

N.B. 2ry hypertension due to Cushing's $,


++ cortisol -> ++ vasoconstriction, insulin resistance & mineralocorticoid activity.
++ Vasoconstriction -> Hypertension.
++ insulin resistance -> Hyperglycemia & weight gain.
++ mineralocorticoid activity -> Hypokalemia.
Proximal ms weakness, central adeposity, thinning of skin, weight gain are common.
Psychiatric problems (Sleep disturbances, depression & psychosis) are also common.
Cushing's $ may be due to ADRENAL CORTICAL (Not medullary) HYPERPLASIA,
ACTH producing pituitary adenoma (Cushing's disease), ectopic ACTH production,
or exogenous steroids.

.
.
.
.
.
.

N.B.
Renovascular hypertension
sh'd be suspected in all pts with resisant hypertension in addition to
diffuse atherosclerosis - asymmetric kidney size - recurrent flash pulmonary edema.
or elevation in serum creatinine > 30 % from baseline after starting ACEIs or ARBs.
The presence of continous abdominal bruit has a high specificity for renovascular HTN.

.
.
.
.
.
.
.
.

N.B.
POLYCYSTIC KIDNEY DISEASE (PKD) -> Autosomal dominant inheritance.
Bilateral cystic dilatation of the renal tubules.
Hypertension is one of the earliest manifestations.
Hematuria is often present.
Flank or abdominal masses with pain & 2ry eryhthrocytosis.
PKD is associated with cerebral aneurysms.
Family H/O of stroke or sudden death.

. Dx -> Abdominal U/$.


. HYPERTENSIVE CRISIS:
______________________
. Hypertensive crisis is not defined as a specific level of blood pressure !
. It is defined as hypertension associated with END-ORGAN DAMAGE.
. End organ damage = CONFUSION - BLURRY VISION - DYSPNEA - CHEST PAIN.
{1} HYPERTENSIVE URGENCY:
__________________________
. Severe hypertension (> 180/120 mmHg) .. BUT .. No symptoms or acute end organ damage.
{2} HYPERTENSIVE EMERGENCY:
____________________________
. Severe hypertension (> 180/120 mmHg) + Acute, life threatening end organ complications.
. Malignant hypertension -> Severe HTN + Retinal hemorrhages + exudates + Papilledema.
. Hypertensive encephalopathy -> Severe HTN + cerebral edema + Non localizing neuro signs
. The best initial therapy for hypertensive crisis is LABETALOL or NITROPRUSSIDE.
. INTRAVENOUS LABETALOL is preferred to Nitroprusside (Needs monitoring).
. Do NOT lower blood pressure in HTN crisis to NORMAL, or you may PROVOKE A STROKE.
. N.B. ANTI-HYPERTENSIVE DRUGS SIDE EFFECTS:
____________________________________________
. THIAZIDE DIURETICS "HYDROCHLOROTHIAZIDE":
____________________________________________
. Metabolic side effects -> Hyperglycemia, ++ triglycerides & ++ LDL cholesterol.
. Electrolyte side effects -> Hyponatremia, hypokalemia & hypercalcemia.
. DIRECT RENIN INHIBITOR:
__________________________
. Enhances natriuresis.
. -- serum angiotensin 2 concentration.
. -- Aldosterone production.

. N.B. CLUES FOR CAUSES OF SECONDARY HYPERTENSION:


__________________________________________________
* RENAL PARENCHYMAL DISEASE:
_____________________________
. ++ serum creatinine.
. Abnormal urinalysis (proteinuria & RBCs casts).
* RENO-VASCULAR DISEASE:
_________________________
. Severe hypertension (> 180 mmHg systolic & 120 mmHg diastolic) after age 55.
. Possible recurrent flash pulmonary edema or resistant heart failure.
. Unexplained rise in serum creatinine.
. Abdominal bruit.

* PRIMARY HYPERALDOSTERONISM = CONN's $:


_________________________________________
. Due to adrenal adenoma or bilateral adrenal hyperplasia.
. Easily provoked hypokalemia.
. Slight hypernatremia.
. Metabolic alkalosis (High serum BICARBONATE HCO3).
. ++ ALDOSTERONE & -- RENIN.
. Hypertension with adrenal incidentaloma.
* PHEOCHROMOCYTOMA:
____________________
. Paroxysmal ++ Blood pressure with tachycardia.
. Pounding headaches, palpitations & diaphoresis.
. Hypertension with adrenal incidentaloma.
* CUSHING's $YNDROME:
______________________
. Central obesity & facial plethora.
. Proximal muscle weakness.
. Ecchymosis, amenorrhea & erectile dysfunction.
. Hypertension with adrenal incidentaloma.
* HYPOTHYROIDISM:
__________________
. Fatigue, dry skin & cold intolerance.
. Constipation, weight gain & bradycardia.
* PRIMARY HYPERPARATHYROIDISM:
_______________________________
. Hypercalcemia (Polyuria & polydipsia).
. Renal stones.
. Neuropsychiatric (Confusion, depression & psychosis).
* AORTIC COARCTATION:
______________________
. Differential hypertension with brachial femoral pulse delay.

. MISCELLANEOUS TOPICS:
_______________________
_______________________
. NOREPINEPHRINE INDUCED VASOSPASM:
___________________________________
. Caused by vasopressors used in emergency bleeding episodes.
. Norepinephrine is an alpha 1 agonist causing vasoconstriction.
. Norepinephrine causes ischemia of the distal fingers & toes 2ry to vasospasm.
. SYMMETRICAL duskiness & coolness of all fingertips.
. A similar condition may occur in intestines (Mesenteric ischemia) or kidneys (RF).
. BLUE TOE $YNDROME:
____________________

. Due to cholesterol emboli in pts with atherosclerosis.


. Affect the distal portions of the digits.
. ABSENCE of symmetry of digital involvement.
. SUBCLAVIAN VEIN THROMBOSIS:
_____________________________
. Used for total parenteral nutrition.
. Continous irritating hyperosmolar fluid can traumatize the veins leading to thrombosis.
. Prolonged placement of central lines can lead to subclavian vein thrombosis.
. It results in arm swelling & pallor.
. CATHETER REMOVAL IS MANDATORY.
. Duplex is ordered to document thrombus & the need for anticoagulation.
. HEAT STROKE:
______________
. Temperature above 40.5 c (105 F).
. Exertional heat stroke occurs in healthy individuals exercising in extreme heat.
. Dehydration, hypotension, tachycardia & tachypnea are common.
. Systemic effects like seizures, ARD$, DIC & hepatic or renal failure may occur.
. COCAINE USE:
______________
. Atrophic nasal mucosa.
. Sympathetic hyperactivity (Tachycardia, hypertension, dilated pupils).
. Chest pain due to coronary vasospasm.
. Psychomotor agitation & siezures.
. Complications (Acute MI - Aortic dissection - Intracranial hemorrhage).
. Management of chest pain:
-> IV BENZODIAZEPINES for blood pressure & anxiety.
-> Aspirin.
-> Nitroglycerin & CCBs for pain.
-> BBs are CONTRAINDICATED.
-> Fibrinolytics are NOT PREFERRED due to ++ risk of intracranial hemorrhage.
-> Immediate cardiac catheterization with reperfusion when indicated.
. LATEX ANAPHYLACTIC SHOCK:
___________________________
. Exposure to latex-containing products such as surgical gloves & condoms.
. Development of hives after protected sex due to sensitization to latex condoms.
. Latex gloves used by surgeonsprovokes anaphylactic reaction with hypotension & rash.
. Health care workers & pts with atopic disease are at high risk of latex allergy.
. CHAGAS DISEASE:
_________________
. A potozoal disease caused by Trypanosoma cruzi.
. It is endemic to Latin America "Brazil".
. Two primary manifestations -> MEGACOLON/MEGAESOPHAGUS & CARDIAC DISEASE.
. Megacolon or megaesophagus (focal GI dilatation) is 2ry t destruction of the nerves,
. controlloing the gastrointestinal smooth muscles leading dialatation.
. Congestive heart failure occurs (Pedal edema - JVD - S3 gallop - cardiomegaly).
. ACUTE LIMB ISCHEMIA:
______________________
. Pain - Pulselessness - Paresthesia - Poikilothermia "Coldness" -& Pallor.

.
.
.
.

Angiography -> Abrupt cut-off of arterial blood flow.


IV Heparin sh'd be started immediately upon suspicion.
Definitive ttt -> Surgical embolectomy,
or Intra-arterial fibrinolysis & mechanical embolectomy.

. ARD$:
_______
. Acute onset.
. Bilateral patchy airsapce disease on CXR.
. PCWP < 18.
. No ++ in LVEDP.
. PaO2/FiO2 < 200.
. GASTRO-ESOPHAGEAL MURAL INJURY CHARACTERISTICS:
_________________________________________________
_________________________________________________
(A) MALLORY WEISS $YNDROME:
____________________________
. Upper gastro-intestinal MUCOSAL TEAR.
. Caused by forceful retching (++ pressure).
. Submucosal arterial or venule plexus bleeding.
. Vomiting, retching, hematemesis & epigastric pain.
. Dx -> EGD confirms diagnosis.
. Most tears heal spontaneously.
. Endoscopic therapy for continous bleeding.
(B) BOERHAAVE $YNDROME:
________________________
. Esophageal TANS-MURAL tear.
. Caused by forceful retching (++ pressure).
. ESOPHAGEAL AIR/FLUID LEAKAGE into nearby areas e.g. pleura.
. Vomiting, retching, chest & upper abdominal pain.
. Odynophagia, fever, dyspnea & septic shock may occur.
. Subcutaneous emphysema may be seen.
. Dx -> CT or CONTRAST ESOPHAGOGRAPHY with GASTROGRAFIN confirms diagnosis.
. CXR -> Pneumo-mediastinum & pleural effusion.
. Pleural fluid analysis -> EXUDATIVE, LOW pH, VERY HIGH AMYLASE > 2500 IU.
. Tx -> Surgery for thoracic perforations.
. Conservative measures e.g. antibiotics for cervical perforation.
. IMPORTANT AUSCULTATORY MEDIA Qs:
__________________________________
__________________________________
.
.
.
.
.
.

S3
Extra-sound heared just after S2.
(ken-tuc-"KY") -> With S3 corresponding to the last syllabus.
Result when inflow from Lt atrium strikes blood already in Lt ventricle.
It is a sign of Lt ventricular failure.
Best initial ttt of LVF is IV diuretics.

. S4
. Additional diastolic sound just prior to S1.
. ("TEN"-nes-see) -> With S4 corresponding to the first syllabus.

. It is indicator of a stiff left ventricle.


. Causes: prolonged systemic hypertension or restrictive cardiomyopathy.
. AR MURMUR
. Early diastolic murmur.
. Associated clinical finding -> Water hammer pulse = Bounding pulse.
. MR MURMUR
. Holo-systolic murmur.
. Heard best over the apex with radiation to the axilla.
. IMPORTANT KEY WORDS:
______________________
. FIXED SPLITTING od S2 = ASD.
. PULSUS PARADOXUS (Fall in BP > 10 mmHg with inspiration) = CARDIAC TAMPONADE.
. PULSUS PARVUS ET TARDUS (Week & delayed pulse stroke) = Aortic stenosis.
. OPENING SNAP with loud S1 = MS.
. CAPILLARY PULSATIONS in fingers & lips = AR.
. PERIPHERAL EDEMA DIFFERENTIAL DIAGNOSIS:
__________________________________________
__________________________________________
{1} CARDIAC:
_____________
. Bilateral.
. Congestive heart failure.
. Dyspnea & orthopnea.
. Jugular venous distension & hepatomegaly.
{2} HEPATIC:
_____________
. Bilateral.
. Hepatic cell failure.
. Ascites dominates over LL edema.
. Abnormal liver function tests (++ ALT & AST)
. Hypoalbunimea & hyperbilirubinemia.
. Spider nevi, gynecomastia, palmer erythema.
{3} RENAL:
___________
. Bilateral.
. Nephrotic $yndrome (Due to massive proteinuria).
. Nephritic $yndrome (Due to fluid retention).
. Proteinuria, hypoalbuminemia.
. Abnormal renal function tests (++ urea & creatinine).
{4} NUTRITIONAL:
_________________
. Bilateral.
. H/O of metabolic problems.
. Rare in adults.
{5} MEDICATIONS SIDE EFFECTS:

______________________________
. Bilateral.
. Ex. Dihydropyridine Ca channel antagonists (Amlodipine).
. Amlodipine dilate peripheral blood vessels.
{6} VENOUS INSUFFECIENCY:
__________________________
. Unilateral.
. Varicose veins.
. Skin changes & ulcers.
. N.B. TRICKY CASE:
___________________
. 53 ys old pt - 2 days H/O of Rt calf pain & swelling.
. Constant pain - exacerbated by knee flexion.
. H/O of past IV drug abuse, endocarditis & stroke.
. He is currently wheel-chair bound 2ry to stroke related Lt sided hemiparesis.
. BP 140/90 - HR 100/mon.
. No JVD - No hepato-jugular reflux.
. Clear chest on ausculatation.
. Enlarged abdomen with shifting dullness & fluid wave suggestive of ascites.
. Palpable liver 3 cm below the Rt. costal margin - splenomegaly.
. Rt sided calf swelling & tendrness.
. What is the cause of ascites ??
. Pulmonary embolism/paradoxical embolism/Rt sided HF/Chronic liver disease/Nephrotic $ ?
. The answer is -> CHRONIC LIVER DISEASE.
.
.
.
.
.

The pt's H/O of IV drug abuse ++ risk for cirrhosis 2ry to infection e' HBV & HCV.
The findings of hepatosplenomegaly & ascites point toward the diagnosis.
Most common cause of ascites is hepatic cirrhosis.
The pt's leg swelling may be due to DVT resulting from impaired mobility.
ASCITES is NOT related to DVT !!

.
.
.
.

N.B. Hepato-jugular reflux:


A useful clinical tool that can differentiate bet. cardiac & liver causes of LL edema.
Cardiac cause (Heart failure) -> ++ Jugular venous pressure & +ve Hepatojugular reflux.
Hepatic cause (Liver cirrhosis) -> -- or Normal JVP & -ve Hepatojugular reflex.

.
.
.
.
.

N.B.
Chronic venous insuffeciency is a common cause of peripheral edema.
Sh'd be suspected in pts with isolated lower limb edema & or dilated veins,
with otherwise normal physical examination.
Initial ttt -> Conservative measures with leg elevation, exercise & compression therapy

. PATHO-PHYSIOLOGY of CAUSES of PERIPHERAL EDEMA:


_________________________________________________
. ++ CAPILLARY HYDROSTATIC PRSSURE:
____________________________________
. Heart failure (Lt ventricular & cor pulmonale).
. Primary renal Na retention (Renal diseases & drugs).
. Venous obstruction (Cirrhosis & venous insuffeciency).

. -- CAPILLARY ONCOTIC PRESSURE = HYPOALBUMINEMIA:


___________________________________________________
. Protein loss (Nephrotic $).
. -- Albumin synthesis (Cirrhosis & Malnutrition).
. ++ CAPILLARY PERMEABILITY:
_____________________________
. Burns, trauma & sepsis.
. Allergic reactions.
. ARD$.
. Malignant ascites.
. ++ INTERSTITIAL ONCOTIC PRESSURE = LYMPHATIC OBSTRUCTION:
____________________________________________________________
. Malignant ascites.
. Hypothyroidism.
. Lymph node dissection.
. PARAMETERS OF DIFFERENT TYPES OF SHOCKS:
__________________________________________
__________________________________________
. TYPE OF SCOCK __ COP __ SVR __ BP __ HR __ PCWP(CVP) __ Tx
. CARDIOGENIC __ - __ + __ - __ + __ +

__ TTT cardiac problem

. HYPOVOLEMIC __ - __ + __ - __ + __ -

__ Fuids & pressors

. SEPTIC

__ + __ - __ - __ + __ -

. NEUROGENIC

__ - __ - __ - __ + __ -

__ Fluids, pressors & Antibiotics


__ Fluids & pressors

. BP -- in all types of shock.


. HR ++ in all types of shock.
. PCWP ++ only in cardiogenic shock.
. COP ++ only in septic shock.
. SVR ++ in hypovolemic & cardiogenic shocks.
. Skin is pale & cool in hypovolemic & cardiogenic shocks.
. Skin is warm & faint in septic shock.
.
.
.
.

The
The
The
The

most
most
most
most

common
common
common
common

cause
cause
cause
cause

of
of
of
of

cardiogenic shock is myocardial infarction.


hypovolemic shock is massive hemorrhage.
septic shock is E-coli & staphylococcal infection.
nerogenic shock is cervical or thoracic spinal cord injury.

Dr. Wael Tawfic Mohamed


_________________________

DERMATOLOGY TIKI TAKA


----------------------. CELLULITIS:
-----------. Cellulitis with systemic manifestations e.g. fever,rigors,chills & confusion is ttt by
I.V.
NAFICILLIN or Cefazolin.
. caused by staph or strept.
. Generalized swelling which is erythematous "linear streaks", warm, tender but less well
demarcated than Erysipelas.
. An associated fungal infection may acts as a portal of entry.
. Tinea Corporis:
----------------. Ring shaped scaly patches with central clearin & scaly borders.
. Dx: KOH -----> Hyphae. . Tx: Local Terbinafine or systemic Griseofluvin.
. Tinea Versicolor:
------------------. Pale velvety pink or whitish hypopigmented macules that DON'T TAN !
. SCALE ON SCRAPING.
. Dx: KOH preparation ----> Spaghetti & meat ball appearance.
. Tx: Selenium sulfide.
. NECROTIZING FASCIITIS:
-----------------------. Severe pain & swelling.
. H/O of recent trauma.
. High fever > 39 c.
. Edematous limb with PURPLISH DISCOLORATION of the injured area "denoting start of
gangrene!".
. Surgical debridement of all necrotic tissue.
. Empiric IV Antibiotics e.g AMPICILLIN + SULBACTAM + CLINDAMYCIN.
. Bullae & seroanguinous discharge.
. Seborrheic dermatitis:
-----------------------. Fine loose waxy scales with underlying erythema.
. On scalp, eye brows.
. Associated with HIV or parkinsonism.
. PRIMARY BILIARY CIRRHOSIS:
---------------------------. Pruritis, jaundice, steatorrhea, HSM, ++ ALP, ++ Bilirubin.
. +ve Anti-mitochondrial Antibodies.
. Immune mediated destruction of intra hepatic bile ducts ---> Bile stasis & cirrhosis.
. Cutaneous association ---> XANTHELASMA
"Yellowish, soft plaques on the medial aspects of the eyelids bilaterally".
. CHALAZION:
-----------. Painful swelling that progresses to a nodular rubbery lesion.

. due to MEIBOMIAN gland obstruction.


. Recurrent chalazion may be due to meibomian gland carcinoma !
. U can't differentiate bet. PERSISTENT CHALAZION & BASAL CELL CARCINOMA except through
HISTOPATHOLOGICAL exam.
. MOLLUSCUM CONTAGIOSUM is caused by POX VIRUS not HPV !!!!!!! (REPEATED FAULT) !!
. BASAL CELL CARCINOMA:
----------------------. Fair skinned individual.
. Prolonged sun exposure.
. Slowly growing nodule with rolled border.
. SHINY PEARL.
. Most common location is the lower eyelid margin.
. Seborrheic Keratosis:
----------------------. OLD AGE.
. WAXY - "STUCK ON" - well circumscribed lesion.
. Not pre-cancerous.
. No therapy is required.
. Surgical removal for cosmetic purpose.
. MELANOMA -----------> Excisional biopsy " FULL THICKNESS".
. Varicella Zoster virus = Shingles:
-----------------------------------. Vesicular eruption that occurs in a dermatomal distribution.
. Preceided by pain.
. The 1ry disease in children is termed "chickenpox".
. ANGIO-EDEMA:
-------------. H/O of ICU pt on ACEIs e.g ENALAPRIL.
. Edema in the face, mouth, lips.
. Laryngeal edema may occur causing airway obstruction.
. occurs due to BRADYKININ release.
. it may occur at any time not just at the start of drug intake.
. Dx----> Low levels of C2 & C4.
. Tx----> STOP ACEIs + FRESH FROZEN PLASMA + Secure the airway.
. HERIDITARY angioedema:
-----------------------. C1 esterase inhibitor defeciency.
. Drug induced PHOTOTOXICITY:
----------------------------. The most common drug is DOXYCYCLINE (TETRACYCLINE).
. Manifest as exaggerated sunburn reactions with erythema ,edema & vesicles over sunexposed areas.
. WARFARIN induced skin necrosis:
--------------------------------. More common in females.
. Common sites: Breasts, buttocks, thighs & abdomen.

. Initial complaint is pain followed by bullae formation & skin necrosis.


. Occurs within weeks after starting therapy.
. Tx: Discontinue WARFARIN & Give Vit. K & maintain anticoagulation using Heparin.
. Dermatitis Herpetiformis:
--------------------------. Pruritic papules & vesicles over the extensor surfaces.
. Presence of anti-endomysial antibodies.
. Tx----------------> DAPSONE.
. Dermatitis Herpetiformis:
--------------------------. Ass. with celiac disease.
. Erythematous papules, vesicles & bullae that occur bilaterally, symmetrically & in
"herpetiform" arrangement.
. On the extensor sufraces of the elbows,knees,buttocks.
. Tx: Gluten free diet & DAPSONE.

groups

. Pemphigus Vulgaris:
--------------------. Thin & fragile large wide bullae.
. +ve Nikolsky sign.
. Mouth is involved.
. H/O of ACE Is use.
. Deposition of IgG in the epidermis.
. Bullous Pemphigoid:
--------------------. Thick & intact small & narrow bullae.
. Bullae don't rupture easily.
. No mouth involvement.
. H/O of SULFA drugs use.
. Deposition of IgG & C3 in the epidermis.
. ROSACEA:
---------. 30 - 60 ys old pt.
. TELANGECTASIA over the cheeks, nose & chin.
. Flushing of these area is precipitated by hot drinks,heat,emotion.
. Tx: initial ttt is METRONIDAZOLE.
. Toxic Epidermal Necrolysis "TEN":
---------------------------------. Much more surface area involved.
. Higher mortality rate.
. +ve NIKOLOSKY sign.
. Although the most common cause of death is sepsis, Anitibiotics is not indicated.
. TEN has similar features to SSSS, however, TEN is DRUG INDUCED but SSSS is caused by a
toxin.
. Vilitiligo (Leukoderma):
-------------------------. Young 20-30 ys.
. Pale whitish macules with hyperpigmented borders.
. Around body orifices.

. Auto-immine destruction of melanocytes.


. STEVENS JOHNS $YNDROME:
------------------------. Immune complex mediated hypersensitivity.
. H/O of SULFONAMIDES, NSAIDs & PHENYTOIN intake.
. Characteristic "TARGET" appearance.
. Fever, conjunctivitis, ++HR, --BP, altered consciousness, coma, convulsions may occur.
. RUBELLA:
--------. Middle aged female.
. Maculo-papular rash starting on the face & extends to involve the trunk & extremeties
involving th palms & soles).
. Tender lymphadenopathy (Post. auricular & post. cervical LNs).
. Poly-arthritis.
. Secondary $yhphilis:
---------------------. Maculopapular rash (involving the palms & soles).
. The papules may coalese to form CONDYLOMA LATA in severe cases!
. NICKEL jewelry can cause allergic contact dermatitis (Type 4 hypersensitivity).
. Frost-bite injury:
-------------------. Rapid re-warming with warm water.
. Dead Tissue debridement is WRONG.
. Rapid re-warming with fry heat or fan is WRONG.
. SQUAMOUS CELL CARCINOMA:
-------------------------. Non-melanoma skin cancer.
. Second most common skin cancer after Basal cell carcinoma.
. Aggressive due to distant mateastasis.
. EXPOSURE to SUN LIGHT is the most imp. risk factor.
. MILD ACNE & NON-INFLAMMATORY COMEDONES -----> Topical retinoids.
. MILD INFLAMMATORY ACNE ---------------------> Topical benzyl peroxide.
. MODERATE to SEVERE ACNE (NODULO-CYSTIC) ----> ORAL ISOTRETINOIN.
. GRAFT VERSUS HOST DISEASE (GVHD):
----------------------------------. in pts with bone marrow transplantation.
. due to activation of the DONOR "T" lymphocytes.
. Skin ---> Maculopapular rash.
. Intestine ---> Bloody diarrhea.
. Liver ---> Abnormal LFTs & jaundice.
. Drug induced type 1 hypersensitivity reaction:
----------------------------------------------. IMMEDIATE ONSET.
. Mediated by IgE & Mast cells.
. Urticaria & pruritis without systemic symptoms.
. Tx: ANTI-HISTAMINICS & dis-continue the offending drug !

(Not

. MELANOMA criteria (ABCDEs):


----------------------------. Assymetry.
. Border irregularities.
. Color variation.
. Diameter > 6mm.
. Evolving: lesion changing in size, shape or color; new lesion.
. The most concerning sign for malignancy is ZONES OF DIFFERENT SKIN COLORS !
. PORPHYRIA CUTANEA TARDA:
-------------------------. photosensitivity reaction to accumulating porphyrins.
. Painless blisters on the dorsum of the hand.
. Hypertichosis on the face & Hyperpigmentation.
. H/O of liver disease (HCV) or OCP use.
. Dx: Urinary uroporphyrins.
. Tx: Phlebotomy & Deferoxamine.
. SQUAMOUS CELL CARCINOMA:
-------------------------. isolated solitary ulcer.
. in the Vermilion area of the lip.
. H/O of sun exposure (FARMER).
. Histologically: INVASIVE CORDS OF SQUAMOUS CELLS WITH KERATIN PEARLS.
. BASAL CELL CARCINOMA:
----------------------. INVASIVE CLUSTERS OF SPINDLE CELLS SURROUNDED BY PALISADED BASAL CELLS.
. CHERRY HEMANGIOMA:
-------------------. Small vascular bright red papular lesion.
. 30-40 ys & ++ in no with age "Senile hemangioms".
. Don't regress spontaneously.
. Sharply circumscribed areas of congested capillaries.
. ACTINIC KERATOSIS:
-------------------. Erythematous papule with a central scaling.
. Sand paper like texture.
. H/O of chronic sun exposure.
. Pre-cancerous ----> may convert to squamous cell carcinoma.
. Molluscum Contagiosum (Pox virus):
-----------------------------------. Firm, flesh colored, dome-shaped, umbilicated papules.
. Transmitted through sexual contact.
. Due to CELLULAR immunodefeciency.
. Associated with HIV.
. SHINGLES (HZV) may develop due to "INFLIXIMAB" therapy causing immunodefeciency.
. Allergic contact dermatitis:

-----------------------------. Type 4 hypersensitivity reaction.


. Prurutic erythematous rash with vesicles.
. Bilateral distribution.
. H/O of cutting woods (Poison Sumac).
. Vesicular fluid is sterile and grows coagulase -ve staphylococci (S. Epidermidis).
. May be 2ry infected staph or strept !
. ACANTHOSIS NIGRICANS:
----------------------. Symmetrical, hyperpigmented, velvety plaques in the axilla, groin & neck !
. Ass. with INSULIN RESISTANCE in YOUNG pts e.g. DM & PCO.
. Ass. with GIT malignancy in OLD pts.

ENT
----. TEMPORO-MANDIBULAR JOINT DYSFUNCTION:
--------------------------------------. H/O of teeth grinding at night.
. Referred pain to ear.
. Worse with eating.
. Peri-tonsillar abscess:
------------------------. Muffled voice make one consider other diagnosis than simple tonsillitis or pharyngitis. .
Deviation of the Uvula + Unilateral lymphadenopathy = Peri-tonsillar abscess.
. Tx: URGENT NEEDLE ASPIRATION + IV Antibiotics.
. PRESBYCUSIS:
-------------. Old pt in 60s.
. Sensori-neural hearing loss.
. HIGH frequency BILATERAL hearing loss.
. Difficult hearing in noisy crowded places.
. OTOTOXIC drugs:
----------------. Aminoglycosides antibiotics.
. Loop diuretics e.g. Furosemide. NOT THIAZIDEs !!
. Aspirin.
. REMEMBER:
----------. SE of BBs -------> Bronchoconstriction - Bradycardia - fatigue - depression.
. SE of ACEIs -----> Cough - Hyperkalemia - Angioedema.
. SE of Aspirin ---> Tinnitus.
. SE of Thiazides -> Orthostatic Hypotension - photosensitivity - hypercalcemia.
. Serous O.M.
------------. is associated with HIV pts manifesting as middle ear effusion without infection.
. causing dull hypomobile tympanic membrane.
. NASAL POLYP:
-------------. is associated with chronic rhino-sinusitis, asthma.
. H/O of aspirin or NSAIDs induced broncho-spasm
. (ASPIRIN EXACERBATED RESPIRATORY DISEASE).
. Symptoms of bilateral nasal obstruction, nasal discharge & anosmia.
. LEKOPLAKIA:
------------. Hard to remove white patches in the floor of the mouth.
. due to chronic irritation by smoking or alcohol.
. May lead to squamous cell carcinoma.

. CANDIDIASIS:
-------------. Diabetic pt. with poor control.
. Whitish plaques with underlying erythema.
. Easily scrapped off with a tongue depressor.
. Epiglottitis:
--------------. High fever.
. Severe sore throat.
. Odynophagia.
. DROOOOOOOOOOOOOOOOLING.
. Progressive airway obstruction.
. HARSH SHRILL.
. Causative organisms: Haemophilus influenzae & Streptococcus pyogenes.
. Retro-pharyngeal space is the most common neck space susceptible to infection.
. Otosclerosis:
--------------. is the most common cause of conductive hearing loss in middle aged adults 20-30s.
. AMINOGLYCOSIDES - GENTAMYCIN is the most common cause of sensorineural hearing loss.
. Retro-pharyngeal abscess:
--------------------------. Fever & sore throat.
. Dysphagia & Odynophagia.
. Trismus (pain on mouth opening).
. Pain on neck EXTENSION (pain on neck flexion = Meningitis).
. H/O of local trauma to the pharynx e.g. FISH BONE.
. MENIERE's disease:
-------------------. Vertigo = Severe spinning sensation + nausea.
. The type of vertigo is peripheral not central as it last just for 1-2 hours with the
presence
of ear fullness & H/O of excessive cell phone use.
. EAR FULLNESS suggests Meniere's disease
. from an abnormal accumulation of endo-lymph within the inner ear.
. Meniere's dis. is an INNER ear disease.
. Simply exclude CEREBELLAR dis. by absence of inco-ordination or gait disturbances!
. MALIGNANT OTITIS EXTERNA:
--------------------------. caused by PSEUDOMONAS AERUGINOSA.
. Elderly with poorly controlled DM.
. Ear pain, discharge.
. GRANULATION TISSUE within the ear canal on otoscopy.
. Progression to the base of the skull may damaga the facial nerve causing facial palsy. . Dx:
CT skull base.
. Tx: IV CIPROFLOXACIN. not surgery !!
. Meniere's disease:
-------------------. Vertigo + Ear fullness + Hearing loss.
. 1st line ttt ----> LOW SALT DIET.

. ASSESSMENT a case of hearing loss:


-----------------------------------. A louder tone is heard when a TF is placed on the pt's Rt mastoid process
& a softer tone when the TF is placed near her Rt. external auditory meatus
-> Abnormal Rinne
-> CONDUCTIVE hearing loss
-> bec. the diseased ear hears better due to obscuring of the external noise.
. When the TF is placed on the middle of the forehead, she feals the vibration better in
ear than the Lt
-> Abnormal Weber test
-> The sound lateralizes to the diseased ear due to better bone conduction.
-> CONDUCTIVE hearing loss.
. APHTHOUS ULCERS:
----------------. CROHN's disease can involve any part of the GIT from the MOUTH to the ANUS.
. APTHOUS ulcers in the mouth can be seen as extra-intestinal manifestation.
. Pathology: GRANULOMATOUS inflammation.

her Rt

GASTROENTEROLOGY TiKi TaKa


---------------------------. U should exclude MI with AS in a pt with epigastric pain radiating to the mid-scapulae.
---------------------------------------------------------------------------------------. When u r given a complaint of acute epigastric pain,
. radiating to the back inbet. the scapulae,
. with H/O of coronary artery disease,
. in addition to suspicion in pancreatitis & PUD,
. the 1st step to do is EKG to exclude MI !!!!!!!!!
. Chronic mesenteric ischemia:
---------------------------. Un-explained chronic abd. pain.
. weight loss.
. Food fear & avoidance of eating.
. Associated atherosclerotic disease.
. Abd. ex. may reveal a bruit.
. Dx: Doppler U/S.
. Mallory Weiss $:
----------------. is hematemesis due to ++ intra-abd. pressure with vomiting,
. leading to rupture of submucosal ARTERIES at the distal esophagus.
. Portal hypertension:
--------------------. leads to hematemesis due to rupture of esophageal varices (submucosal VEINS).
. Diverticulitis:
---------------. LLQ pain + Constipation + Fevr + vomiting.
. The most appropriate test to confirm acute diverticulitis is Abd. CT.
. Sigmoidoscopy is contra-indicated for fear of perforation.
. ULCERATIVE COLITIS:
-------------------. Young pt.
. Abd. pain.
. Bloody diarrhea.
. Rectal urgency.
. Anemia & reactive thrombocytopenia.
. Rectal tenderness.
. Stool mixed with mucous & blood.
. ++ WBCS with nausea ----> Toxemia.
. U should suspect TOXIC MEGACOLON.
. DO ABDOMINAL X-rays to search for Toxic Megacolon.
. Chron's disease:
-----------------. Young pt.

.
.
.
.
.
.
.

Chronic diarrhea.
Abd. pain.
Weight loss.
Mouth ulcers.
Rt upper Q. tenderness without rebound.
Gas in small & large intestines.
Reactive thrombocytosis & anemia.

. DIVERTICULOSIS:
---------------. Old pt.
. is associated with constipation not diarrhea,
. with Left lower Q. pain.
. CANCER HEAD PANCREAS:
----------------------. Enlarged (NON TENDER) Gall bladder.
. Weight loss.
. evidence of biliary obstuction (++ALP disproportionate with +AST & +ALT).
. Dx: Abd. CT !
. The most common cause of iron defeciency anemia in an elderly pt is GIT bleeding.
----------------------------------------------------------------------------------. The next step is COLONOSCOPY.
. A single -ve occult blood test doesn't exclude GIT bleeding.
. DYSPHAGIA:
-----------. Both sloids & liquids = Motility disorder e.g. ACHALASIA.
. Solids then progressing to liquids = Obstructing lesion e.g. esophageal
so .. You should perform BARIUM SWALLOW 1st before endoscopy.

adenocarcinoma. .

. Spontaneous bacterial peritonitis:


-----------------------------------. should be considered in any pt. with cirrhosis & ascites,
. accompanied by fever or change in mental status.
. Paracentesis is the test of choice, with a +ve ascitic fluid cultue & PMN > 250 cells.
. Step wise approach of ttt of Ascites:
--------------------------------------1. Sodium & water retention.
2. Spironolactone.
3. Loop diuretic (Furosemide).. But not more than 1 L/day of diuresis.
4. Frequent abd. paracentesis (2-4 L/day).
. Aggressive diuresis > 1 L/day may worsen encephalopathy or precipitate hepato-renal $.
. Liver cirrhosis ----> Renal hypo-perfusion -----> Hepato-Renal $ !
------------------------------------------------------------------. manifested by HIGH UREA & CREATININE.
. Very low urine Na < 10 meq = Pre-renal cause.
. No protein & No blood in dipstick urine = Not intrinsic glomerular cause.
. CARCINOID $ triad:
-------------------

.
.
.
.
.
.
.
.

Flushing.
Valvular heart disease.
Diarrhea.
Ass. e' hepatic metastasis.
++ Serotonin & 5 HIAA in blood & urine.
The precursor of Serotonin is Tryptophan,
which is also utilized in the synthesis of Niacin.
-- Niacin ----> PELLAGRA 4 Ds (Diarrhea-Dermatitis-Dementia-Death).

. Neutrophilic cryptitis is seen on bowel biopsy in pts with IBD.


. CARCINOID $ = Flushing + secretory diarrhea + Wheezing + cardiac problem.
. Intra-abdominal malignancy obstructing the biliary system:
-----------------------------------------------------------. Painless jaundice.
. Conjucated hyper-bilirubinemia.
. Elevated Alkaline phosphatase.
. ex. pancreatic adenocarcinoma.
. Pancreatic choleraa = VIPoma.
------------------------------. The pancreas secretes vaso-active intestinal peptide.
. Hypokalemia (leg cramps).
. chronic diarrhea --> dehydration.
. Abd. pain.
. weight loss.
. facial flushing & redness.
. Dx: Abd. CT.
. FOLIC ACID DEFECIENCY:
-----------------------. A tea & toast type of diet is associated with folic acid defeciency.
. Folic acid is heat sensitive.
. Folic acid defeciency causes macrocytic anemia.
. Corn based diets -> NIACIN defeciency -> PELLAGRA (4Ds):
---------------------------------------------------------. Diarrhea + Dementia + Dermatitis + Death.
. DIVERTICULITIS:
---------------. H/O of costipation & little fiber in diet.
. Left lower quadrant pain & fever.
. Tx: IV Antibiotics.
. If no improvement ---> Abd. CT to detect complications.
. Tropical sprue:
----------------. Endemic tropical area e.g. Puertorico.
. Biopsy of S.I ---> Blunting of villi,
. with infiltration of chronic infl. cells e.g. lymphocytes, plasma cells & eosinophils. .
Malabsorption to Vit. B12 & folic acid ---> Megaloblastic anemia.
. Other signs of malabsorption e.g. glossitis - cheilosis - pallor.

. WHIPPLE's disease:
-------------------. PAS +ve material in the lamina propria of the small intestine is diagnostic.
. Primary HIV infection:
-----------------------. can present with a mononucleosis like syndrome,
. consisting of fever, night sweats, lymphadenopathy, arthralgia & diarrhea.
. JAUNDICE:
----------.Abd. U/S is the best initial investigation for JAUNDICE.
.But .. once u suspect pancreatic cancer , then the best inv. is Abd. CT.
.Manif. of cancer include weight loss , pressure obstruction of CBD,
.leading to ++ direct bilirubin & ++ ALP.
. MEN 1 = 3 Ps:
--------------. Primary Hyperparathyroidism.
. Pituitary tumors.
. Pancreatic tumors (insulinoma-gastrinoma-VIPoma).
. GASTRINOMA = ZOLLINGER ELLISON's $ (Non B-cell pancreatic tumor).
. Endoscopy--> Multiple ulcerations & prominent gastric folds.
. MEN 2 A:
---------. Medullary thyroid cancer & Pheochromocytoma.
. MEN 2 B:
---------. Neuromas & Marfanoid habitus & Pheochromoytoma.
. Pts aged > 55ys with new onset dyspepsia with ALARM SYMPTOMS: -------------------------------------------------------------* weight loss.
* dysphagia.
* persistent vomiting.
* should be evaluated with UPPER ENDOSCOPY.
. Pts < 55ys with no alarm symptoms:
-----------------------------------. should have 1st H.Pylori serology test,
. followed by empiric ttt with PPIs e.g. Omeprazole.
. If failed ----> ENDOSCOPY.
. Hemorrhage is the most common complication of peptic ulcer.
. CRYPTOSPORIDIUM PARVUM:
------------------------. HIV pt. with chronic severe diarrhea with CD4 cells < 100.
. ZOLLINGER ELLISON's $YNDROME:
------------------------------. Endoscopic findings of prominent gastric folds.
. Chronic duodenal ulcer.

. upper jejunal ulcer.


. Serum GASTRIN conc. < 1000 is diagnostic.
. Bacterial overgrowth:
----------------------. Malabsorption in a pt. with a H/O of abdominal surgery.
. Vit. D def. = Hypocalcemia.
. Vit. A def. = Night blinness.
. Vit. B12 def. = Neuropathy.
. LACTOSE INTOLERANCE:
-------------------. Asian American.
. +ve Hydrogen breath test.
. +ve stool test for reducing substance.
. ++ stool osmotic gap.
. -- stool pH.
. No steatorrhea.
. ESOPHAGUS:
---------. ADENO-carcinoma ------------> Chronic GERD & Barret's esophagus.
. Squamous cell carcinoma ----> Smoking & Alcohol.
. Acute Appendicitis:
------------------. VS-VS-VS-VS-VS-VS-VS-VS-VS-VS-VS-VS Visceral followed by somatic pain !!
. ACUTE EROSIVE GASTRITIS:
-------------------------. Massive doses of Aspirin & NSAIDS can cause upper GI bleeding.
. Alcohol can aggravate its effect.
. Mallory Weiss $:
-----------------. occur in the distal esophagus at the gastro-esohageal junction,
. after repeated bouts of retching & vomiting.
. Zinc defeciency:
-----------------. may result from total parenteral nutrition or malabsorption.
. Alopecia,skin lesions,abnormal taste,impaired wound healing.
. Drug induced pancreatitis:
---------------------------. Pts with H/O of VALPROIC ACID ttt for seizure disorder.
. Ulcerative colitis:
--------------------. presents as diarrhea & bloody stools.
. The condition may be complicated by systemic toxicity : fever & weight loss,
. with dilated colon on CXR "TOXIC MEGA-COLON".
. Tx: I.V. fluids + Antibiotics + Bowel rest + I.V. corticosteroids.
. If failed: Emergency surgery with sub-total colectomy with end ileostomy.

. Minimal bleeding per rectum or scant hematochezia:


---------------------------------------------------. Dx -> Office based ANOSCOPY or PROCTOSCOPY.
. GIARDIASIS:
------------. Foul smelling stool.
. Abd. cramps.
. Bloating = MALABSORPTION diarrhea.
. H/O of developing country e.g. South America.
. Tx: METRONIDAZOLE.
. MALIGNANT criteria of a colonic polyp:
--------------------------------------. Villous adenoma.
. Sessile adenoma.
. Size > 2.5 cm.
. MULTIPLE MYELOMA:
------------------. Back pain + Renal dysfunction + High ESR + Anemia.
. MM ----> ++ Ca Hypercalcemia.
. ++ Ca ----> Constipation.
. so .. The cause of constipation in a pt. with MM is ELECTROLYTE DISTURBANCE (++ Ca).
. NON-CASEATING GRANULOMA ----> PATHOGNOMONIC to CHRON's disease !
. N.B. UC always involves the rectum while it is spared in CD.
. Angiodysplasia:
---------------. Pt. > 60 ys. with anemia.
. Painless GIT bleeding.
. Murmur of Aortic stenosis.
. DIVERTICULOSIS:
---------------. The most common cause of painless GIT bleeding.
. Not associated with AS.
. Endoscopy: Multiple out-pouchings of the mucosa through the hypertrophied muscular
layer.
. ZOLLINGER ELLISON's $:
-----------------------. Multiple duodenal ulcers + Single jejunal ulcer resistant to H2 blockers & PPIs.
. GASTRIN producing PANCREATIC TUMOR.
. Un-controlled gastrin production
. ---> Parietal cell hyperplasia
. ----> +++ Stomach acid production
. ----> Inactivation of pancreatic enzymes
. ----> Steatorrhea.
. Inflammatory Bowel disease:
----------------------------. Bloody diarrhea + anemia + elevated ESR + Reactive thrombocytosis.

. The type of diarrhea is INFLAMMATORY.


. Newly diagnosed gastric carcinoma transformation in a gastric ulcer by an endoscopy
warrants an abdominal CT scan to evaluate the extent of the cancer.
. Pharyngo-esophageal (ZENKER's) diverticulum:
---------------------------------------------. is due to motor dysfunction.
. Pt. < 50 ys.
. with oro-pharyngeal dysphagia & neck mass.
. Tx: Crico-pharyngeal Myotomy.
. Diffuse esophageal spasm:
-------------------------. Young female.
. intermittent episodes of chest pain & dysphagia.
. Ba swallow: Cork screw esophagus.
. Causes of ++ BUN / Creatinine ratio:
-------------------------------------. Pre-renal RF.
. GIT bleeding due to reabsorption of blood from the GIT.
. Steroid adminstration.
. Esophageal dysmotility due to SCLERODERMA:
-------------------------------------------. Sticking sensation in the throat.
. Significant -- in LES tone.
. Absence of peristaltic waves in the lower 2/3s of the esophagus.
. CHRONIC PANCREATITIS:
---------------------. H/O of ALCOHOL use.
. Epigastric chronic abd. pain.
. Malabsorption (Steatorrhea & chronic diarrhea).
. Weight loss.
. Type 2 D.M.
. Amylase & Lipase may be normal (Not diagnostic).
. Dx: Abd. CT showing pancreatic calcifications.
.Duodenal ulcers:
-----------------. typically presents with epigastric pain that improves with eating.
. OVER 90% of duodenal ulcers are infected with H. Pylori.
. Tx of H. Pylori ass. ulcers is acid suppression & organism eradication with
line regimen is OCA = OMEPRAZOLE + CLARITHROMYCIN + AMOXICILLIN.
. In upper GIT bleeding:
---------------------. If Hb < 10 ----> PACKED RBCs transf.
. If Ht < 30 ----> PACKED RBCs transf.
. Upper bleeding with coagulopathy:
----------------------------------. is most likely to ruptured gastro-esophageal varices.

antibiotis. . 1st

.
.
.
.
.

Tx: FRESH FROZEN PLASMA: bec.FFP contains all the clotting factors & plasma proteins.
We never use whole blood transfusion.
Cryoppt contains Factor 8, Fibrinogen, Von Willebrand factor & factor 10.
FFP is preferred to cryoppt as it contains all clotting factors.
Platelet transfusion is done when platelets < 50,000 !!

. Achalasia Manometry:
--------------------. esophageal body peristalsis.
. Failure of relaxation of LES.
. Achalasia Ba Swallow:
---------------------. Dilated esophagus.
. Bird's beak deformity of LES.
. HERPES ZOSTER (SHINGLES):
-------------------------. Pt. with Rt. sided abd. pain.
. Light touch to the skin to the Rt. of the Umbilicus elicits intense pain.
. Immunocompromized pt. 2ry to chemotherapy.
. Digoxin side effects:
---------------------. GIT-------> Anorexia, Nause & vomiting.
. Cardiac---> Biventricular Arrhythmia.
. VERAPAMIL ++ the Digoxin's toxicity.
. N.B. Mesenteric ischemia presents with severe abd. pain out of prop. to exam. !!
. Ulcerative colitis:
-------------------. Bloody diarrhea + tenesmus + abd. cramps + weight loss + anemia.
. Extraitestinal manifestations:
-----------------------------.Sclerosing colangitis.
. Uveitis.
. Erythema nodosum.
. Spondyloarthropathy.
. Complications:
-------------. Toxic Megacolon.
. Colon cancer.
. Yearly colonoscopies is recommended for pts with UC,
. beginning 8-10 ys after diagnosis for prevention of cancer colon.
. Irritable bowel $yndrome:
--------------------------. Abd. pain + diarrhea and/or constipation.
. Endoscopy ---> NORMAL COLONIC MUCOSA!
. Dark melanotic stools = Upper GI bleeding.
-------------------------------------------. The most common cause of upper GI bleeding is PUD.
. Duodenal ulcer's pain gets better with eating.
. while Gastric ulcer worsens with eating.

. PEPTIC STRICTURE:
------------------. Slowly progressive dysphagia to solids without anorexia & weight loss.
. As stricture progresses , it can actually block reflux,
. leading to improvement of heart burn symptoms.
. ENDOSCOPY --> SYMMETRIC circumferential narrowing.
. ADENO-CARCINOMA:
---------------. Pt with GERD < 20 ys.
. Weight loss.
. ASYMMETRIC narrowing of the esophageal lumen.
. REMEMBER:
----------. Pt. with fever + chills + Lt upper Q. pain + splenic fluid collection
. = Lt. sided endocarditis with septic emboli to the spleen,
. causing splenic abscess.
. H/O of incarerated pt. with ++ liver enzymes (possible HCV),
. suggesting IV drug use as the cause of infective endocarditis.
. NERD FAULT !!
--------------. Pt. with upper GI bleeding (Hematemesis)
. who have depressed conscioussness level should be intubated with ??
. ENDO-TRACHEAL tube not naso-gastric tube to secure the airway.
. KCL Potassium chloride ----> Drug induced esophagitis.
Dr. Wael Tawfic Mohamed
-------------------------

GENITOURINARY TIKI TAKA


_________________________
. GLOMERULONEPHRITIS common criteria:
_____________________________________
1- RBCs in urine.
2- Red cell casts in urine.
3- Mild degree of proteinuria (< 2 g. / 24 hs.).
4- Edema.
5- May lead to nephrotic $.
6- Most accurate diagnosis by --> RENAL BIOPSY.
* GOOD PASTURE's $YNDROME:
___________________________
. Cough, hemoptysis, shortness of breath & lung findings.
. Dx: Best initial test: Anti-basement membrane Abs.
. Dx: Most accurate test: Renal biopsy -> Linear deposits.
. Tx: PLASMAPHARESIS & steroids.
* CHURG STRAUSS $YNDROME:
__________________________
. ASTHMA, cough, EOSINOPHILIA + Renal abnormalities.
. Dx: Best initial test: CBC for eosinophil count.
. Dx: Most accurate test: Renal biopsy.
. Tx: Glucocorticoids "prednisone".
* WEGENER's GRANULOMATOSIS:
____________________________
. URT infections + LRT infections.
. URT infections -> sinusitis & otitis.
. LRT infections -> cough, hemoptysis, Abnormal CXR.
. It is a systemic vasculitis so it may involve the joint, skin, eye.
. Dx: Best initial test: C-ANCA "Anti-neutrophil cytoplasmic Ab".
. Dx: Most accurate test: Renal biopsy.
. Tx: Steroids & cyclophosphamide.
* POLYARTERITIS NODOSA:
________________________
. Systemic vasculitis.
. Involvement of all organs EXCEPT LUNGS !!!!!
. Renal - myalgia - GI bleeding - purpura - stroke - uveitis - neuropathy.
. MULTIPLE MOTOR & SENSORY NEUROPATHY + PAIN.
. Dx: Best initial test: ESR & inflammation markers.
. Dx: Most accurate test: Renal biopsy or SURAL N. biopsy.
. Test for HEPATITIS B & C (Ass. e' PAN).
. ANGIOGRAPHY showing BEADING can spare the need for biopsy.
. Tx: Steroids & cyclophosphamide.
* IgA NEPHROPATHY = BERGER's DISEASE:
______________________________________
. Painless recurrent hematuria.
. ASIAN pt.

.
.
.
.
.

H/O of very recent viral upper RTI.


Dx: Best initial test: ++ IgA !
Dx: Most accurate test: RENAL BIOPSY IS ESSENTIAL !
Normal complement levels.
Tx: Steroids.

* HENOCH - SCONLEIN PURPURA:


_____________________________
. Adolescent or child.
. Raised, non-tender purpuric skin lesions "buttocks".
. Abdominal pain.
. Possible bleeding.
. Joint pain.
. Renal involvement.
. Dx: Best initial test: CLINICAL SUSPENSE !
. Dx: Most accurate test: R. biopsy "Not necessary".
. Tx: No ttt - Resolves spontaneously.
* POST-STREPTOCOCCAL GLOMERULONEPHRITIS = PSGN:
________________________________________________
. Dark urine "Tea-colored or cola-colored".
. Periorbital edema & hypertension.
. H/O of Throat or skin infections 10 - 20 days ago.
. Dx: Best initial test: Anti-streptolysin O test "ASLO",
. Anti-DNase & Antihyaluronidase.
. Low complement levels.
. Dx: Most accurate test: R. biopsy sh'd n't be done bec. blood tests r suffecient.
. Tx: Antibiotics e.g. PENICILLIN.
. CONTROL HYPERTENSION & FLUID OVERLOAD with diuretics.
* CRYOGLOBULINEMIA:
____________________
. H/O of HEPATITIS "C" with renal involvement.
. Joint pain & pruritic skin lesions & Hepatosplenomegaly.
. Dx: Best initial test: Serum cryoglobulin componet levels,
. immunoglobulins & light chains, IgM.
. Low complement levels esp. "C4".
. Dx: Most accurate test: R. biopsy.
. Tx: Treat HEPATITIS C with INTERFERON + RIBAVIRIN.
* LUPUS (SLE) NEPHRITIS:
_________________________
. H/O of SLE !!
. N.B. Drug induced lupus spares the kidneys & the brain "V.V.V. imp.".
. Dx: Best initial test: ANA & Anti-Ds DNA.
. Dx: Most accurate test: RENAL BIOPSY.
. R. biopsy is v. imp. in cases of SLE to determine the extent of the disease & ttt.
. Tx:
----- Sclerosis only -------------------------------> No ttt.
----- Mild dis., early stage, NON proliferative ----> Steroids.
----- Severe dis. late stage, PROLIFERATIVE --------> MYCOPHENOLATE.
* ALPORT $YNDROME:
___________________
. CONGENITAL with family H/O of renal failure.

. Recurrent episodes of hematuria.


. Eye & ear problems e.g. deafness.
. No specific therapy.
* HEMOLYTIC UREMIC $YNDROME:
____________________________
. H/O of E-coli 0157:H7
. Intra-vascular hemolysis (fragmented cells on smear).
. ++ Creatinine.
. -- platelets.
* THROMBOTIC THROMBOCYTOPENIC PURPURA "TTP":
_____________________________________________
. HU$ +
. Fever +
. Neurological abnormalities.
. Tx: Plasmapharesis in severe cases.
___________________________
. ARF : PRE-RENAL AZOTEMIA:
___________________________
.. Presentation:
_________________
... Elderly pt with poor oral intake living in nursing homes taking medications e.g.,
... NSAIDs, ACE Is & diuretics causing intravascular volume depletion.
... leading to renal glomerular vasoconstriction.
.. Causes:
___________
. 1- Hypotension "SBP <90 mmHg".
. 2- Hypovolemia "dehydration or blood loss".
. 3- Low oncotic pressure " -- Albumin".
. 4- Congestive heart failure.
. 5- Constrictive pericarditis.
. 6- Renal artery stenosis.
.. Dx:
_______
... BUN:Creatinine ratio > 20:1.
... Urinary Na is low < 20.
... Fe Na < 1.
... Urine osmolality > 500.
___________________________________________________
. ARF : POST-RENAL AZOTEMIA = OBSTRUCTIVE UROPATHY:
___________________________________________________
.. Causes:
___________
. 1- Stone in the bladder or ureter.
. 2- Strictures.
. 3- Cancer of the bladder, prostate or cervix.
. 4- Neurogenic bladder "Atonic or non-contracting due to MS or DM".

.. Dx:
_______
... Similar to pre-renal azotemia.
... Distended bladder on exam.
... Large volume diuresis after passing a urinary catheter.
... Bilateral hydronephrosis on U/$.
______________________________________________________
. ARF : INTRA-RENAL AZOTEMIA = ACUTE TUBULAR NECROSIS:
______________________________________________________
.. Dx:
_______
... BUN/Creatinine ratio 10:1.
... Urinary Na > 40.
... Urine osmolality < 350.
* TOXIN INDUCED RENAL INSUFFECIENCY:
_____________________________________
. Aminoglycosides: Gentamycin, tobramycin, Amikacin (--Mg is suggestive).
. Amphotericin.
. Contrast agents (--Mg is suggestive).
. Chemotherapy e.g Cisplatin.
. Urinalysis: MUDDY BROWN or GRANULAR CASTS.
* ALLERGIC INTERSTITIAL NEPHRITIS:
___________________________________
. Hypersensitivity reaction to medications e.g. Penicillin or Sulfa drugs.
. Phenytoin, Allopurinol, Cyclosporin, Quinidine & Rifampin.
. FEVER & RASH & ARTHRALGIA.
. Dx: WRIGHT stain or HANSEL's STAIN of the urine ---> EOSINOPHILIA.
. WBCs casts are common but RBCs cast are rare.
. Tx: Discontinue the offending drug.
* RHABDOMYOLYSIS:
__________________
. Large volume muscular necrosis.
. causes direct toxic effect of myoglobin on the kidney tubule.
. H/O of crush injury or seizure.
. H/O of prolonged immobility.
. H/O of recent start of STATIN for hyperlipidemia.
. Best initial test: Urinalysis -> Large amounts of blood with no cells.
. Relative absence of RBCs on urine microscopy.
. ++ CPK (MOST SPECIFIC FINDING).
. Most accurate test: Urine myoglobin > 20000.
. Rhabdomyolysis --> ++ K & -- Ca.
. In case of hyperkalemia .. Do EKG to exclude arrhythmia.
. Tx hyperkalemia with IV Ca gluconate, insulin & glucose.
. Tx: BOLUS OF NORMAL SALINE, MANNITOL.
. ALKALINIZATION OF URINE.
* OXALATE CRYSTAL INDUCED RENAL FAILURE:
_________________________________________
. H/O of suicide trial by anti-freeze ingestion "ethylene glycol".

. intoxication due to metabolic acidosis & ++ in anion gap.


. Best initial Dx: Urinalysis --> ENVELOPE SHAPED OXALATE CRYSTALS.
. Best initial Tx: ETHANOL or FOMEPIZOLE with immediate dialysis.
* URIC ACID CRYSTAL INDUCED RENAL FAILURE:
___________________________________________
. H/O of chemotherapy for lymphoma causing tumor lysis $.
* CONTRAST INDUCED RENAL FAILURE:
__________________________________
. H/O of radiological procedure with contrast.
. H/O of elderly pt with DM or HTN.
. CREATININE just above normal 1.5 - 2.5.
. Tx: HYDRATION with Normal saline & Bicarbonate & N-Acetyl cysteine.
. NON-IONIC contrast agent is associated with less severity of nephropathy.
* NSAIDs INDUCED NEPHROPATHY Mechanism:
________________________________________
. Direct toxicity & ATN.
. Allergic interstitial nephritis with eosinophils in the urine.
. Nephrotic $.
. Afferent arteriolar VC.
. NEPHROTIC $YNDROMES & THEIR ASSOCIATIONS:
___________________________________________
. CHILDREN -------------------------------> Minimal change disease.
. ADULTS & CANCERS "LYMPHOMA" ------------> MEMBRANOUS.
. HEPATITIS C ----------------------------> MEMBRANOPROLIFERATIVE.
. HIV, HEROIN USE ------------------------> FOCAL SEGMENTAL.
. UN-CLEAR -------------------------------> MESANGIAL.
. STEPS FOR PROTEINURIA EVALUATION:
___________________________________
. Repeat the urine analysis.
. Evaluate for orthostatic proteinuria.
. Get a protein/creatinine ratio.
. Perform a renal biopsy.
. INDICATIONS OF DIALYSIS:
__________________________
. Hyperkalemia.
. Metabolic acidosis.
. Uremia with encephalopathy.
. Fluid overload.
. Uremia with pericarditis.
. Toxicity with a dialyzable drug e.g. Lithium , ethylene glycol or Aspirin.
. URGE INCONTINENCE:
____________________
. Pain followed by urge to urinate.
. Not related to coughing, laughing or standing.
. Dx: Urodynamic pressure monitoring.
. Tx: Behaviour modification + Anti-cholinergics.
. STRESS INCONTINENCE:

______________________
. NO PAIN.
. Follow coughing or laughing.
. Dx: Observe leakage with coughing.
. Tx: KEGEL exercise + Estrogen cream.
. SEVERE HYPERKALEMIA:
______________________
. Denoted by PEAKED T waves on EKG.
. Tx: I.V. CALCIUM GLUCONATE.
. NEPHROLITHIASIS:
__________________
. Sudden onset flank pain.
. Colicky, may be referred to the scrotum.
. Nause, vomiting.
. Cola colored urine.
. Dx: Non contrast CT Abdomen (Preferred to X-ray as it detects Radio-lucent stones).
. Tx: Relieve the pain by NSAIDs.
. Tx: Stones < 5 mm -> pass spontaneously with conservative ttt.
. Best conservative ttt is FLUID INTAKE > 2 LITERS / day.
. DEHYDRATION:
______________
. Altered mental status.
. Dry oral mucosa.
. ++ Na & ++ K.
. BUN / Creatinine > 20 "Pre-renal azotemia".
. More common in old age due to -- thirst response to dehydration.
. Tx: I.V. sodium containing CTYSTALLOIDS = NORMAL SALINE = 0.9 % NaCl.
. HERNIATED INTERVERTEBRAL DISK may cause URINE RTENTION due to SEVERE PAIN:
____________________________________________________________________________
. Unilateral radicular pain in a dermatomal distribution.
. Bk tendrness due to spasm of the paraspinous muscles.
. Cauda Equina $ can be excluded by absence of saddle anesthesia & intact sphincter tone.
. There will be pain on coughing or chest movement.
. So, severe pain in a pt.with a mild urinary obstruction, such as BPH,
. may cause urinary retention due to inability to Valsalva.
. CHLAMYDIAL URETHRITIS:
________________________
. Middle aged female.
. H/O of mutliple sex parteners.
. Dysuria & urinary frequency.
. Urinalysis: Absent bacteriuria.
. Urine culture < 100 colonies.
. HONEYMOON CYSTITIS:
_____________________
. Urinary infection most commonly arises by an ascending route.
. Sexual intercourse is one of the most imp. risk factors of un-complicated UTIs.
. due to its mechanical effect of introducing uropathogens into the bladder.
. RENAL CELL CARCINOMA:

_______________________
. Triad of flank pain, hematuria & palpable abdominal renal mass.
. Scrotal varicoceles "Lt sided" r seen in 10 % of pts.
. Varicoceles typically fail to empty when the pt is recumbent due to tumor obstruction.
. So presence of non emptying varicocele make you suspect mass obstruction by a tumor !
. Para-neoplastic symptoms e.g. Thrombocytosis, hypercalcemia & cachexia.
. Dx: Abd. CT .
. BENIGN PROSTATIC HYPERPLASIA = BPH:
_____________________________________
. Lower urinary tract symptoms e.g. frequency. nocturia, hesitancy & weak stream.
. Hypertrophy usually starts at the CENTRAL part of the prostate.
. Rectal exam: Smooth & firm enlargement of the prostate.
. N.B. prostate cancer rectal ex: (prostate nodules - induration - asymmetry).
. 1st initial step of management is placement of a FOLEY's catheter.
. Tx of BPH: Alpha blockers.
. Tx of severe cases: Surgery TURP.
. Current recommendations: All BPH pts sh'd have urinalysis & serum creatinine,
. to assess for urinary infection, obstruction or hematuria.
. If there is woresening of creatinine,
. Abdomial ULTRA$OUND is the initial test of choice to assess for HYDRONEPHROSIS.
. Hydronephrosis is caused by urinary obstruction & renal failure.
. HYPERKALEMIA (++ K > 5):
__________________________
. Drugs ++ K (ACE Is - NSAIDs - K sparing diuretics e.g. spironolcatone & Amiloride).
. Pseudohyperkalemia (Hemolyzed sample during venipuncture).
. Hyperkalemia (K > 6.5) may cause cardiac toxicity .
. EKG -> Peaked T waves & progressive widening of the QRS complex.
. Tx: IV CALCIUM GUCONATE.
. Tx: Insulin - B2 agonists.
. Tx: Na HCO3.
. Dialysis in severe cases.
. REMOVAL OF K FROM THE BODY -----> KAYEXALATE !
. The most common cause of death in RENAL DALYSIS & TRANSPLANTATION:
____________________________________________________________________
. is CARDIOVASCULAR complications.
. ANALGESIC NEPHROPATHY:
________________________
. Woman with chronic headaches on NSAIDs.
. Presenting with painless hematuria.
. NSAIDs -> VC of renal medulla vessels -> RENAL PAPILLARY NECROSIS.
. CHRONIC TUBULO-INTERSTITIAL NEPHRITIS.
. AUTOSOMAL DOMINANT POLYCYSTIC KIDNEY DISEASE (ADPCKD):
________________________________________________________
. HYPERTENSION + PALPABLE kidneys "BILATERALLY".
. Multiple renal cysts & intermittent flank pain.
. Liver enlargement due to cystic involvement "Most common extra-renal manifestation".
. Hematuria, UTIs & nephrolithiasis.
. Death may occur due to intracranial bleeding caused by rupture of berry anurysm.
. GLOMERULOPATHY associated diseases:

_____________________________________
. Lymphoma ----------------------------> Membranous nephropathy.
. Lymphoma complicated by nephrotic $ -> Minimal change nephropathy.
. HIV ---------------------------------> Focal & segmental glomerulosclerosis.
. AFRICAN AMERICANS -------------------> Focal & segmental glomerulosclerosis.
. OBESE -------------------------------> Focal & segmental glomerulosclerosis.
. HEROIN ADDICTS ----------------------> Focal & segmental glomerulosclerosis.
. MULTIPLE MYELOMA:
___________________
. Old age pt 65 ys with anemia, fatigue & bony pains (back & chest).
. Renal insuffeciency due to obstruction of the distal & collecting tubules by,
. BENCE JONES PROTEINS "PARA-PROTEINS".
. Old pt + bony pain + renal failure + Hypercalcemia = Multiple myeloma.
. AMITRIPTYLINE INDUCED URINE RETENTION:
________________________________________
. Amitriptyline is TCA with anticholinergic properties,
. it will lead to -- dterusor ms contraction & prevent urethral sphincter relaxation.
. leading to urine retention.
. Tx: Discontinue Amitriptyline + urinary catheterization.
. OVER-FLOW IN-CONTINENCE:
__________________________
. May be due to DM autonomic neuropathy causing a denervated bladder -> urine retention.
. The a-contractile hypotonic bladder gradually overdistends,
. When the bladder pressure rises above the urethral pressure,
. Urine is lost until the pressure equalizes !
. These events occur in a cyclic manner occuring at day & night.
. Exam may reveal a distended bladder.
. post-voidal residual urine volume is high.
. Associated other D.M. manifestations e.g. gastropathy, nephropathy & retinopathy.
. D.M. is the 1st leading cause of nephropathy, kidney biopsy will show:
________________________________________________________________________
. GLOMERULAR HYPERFILTRATION is the EARLIEST renal abnormality detected. (UW Q!).
. ++ extracellular matrix, basement membrane thickening, mesangial expansion & fibrosis.
. DIABETIC MICRO-ANGIOPATHY. (UW Q!)
. Proteinuria & progressive -- in GFR.
. Glomerulosclerosis. (UW Q!).
. HTN is the 2nd leading cause of nephropathy, kidney biopsy will show:
_______________________________________________________________________
. Arterio-sclerotic lesions of the afferent & efferent renal arterioles & capillaries.
. NO proteinuria.
. HEMATURIA:
____________
. Initial "Beginning of urination" -----> Urethral lesion e.g. Urethritis.
. Terminal "At the end of voiding"
-----> Prostatic or Bladder lesion e.g. cystitis.
. Total "during the entire process" ------> Ureters or kidneys lesion.
. The presence of clots in urine is more consistent with bladder not renal lesion.
_______________________

. URINARY TRACT STONES:


_______________________
1.CALCIUM OXALATE STONES:
__________________________
. Radio-opaque.
. envelope shaped on microscopy.
. Small bowel disease, surgical resection or chronic diarrhea,
. may lead to malabsorption of fatty acids & bile salts,
. which are important for chelating calcium,
. so, when Calcium is free, it binds with oxalic acid,
. forming Ca Oxalate stones.
2.CALCIUM PHOSPHATE STONES:
____________________________
. common in primary hyperparathyroidism.
3.URIC ACID STONES:
____________________
. When urine is acidic.
. When there is ++ cell turnover.
. Radio-lucent on X-ray.
. Tx: Hydration.
. Tx: Alkalinization of urine to pH > 6.5 by oral POTASSIUM CITRATE.
4.CYSTEINE STONES:
___________________
. ++ cysteine "Inborn error of metabolism".
. +ve family H/O.
. Recurrent stones since childhood.
. HARD & RADIO-OPAQUE stones.
. HEXAGONAL CRYSTALS on urine analysis.
. +ve Urinary cyanide nitroprusside test.
5.STRUVITE STONES:
___________________
. Formed when urine is ALKALINE.
. Bec. of infection with urease producing bacteria e.g. PROTEUS.
. H/O of recurrent UTI.
. NEPHROTIC $YNDROME:
______________________
. Proteinuria ( > 3- 3.5 g/day - most imp. criterion).
. Hypoalbuminemia.
. Edema.
. Hyperlipidemia & lipiduria.
. Pathology: Altered permeability of the glomerular membrane.
. Children : Minimal change disease.
. Adults : Membranous glomerulopathy.
. Complicated by HYPERCOAGULABILITY -> Thrombo-embolic manifestations.
. Accelerated atherosclerosis.
. Venous or arterial thrombosis & even pulmonary embolism.
. Other complications: Ptn malnutrition - iron resistant microcytic hypochromic anemia.
. Other complications: ++ susceptibility to infections & vitamin D defeciency.

. POST-OPERATIVE OLIGURIA:
__________________________
. Low urine out-put volume with lower abdominal pain.
. Most common cause is post-renal i.e. bladder out-let obstruction.
. Placement of a bladder catheter can rapidly improve symptoms "1st step done".
. Never to start fluids before catheterization as it may worsen the condition.
. BLUE TOE $YNDROME = CHOLESTEROL EMBOLIZATION:
_______________________________________________
. H/O of cardiovascular disease with recent surgical intervention or angiography.
. Atherosclerotic plaque may break off & enter the circulation.
. Abdominal pain & nausea.
. Livedo reticularis = cyanotic dicolouration of the skin
. ARF may occur due to renal artery embolization.
. ++ urea & creatinine levels.
. -- complement levels.
. ++ eosinophils.
. N.B. BLUE TOE $ sh'd n't be mis-diagnosed with CONTRAST INDUCED NEPHROPATHY:
______________________________________________________________________________
. Absence of livedo reticularis, abd. symptoms, high eosinophils & -- complement.
. ACUTE PYELONEPHRITIS:
_______________________
. Acute febrile illness.
. Costo-vertebral angle tendrness.
. Pyuria & bacteriuria.
. Initial ttt -> Blood cultures followed by Empirical I.V. Antibiotics.
. No response within 72 hours -> Do imaging e.g. U/$ or CT,
. to search for underlying pathologies (e.g.obstruction) or complications (e.g.abscess).
. Prazosin & TRAZODONE cause PRIAPISM !
. Diabetic pts with renal failure on METFORMIN should stop it as it ++ lactic acidosis.
. SIMPLE RENAL CYST:
____________________
. Age > 50 ys.
. Benign, don't require ttt.
. Reassurance.
. Both IgA Nephropathy & PSGN are major causes of hematuria after an upper RTI:
_______________________________________________________________________________
. IgA nephropathy: begins (1-5days) after URTI with normal serum complement.
. PSGN : begins 10-15 days after URTI with low serum complement.
. MEMBRANO-PROLIFERATIVE GLOMERULONEPHRITIS:
____________________________________________
. caused by persistent activation of the alternative complement pathway.
. Microscopy: Dense intra-membranous deposits that stain for C3.
. Dipsticks findings in case of UTI:
____________________________________
. Leukocyte esterase -> Pyuria.

. Positive nitrites -> Enterobacteriaceae.


. The most common culprit organism in UTIs is E-Coli.
. ERYTHROPOIETIN THERAPY in cases of ESKD:
__________________________________________
. ESKD presents with normocytic normochromic anemia due to -- erythropoietin.
. Tx of choice is recombinant erythropoietin.
. Started if Hb < 10 g/dl.
. Most common side effect is WORSENING OF HYPERTENSION.
. Other SE: Headaches & flu-like symptoms.
. ESKD ttt options:
___________________
. Dialysis or renal transplantation.
. Renal transplantation is more preferred due to better survival rate.
. A living related donor is always preferred.
. Management of CALCIUM OXALATE STONES:
_______________________________________
. 1- ++ fluid intake.
. 2- -- Na intake.
. 3- THIAZIDE DIURETIC.
. 4- -- protein & oxalate intake.
. GUESS WHAT ???!!------------> Calcium restriction is not required
. ACUTE EPIDIDYMITIS:
_____________________
. Fever.
. Painful enlargement of the testes.
. Irritative voiding symptoms.
. Two types of epididymitis: Sex-transmitted related & non-related.
. Sex-transmitted: more common in young pts & associated with urethritis.
. Sex-transmitted: pain at the tip of the penis & urethral discharge.
. Sex-transmitted: caused by Chlamydia trachomatis & Neisseria Gonorrhea.
. NON-sex-transm.: more common in elderly & associated with a UTI.
. NON-sex-transm.: No pain at the penile tip - No urethral discharge.
. NON-sex-transm.: caused by gram -ve rods e.g. E-coli.
. ACUTE PROSTATITIS:
____________________
. Fever, chills, ++ WBCs with bandemia.
. Urinary urgency, dysuria & +ve leukocyte esterase.
. Pain in the perineal region.
. Tender boggy prostate..
. Obtaining a mid-stream urine sample is the 1st step sh'd be done.
. Prostatic massage sh'd be avoided as it may lead to infectious spread.
. EXOGENOUS ANABOLIC STEROID USE can produce INFERTILITY in MEN:
________________________________________________________________
.By suppressing the production of GnRH, LH & FSH.
. ACYCLOVIR -> CRYSTALLURIA with RENAL TUBULAR OBSTRUCTION:

___________________________________________________________
. Acyclovir doesn't produce interstitial nephritis. (Take care - common mistake).
. Acyclovir is poorly soluble in urine & easily precipitates in renal tubules.
. It causes tubular obstruction with acute renal failure.
. It is due to large parenteral doses of Acyclovir.
. Inadequate hydration is a common predisposing factor.
. FIBROMUSCULAR DYSPLASIA -> RENAL ARTERY STENOSIS -> RENOVASCULAR
HYPERTENSION:
________________________________________________________________________________
. Young adult.
. Headache, hypertension & renal bruit.
. Medical therapy only with ACEIs is NOT effective.
. Tx of choice is: Percutaneous ANGIOPLASTY + STENT.
. PERICARDITIS is a common complication of UREMIA (RF):
_______________________________________________________
. Chest pain (Non radiating - Retrosternal - Relieved by leaning forward).
. EKG (Diffuse ST elevation - PR segment depression).
. Pericardial friction rub.
. Tx: Hemodialysis.
. CASTS in NEPHROLOGY:
______________________
. Muddy brown granular casts -> Acute tubular necrosis.
. Broad & waxy casts ---------> Chronic renal failure.
. RBCs casts -----------------> Glomerulonephritis.
. WBCs casts -----------------> Interstitial nephritis & pyelonephritis.
. Fatty casts ----------------> Nephrotic $.
. RENAL AMYLOIDOSIS:
____________________
. H/O of Rheumatoid arthritis (predisposes to amyloidosis).
. Enlarged kidneys & hepatomegaly.
. Renal biopsy -> Amyloid deposits with APPLE GREEN BIREFRINGENCE under polarized light.
. RENAL DISEASE -----> RENAL BIOPSY FINDING:
____________________________________________
. AMYLOIDOSIS -> Amyloid deposits with APPLE GREEN BIREFRINGENCE under polarized light.
. RPGN "Rapid progressive GN" -> Crescent formation.
. GOOD PASTURE's $ -> Linear immunoglobulin deposits (Ani-glomerulat b. membrane Abs).
. LUPUS NEPHRITIS -> Granular deposits.
. NEPHROTIC $ "MINIMAL CHANGE DISEASE" -> NORMAL LIGHT MICROSCOPY.
. IMPORTANT DRUG SIDE EFFECTS:
______________________________
. TACROLIMUS: Nephrotoxicity - hyperkalemia - hypertension - tremors.
. CYCLOSPORINE: Same as Tacrolimus + Hirsutism & Gum hypertrophy.
. AZATHIOPORINE: Diarrhea - leukopenia - hepatotoxicity.
. "M"YCOPHENOLATE -> Bone "M"arrow depression.
. RENAL VEIN THROMBOSIS:
________________________
. Important complication of Nephrotic $.
. caused by MEMBRANOUS GLOMERULONEPHRITIS (Not minimal change dis.).

. Due to loss of ANTITHROMBIN 3 in urine.


. Sudden onset of abdominal pain, fever & hematuria.
. ACUTE CYSTITIS:
_________________
. Healthy, young, non-pregnant woman. " Un-complicated".
. Pregnant, v.young, v.old, D.M.,immunocompromized,anatomical abnormality. "Complicated".
. Dysuria, frequency, supra-pubic pain & or hematuria (Hemorrhagic cystitis).
. Tx of un-complicated cystitis: NITROFURANTOIN or Oral TMP-SMX.
. Tx of complicated cystitis: Levofloxacin or ciprofloxacin.
. RENAL TRANSPLANT DYSFUNCTION:
_______________________________
. Oliguria - hypertension - ++ creatinine/urea.
. Causes:
_________
1- Ureteral obstruction.
2- Acute rejection.
3- Cyclosporine toxicity.
4- Vascular obstruction.
5- Acute tubular necrosis.
. Acute rejection is best treated with INTRAVENOUS STEROIDS.
. NON-INFLAMMATORY CHRONIC PROSTATITIS:
_______________________________________
. Afebrile pts.
. Irritative voiding symptoms (frequency, urgency, suprapubic or perineal discomfort).
. Normal physical exam.
. Normal urine analysis.
. Expressed prostatic secretions show NORMAL number of leukocytes.
. culture of the expressed secretionsis NEGATIVE for bacteria.
. No past H/O of UTIs.
. HEPATO-RENAL $YNDROME:
________________________
. Complication of end stage LIVER disease (e.g. Cirrhosis).
. -- GFR in absence of shock, proteinuria or other causes of renal dysfunction.
. Failure to respond to 1.5 liters of normal saline.
. Most common causes of death are infection & hemorrhage.
. Tx: LIVER "NOT KIDNEY" TRANSPLANTATION.
. RIRFAMPICIN (Anti-Tuberculous drug):
______________________________________
. Reddish discolouration of urine, saliva, sweat & tears.
. Benign drug effect.
. Reassure the patient.
. UREMIC COAGULOPATHY:
______________________
. Complication of CRF.
. Echymosis & epistaxis are the most common presentations.
. The main cause is PLATELET DYSFUNCTION.
. PT,PTT,Platelet count -> NORMAL.
. Bleeding time is prolonged.

. Tx: DDAVP ++ the release of factor 8 (Von Willebrand f) from endothelial storage sites.
. PLATELET TRENSFUSION has NOOOOOO EFFECT as they quickly become INACTIVE.
. SICKLE CELL TRAIT:
____________________
. YOUNG BLACK MALE with PAINLESS HEMATURIA.
. Painless hematuria in EPISODES !
. Caused by PAPILLARY ISCHEMIA.
. Reassurance.
. DETRUSOR MUSCLE INACTIVITY:
_____________________________
. May be caused by 1st generation ANTI-HISTAMINICs due to their ANTI-CHOLINERGIC effects.
. They inhibit the action of Acetyl-choline on Muscarinic receptors.
. Urine retention occurs due to detrusor ms. failure of contraction.
. Other SEs: Dryness of eyes, oral mucosa & rspiratory passags.
. GROSS PAINLESS HEMATURIA in an ADULT = BLADDER MASS TUMOR:
____________________________________________________________
. Do a contrast CT abominal scan or IVP to detect the mass.
. The presence of erythrocytes in urine sh'd be confirmed microscopically,
. to exclude myoglobinuria, hemoglobinuria or porphyria.
. Other causes: BEETS large amounts ingestion or Rifampicin ttt.

Dr. Wael Tawfic Mohamed


_________________________

HEMATOLOGY & ONCOLOGY TiKi TaKa


_________________________________
. ANEMIA presentations:
_______________________
. Mild -> Fatigue - loss of energy - tiredness - malaise.
. Severe -> Shortness of breath - lightheadedness - confusion.
. Diseases with similar presentations (Hypoxia - CO poisoning - Methemoglobinemia).
. Pallor - flow murmur - pale conjunctiva.
. MICROCYTIC ANEMIA:
____________________
____________________
(1) IRON DEFECIENCY ANEMIA:
____________________________
. The most common cause is chronic blood loss (Bleeding ulcer).
. ++ platelet count.
. Dx -> Best initial test -> IRON STUDIES:
- ____________________________________________
- | -- iron.
.|
- | -- ferritin.
.|
- | -- iron saturation.
.|
- | ++ TIBC (Total iron binding capacity). .|
|--------------------------------------- .|
- | ++ Red cell distribution of width (++ RDW).|
- ____________________________________________
. Dx -> Most accurate test -> Bone marrow biopsy.
. Tx -> Ferrous sulfate orally (May turn the stool BLACK but Guaiac test -ve).
. PICA -> Appetite for non-nutritive substances (ice - clay - dirt).
(2) ANEMIA OF CHRONIC DISEASE:
_______________________________
. Rheumatoid arthritis.
. End Stage Kidney Disease (ESKD).
. Any chronic infectious, inflammatory or connective tissue disease.
. . Dx -> Best initial test -> IRON STUDIES:
- _______________________________________
- | -- iron.
.|
- | ++ ferritin.
.|
- | Normal or -- iron saturation.
.|
- | -- TIBC (Total iron binding capacity).|
|--------------------------------------.|
- _______________________________________
* IRON IS LOCKED IN STORAGE OR TRAPPED IN MACROPHAGES OR IN FERRITIN.
. Osteoarthritis doesn't cause anemia of chronic disease !
. Exclude anemia of chronic disease if there is NORMAL ESR !
. The only form of anemia that responds to erythropoietin is ESKD.
. Tx -> tt the underlying cause e.g. (METHOTREXATE or INFLIXIMAB for RA).
. Although iron concentration is low, Iron supplementation is not benificial.
. IRON IS LOCKED IN STORAGE OR TRAPPED IN MACROPHAGES OR IN FERRITIN.

(3) THALASSEMIA:
_________________
. Very small MCV with few or no symptoms. (55 - 75 ml).
. TARGET cells.
. Dx -> NORMAL iron studies.
. Most accurate test -> HEMOGLOBIN ELECTROPHORESIS:
- Beta -> ++ Hg A2 & Hg F.
- Alpha -> Normal (Most accurately diagnosed by DNA sequencing).
- Hg H has Beta 4 tetrads with 3 gene deleted Alpha thalassemia.
. B thalassemia major -> Both B hemoglobin are deffective.
. Characterized by severe anemia & transfusion dependence at an early age).
. B thalassemia minor occurs in people heterozygous for the B hemoglobin gene.
. The only microcytic anemia with a HIGH reticulocytic count is Hg H.
. Microcytic anemia non responsive to iron supplementation.
. No ttt !
(4) SIDEROBLASTIC ANEMIA:
__________________________
. Defect in heme-synthesis.
. Alcoholic pt.
. Isoniazid intake without Vit. B6 "pryridoxine" supplementation.
. Lead exposure.
. Dx -> Iron studies -> HIGH IRON & -- TIBC.
. Dx -> Bone marrow biopsy -> RINGED SIDEROBLASTs.
. Most accurate test -> PRUSSIAN BLUE STAIN.
. Tx -> Minor -> Pyridoxine replacement.
. Tx -> Major -> Remove the toxin exposure.
. "IRON DEFECIENCY ANEMIA" - "ALPHA THALASSEMIA MINOR" - "BETA THALASSEMIA
MINOR":
______________________________________________________________________________________
. Hematocrit < 30 %
- Hematocrit > 30 %
- Hematocrit > 30 %
. RDW ++
- RDW normal
- RDW normal
. RBC count -- RBC count normal
- RBC count normal to ++
. No target cells
- TARGET cells on smear
- TARGET cells on smear
. -- serum iron & ferritin - Nor. to ++ iron & ferritin - Nor. to ++ iron & ferritin
. ++ TIBC
- Normal TIBC
- Normal TIBC
. Responds to iron supplem. - No response to iron supplementation
. Normal Hb Electrophoresis - Normal Hb Electrophoresis - ++ Hb A2 on Hb Electrophor.
. N.B. MECHANISMS OF ANEMIA IN MICROCYTIC HYPOCHROMIC DISEASES:
_______________________________________________________________
. -- fe intake & ++ blood loss -> IRON DEFECIENCY ANEMIA.
. Defective utilization of storage iron -> ANEMIA OF CHRONIC DISEASES.
. -- globin production -> THALASSEMIA.
. -- Heme synthesis -> LEAD POISONING & SIDEROBLASTIC ANEMIA.
. MICROCYTIC HYPOCHROMIC DISEASE -> FINDINGS IN IRON STUDIES:
_____________________________________________________________
. IRON DEFECIENCY ANEMIA -> -- fe, -- ferritin & ++ TIBC.
. THALASSEMIA -> Normal to ++ fe & ferritin levels.
. ANEMIA OF CHRONIC DISEASE -> -- TIBC.
. SIDEROBLASTIC ANEMIA -> Normal to ++ fe & ferritin levels.

. MACROCYTIC ANEMIA:
____________________
____________________
. Extravascular hemolysis occurs in spleen & liver so u can't see it on a smear !
. Caused by Vit. B12 or floate defeciency.
. Vit. B12 def. -> neurological findings (Mostly peripheral neuropathy- Least dementia).
. Folate defeciency is NOT associated with neurological findings.
. Suspect Vit B12 def. in STRICT VEGETERIANS with anemia & neurologic complications.
. METFORMIN blocks B12 absorption.
. B12 def. -> Smooth tongue (Glossitis) & diarrhea.
. -- B12 -> -- production of INTRINSIC FACTOR by gastric parietal cells,
. -- B12 -> Atrohic gastritis -> DOUBLE RISK of GASTRIC CANCER.
. Dx -> Best initial test -> CBC with peripheral smear -> HYPERSEGMENTED NEUTROPHILs.
. Dx -> Hypersegmented neutrophils + Large MCV = Megaloblastic anemia.
. Dx -> Anisocytosis - poikilocytosis - basophilic stippling.
. Dx -> ++ Bilirubin & LDH.
. Dx -> -- Reticulocytic count.
. Dx -> Most accurate test -> -- Vit. B12 & Folic acid levels.
. If u suspect B12 defeciency but B12 level is normal -> Order Methyl Malonic acid (MMA).
. Dx -> HOMOCYSTEINE is ++ in both Vit. B12 & folate defeciency.
. Dx -> Most accurate test -> Anti-parietal cell Abs & anti-intrinsic factor Abs.
. Tx -> REPLACEMENT of Vit B12 & folate.
. Tx -> Folic acid will correct the hematological problems.
. Tx -> Vit. B12 will correct the neurological problems.
. Watch for low potassium after ttt B12 def. !
. N.B. (1):
___________
. Folate & cobalamine (B12) are involved in the conversion of homocysteine to methionine.
. -- Vit. B12 or folic acid -> ++ Homocysteine level.
. N.B. (2):
___________
. Folic acid & vit. B12 defeciency can be distinguished by measuring Methyl Malonic acid!
. Vit B12 (Not folic acid) is involved in the conversion of MMA to succinyl coA.
. -- Vitamin B12 -> ++ MMA.
. -- Folic acid -> NORMAL MMA.
. N.B. (3):
___________
. Laboratory tests ------------> Prinicious anemia -------------> Folic acid defeciency
________________________________________________________________________________________
. Serum B12 level ------------->
(--)
-------------> Normal.
. Serum folic acid level ------>
(Normal)
-------------> (--).
. Serum LDH level ------------->
(++)
-------------> Normal.
. Achlorhydria ---------------->
present
-------------> Absent.
. Schilling test -------------->
+ve
-------------> -ve.
. Methyl malonyl acid MMA ----->
++
-------------> Absent.
. Neurological signs ---------->
++
-------------> Absent.
. N.B. (4):
___________
. Alcohol abuse is the most common cause of nutritional folate defeciency in USA.
. N.B. (5):

___________
. Anemia of chronic kidney disease is due to erythropoietin defeciency.
. One must be careful to ensure adequate iron stores prior to erthropoietin replacement,
. Bec. the erythropoietin induced surge in RBCs production may consume much iron,
. precipitating an iron defecient state.
. N.B. (6):
___________
. A case of microcytic hypochromic anemia with -- iron & ferritin,
. The most common cause of iron defeciency anemia is GIT blood loss.
. Dietary iron defeciency & malabsorption of iron are rare causes.
. Iron supplementation helps to restore iron reserves but u must detect the cause !
. So .. perform test for occult blood in the stool.
. N.B. (7):
___________
. The most common cause of folic acid defeciency is NUTRITIONAL.
. Either due to poor diet or Alcoholism.
. May be caused by some drugs either by impairing its absorption (PHENYTOIN),
. or antagonizing its physiologic effect (Methotrexate - Trimethoprim).
. N.B. (8):
___________
. PERINICIOUS ANEMIA:
______________________
. Most common cause of vit. B12 defeciency.
. Auto-antibodies against the gastric intrinsic factor required for B12 absorption.
. More in Northern Europeans.
. Associated other auto-immune diseases e.g. Autoimmune thyroiditis & Vitiligo.
. Shiny tongue due to atrophic glossitis.
. Shuffling broad-based gait ataxia.
. Neurological abnormalities with loss of pain & vibration sense.
. Peripheral blood smear -> Macro-ovalocytes, megaloblasts & hyper-segmented neutrophils.
. Dx -> Detection of Anti-intrinsic factor Abs.
. N.B. (9):
___________
. Total body stores of Vit.B12 in humans are 2-5 mg with min. daily requirement 6-9 micg.
. Animal products (meat & dairy) are the only dietary sources of vit. B12.
. It would take 4-5 years of a pure vegan diet to cause dietary defeciency.
. In contrast, Folate stores are smaller, clinical defeciency occur within 4-5 months.
. N.B. (10):
____________
. SCHILLING TEST:
__________________
. Used to detect the cause of vit. B12 defeciency.
. Used to differentiate dietary defeciency from perinicious anemia & malabsorption.
. In dietary defeciency,
. Oral radiolabelled Vit B12 is absorbed in the gut & excr. by kidneys in NORMAL amounts.
. But In Malabsorption,
. Oral radio-labelled Vit B12 is excreted in sub-normal amounts.
. HEMOLYTIC ANEMIA:
___________________

___________________
. SUDDEN ONSET of hematological manifestations.
. Dx -> Hemolysis ->
-> ++ indirect bilirubin.
-> ++ reticulocyte count.
-> ++ LDH level.
-> -- Haptoglobin.
. Dx -> Intravascular hemolysis ->
-> Abnormal peripheral smear (schistocytes - helmet cells - fragmented cells).
-> Hemoglobinuria.
-> Hemosiderinuria.
(1) SICKLE CELL ANEMIA:
________________________
. Very severe pain in the back, chest & thighs.
. Retinal infarction.
. Flow murmur from anemia.
. Splenomegaly in children & asplenia in adults.
. Rales or consolidation from lung infection or infarction.
. Skin ulcers.
. Aseptic necrosis of hip.
. Neurological strokes.
. Cause of Osteomyelitis in sickle cell anemia is SALMONELLA !
. Cause of BM suppression in sickle cell anemia is PARVO B19 !
. Tx -> OXYGEN - HYDRATION with NORMAL SALINE - Pain relief.
. If there is fever -> (Emergency - No spleen) -> Ceftriaxone -levofloxacin.
. Tx -> EXCHANGE TRANSFUSION if ->
-> Eye -> Visual disturbance from retinal infarction.
-> Lungs -> Pulmonary infarction with pleuritic pain.
-> Penis -> Priapism from infarction of prostatic plexus of veins.
-> Brain -> Stroke (weakness & aphasia due to sludging & occlusion of cerebral vessels).
. N.B. -> SUDDEN DROP IN HEMATOCRIT can be caused by FOLATE DEFECIENCY or PARVO B19
INF:
. Folate defeciency in SCD due to ++ RBCs turnover & consumption of folate in the BM.
. Daily folic acid supplementation is recommended in all SCD.
. If the pt is already on Folate replacement -> The cause is PARVO B19 viral infection !
. PARVO B19 may invade the bone marrow & causes aplastic anemia.
. Detect Parvo B19 infection by PCR for DNA of Parvo B19.
. On discharge -> Give folate replacement - Pneumococcal vaccine.
. Hydroxyurea (++ Hb F) is given to prevent further crisis.
(2) HEMOGLOBIN SICKLE CELL DISEASE:
____________________________________
. African American pt.
. Family H/O of blood disrder.
. Mild version of sickle cell anemia.
. Visual distrurbance is frequent.
. No painful crisis.
. RENAL problems are common,
. (Hematuria - Isosthenuria "inablility to dilute urine or concentrate urine" - UTIs).

. Isosthenuria due to RBCs sickling in the vasa rectae of the inner renal medulla.
. The pt have to wake to urinate 2 - 3 times per night despite restricting fluid intake.
. No ttt.
(3) AUTOIMMUNE HEMOLYSIS:
__________________________
. H/O of other autoimmune diseases e.g. SLE (peripheral immune destruction of all cells).
. H/O of CLL or lymphoma (Enlarged Non tender axillary LNs bilaterally!).
. H/O of penicillin - alpha methyl dopa - quinine or sulfa drug intake.
. Dx -> ++ LDH - ++ indirect bilirubin - ++ reticulocyte count - -- Haptoglobin.
. Dx -> peripheral smear -> spherocytes.
. Dx -> Most accurate test -> Coombs test. (WARM Abs = IgG Abs).
. Tx -> Best initial ttt -> Steroids e.g. prednisone.
. If there is recurrent episodes of hemolysis -> SPLENECTOMY is the most effective ttt.
. If no response to prednisone or splenectomy -> Give IVIG.
(4) COLD INDUCED HEMOLYSIS = COLD AGGLUTININs:
_______________________________________________
. H/O of mycoplasma or EBV infection.
. Coomb's test -> -ve.
. Complement test-> +ve.
. No response to prednisone, spenectomy or IVIG.
. Tx -> RITUXIMAB.
(5) GLUCOSE - 6 - PHOSPHATE DEFECIENCY (G6PD):
_______________________________________________
. X-linked disorder - Only MALE pt. (African American).
. ENZYME defeciency (Glucose 6-phosphate dehydrogenase).
. Sudden onset of hemolysis.
. INFECTION is the most common predisposing factor.
. H/O of intake of Oxidizing drugs (Sulfa - primaquine - Dapsone).
. H/O of ingestion of FAVA BEANS.
. Mechanism of cell damage -> OXIDATIVE STRESS.
. Dx -> Best initial test -> Heinz body test & BITE CELLs.
. +ve prussian blue is indicative of hemosiderin found in urine during hemolytic episode.
. Heinz bodies are collections of oxidized precipitated Hb embedded in the RBC membrane.
. Bite cells appear when pieces of the red cell membrane have been removed by the spleen.
. Most accurate test -> G6PD level after 2 months of symptoms !
. A normal level of G6PD immediately after an episode of hemolysis doesn't exclude it !
. Tx -> AVOID OXIDANT STRESS.
. N.B. PYRUVATE KINASE DEFECIENCY:
___________________________________
-> can lead to similar presentation as G6PDD .. BUT:
-> Hemolysis not precipitated by sulfa drugs !
-> No bite cells on peripheral smear !
(6) HERIDITARY SPHEROCYTOSIS:
______________________________
. Northern European descent.
. TRIAD of HEMOLYTIC ANEMIA + JAUNDICE + SPLENOMEGALY.
. Autosomal dominant disorder of spectrin (Ptn that provides scaffolding for RBCs).
. RBCs are not deformable -> Gets trapped in the fenesterations of the spleen's red pulp.
. Recurrent episodes of hemolysis.
. Splenomegaly.

.
.
.
.
.
.
.
.

Bilirubin gall stones (pigmented Ca bilirubinate) -> Acute cholecystitis.


Dx -> +++ MCHC (Mean cell hemoglobin concentration = RBC membrane loss & dehydration).
Dx -> Peripheral smear -> Spherocytes.
Dx -> -ve Coomb's test (+ve in AIHA = Autoimmune hemolytic anemia).
Dx -> Most accurate test -> OSMOTIC FRAGILITY test.
Tx -> SPLENECTOMY will prevent hemolysis as spherocytes are destroyed in the spleen.
Folate supplementation is important.
There is a risk for infection with Parvovirus B19 causing aplastic anemia.

(7) HEMOLYTIC UREMIC $YNDROME (HU$):


_____________________________________
. H/O of E-coli 0157:H7 infection.
. HUS TRIAD (ART):
-> Autoimmune hemolysis -> intravascular hemolysis (Anemia - indirect hyperbilirubinemia)
-> Renal failure -> ++ BUN & creatinine.
-> Thrombocytopenia -> -- platelets.
. Dx -> Peripheral smear -> Schistocytes.
. Tx -> Most cases resolve on their own.
. Tx -> Severe cases are ttt with plasmapharesis or plasma exchange.
. Steroids don't help !
. Antibiotics worsen the case !
. Platelet transfusion worsen the case !
(8) THROMBOTIC THROMBOCYTOPENIC PURPURA (TTP):
_______________________________________________
. H/O of TICLOPIDINE use.
. HUS TRIAD + FN (Fever & Neurolgical abnormailities).
. Dx -> Peripheral smear -> Schistocytes.
. Same ttt as HUS (PLASMAPHARESIS or PLASMA EXCHANGE).
(9) PAROXYSMAL NOCTURNAL HEMOGLOBINURIA (PNH):
_______________________________________________
. Pancytopenia.
. Recurrent attacks of dark urine "Specially in the morning !".
. Most common cause of death -> LARGE VESSEL VENOUS THROMBOSIS "PORTAL VEIN
THROMBOSIS".
. PNH may transform to alpastic anemia or acute myelogenous leukemia (AML).
. PORTAL VEIN THROMBOSIS may be the key to answer PNH case !
. Dx -> Most accurate test -> Flow cytometry -> Cd 55 & CD 59 antibodies.
. Tx -> Best initial ttt -> Glucocorticoids (prednisone).
. Transfusion dependent pts may be ttt with Eculizomab -> inhibit C5 complement.
. Hemolytic anemia + Venous thrombosis = PNH.
(10) MACROVASCULAR TRAUMATIC HEMOLYSIS:
________________________________________
. ++ reticulocytes - ++ LDH - -- Haptoglobin ("intra-vascular hemolytic anemia).
. Fragmented RBCs.
. Chronic hemolysis -> iron loss -> Microcytic anemia.
. Due to mechanical trauma from artificial valves or calcified aortic valves.
. N.B. HERIDITARY SPHEROCYTOSIS & AUTOIMMUNE HEMOLYTIC ANEMIA:
______________________________________________________________
. Peripheral blood smerar in both conditions -> Spherocytes without central pallor.
. Both cause extravascular hemolytic anemia.
. Heriditary spherocytosis -> Autosomal dominant heriditary condition.

. AIHA -> Acquired condition.


. Spherocytosis -> +ve family H/O & -ve Coomb's test.
. AIHA -> -ve family H/O & +ve Coomb's test.
. N.B. PATHO-PHYSIOLOGIC MECHANISMS of ANEMIA in variable diseases:
___________________________________________________________________
. Impaired DNA & purine synthesis -> Vitamin B12 defeciency.
. RBC membrane instability -> Heriditary spherocytosis.
. Impaired Hb synthesis -> Iron defeciency anemia, sickle cell anemia & Thalassemia.
. Impaired glutathione synthesis -> G6PDD.
. Mechanical injury to RBCs -> Hemolysis with artificial heart valves.
. MET-HEMOGOLBINEMIA:
_____________________
. Shortness of breath for no clear reason.
. Clear lungs on examination.
. Blood locked up in an oxidized state that can't pick up oxygen.
. H/O of exposure to drugs such as nitroglycerin, amyl nitrate, nitroprusside & dapsone.
. H/O of exposure to anesthetic drugs e.g lidocaine.
. H/O of brown blood !!
. Tx -> Methylene blue !!
. CARBOXYHEMOGLOBINEMIA = CARBON MONOXIDE (CO) POISONING:
_________________________________________________________
. H/O of environmental risk (Pt working in an enclosed space -> underground parking lot).
. Present with headache, nausea & dizziness.
. Exposure to CO from automobile exhaust.
. CO binds Hb with an affinity app. 250 times that of oxygen.
. -- in blood carrying oxygen capacity.
. As a compensation -> the body ++ RBCs production (++ HCT).
. OBSTRUCTIVE SLEEP APNEA (OSA) -> ++ ERYTHROPOIETIN PRODUCTION:
________________________________________________________________
. Recurrent transient obstruction of the upper airway due to pharyngeal collapse.
. Obese or over-weight pts have excessive snoring, day time sleeping & morning headaches.
. Transient episodes of hypoxia -> sensed by the kidneys -> ++ Erythropoietin production.
. Erythropoietin ++ RBCs -> Polycythemia.
. OSA doesn't cause carboxyhemoglobinemia ! xxxxxxxxxxxxxx
. LEUKEMIA:
___________
___________
(A) ACUTE LEUKEMIA:
____________________
. Pancytopenia (Fatigue - bleeding - infections).
. Excess WBCS but don't work (Functional immunodefeciency).
. Dx -> Peripheral smear -> BLASTS.
. AUER RODS = AML ACUTE MYELOID LEUKEMIA.
. DIC is asscociated with M3 (Acute pro-myelocytic leukemia).
. Cytogenetics is used for prognosis & relapse prediction.
. If the pt is at high risk for relapse after chemotherapy,
. BM transplantation sh'd be performed as chemotherapy induces remission.
. Tx -> Chemotherapy (idarubicin & daunorubicin) -> Acute myelogenous leukemia.
. Tx -> Add ATRA (All Trans-Retinoic Acid) for -> M3 Acute pro-myelocytic leukemia.

. Tx -> Ass intra-thecal Methotrexate for ALL Acute Lymphocytic leukemia.


.
.
.
.

N.B. Acute leukemia can sometimes present with extremely HIGH WBCs count,
When the WBCs > 100000 -> sludging of WBCs in the blood vessels of eye & lungs & brain.
LEUKOSTASIS is ttt with LEKOPHARESIS (Removing of WBCs via blood centrifugation).
Hydroxyurea is used to lower the cell count.

(B) MYELODYSPLASIA:
____________________
. Mild slowly progressive pre-leukemia $.
. May progress to acute leukemia.
. MOST COMMON CAUSE OF DEATH -> INFECTION or BLEEDING.
. Elderly pt.
. Pancytopenia (Fatigue - bleeding - infections).
. ++ MCV (Macro-ovalocytes).
. -- Reticulocyte count.
. Dx -> Special neutrophil with 2 lobes (PELGER-HUET) cells.
. Tx -> Supportive with transfusions as needed & AZAcitadine.
. Those with 5q minus $ are ttt with Lenalidomide.
. MYELO-PROLIFEARTIVE DISORDERS:
________________________________
(A) CHRONIC MYELOGENOUS LEUKEMIA (CML):
________________________________________
. ++ WBCs (NEUTROPHILs predominant).
. SPLENOMEGALY.
. Dx -> Best initial -> -- Leukocyte Alkaline Phosphatase (LAP).
. Dx -> Most accurate -> PHILADELPHIA CHROMOSOME (Cytogenic study:Abnormal
chromosome 22)
. formed by reciprocal translocation of chromosom 9 & 22.
. Reciprocal translocation -> BCR-ABL fusion gene -> ABNORMAL TYROSINE KINASE ACTIVITY !
. {-- LAP} + {++ WBCs} = CML.
. {++ LAP} + {++ WBCs} = LEUKEMOID REACTION = Reactivity from infection.
. Tx -> Best initial -> IMATINIB (Gleevac) " Tyrosine kinase inhibitor".
. BMT is the only way to cure CML (Never best initial ttt).
(B) CHRONIC LYMPHOCYTIC LEUKEMIA (CLL):
________________________________________
. ++ WBCs (LYMPHOCYTES predominant).
. Pt > 50 ys.
. Asymptomatic - found on routine testing.
. Dx -> Peripheral blood smear -> SMUDGE cells "Ruptured nuclei of lymphocytes".
.
.
.
.
.

Stage
Stage
Stage
Stage
Stage

0
1
2
3
4

-> ++ WBCs.
-> Enlarged LNs.
-> Enlarged Spleen.
-> Anemia.
-> -- platelets.

.THE PRESENCE OF THROMBOCYTOPENIA = STAGE 4 -> POOR PROGNOSIS.


. Tx -> Stage 0 & 1 -> No ttt.
. Other stages -> FLUDARABINE + RITUXIMAB.

(C) HAIRY CELL LEUKEMIA:


_________________________
. Middle aged pt with Pancytopenia & massive splenomegaly.
. Dx -> Tartarate resistant acid phosphatase (TRAP) smear -> HAIRY CELLS !
. Named so bec. the lymphocytes have fine hair like irregular projections.
. The bone marrow may be become fibrotic -> Un-successful aspirates (DRY-TAP).
. Tx -> CLADRIBINE.
(D) MYELOFIBROSIS:
___________________
. Same as hairy cell leukemia -> BUT -> NORMAL TRAP level !
. Dx -> TEAR DROP shaped cells on smear.
. Tx -> LENALIDOMIDE or THALIDOMIDE , BMT.
(E) POLYCYATHEMIA VERA (Pvera):
________________________________
. Plethoric face.
. Headache - Blurring of vision - dizziness - fatigue.
. Pruritis happening after a hot bath or shower.
. pruritis due to release of histamine from basophils.
. Hypertension can be one of the presentations of polycythemia.
. ++ incidence of peptic ulcerations (++ Histamine release from basophils).
. ++ incidence of gouty arthritis ( ++ cell turnover).
. Dx -> CBC "MARKEDLY HIGH HEMATOCRITE & low MCV" !!!
. Dx -> ABG " Absence of hypoxia".
. Dx -> -- Erythropoietin (Most important).
. ++ WBCs & ++ Platelets.
. ++ B 12 & ++ LAP levels.
. MARKEDLY HIGH HEMATOCRITE may lead to THROMBOSIS.
. Ass. with JAK 2 mutation.
. Tx -> PHLEBOTOMY. (HCT < 45%).
. Tx -> Hydroxyurea to -- cell count.
. Tx -> Give daily Aspirin to prevent Thrombosis.
. High risk of thrombosis -> Due to ++ platelet count.
. High risk of bleeding -> Due to impaired platelet function
. N.B. . HERIDITARY TELANGIECTASIA = OSLER - WEBER - RENDU $YNDROME:
____________________________________________________________________
. Diffuse telangiectasia + Recurrent epistaxis + Wide spread AV MALFORMATIONs.
. AV malformations in lung -> Blood shunt from Rt to Lt side of the heart.
. AV shunts -> Chronic hypoxemia -> Reactive polycythemia (++ HCT).
(F) ESSENTIAL THROMBOCYTOPENIA:
________________________________
. ++ PLATELET cell count.
. Headache - visual disturbances - pain in hands (Erythromyelalgia).
. Most common cause of death -> Bleeding & thrombosis.
. Tx -> Hydroxyurea to -- cell count.
. Tx -> Give daily Aspirin to prevent Thrombosis.
. PLASMA CELL DISORDERS:
________________________
. ALL are characterized by ++ serum protein with normal albumin (GAMMA GAP).
_____________________________________________________________________________

(A) MULTIPLE MYELOMA:


______________________
. Monoclonal proliferation of plasma cells.
. CRAB -> Calcium ++ & Renal impairment & Anemia & Bones pain, lytic lesions & #s.
-> Skeletal survey -> Punched out osteolytic lesions (if osteoblastic = Prostate cancer).
-> Serum immune electrophoresis -> Abnormal M spike.
-> Serum protein electrophoresis (SPEP) -> ++ Mono-clonal Antibodies (IgG).
-> Urine protein electrophoresis (UPEP) -> BENCE-JONES proteins.
-> Peripheral smear -> ROULEEAUX formation of blood cells.
-> ++ Ca.
-> ++ ESR > 100.
-> B2 micro-globulin (detect prognosis).
-> ++ BUN & Creatinine (RF).
-> Most specific test -> Bone marrow biopsy -> ++ PLASMA cells > 10 % !
. Most common cause of death -> INFECTIONS & RENAL FAILURE.
. susceptible to infections due to -- WBCs & -- ability to produce effective antibodies,
. due to BM infiltration with malignant plasma cells.
. Tx -> Melphalan & steroids.
. Tx -> Most effective ttt -> Autologous stem cells bone marrow transplantation.
. Tx -> Hypercalcemia -> Hydration & diuresis.
. Tx -> Bone #s -> Biphosphonates.
. Tx -> RF -> hydration.
. Tx -> Anemia -> Erythropoietin.
. Tx -> Prophylaxis against infections with vaccinations (Pnemovax).
(B) MONO-CLONAL GAMMOPATHY OF UNKNOWN SIGNIFICANCE (MGUS):
___________________________________________________________
. Asymptomatic ++ of IgG on SPEP. (due to ++ ptn in old age).
. Elderly pt > 70 ys.
. MGUS require METASTATIC SKELETAL BONE X-RAY to exclude lytic lesions sugesting MM !
. No ttt.
(C) WALDENSTROM's MACRO-GLOBULINEMIA:
______________________________________
. ++ Ig M -> Hyperviscosity.
. Blurry vision (Dilated segmented tortuous retinal veins)
. confusion & headache.
. Enlarged Lns & Hepatosplenomegaly.
. Dx -> Serum viscosity level -> ++.
. Dx -> SPEP -> ++ Ig M SPIKE !
. Tx -> PLASMAPHARESIS.
. Same ttt as CLL -> Fludarabaine.
(D) APLASTIC ANEMIA:
_____________________
. PANCYTOPENIA with no identified etiology.
. Tx -> Pt < 50 ys & has a match - > BMT.
. Tx -> Pt > 50 ys & NO match -> Anti-thymocyte globulin & cyclosporine.
. N.B. MONO-CLONAL GAMMOPATHY OF UNKNOWN SIGNIFICANCE Vs MULTIPLE MYELOMA:
__________________________________________________________________________
(MGUS)
(MM)
. Absence of anemia.

. Anemia.

.
.
.
.
.

Hypercalcemia.
. Hypercalcemia.
Renal insuffeciency.
. Renal insuffeciency.
Lytic lesions in bones.
. Lytic lesions in bones.
Serum monoclonal protein < 3 g/dl.
. Serum monoclonal protein > 3 g/dl.
< 10 % plasma cells in the bone marrow.
. > 10 % plasma cells in the bone marrow.
. ++ B 2 microglobulin.

. MGUS require METASTATIC SKELETAL BONE X-RAY to exclude lytic lesions sugesting MM !
. LYMPHOMA:
___________
. ENLARGED LNs (Mostly CERVICAL).
. Anemia due to BONE MARROW INFILTRATION by CANCEROUS CELLS !
. Two types_________________________________
|
____________________________________________________
|
|
.HODGKIN's LYMPHOMA
.NON-HODGKIN's LYMPHOMA
.__________________
._______________________
. Best initial test -> EXCISIONAL LN BIOPSY.
. SAME.
. REED STERNBERG CELSS.
. NO !
. STAGING by Contrast CT & CXR & BM biopsy:
. SAME.
-> 1 -> Single LN group.
-> 2 -> 2 LN groups on one side of diaphragm.
-> 3 -> LN invovement on both sides of diaphragm.
-> 4 -> Wide-spread disease = B symptoms (fever - weight loss - Night sweats).
. 90 % of cases present with Stages 1 & 2.
. 90 % of cases present with stages 3 & 4.
. Tx -> Stages 1 & 2 with no B symptoms -> radiotherapy.
. Tx -> Stages 3 & 4 with B symptoms ----> chemotherapy.
. "ABVD" ->
. "CHOP" ->
- Adriamycin.
- Cyclophosphamide.
- Bleomycin.
- Hydroxyadriamycin.
- Vinblastine.
- Oncovin.
- Dacarbazine.
- Prednisone.
. TUMOR LYSIS $YNDROME:
_______________________
. Ass. with tumors with high cell turn-over.
. Ex: Poorly differentiated lymphoma "Burkitt's lymphoma" & Leukemias "ALL".
. HYPER {phosphatemia - kalemia - uricemia}.
. HYPO {Calcemia}.
. Both potassium & phosphate r intra-cellular ions, so cell break down -> their release.
. Released phosphate binds calcium leading to hypocalcemia.
. Degradation of cell proteins -> ++ uric acid levels.
. TL$ may lead to fatal arrhythmias, ARF & sudden death.
. Tx -> Allopurinol greatly reduces the possibility of acute urate nephropathy.
. ASYMPTOMATIC LOCALIZED LYMPHADENOPATHY:
_________________________________________
. Several non-tender rubbery cervical Lymph nodes.
. Most commonly in upper respiratory tract infection.
. Commonly observed in children & youn adults.

. Tx -> OBSERVATION !
. GASTRIC MALT = MUCOSA ASSOCIATED LYMPHOID TISSUE MANAGEMENT:
______________________________________________________________
. ERADICATION OF HELICOBACTER-PYLORI !
. TRIPLE THERAPY (OMEPRAZOLE - CLARITHROMYCIN - AMOXICILLIN).
. INFECTIOUS MONONUCLEOSIS (IM):
________________________________
. Acute, benign, self limiting lymphoproliferative condition.
. Caused by Epstein Barr virus (EBV).
. EBV is transmitted by close contact to infected oro-tracheal secretions.
. Extreme fatigue - malaise - sore throat - fever - generalized maculopapular rash.
. Posterior cervical lymphadenopathy & palatal petichae.
. Splenomegaly is common.
. Contact sports sh'd be avoided to prevent splenic rupture.
. Leukocytosis with variant lymphocytes (Atypical lymphocytes).
. Dx -> HETEROPHIL ANTIBODIES (SENSITIVE & SPECIFIC).
. HETEROPHIL ANTIBODIES may be NEGATIVE in EARLY disease.
. -ve HETEROPHIL ANTIBODIES doesn't exclude IM.
. COAGULATION DISORDERS:
________________________
________________________
. 1 . VON WILLEBRAND's DISEASE (VWD):
______________________________________
. NORMAL PALTELET COUNT & ABNORMAL PLATELET FUNCTION.
. -- VWF -> -- platelet activation.
. Superficial bleeding from the skin & mucosal surfaces e.g. gingiva, gums & vagina.
. Epistaxis is a common presentation & worsens with the use of aspirin.
. -- VWF -> destabilizes factor 8 -> ++ aPTT.
. Dx -> Most accurate -> RISTOCETIN COFACTOR ASSAY & VWF level.
. If the level of VWF is normal -> RISTOCETIN test will tell if it is working properly.
. Tx -> DESMPORESSIN or DDAVP -> Will release sub-endolthelial stores of VWF & factor 8.
. If DDAVP failed -> Factor 8 replacement.
. TYPES OF BLEEDING:
____________________
____________________
. A -> PLATELET TYPE BLEEDING:
______________________________
. petichae - epistaxis - purpura - gingiva - gums - vagina.
. B -> FACTOR TYPE BLEEDING:
____________________________
. Hemoarthrosis - Hematoma.
. A -> PLATELET TYPE BLEEDING:
______________________________
. 1 . IDIOPATHIC IMMUNE THROMBOCYTOPENIA (ITP):
________________________________________________
. Immune destruction of platelets.

.
.
.
.
.
.
.
.
.
.
.
.
.
.
.
.
.
.
.

Platelets count < 50000.


Ig G auto-antibodies against the paltelet membrane glycoproteins 2B & 3A receptors.
Mostly occur in children.
Pt may be asymptomatic.
Mucocutaneous bleeding, ecchymosis & or petichiae.
May follow acute viral infection.
Dx -> ISOLATED THROMBOCYTOPENIA < 100000/ml.
Dx -> Peripheral smear sh'd be done to exclude other causes e.g. TTP.
Dx -> Bone marrow -> ++ Megakaryocytes (platelet precursors).
Dx -> U/$ -> NORMAL SPLEEN.
Dx -> Check for hepatitis C & HIV infections.
Dx -> Diagnosis of exclusion.
Tx -> Platelets > 30000 , without bleeding -> Observe !
Tx -> Platelets < 30000 , or bleeding -> Corticosteroids Prednisone (1st line therapy).
Tx -> Platelets < 20000 ! with eistaxis, melena & intracranial hemorrhage -> IVIG.
IVIG: INTRAVENOUS IMMUNOGLOBULIN -> when platelet < 20000 & life threatening condition.
Recurrent episodes -> SPLENECTOMY.
No response to splenectomy -> Romiplostim.
N.B. HCV & HIV tests as ITP may be the initial presentation of HIV infection.

. 2 . UREMIA INDUCED PLATELET DYSFUNCTION:


___________________________________________
. Uremia prevents palatelets from degranulation.
. NORMAL PLATELET COUNT + PLATELET TYPR BLEEDING + RENAL FAILURE PT.
. NORMAL Ristocetin test & VWF level.
. Tx -> Desmopressin - DDAVP.
. B -> FACTOR TYPE BLEEDING = COLTTING FACTOR DEFECIENCIES:
___________________________________________________________
* FACTOR 8 = HEMOPHILIA A:
___________________________
. Joint bleeding (Hemarthrosis) or hematoma on a MALE child.
. Dx -> Mixing study then specific factor level.
. Tx -> Minor defeciency -> DDAVP.
. Tx -> Severe defeciency -> Factor 8 replacement.
* FACTOR 9 = HEMOPHILIA B:
___________________________
. Joint bleeding (Hemarthrosis) or hematoma (less common than factor 8 def.).
. Dx -> SAME.
. Tx -> Factor 9 replacement.
* FACTOR 11:
_____________
. Rare bleeding with trauma or surgery.
. Dx -> Same.
. Tx -> FFP Fresh frozen plasma.
. FACTOR 12:
____________
. No bleeding.
. Dx -> Same.
. Tx -> No ttt necessary.
. SENILE PURPURA:

_________________
. occurs in areas susceptible to traumas in elderly (Dorsum of the hands & forearms).
. Due to PERIVASCULAR CONNECTIVE TISSUE ATROPHY .
. Lesions rapidly resolve leaving a brownish discolouration from hemosiderin deposition.
. Requires no ttt.
. HEPARIN INDUCED THROMBOCYTOPENIA (HIT):
___________________________________________
. HEPARIN exposure > 5 days then,
. Platelet count reduction > 50 % from baseline.
. Despite -- platelet count ->HIT is highly thrombogenic -> Arterial & venous thrombosis.
. Necrotic skin lesions at heparin injection sites.
. Acute systemic anaphylactoid recation after heparin.
. Dx -> SEROTONIN RELEASE ASSAY (GOLD STANDARD).
. TTT is started on clinical suspicion before serotonin assay.
. Tx -> CESSATION OF ALL HEPARIN PRODUCTS !
. Tx -> Start Argatroban (direct thrombin inhibitor).
. WARFARIN INDUCED SKIN NECROSIS:
_________________________________
. More common in females.
. Common sites: breasts , buttocks, thighs & abdomen.
. Initial complaint is PAIN FOLLOWED BY BULLAE FORMATION & SKIN NECROSIS.
. Occurs within weeks after starting therapy.
. Tx -> Discontinue warfarin - Give heparin to continue anticoagulation - Give Vit. K.
. Acetaminophen, NSAIDs & Amiodarone may potentiate the anticoagulant effect of warfarin.
. VITAMIN K DEFECIENCY BLEEDING:
________________________________
. Pt kept NPO NOTHING PER ORAL for a prolonged period of time & receiving Antibiotics.
. OR Newborn hadn't received prophylactic vit. K (Home-born) for prevention of hemorrhage
. ++++ PT > ++ PTT.
. THROMBOPHILIA = HYPERCOAGULABLE STATES:
_________________________________________
_________________________________________
(1) LUPUS ANTI-COAGULANT or ANTI-CARDIOLIPINS ANTIBODIES:
__________________________________________________________
. Venous thrombosis.
. ++ aPTT (PROLONGED PARTIAL THROMBOPLASTIN TIME).
. Normal PT, BT, platelet count & VWF.
. Spontaneous abortion. (Due to promotion of arterial & venous thrombosis).
. False +ve VDRL & True -ve FTA-ABS (Excluding $yphilis as a cause of recurrent abortions
. Dx -> Mixing study.
. Dx -> Russel Viper venom test (Most accurate).
. Tx -> Heparin (LMWH) followed by warfarin.
(2) PROTEIN C DEFECIENCY:
__________________________
. Skin necrosis with the use of Warfarin.
. Dx -> Ptn C level.
. Tx -> Heparin followed by Warfarin.

(3) FACTOR 5 LEIDEN MUTATION:


______________________________
. Most common cause of thrombophilia.
. Dx -> Factor 5 mutation test.
. Tx -> Heparin followed by Warfarin.
(4) ANTI-THROMBIN DEFECIENCY:
______________________________
. No change in the aPTT with a blous of IV heparin !
. Dx -> Anti-thrombin 3 level.
. Tx -> Large amounts of heparin or direct thrombin inhibitor followed by warfarin.
. DIC = DISSEMINATED INTRAVASCULAR COAGULATION:
_______________________________________________
. Doesn't occur in healthy pts.
. Ass. with sepsis - burns - snake bites - cancer - Abruptio placenta or AF Embolism.
. BLEEDING related to CLOTTING FACTORS DEFECIENCY & THROMBOCYTOPENIA !!
. ++ both PT & PTT.
. -- platelet count.
. ++ D-dimer & FDP (fibrin degradation products).
. -- fibrinogen level (consumed).
. Tx -> REPLACEMENT by FFP FRESH FROZEN PLASMA (Contains both palatelets & clot.factors).
. TRANSFUSION REACTIONS:
________________________
________________________
. 1 . ABO INCOMPATIBILITY:
___________________________
. Acute symptoms of hemolysis WHILE the transfusion is occuring.
. Ex -> DURING a transfusion, the pt becomes hypotensive & tachycardic.
. Back & chest pain & dark urine.
. ++ LDH & bilirubin.
. -- Haptoglobin.
. 2 . TRANSFUSION RELATED ACUTE LUNG INJURY (TRALI) = LEUKO-AGGLUTINATION
REACTION:
____________________________________________________________________________________
. Acute Shortness of breath from antibodies in the donor blood against the repient WBCs.
. Ex -> 20 mins after a pt. receives a blood transfusion, the pt becomes short of breath.
. Transient infiltrates on CXR.
. All symptoms resolve spontaneously.
. 3 . IgA DEFECIENCY:
______________________
. presents with anaphylaxis !
. In the future, use blood donations from an IgA defecient donor or washed RBCs.
. Ex -> As soon as the pt. received transfus., he becomes SOB, hypotensive & tachycardic.
. NORMAL LDH & BILIRUBIN.
. 4 . MINOR BLOOD GROUP INCOMPATIBILITY:
_________________________________________
. To kell, Duffy, Lewis or Kidd antigens or Rh incompatibility !
. Delayed jaundice.
. No specific therapy.

. Ex -> A few days after transfusion, the pt becomes jaundiced.


. The hematocrit doesn't rise with transfusion & he is generally without symptoms.
. 5 . FEBRILE NON-HEMOLYTIC REACTION:
______________________________________
. Small rise in temperature.
. No ttt required.
. Reaction against donor WBCs antigens.
. prevented by using filtered blood transfusions in the future to remove WBCs antigens.
. Ex -> A few hours after transfusion, the pt becomes febrile with rise 1 degree in temp.
. No evidence of hemolysis.
. N.B. HYPOCALCEMIA MANIFESTATIONS (PARESTHESIA) AFTER BLOOD TRANSFUSION:
_________________________________________________________________________
. Occurs in pts who receive more than one blood volume of blood transfusion or packed RBCs
over 24 hours may develop ++ plasma level of CITRATE (A SUBSTANCE ADDED TO STORED
BLOOD) -> CHELATION of Ca & Mg by citrate -> -- Ca -> Paresthesia.
. MIGRATORY SUPERFICIAL THROMBOPHLEBITIS = TROUSSEAU's $YNDROME:
________________________________________________________________
. Hypercoagulable disorder.
. Un-explained superficial venous thrombosis at un-usual sites e.g. arm & chest area.
. Associated with OCCULT VISCERAL MALIGNANCY !
. Ex. Cancer involving the pancreas "most common".
. The tumor releases mucin that react with platelets forming platelet rich micro-thrombi.
. H/O of heavy smoking - abdominal pain - migrattory thrombophlebitis.
. Dx -> CT Abdomen -> searching for an occult tumor (pancreatic carcinoma).
. GLUCAGONOMA:
______________
. Necrolytic migratory erythema:
---------------------------------> Erythematous papules / plaques on face, perineum & extremities.
-> Enlarge & coalese with central clearing & blistering & crusting & scaling.
. Diabetes Mellitis:
---------------------> Mild hyperglycemia easily controlled with diet & oral agents.
-> Don't require insulin !
. GIT symptoms:
----------------> Diarrhea - anorexia abdominal pain - occasional constipation.
. Other findings:
------------------> Weight loss.
-> Neuropsychiatric (ataxia - dementia - proximal ms weakness).
-> Associated wih venous thrombosis !
. Dx -> Hyperglycemia with ++ GLUCAGON > 500 mg/dl.
. Dx -> Normocytic Normochromic anemia.
. Dx -> CT or MRI to detect the tumor.
. SMOKER + HORNER's $ = LUNG CANCER:
____________________________________
. Horner's $ -> Miosis, ptosis & anhydrosis.
. Simple CXR is the best next step to detect lung cancer.

. SMALL CELL (OAT) LUNG CARCINOMA:


__________________________________
. associated with $yndrome of inappropriate ADH secretion & ACTH production.
. SIADH (Hyponatremia & -- serum osmolality & ++ urine osmolality).
. Metastasis is already present at the time of diagnosis !
. Tx of SIADH -> Mild (Fluid restriction) or Severe (Hypertonic saline).
. ++ ACTH -> Hypertension - Hypokalemia - Metabolic alkalosis - Hyperpigmentation.
. e'out other manifests of Cushing $ (Moon face - Dorsal hump - central obesity - striae)
. SQUAMOUS CELL CARCINOMA OF THE LUNG:
______________________________________
. Significant smoking H/O.
. HYPERCALCEMIA ++ Ca -> (sCa++mous) !
. Hilar mass.
. ADENOCARCINOMA OF THE LUNG:
_____________________________
. Least association with smoking.
. Located peripherally.
. Consists of columnar cells growing along the septa.
. Presents as a sloitary nodule !
. May be detected incidentally.
. FEBRILE NEUTROPENIA:
______________________
. Single temperature > 38.3c or sustained temp. > 28 c for > 1 hour in a neutropenic pt.
. Neutropenia = Absolute neutrophil count < 1500 cells/ml.
. Mild (<1500) - Moderate (<1000) - Severe (<500).
. All require antibiotics.
. Moderate & severe neutropenia require hospitalization.
. The most common invading organism is PSEUDOMNAS AERUGINOSA.
. Tx -> HOSPITALIZATION,
. BLOOD CULTURES followed by IV CEFEPIME or PIPERACILLIN TAZOBACTAM.
. SQUAMOUS CELL CARCINOMA OF THE MUCOSA OF THE HEAD & NECK:
___________________________________________________________
. ALCOHOLIC SMOKER.
. HARD UNILATERAL NON-TENDER CERVICAL OR SUBMANDIBULAR lymph node.
. Dx -> Best initial -> PAN-ENDOSCOPY.
. Tx -> Radical neck dissection.
. TESTICULAR TUMOR:
___________________
. Painless hard mass in testicle.
. Suggestive ultrasound.
. Tx -> Orchiectomy (Removal of the testicle & its associated cord).
. FNAC or trans-rectal biopsy are contr'd bec. the risk of spillage of cancer cells.
. FIBROCYSTIC DISEASE OF THE BREAST:
____________________________________
. Rubbery, firm, mobile & pinful mass in a young pt.
. More pain during menses.
. Aspiration of the cyst -> Clear fluid with diasppearance of the mass.

. Tx -> Observation of the pt 4 - 6 weeks.


. Only send the fluid for cytology if their is blood or foul smelling.
. INVASIVE DUCTAL BREAST CARCINOMA:
___________________________________
. TNM staging is the single most important prognostic tool for breast cancer.
. Prognosis is best detected using ONCOGENE AMPLIFICATION by FISH !
. FISH = Fluorescent in situ hybridization.
. Over-expression of the oncogene HER2 can be detected by FISH = (Worse prognosis).
. Positivity predicts a +ve response to TRASTUZUMAB & ANTHRACYCLINE chemotherapy.
. ER +ve & PR +ve are GOOD prognostic features.
. Tx -> TRASTUZUMAB (Herceptin) is used to ttt breast cancer that is HER2 +ve.
. Trastuzumab side effect -> CARDIOTOXICITY.
. ECHOCARDIOGRAPHY is recommended before ttt to assess the ejection fraction.
. INFLAMMATORY CARCINOMA OF THE BREAST:
_______________________________________
. Breast cancer sh'd be considered whenever a pt. without a prior H/O of skin disease,
. develops a breast rash that is non-responsive to standard ttt !
. When invasive ductal carcinoma is severe, it can infiltrate into the dermal lymphatics,
. resulting edema - erythema - warmth of the entire breast (inflammatory carcinoma).
. When the rash is localized to the nipple & has an ulcerating eczymatous appearance,
. the primary cosideration should be PAGET's DISEASE of the breast.
. 5 % of pts with PAGET's disease have an underlying breast cancer "ADENO-CARCINOMA".
. Skin biopsy -> Large cells surrounded by clear halos.
. TAMOXIFEN:
____________
. Has mixed agonist & antagonist activity on Estrogen receptors.
. It is used as an adjuvant therapy for early stage breast cancer,
. It reduces the risk of recurrence of the original cancer,, BUT,,
. ++ risk of developing of another cancer in the other breast !
. Estrogenic effects ++ risk of ENDOMETRIAL cancer & Venous thrombosis.
. SQUAMOUS CELL CARCINOMA of the SKIN:
______________________________________
. Any scar that develops into a non-healing, painless, bleeding ulcer.
. Sun-exposed or burned areas are typically involved.
. Rough scaly nodules that can ulcerate & metastasize.
. Tar derivatives (tobacco smoke) & chronic radiation exposure are predisposing factors. . Dx > PUNCH BIOPSY.
. Tx -> Surgical removal with wide excision of the skin around the tumor.
. BASAL CELL CARCINOMA of the SKIN:
___________________________________
. Most common form of skin cancers in USA.
. Open sore that bleeds, oozes or crusts & remains open for 3 or more weeks.
. Reddish patch or irritated area, shiny, waxy, scar like with elevated rolled borders.
. Remains local - Never spreads.
. Tx -> Mohs surgery (Microscopic shaving) -> 1-2 mm of clear margins are excised.
. Highest cure rate with Mohs surgery.
. Indicated in lesions located at critical areas e.g. perioral region, nose, lips & ears.
. ESOPHAGEAL CARCINOMA:
_______________________

.
.
.
.
.
.

Heart burn - significant weight loss - Regurgitation of food - fatigue - smoking H/O.
Age > 50 ys.
Histological types -> Squamous cell carcinoma & Adenocarcinoma.
SCC -> Ass. with smoking & alcohol consumption.
Adenocarcinoma -> Barret's esophagus (GERD complication).
Dx -> BARIUM SWALLOW followed by ENDOSCOPY.

. MYASTHENIA GRAVIS:
____________________
. Ptosis & double vision by the end of the day.
. Dx -> EMG -> Decremental response in compound action potential.
. Dx -> Acetyl choline receptor antibody test +ve.
. Dx -> CT scan chest sh'd be done in all newly diagnosed MG pts searching for a THYMOMA.
. COLON CANCER SCREENING:
_________________________
* ROUTINE:
___________
-> Colonoscopy at 50 ys then every 10 ys.
* SINGLE FAMILY MEMBER WITH COLON CANCER:
__________________________________________
-> Colonoscopy at 40 ys,
-> or .. 10 ys earlier than the age at which the family member had cancer,
-> whichever is earlier then every 10 ys.
* HNPCC 3 family members & 2 generations & 1 premature (<50ys):
________________________________________________________________
. Colonoscopy at 25 then every 1-2 ys.
. HNPCC is ass. with high risk of extra-colonic tumors (ENDOMETRIAL CARCINOMA).
. It is a part of LYNCH $ type 2.
* FAMILIAL ADENOMATOUS POLYPOSIS:
__________________________________
. Sigmoidoscoy at 12 then every 1-2 ys.
. 100 % risk of cancer.
. ELECTIVE PROCTO-COLECTOMY sh'd be done.
* JUVENILE POLYPOSIS - PEUTZ JEHGERs - TURCOT's $ - GARDNER's $:
_________________________________________________________________
. No additional screening.
. ULCERATIVE COLITIS:
_____________________
. Colonoscopy once the diagnosis is established for 8 ys then repeated every 1 - 2 ys.
. MEN 1 = MULTIPLE ENDOCRINE NEOPLASIA 1:
_________________________________________
. Paratyhroid (>90%) -> parathyroid hyperplasia -> primary hyperparathyroidism -> ++ Ca.
. Pituitary -> prolactinoma.
. Pancreas / GIT -> Gastrinoma "ZE $" - insulinoma - VIPoma - Glucagonoma.
. KEY WORDS-> INTRACTABLE ULCERATION + HYPERCALCEMIA -> ZE$ & Hyperparathyroidism
= MEN1.

. POST-SPLENECTOMY SEPSIS:
__________________________
. Asplenic pt have defective PHAGOCYTOSIS !
. impaired antibody mediated opsonization in phagocytosis.
. High risk of overwhelming infection by encapsulated organisms,
. e.g. Strept. pneumoniae, N. menengitidis & H. influenzae.
. DEEP VENOUS THROMBOSIS = DVT MANAGEMENT:
__________________________________________
. Presents with pain, swelling & discoloration.
. D.D. -> Ruptured Baker's cyst - venous insuffeciency - post-thrombotic $ & cellulitis.
. Failure to anti-coagulate the pt may lead to pulmanary embolism.
. Modified Wells criteria is a pretest propability of DVT:
-> Previous DVT.
-> Active cancer.
-> Recent immobilization.
-> Localized tendrness along vein distribution.
-> Swollen leg.
-> Pitting edema.
.
. Pre-test propability of DVT using WELLs criteria
.__________________________________________________
.|
._________________________________________
.|
.|
. Not likely
. Likely
.____________
.________
|
.|
. D-Dimer test (+)--------------->. Compression Ultrasonography
.______________
._____________________________
.|
.|
.|
._________________________________
.|
.|
.|
(-)
(+)
(-)
.|
.|
.|
. Un-likely to have DVT ! (TTT with Heparin & Warfarin) (Contrast Venography)
. Clinical features of METASTATIC BRAIN CANCER:
_______________________________________________
. Incidence -> Lung > Breast > Un-known primary > Melanoma > Colon.
. Primary solitary brain metastases -> BREAST - COLON - RENAL CELL CARCINOMA.
. Multiple brain metastasis -> LUNG - MALIGNANT MELANOMA.
. Brain metastasis are the most common intracranial tumors.
. Headache - nausea&vomiting - seizures & focal neurological symptoms (weakness-aphasia).
. TYPES OF THERAPIES !
______________________
. ADJUVANT -> TTT given in addition to standard therapy.
. INDUCTION -> Initial dose of ttt to rapidly kill tumor cells.
. CONSOLIDATION -> TTT given after induction therapy to -- the tumor burden.
. MAINTENANCE -> Given after induction & consolidation ttt to kill residual tumor cells.
. NEO-ADJUVANT -> Given before the standard therapy for a particular disease.
. SALVAGE -> TTT for a disease when the standard ttt fails.
. MECHANISM OF ACTION of IMP. DRUGS:

____________________________________
. HEPARIN -> "Anti-coagulant" -> ++ ANTI-THROMBIN 3 -> -- Thrombin, 9 & 10.
. WARFARIN -> "Anti-coagulant" -> -- synthesis of Vit. K dep. factors 1972, ptn C & S.
. ASPIRIN -> "Anti-platelet" -> -- cyclo-oxygenase 1 -> -- TXA 2 synthesis.
. CLOPIDOGREL ->"Anti-platelet" -> block platelet sufrace receptors -> -- platelet activ.
. ANDROGEN ABUSE:
_________________
. Atheletes commonly abuse androgen to enhance performance in competetive sports.
. Ex: testosterone & synthetic androgen.
. ++ Muscle mass & strength & ++ physical exercise intolerance.
. Men SEs -> -- testicular function - -- sperm production - testicular atrophy.
. Men SEs -> Gynecomastia - mood distaurbance - agrressive behavior.
. Women SEs -> ++ Acne - Hirsutism - deepening of voice - menstrual irregularities.
. Labs -> Erythrocytosis & ++ HCT - Hepatotoxicity - Dyslipidemia ( -- HDL & ++ LDL).
. SOLITARY PULMONARY NODULE APPROACH:
_____________________________________
. 3 cm or less coin-shaped lesion,
. in the middle to lateral one third of the lung.
. Surrounded by normal parenchyma.
. Most of them are benign !
. Calcifications of the nodule favors a benign lesion !
. POP CORN calcification -> Hamartoma.
. BULLS EYE -> Granuloma.
. Low risk pts (< 40 ys & non smokers) -> Not a sign of immediate alarm.
. Best approach -> ASKING FOR AN OLD X-RAY !
. If no change in it for the last 12 months -> Benign.
. Followed by CXR every 3 months for the next 12 months -> If no growth or syms -> Leave!
. High risk pts (> 40 ys & smokers) -> Full investigation work up !
. GIANT CELL TUMOR OF BONE:
___________________________
. Dx -> X-ray -> Expansile & eccentric lytic area with SOAP & BUBBLE APPEARANCE.
. Benign but locally aggressive skeletal neoplasm.
. Young adults.
. pain, swelling & -- range of motion at the involved site.
. Pathologic #s are common due to thinning of the bone cortex in weight bearing areas.
. Affect the epiphysis of the long bones (distal femur & proximal tibia).
. MRI -> Tumor containing both cystic & hemorrhagic regions.
. Tx -> Surgery.
. COMPRESSION OF THE THECAL SAC or SPINAL CORD by a TUMOR:
__________________________________________________________
. Ex -> H/O of prostatic cancer + Acute onset back pain.
. Rapid recognition is crucial to avoid paralysis.
. Point tenderness over the spine process L5 & S1.
. Imbalance, ms weakness & week rectal sphincter tone.
. Dx -> MRI spine BUTTTTTTTTTTTTTTTTTTTTTTTTTTTTTTTTTTTTTT
. Initial management with glucocorticoids (dexamethasone) is crucial to -- swelling,
. as an attempt to preserve neurological function while awaiting the results of imaging.
. POST-SPLENECTOMY RECOMMENDATIONS:
___________________________________
. Risk for sepsis after splenectomy is present UP TO 30 years & more !

. Anti-pneumococcal, Haemophilus & meningococcal vaccine sh'd be given,


. several weeks before splenectomy.
. Daily oral penicillin prophylaxis for 3 - 5ys following splenectomy.
. CANCER OVARY:
_______________
. NO screening tests !
. Serum CA 125 & pelvic U/$ may help in diagnosis.
. ++ HOMOCYSTEINE levels -> HYPERCOAGULABILITY:
_______________________________________________
. Vit. B6, Folate & Vit. B12 are involved in the metabolism of homocysteine.
. Vit. B6 lowers homocysteine levels by acting as a cofactor for cystathionine B-synthase
. which metabolizes homocysteine to cystathionine.
. PAIN CONTROL IN CANCER PATIENTS:
__________________________________
. Give the appropriate pain medication to cancer pts e.g. Ibuprofen.
. If no response .. Use a narcotic drug if it is the most appropriate.
. Use SHORT ACING IV MORPHINE.
. MANAGEMENT OF CHEMOTHERAPY INDUCED NAUSEA & VOMITING:
_______________________________________________________
. Volume repletion + SEROTONIN ANTAGONIST (Block 5HT3 recptors).
. MANAGEMENT OF CANCER RELATED ANOREXIA & CACHEXIA:
___________________________________________________
. PROGESTERONE ANALOG -> ++ Appetite & ++ weight gain.
. PROSTATE CANCER with BONE METASTASIS:
_______________________________________
. Tx -> FOCAL EXTERNAL BEAM RADIATION (If the pt under-went orchiectomy !).
. BRAIN METASTASIS:
___________________
. SOLITARY -> SURGICAL RESECTION followed by whole brain radiation.
. Multiple -> Palliative whole brain radiation.
. MANAGEMENT OF HYPERCALCEMIA DUE TO UNDERLYING MALIGNANCY:
___________________________________________________________
. Asymptomatic or mild -> Ca < 12 mg/dl:
_________________________________________
. Just avoid thiazides, lithium use, volume depletion or prolonged bed rest.
. Severe -> Ca > 14 mg/dl:
___________________________
. Immediate ttt -> Normal saline hydration + Calcitonin.
. Long term ttt -> Biphosphonate (Zoledronic acid).
. D.D. of ANTERIOR MEDIASTINAL MASS (4 Ts):
___________________________________________
. Thymoma - Teratoma - Thyroid neoplasm - Terrible lymphoma.
. Germ cell tumors are classified into seminomatous & non-seminomatous.
. Non-seminomatous (Yolk sac tumor - choriocarcinoma - embryonal carcinoma).
. A mixture of all types of non seminomatous germ cell tumors = MIXED GERM CELL TUMOR.

. Seminoma -> ++ B-HCG & Normal AFP.


. Non seminoma (MIXED) -> ++ B-HCG & ++ AFP.
. HEMOCHROMATOSIS:
__________________
. NEW-ONSET DIABETES MELLITUS + ARTHROPATHY + HEPATOMEGALY.
. Due to abnormal ++ intestinal absorption of iron -> ++ iron deposition in tissues.
. Damage to organs ex. liver, pancreas , heart & pituitary.
. Liver -> Hepatomegaly -> Liver cirrhosis -> Hepatocellular carcinoma.
. Pancreas -> Bronze D.M.
. Pituitary -> Hypogonadism.
. Heart -> Restrictive heart failure.
. Joints -> Arthropathy.
. Skin -> Hyperpigmentation.
. Dx -> Iron studies -> ++ fe, ++ ferritin, ++ transferrin saturation.
. Dx -> Liver biopsy -> confirm the diagnosis.
. LEAD POISONING:
_________________
. Lead bind to erythrocytes & disrupts hemoglobin synthesis -> Microcytic anemia.
. Due to chronic lead exposure & toxicity.
. Acute exposure -> Abdominal pain & constipation.
. Chronic exposure -> Fatigue, iiritability & insomnia.
. Hypertension - Sensori-motor neuropathies - Neuropsychiatric disturbances - Nephropathy
. OCCUPATIONAL HISTORY IS VERY IMPORTANT !
. BATTERY MANUFACTURING - PLUMBING - MINING - PAINTING - PAPER HANGING & AUTOREPAIR.
. Dx -> Blood Lead level.
. Dx -> Peripheral smear -> BASOPHILIC STIPPLING.
. Tx -> CHELATION THERAPY.

Dr. Wael Tawfic Mohamed


_________________________

HEPATOLOGY TiKi TaKa


______________________
. LIVER DISEASES:
_________________
_________________
. ACUTE HEPATITIS:
__________________
__________________
. Jaundice - fatigue - weight loss - drak urine (bilirubin in urine).
. Hepatitis B & C present with serum sickness phenomena (arthralgia - urticaria - fever).
. HBV is associated with polyarteritis nodosa (PAN) !
. HCV is associated with Cryoglobulinema.
. HEV is most severe in PREGNANT WOMEN .. It can be fatal.
. Dx -> ++ CONJUGATED (DIRECT) BILIRUBIN.
. Dx -> ++ ALLLLLLLLT -> VIRALLLLLLLLL hepatitis.
. Dx -> ++ ASSSSSSSST -> DRUGSSSSSSSSS hepatitis.
. Dx -> AST:ALT = 2:1 -> ALCOHOLIC hepatitis.
. Dx -> Most accurate test -> Serology (IgG & IgM).
. N.B. EVALUATION OF LIVER DAMAGE:
___________________________________
-> ACUTE HEPATITIS -> Liver function tests & Viral serology.
-> CHRONIC HEPATITIS -> Liver biopsy.
# HEPATITIS B:
_______________
. HEPATITIS B DIAGNOSIS -> SEROLOGY:
____________________________________
. The 1st test to become ABNORMAL in ACUTE HB infection is SURFACE ANTIGEN (HBsAg).
. ++ ALT, e-antigen & symptoms all occur after the appearance of HBsAg.
...................... Surface Ag ........ e-Ag ......... Core Ab .......... Surface Ab
. ACUTE disease ----->

++

. WINDOW PHASE ------>


(RECOVERING)
. VACCINATED -------->

........ ++ .........

---

. HEALED RECOVERED -->

++ ..........

........ -- ......... (++) ..........

(--)
--

........ -- .......... -- ........... (++)


--

........ -- .......... (++) ........... (++)

. CHRONIC disease ---> Same as acute disease but based on persistance of HbsAg > 6 months
. Anti HBc Ig "G" Ab -> Denotes CHRONICITY !
. Best means of screening for HBV infection -> HBsAg & IgM Hbc Ab.
. N.B. ACUTE VIRAL REPLICATION indicators:
__________________________________________
. Hepatitis B DNA plymerase = e-Antigen = Hepatitis B PCR for DNA.
. CHRONIC HEPATITIS B TREATMENT:
________________________________
. Chronicity = More than 6 months with +ve serology.
. Tx -> Anti-viral therapy -> LAMIVUDINE + INTERFERON.
# HEPATITIS C:
______________
. HEPATITIS C DIAGNOSIS:
________________________
. Best initial test -> Hepatitis C antibody.
. Most accurate test ->
1 - Hepatitis C PCR for RNA:determine the degree of viral activity & response to therapy.
2 - Liver biopsy: determine the seriousness of the disease i.e. extent of liver damage.
. Chronic HCV classically presents with waxing & waning transaminases levels & few syms.
. Pts may complain of arthralgias or myalgias.
. Extra-hepatic sequlae: Cryoglobulinemia - porphyria cutanea tarda & glomerulonephritis.
. CHRONIC HEPATITIS C TREATMENT:
________________________________
. All chronic hepatitis C pts with ++ ALT, detectable HCV RNA & histologic evidence.
. Tx -> Anti-viral therapy -> RIBAVIRIN + INTERFERON.
. Chronic HCV pts with persistently NORMAL liver enzymes & MINIMAL histological findings,
. NO NEED TO BE TTT WITH INTERFERON OR ANTI-VIRAL DRUGS.
. JUST follow up with yearly liver function tests.
.
.
.
.

All chronic HCV pts sh'd receive vaccinations to Hepatitis A & B if not already immune.
Both vaccinations are safe during pregnancy.
Incidence of vertical transmission is very low 2-5 % (No need for C.S. for pregnants).
HCV infected mothers should NORMALLY BREAST-FEED their babies.

. SE of Ribavirin -> Anemia.


____________________________
. SE of Interferon -> Arthralgia - myalgia - flu-like $ - thrombocytopenia - depression.
________________________________________________________________________________________
. VACCINATION:
______________
. Vaccination for both hepatitis A & B are done universally in childhood.
. No vaccine & No post-exposure prophylaxis for hepatitis C.
. INDICATIONS for HEPATITIS A & B:

___________________________________
1 - CHRONIC LIVER DISEASE -> Cirrhosis.
2 - HOUSE HOLD CONTACTS -> of pts with hepatitis A & B.
3 - HOMOSEXUAL MEN !
4 - Chronic recepients of blood products.
5 - Injection drug users.
. SPECIFIC INDICATIONs FOR HEPATITIS A & B VACCINE:
___________________________________________________
. A -> TRAVELERS.
. B -> Health care workers & patients on dialysis.
. POST-EXPOSURE PROPHYLAXIS FOR HEPATITIS B:
____________________________________________
. Health care worker got stucked with a needle contaminated with blood from HBV pt.
. A child born to a mother with chronic hepatitis B.
. GIVE -> HEPATITIS B IMMUNOGLOBULIN + HEPATITIS B VACCINE.
. If the person had already been vaccinated,
. Check the levels of protective HBsAb (surface antibodies).
. If protective antibodies are ALREADY present -> No further ttt.
. FULMINANT HEPATIC FAILURE:
____________________________
. Hepatic encephalopathy developing within 8 weeks of the onset of acute liver failure.
. More common in pts using heavily using acet5aminophens & alcohols.
. Mostly their is co-infection of hepatitis B & D.
. Markedly ++ ALT, ++ PT & coagulopathy.
. HIGH PRIORITY CANDIDATES FOR LIVER TRANSPLANTATION. (The only effective ttt!).
. N.B. PROTHROMBIN TIME IS THE SINGLE MOST IMPORTANT TEST TO ASSESS LIVER FUNCTION
!
_____________________________________________________________________________________
. ASYMPTOMATIC ++ OF LIVER FUCTION TESTS (AST & ALT) APPROACH:
______________________________________________________________
.1. Ask for H/O of drug or alcohol intake, travel outside USA, blood transfusion or sex.
.2. Drugs (NSAIDs - Antibiotics - Anti-epileptic - Anti-tuberculous).
.3. Repeat the tests again to confirm the elevations !
.4. If elevation persists > 6 months -> Chronic.
.5. So .. test for HBV & HCV, Hemochromatosis & fatty liver.
. INH (ISONIAZID) INDUCED HEPATITIS:
____________________________________
. INH may induce sub-clinical hepatitis.
. Mild elevation of ALT & AST (< 100 IU/L).
. The hepatic injury is typicall self-limited.
. No ttt is needed .. The condition will resolve spontaneously.
. CHRONIC HEPATITIS -> CIRRHOSIS:
_________________________________
_________________________________
. CHRONIC ALCOHOL ABUSE & VIRAL HEPATITIS ARE THE MOST COMMON CAUSES.
. GENERAL FEATURES OF CIRRHOSIS:

_________________________________
1 - Edema -> from low oncotic pressure -> Tx: Spironolactone & diuretics.
2 - Gynecomastia.
3 - Palamar erythema.
4 - Splenomegaly.
5 - Thrombocytopenia (Caused by splenic sequestration).
6 - Encephalopathy (Tx: lactulose).
7 - Ascites (Tx: spironolactone).
8 - Esophageal varices (Tx: propranolol will prevent bleeding - Banding if rebleed).
. HYPER-ESTROGENISM in Cirrhosis:
_________________________________
. Due to -- clearance of Estrogen due to ++ portosystemic shunt.
. -- sex hormone binding globulin synthesis.
. Gynecomastia - testicular atrophy - spider angiomata - palmar eryhtema & -- body hair.
. ASCITES:
___________
. Paracentesis if (New ascites - pain, fever & tenderness).
. SERUM TO ASCITES ALBUMIN GRADIENT (SAAG):
___________________________________________
-> < 1.1 -> EXUDATE -> NO portal hypertension.
-> > 1.1 -> TRANSUDATE -> PORTAL HYPERTENSION (++ Capillary hydrostatic pressure).
. SPONTANEOUS BACTERIAL PERITONITIS:
____________________________________
-> Pt with cirrhosis & ascites presenting with low grade fever, abd. pain & or confusion.
-> Dx -> Best initial test -> Cell count > 250 neutrophils.
-> Dx -> Most accurate -> Fluid culture.
-> Tx -> CEFOTAXIME.
-> Prevent recurrence -> TMP-SMX.
. HEPATIC HYDRO-THORAX:
_______________________
. Transudative pleural effusions in pts with cirrhosis,
. who have no underlying cardiac or pulmonary disease.
. Results in a RT-sided pleural effusion.
. Tx -> Salt restrictions & diuretics.
. TIPS "Trans-jugular Intra-hepatic Porto-systemic Shunt" placement in refractory cases.
. CHRONIC LIVER DISEASES (CAUSES OF CIRRHOSIS):
_______________________________________________
{1} ALCOHOLIC CIRRHOSIS:
_________________________
. Diagnosis of exclusion !!
. H/O of long-standing alcohol abuse.
{2} PRIMARY BILIARY CIRRHOSIS (PBC):
_____________________________________
. Not related to IBD !!
. Middle-aged woman.
. ITCHING !
. XANTHELASMA (Cholesterol deposits).

.
.
.
.
.

H/O of other autoimmune diseases.


Best initial test -> ++ ALP Alkaline phosphatase with NORMAL BILIRUBIN level.
++ IgM.
Most accurate test -> Anti-mitochondrial antibody (AMA) & Liver biopsy.
Tx -> Ursodeoxycholic acid.

{3} PRIMARY SCLEROSING CHOLANGITIS (PSC):


__________________________________________
. Inflammatory bowel disease (IBD) accounts for 80% of causes (UC > CD).
. ITCHING !
. Best initial test -> ++ ALP Alkaline phosphatase with ++ BILIRUBIN level.
. Most accurate test -> ERCP -> BEADING of the biliary system.
. +ve ASMA (Anti-smooth muscle antibody).
. +ve ANCA.
. Tx -> Ursodeoxycholic acid.
{4} WILSON's DISEASE = HEPATO-LENTICULAR DEGENERATION:
_______________________________________________________
. -- Ceruloplasmin -> ++ Copper.
. Abnormal Copper deposition in liver, basal ganglia & cornea.
. Young pt < 30 ys.
. Cirrhosis + Choreiform movement disorder + Neuropsychiatric abnormalities.
. May present with hemolysis.
. Best initial test -> Slit lamp (Keiser Fleischer rings) & -- Ceruloplasmin level < 20mg
. Most accurate test -> Liver biopsy -> Copper level > 250 mcg/gram.
. Tx -> Penicillamine.
{5} HEMOCHROMATOSIS:
_____________________
. Genetic disorder -> over-absorption of copper.
. Iron deposits in various body tissues.
. Heart -> Restrictive cardiomyopathy.
. Skin -> Darkening & pigmentation.
. Joint -> Psedogout & CPPD.
. Pancreas -> Bronze Diabetes.
. Pituitary -> Panhypopituitarism.
. Genitalia -> Infertility.
. Infections -> LISTERIA, VIBRIO VULNIFICUS & YERSINIA ENTEROCOLITICA.
. Liver -> HEPATOMA & cirrhosis -> HEPATOCELLULAR CARCINOMA (Most common cause of
death).
. Dx -> Best initial test -> ++ serum iron & ferritin levels & -- TIBC.
. Dx -> Most accurate test -> Liver biopsy - MRI liver - HFe gene mutation detection.
. Tx -> Phlebotomy.
{6} AUTO-IMMUNE HEPATITIS:
___________________________
. Young woman with other auto-immune diseases (Coomb's +ve AIHA, thyroiditis & ITP).
. Best initial test -> ANA & ASMA & Liver/kidney microsomal antibody.
. Serum protein electrophoresis (SPEP) -> Hyper-gamma globulinemia.
. Most accurate test -> Liver biopsy.
. Tx -> prednisone.
{7} NON-ALCOHOLIC STEATOHEPATITIS (NASH):
__________________________________________
. Associated with obesity - DM - Hyperlipidemia.

.
.
.
.
.
.
.

Hepatomegaly.
Patho-physiology -> INSULIN RESISTANCE.
Dx -> Best initial test -> ALT > AST.
Dx-> Most accurate test -> Liver biopsy (Fatty infiltration).
The liver biopsy looks alike Alcoholic liver disease !!
Tx -> No specific therapy.
Control the underlying cause e.g. weight loss - DM control.

{8} ALPHA - 1 ANTI-TRYPSIN DEFECIENCY:


_______________________________________
. PAN-ACINAR EMPHYSEMA + CIRRHOSIS.
. Co-existing lung involvement.
. Family H/O of cirrhosis.
{9} CARDIAC CIRRHOSIS:
_______________________
. Co-existing right-sided heart failure.
. MANAGEMENT OF CIRRHOSIS:
__________________________
__________________________
{A} . PERIODIC SURVEILLANCE OF Liver Function Tests (INR - Albumin - Bilirubin):
_________________________________________________________________________________
{B} . COMPENSATED:
___________________
. U/$ surveillance for Hepatocellular carcinoma & Alpha feto-protein every 6 months.
. Esophageal endoscopy for varices surveillance.
{C} . DECOMPENSATED -> ASSESS COMPLICATIONS:
_____________________________________________
. VARICEAL HEMORRHAGE -> Start non selective BB - Repeat esophageal endoscopy every year.
. Ascites -> Dietary sodium restriction, diuretics, paracentesis & Alcohol abstinence.
. Hepatic encephalopathy -> identify the cause (infection - GIT bleeding) & Lactulose.
. VARICEAL BLEEDING MANAGEMENT:
_______________________________
. The 1st step in ttt of ACUTE VARICEAL BLEADING is to establish vascular access,
. with TWO LARGE BORE INTRAVENOUS NEEDLES OR CENTRAL LINE.
. Then .. Control the bleeding itself !
. In 50 % of cases, bleeding ceases on its own.
. Other 50 % -> Vasoconstrictors (Terlipressin) = Synthetic analogue of vasopressin.
. HEPATIC ENCEPHALOPATHY MANAGEMENT:
____________________________________
. Decline in neurologic function due to hepatic damage.
. Due to ++ AMMONIA level in circulation & production of false neurotransmitters.
. TTT principles -> Correction of ppt factor & lowering of blood ammonia concentration.
. PPT factors -> hypovolemia - GIT bleeding - infection - hypoxia - hypoglycemia & -- K.
. Lower blood ammonia using DISACCHARIDE (LACTULOSE) -> Ammonia trap.
. Add NEOMYCIN -> Destroy ammonia producing colonic bacteria (OTO & NEPHRO-TOXIC).
. Lower protein in diet (BUT .. NOT PTN FREE DIET xxxx) !

. COAGULOPATHY MANAGEMENT:
__________________________
. Bleeding disorders occur as the liver synthesizes all clotting factors except factor 8.
. Chief among these are Vit. K dependent factors 1972.
. Acute bleeding is best ttt with FFP FRESH FROZEN PLASMA.
. FFP contains all clotting factors.
* AMEBIC LIVER ABSCESS:
_______________________
. Amebiasis is aprotozoal disease caused by ENTAMOEBA HISTOLYTICA.
. H/O of travel to endemic area e.g. MEXICO.
. Followed by dysentery, RUQ. pain & diarrhea.
. The primary infection is the colon leading to bloody diarrhea,
. Ameba may be transported to the liver by portal circulation -> Amebic liver abscess.
. Fever up to 39.5 c.
. ++ WBCs & ++ ALP.
. Generally SINGLE & located in the RT lobe.
. Dx -> Stool exam. -> trophozites.
. CT -> Liver mass (cystic not solid lesion).
. Needle aspiration is not performed due to risk of bleeding & peritonitis.
. The fluid inside is called "anchovy paste" - STERILE & odourless unless 2ry infected.
. Tx -> ORAL METRONIDAZOLE. (NOT SURGICAL RESECTION!).
* HYDATID DISEASE:
___________________
. Hydatid cyst in liver.
. Caused by ECHINOCOCCUS GRANULOSUS.
. Defnitive host is DOG.
. Unilocular cystic lesions in liver, lungs, muscles & bones.
. Most pts are asymptomatic.
. Symptoms are due to compression of the surrounding tissues.
. CT -> EGG SHELL CALCIFICATION of hepatic cyst.
. Aspiration isn't indicated -> anaphylactic shock 2ry to spelling of cyst contents.
. Tx -> Surgical resection under the cover of ALBENDAZOLE.
* ISCHEMIC HEPATIC INJURY = SHOCK LIVER:
_________________________________________
. Accompanies severe hypotension or shock.
. Rapid massive +++ in transaminases.
. Minimal ++ in bilirubin & ALP.
. Once the predisposing factor disappears, the transaminases return to normal.
. LIVER MALIGNANCIES:
_____________________
_____________________
* LIVER METASTASIS:
____________________
. 20 times more common than HCC.
. Tumors of GIT, lung & breast are the most common culprits.
. May be asymptomatic & discovered accidentally.
. If symptomatic: Hepatomegaly, jaundice, cholestasis & ++ alkaline phosphatase (ALP).
. NORMAL ALPHA FETO-PROTEIN (AFP).
. Dx -> CT -> Multiple hepatic nodules of varying sizes.

. Confirmed by liver biopsy.


. Multiple liver masses are much more likely to be the result of a metastatic disease.
. Mostly -> CANCER COLON -> DO COLONOSCOPY.
* HEPATOCELLULAR CARCINOMA (HCC):
__________________________________
. Less common than metastasis.
. ++++ ALPHA FETOPROTEIN.
. CT -> SOLITARY large lesion.
* HEPATIC ADENOMA:
___________________
. Benign rare liver tumor.
. Young & middle aged women with H/O of OCP intake.
. Palpable liver mass.
. Liver biopsy -> Mildly atypical hepatocytes containing glycogen & lipid deposits.
. Normal liver finction tests.
. Normal AFP.
. ++ ALP & GGT.
. Complications -> Severe intra-tumor hemorrhage & malignant transformation.
. ALCOHOLIC LIVER DISEASES:
___________________________
. Females are more susceptible than males to alcoholic liver diseases !!
. Alcoholic fatty liver is reversible condition.
. AST:ALT -> 2:1.
. 80 % of alcoholics will develop fatty liver.
. 20 % of alcoholics will develop hepatitis.
. 50 % of alcoholics will develop cirrhosis.
.
.
.
.

H/O of heavy alcohol use.


Hepatomegaly (Liver span > 12 cm in MCL).
++ MCV.
++ AST : ALT = 2:1.

. Stages -> 1. Fatty liver "steatosis" - 2. Hepatitis - 3. Fibrosis "Cirrhosis".


. Fatty liver is due to short alcohol use.
. Hepatitis & cirrhosis require long term use.
. Alcohol Hepatitis is characterized by:
-> MALLORY BODIES.
-> Neutrophils infiltration.
-> Liver cell necrosis.
-> Peri-venular inflammation.
. On CESSATION of alcohol intake:
-> Steatosis & hepatitis & early fibrosis are REVERSIBLE.
-> True cirrhosis (with generation nodules) are IRREVERSIBLE, regardless of abstinence.
. HYPER-BILIRUBINEMIA APPROACH:
_______________________________
_______________________________

{A} MAINLY UN-CONJUGATED:


__________________________
. Over-production (hemolysis).
. Reduced up-take (drugs & porto-systemic shunt).
. Conjugation defect (Gilbert's $).
{B} MAINLY CONJUGATED -> EVALUATE LIVER ENZYME PATTERN:
________________________________________________________
. {1} Predominantly ++ AST & ALT:
__________________________________
. Viral Hepatitis.
. Autoimmune hepatitis.
. Drug induced hepatitis.
. Hemochromatosis.
. Ischemic hepatitis.
. Alcoholic hepatitis.
. {2} NORMAL AST & ALT & ALP:
______________________________
. DUBIN JOHNSON's $YNDROME.
. ROTOR's $YNDROME.
. {3} Predominantly ++ ALP -> Do abdominal imaging U$ or CT & Antimitochondial Abs:
____________________________________________________________________________________
. Cholestasis of pregnancy.
. Malignancy (pancreas - ampullary).
. Cholangiocarcinoma.
. Primary biliary cirrhosis.
. Primary sclerosing cholangitis.
. Choledocholithiasis.
. N.B. U/$ -> Biliary dilatation = Extra-hepatic cholestasis.
. N.B. U/$ -> No biliary dilatation = Intra-hepatic cholestasis.
. UN-CONJUGATED HYPER-BILIRUBINEMIA FAMILIAL DISORDERS:
_______________________________________________________
{1} GILBERT's $YNDROME:
________________________
. Disorder in conjugation.
. -- Glucuronyl transferase (enzyme that mediates glucuronidation).
. Mild jaundice.
. In-direct bilirubin < 3 mg/dl.
. Tx -> Un-necessary.
{2} CRIGLER NAJJAR TYPE - 1 $YNDROME:
______________________________________
. Disorder in metabolism.
. Severe jaundice.
. Bilirubin encephalopathy (Kernicterus).
. In-direct bilirubin 20 - 25 mg/dl. may reach 50 mg/dl.
. Normal liver enzymes & liver histology.
. IV PHENOBARBITAL -> NO CHANGE in serum bilirubin.

. Tx -> Phototherapy or plasmapharesis the LIVER TRANSPLANT.


{3} CRIGLER NAJJAR TYPE - 2 $YNDROME:
______________________________________
. Disorder in metabolism.
. Severe jaundice (less severe than type 1).
. NOOOO Bilirubin encephalopathy (Kernicterus).
. In-direct bilirubin <<<< 20 mg/dl.
. Normal liver enzymes & liver histology.
. IV PHENOBARBITAL -> ----------- in serum bilirubin.
. Tx -> Not necessary (just phenobarbital to -- bilirubin).
. CONJUGATED HYPER-BILIRUBINEMIA FAMILIAL DISORDERS:
____________________________________________________
{1} ROTOR's $YNDROME:
______________________
. Defect in hepatic storage of conjugated bilirubin.
. Normal liver function tests.
. TTT is not necessary.
{2} DUBIN JOHNSON's $YNDROME:
______________________________
. Defect in hepatic bile secretion.
. Liver biopsy -> dark granular pigment in hepatocytes (Not seen in Rotor's $).
. PANCREATIC DISEASES:
______________________
______________________
. ACUTE PANCREATITIS:
_____________________
. Severe mid-epigastric abdominal pain radiating to the back.
. Vomiting without blood - Anorexia - Tendrness in the epigastric area.
. Main causes are ALCOHOLISM & GALL STONES.
. Other causes -> Hypertiglyceridemia - trauma - infection - iatrogenic ERCP.
. Dx -> Best initial test ->
* ++ Amylase & lipase (most sensitive & specific) -> ++ Amylase/lipase 3 times.
* ABDOMINAL ULTRA$OUND -> Diffusely enlarged hypoechoic pancreas.
. Dx -> Most accurate test -> Abdominal CT scan:
* Detect dilated common bile ducts.
* Comment on intra-hepatic ducts.
. Dx -> N.B. -> ++ ALT > 150 & ++ ALP -> Biliary pancreatitis.
. MRCP -> Detects causes of biliary & pancreatic duct obstruction not found on CT scan.
. ERCP -> If there is dilatation of the common bile duct without a pancreatic head mass.
. ERCP -> Detect stones or strictures in the pancreatic duct system & remove them.
. Tx -> NPO - Bowel rest - Hydration - pain medications.
. N.B. (1):
____________
. If the cause of acute pancreatitis was gall stones not alcoholism,
. Once the pt. recovers with normalization of the pancreatic enzymes & medically stable,
. CLOLECYSTECTOMY IS A MUST !

. N.B. (2):
____________
. Acute pancreatitis in pts without gall stones or a H/O of alcohol use.
. HYPER-TRIGLYCERIDEMIA > 1000 mg/dl -> Acute pancreatitis.
. Eruptive xanthoma on exam.
. Dx -> FASTING LIPID PROFILE.
. COMPLICATIONS OF SEVERE PANCREATITIS:
________________________________________
. 1 . Pseudocyst.
. 2 . Peri-pancreatic fluid collection.
. 3 . Necrotizing pancreatitis.
. 4 . ARD$.
. 5 . ARF.
. 6 . GIT bleeding.
. SEVERE PANCREATITIS:
______________________
. Pancreatitis with failure of at least 1 organ !
. Predisposing factors: Age > 75 ys, Alcoholism & obesity.
. CULLEN SIGN -> Peri-umbilical bluish coloration indicating hemoperitoneum.
. GREY-TURNER SIGN -> Reddish brown coloration around flanks = retroperitoneal bleeding.
. ++ CRP > 150 mg/dl in the 1st 48 hs.
. ++ Urea & creatinine in the 1st 48 hs.
. Severe cases -> (-- BP, -- Ca, -- O2, -- pH) & (++ WBCs, ++ glucose).
. Hypotension, Hypoxia, Metabolic Acidosis, Hypocalcemia, Leukocytosis & Hyperglycemia.
. Hypocalcemia due to fat malabsorption.
. severe pancreatitis may lead to release of activated pancreatic enzymes,
. that enter the vascular system & ++ the vascular permeability,
. so, large volumes of fluid migrate from the vascular system to surrounding peritoneum,
. resulting in widespread vasodilatation, capillary leak, shock & end organ damage.
. Dx -> CT or MRCP to detect pancreatic necrosis & extra-pancreatic inflammation.
. Tx -> Supportive with several liters of IV fluids.
. NECROTIZING PANCREATITIS:
___________________________
. Dx -> CT.
. Tx -> If > 30 % necrosis -> IV Antibiotics (Imipenem) & CT guided biopsy.
. If the biopsy showed infected necrotic pancreatitis -> SURGICAL DEBRIDEMENT.
. Surgical debridement is done to prevent ARD$ & death.
. PANCREATIC PSEUDOCYST:
________________________
. Palpable mass in the epigastrium 4 weeks after the onset of acute pancreatitis.
. Not true cysts as they lack an epithelial lining just walled by a thick fibrous capsule
. The pseudocyst is compromized of inflammatory fluid, tissues & debris.
. The fluid contains high levels of amylase, lipase & enterokinase.
. Dx -> U/$.
. Tx -> Usually resolves spontaneously.
. Tx -> Drainage if persisting > 6 weeks or > 5 cm in diameter or becomes 2rly infected.
. May be complicated by severe hemorrhage if eroded into a blood vessel.
. DRUG INDUCED PANCREATITIS:
____________________________

.
.
.
.
.
.
.
.

Mild & usually resolves with supportive care !


CT scan is diagnostic.
Pt on diuretics -> Furosemide & thiazides.
Pt on antibiotics -> Metronidazole & tetracyclines.
Pt with IBD -> Sulfasalazine.
Pt on immunosuppressives -> Azathioporine.
Pt with seizures or bipolar disorder -> Valproic acid.
Pt with AIDS -> Didanosine & pentamidine.

. CHRONIC PANCREATITIS:
_______________________
. Due to alcohol abuse - cystic fibrosis (Children) - Autoimmune causes.
. Epigastric chronic abdominal pain.
. Intermittent pain free intervals.
. Malabsorption -> chronic diarrhea & steatorrhea.
. Weight loss & DM may occur lately.
. AMYLASE & LIPASE may be normal .. Non diagnostic.
. Plain film or CT scan -> Pancreatic calcifications. (DIAGNOSTIC).
. If x-ray & CT are -ve for calcifications -> ERCP or MRCP.
. Tx -> Pain management with frequent small meals & pancreatic enzymes supplement.
. Alcohol & smoking cessation.
. PANCREATIC CARCINOMA:
_______________________
. More in males & black race & age > 50 ys.
. Risk factors -> Chronic pancreatitis, smoking & DM.
. CIGARETTE SMOKING is the MOST CONSISTENT RISK FACTOR.
. Dull upper abdominal pain radiating to the back, weight loss & jaundice.
. Tumors located in pancreatic body or tail -> pain & weight loss.
. Tumors located in pancreatic head -> Steatorrhea, weight loss & jaundice.
. COURVOISIER's sign -> Palpable, non tender gall bladder at the Rt. costal margin.
. VIRCHOW's NODE -> Left supra-clavicular adenopathy.
. ++ serum bilirubin & ++ ALP.
. ++ CA 19-9 levels (Serum cancer associated antigen).
. Dx -> ABDOMINAL U/$ & CT (if U$ is not diagnostic).
. Tx -> Resection of the involved tissue.
. GUESS WHAT -> ALCOHOLISM & GALL STONES ARE NOT RISK FACTORS OF PANCREATIC
CANCER !!
. PANCREATIC CANCER VS CHRONIC PANCREATITIS:
____________________________________________
. Both may present with epigastric pain.
. (Old age, jaundice & weight loss) favors malignancy.
. Mild elevation of amylase & lipase are consistent with chronic pancreatitis.
. ++ serum Bilirubin & ALP = compression of the intra-pancreatic bile duct = Malignancy.
. Best initial test -> ABDOMINAL U/$ -> DILATED BILE DUCTS & MASS IN HEAD OF PANCREAS.
. CT abdomen is more specific than U/$.
. If CT failed -> i.e. No mass lesion -> Do ERCP.
. If ERCP failed -> due to pancreatic duct obstruction -> Do MRI.
. GALL BLADDER DISEASES:
________________________
________________________

. GALL STONE DISEASE = CHOLELITHIASIS:


______________________________________
. Types of gall stones (Cholesterol - Pigment "Ca bilirubinate" - Mixed).
. Msot common is Cholesterol stone & least common is pigment "Ca bilirubinate".
. 80 % of stones are RADIO-OPAQUE.
. FAT - FERTILE - FEMALE - FORTY - FILTHY !
. Native American - DM - Obesity - OCP & pregnancy are common predisposing factors.
. Bloating & dyspepsia after eating fatty foods.
. RUQ abdominal pain.
. Dx -> Abdominal U/$.
. Tx -> LAPAROSCOPIC CHOLECYSTECTOMY.
. If refused the operation -> Give ursodeoxycholic acid & advise to avoid fatty foods.
. Asymptomatic gall stones should NOT be treated.
. Symptomatic gall stones -> LAPAROSCOPIC CHOLECYSTECTOMY.
. ACUTE CHOLECYSTITIS = ACUTE GALL BLADDER INFLAMMATION:
________________________________________________________
. CHARCOT's TRIAD -> Fever + severe jaundice + RUQ abd pain radiating to the Rt shoulder.
. REYNOLD's PENTAD -> + Confusion + Hypotension -> (Suppurative cholangitis).
. Most commonly due to obstruction of the common bile duct by stone.
. The original incinting event is a gall stone obstructing the CYSTIC DUCT (Not CBD) !!!
. MURPHY's SIGN -> pain on palpation of area of gall bladder fossa on deep inspiration.
. ++ WBCs & ++ ALP (cholestasis & obstruction).
. Dx -> U/$.
. Tx -> Supportive care & broad spectrum antibiotics.
. Most pts recover completely, but despite adequate fluids & antibiotics,
. Some pts continue to have persistent abd. pain, hypotension, high fever & confusion.
. This is an indication of URGENT BILIARY DECOMPRESSION by ERCP.
. ERCP -> Sphincterotomy & stone removal or stent insertion.
. Lap. cholecystectomy won't accomplish drainage of the biliary tree "main concern" !
. Pt will undergo cholecystectomy later on but drainage of the biliary tree is more imp.
. EMPHYSEMATOUS CHOLECYSTITIS:
______________________________
. Due to 2ry infection of the gall bladder with gas forming bacteria e.g. Clostridium.
. Mostly diabetic male pts aged 50 - 70 ys.
. Vascular predisposing factor e.g. obstruction or stenosis of the cystic artery.
. Right upper quadrant pain - nausea - vomiting - low grade fever.
. Crepitus in the abdominal wall adjacent to the gall bladder.
. Complications -> Gangrene & perforation.
. Dx -> Abdominal radiograph -> Air fluid level in the gall bladder.
. Dx -> Abdominal ultrasound -> Curvilinear gas shadowing in the gall bladder.
. Tx -> Immediate fluid & electrolyte resuscitation, cholycystectomy & antibiotics.
. A-CALCULOUS CHOLECYSTITIS:
____________________________
. Acute inflammation of the gall bladder in absence of gall stones.
. Most commonly seen in hospitalized pts wit the following conditions:
. Extensive burns - severe trauma - Prolonged TPN or fasting & mechanical ventillation.
. pathophysiology -> ischemia - biliary stasis - infection or external compression.
. Complications -> Gangrene - perforation - emphysematous cholecystitis.
. Dx -> U/$ -> Signs of cholecystitis but No gall stones.
. CT & HIDA scan are more sensitive & specific.
. PORCELAIN GALL BLADDER:

_________________________
. Due to chronic cholecystitis.
. Calcium laden gall bladder.
. Calcium salts are deposited intra-murally 2ry to chronic irritation from gall stones.
. RUQ. pain with firm non tender mass in the RUQ.
. X-ray -> Rim like calcification in the area of gall bladder.
. CT -> Calcified rim with central bile-filled dark area.
. Mostly develop to GALL BLADDER CARCINOMA.
. Tx -> CHOLECYSTECTOMY.
. GALL BLADDER CARCINOMA = CHOLANGIOCARCINOMA:
______________________________________________
. Rare malignancy.
. More in hispanic or Native american females who have H/O of gall stones.
. Typicallu diagnosed during or after chlecystectomy !
. Can NOT be easily diagnosed pre-operatively.
. CA 19-9 is NOT a specific marker.
. POST-OPERATIVE CHOLESTASIS:
_____________________________
. Benign condition developing after a major surgery.
. Major = Hypotension - extensive blood loss into tissues - massive blood replacement.
. Jaundice by the 2nd or 3rd post-operative day.
. Bilirubin peaks at 10 - 40 mg/dl by the 10th day.
. ALP may be elevated.
. AST & ALT NORMAL.
. POST-CHOLECYSTECTOMY $YNDROME:
________________________________
. Persistent abdominal pain, nause & dyspepsia after cholecystectomy.
. Biliary causes -> Retained common bile duct - cystic duct stone.
. Extra-biliary causes -> Pancreatitis - PUD.
. Dx -> U/$ followed by ERCP.
. POST-CHOLECYSTECTOMY PAIN:
____________________________
. Due to functional etiology e.g. SPHINCTER OF ODDI DYSFUNCTION or CBD stone.
. Normal ERCP & U/$ can rule out CBD stones.
. It is a diagnosis of exclusion.
. Tx of sphincter of Oddi dysfunction -> ERCP with sphincterotomy.
. VANISHING BILE DUCT $YNDROME:
_______________________________
. progressive destruction of the intra-hepatic bile ducts.
. Histological hallmark -> Ductopenia.
. Primary bilary cirrhosis is the most common cause of ductopenia in adults.
. Primary scerosing cholangitis is not related to ductopenia.
. DIFFERENT DIAGNOSTIC TOOLS USED FOR GALL BLADDER DISEASES:
____________________________________________________________
{1} * ABDOMINAL ULTRA$OUND:
____________________________
. Best initial investigation of gall bladder diseases.

{2} * ERCP = ENDOSCOPIC RETRO-GRADE CHOLANGIO-PANCREATOGRAPHY:


_______________________________________________________________
. Best diagnostic & therapeutic tool in evaluation of chronic pancreatitis & CBD disease.
. Most accurate test of detecting causes, location & extent of bile duct obstruction.
. Therapeutic: Stone extraction, sphincterotomy, balloon dilatation & stent placement.
. TTT of choice in case of sphincter of Oddi dysfunction.
{3} * ABDOMINAL RADIOGRAPHS:
_____________________________
. Neither sensitive nor specific.
. > 80 % of gall stones are radio-lucent so can't be visualized.
{4} * HIDA SCAN:
_________________
. Use technitium labelled compounds to demonstarate bile duct obstruction & GB diseases.
. It is superior to U/$ in confirming suspected acute cholecystitis (Acalculus type).
{5} * PTC = PER-CUTANEOUS TRANS-HEPATIC CHOLANGIOGRAPHY:
_________________________________________________________
. study the intra & extra hepatic biliary tree.

. IMMUNOLOGIC BLOOD TRANSFUSION REACTIONS:


__________________________________________
__________________________________________
. 1 . FEBRILE NON-HEMOLYTIC:
_____________________________
. Most common reaction.
. Fever & chills.
. Within 1 - 6 hours of transfusion.
. Caused by cytokine accumulation during blood storage.
. 2 . ACUTE HEMOLYTIC:
_______________________
. Fever, flank pain, hemoglobinuria, renal failure & DIC.
. Within 1st hour of transfusion.
. +ve direct Coomb's test & pink plasma.
. Caused by ABO INCOMPATIBILITY.
. 3 . DELAYED HEMOLYTIC:
_________________________
. Mild fever & hemolytic anemia.
. Within 2 - 10 days of transfuion.
. +ve direct Coomb's test & +ve new antibody screen.
. caused by ANAMNESTIC ANTIBODY RESPONSE.
. 4 . ANAPHYLACTIC:
____________________
. Rapid onset of shock, angioedema, urticaria & respiratory distress.
. Within a few seconds to minutes of the transfusion.
. Caused by RECEPIENT anti-Ig"A" Abs.
. 5 . URTICARIAL = ALLERGIC:

_____________________________
. Urticaria - flushing - angioedema & pruritis.
. Within 2 - 3 hours of transfusion.
. Caused by RECEPIENT Ig"E" Abs & mast cell activation.
. 6 . TRANSFUSION RELATED ACUTE LUNG INJURY:
_____________________________________________
. Respiratory distress & signs of non-cardiogenic pulmonary edema.
. Within 6 hours of transfusion.
. Caused by DONOR ANTI-LEUKOCYTE ANTIBODIES.
. N.B. Individuals who received blood transfusions before 1992 sh'd be screened for HCV.
. N.B. Individuals who received blood transfusions before 1986 sh'd be screened for HBV.

Dr. Wael Tawfic Mohamed


_________________________

INFECTIONS TiKi TaKa


---------------------. Isoniazid side effect:
------------------------. Peripheral neuropathy .. Tx: Vit. B 6 (Pyridoxine)
. CEREBRAL TOXOPLASMOSIS:
------------------------. Multiple ring enhancing lesions causing headache & hemiparesis.
. Prohphylaxis = TMP-SMX = Trimethoprim - Sulfamethoxazole.
. Treatment = SDZ-PMT = Sulfadiazine - Pyrimethamine.
. MYCO-BACTERIAL AVIUM "MAV" complex:
------------------------------------. HIV pt with un-explained fever & cough with CD 4 count < 50.
. AZITHROMYCIN is the best prophylaxis for HIV with MAV.
. FEBRILE NEUTROPENIA:
---------------------. Fever > 38.3 + Neutrophils < 1500.
. Management: Admission + CEFEPIME I.V.
. Mucormycosis:
--------------. caused by fungus RHIZOPUS.
. require aggressive surgical debridement + I.V. AMPHOTERICIN B.
. ASPERGILLOSIS:
---------------. Immunocompromized pt. with pulmonary symptoms.
. CXR: consolidation in the upper lobe.
. CT: HALO sign.
. HISTOPLASMOSIS:
----------------. Pulm. symps.
. CXR: Hilar adenopathy.
. Triad of palatal ulcers + splenomegaly + Thrombocytopenia.
. COCCIDIODIOMYCOSIS:
-------------------. Pulm. symps.
. erythema multiforme or erythema nodosum.
. BLASTOMYCOSIS:
---------------. Immunodefecient pt. with pulm. symps.
. YEAST :)
. COCCIDIOMYCOSIS:
-----------------

. ARIZONA.
. Non specific lung syms + erythema multiforme + erythema nodosum + Arthralgia.
. Any dog bite .. An attempt to capture the dog is tried 1st.
------------------------------------------------------------. If the dog is not captyred .. It is assumed RABID .
. Give post-exposure prophylaxis.
.
.
.
.

If the dog is captured .. but doesn't show any features of Rabies .


Observe for 10 days .
If it developed any Rabies features.
Give post-exp. prophylaxis.

. If the bite involves the head & neck.


. Post exposure prophylaxis is indicated IMMEDIATELY.
. Viral (HSV) Encephalitis:
--------------------------. Fever + confusion.
. Hemiparesis + Hyperreflexia.
. Cranial n. palsies + focal deficits.
. CSF: ++ ptn , ++ WBCS , ++ Lymphocytes.
. Normal glucose.
. Dx: PCR.
. Tx: I.V. ACYCLOVIR.
. Rt. sided endocarditis:
------------------------. should be considered in pts. with H/O of I.V. drug abuse.
. Tx ----------------> VANCOMYCIN.
. directed against MRSA & Streptococci.
. BABESIOSIS:
------------. Tick borne disease.
. Parasite enters the RBC causing hemolysis.
. Symptoms: JAUNDICE .. HEMOGLOBINURIA .. RENAL FAILURE .. DEATH.
. Typical pt: > 40 ys .. without a spleen or immunocompromized.
. LABS: INTRAVASCULAR HEMOLYSIS:
................................ * -- RBCs,--WBCs,--Platelets.
................................ * -- Serum complement.
................................ * ++ ESR, ++ Lymphocytes.
. Dx: GIEMSA stain.
. Tx: ATOVAQUONE - AZITHROMYCIN.
. Any transplant pt. should have TMP-SMX for prophylaxis against (PCP) pnemo-cystis carinii
pneumonia.
. "AIHI" Auto-immune Hemolytic Anemia is one of the complications of INFECTIOUS
MONONUCLEOSIS.
. Empiric antibiotics for bacterial meningitis:
----------------------------------------------. VANCOMYCIN + AMPICILLIN + CEFEPIME + CORTICOSTEROIDS.

. HIV +ve pts are at high risk for T.B.


--------------------------------------. A pt with +ve PPD Tuberculin test i.e. > 5mm induration:
. should have INH (Isoniazid) & Vit.B6 (Pyridoxine) for 9 months as a prophylaxis.
. EHRILICHIOSIS:
---------------. SPOTLESS RMSF.
. TICK BITE.
. Sysytemic symptoms.
. LEUKOPENIA & THROMBOCYTOPENIA.
. ++ ALT & AST.
. Tx: DOXYCYCLINE.
. ENTERO-HEMORRHAGIC E-COLI:
---------------------------. Bloody diarrhea.
. Abd. pain.
. NO FEVER.
. No travel H/O.
. E-Coli = TRAVELER's diarrhea.
. Whenever a health care worker is exposed to HIV:
-------------------------------------------------. Draw his blood for HIV serology.
. Start anti-retro-viral therapy with 3 drugs without delay
. while awaiting the results of HIV serology.
. HIV pts with CD 4 cell count < 50:
-----------------------------------. require prophylaxis against MAV complex with AZITHROMYCIN.
. CRYPTO-COCCAL meningitis:
--------------------------. caused by encapsulated yeast
. HIV pt. with meningitis
. Tx: IV Amphotericin + FLUCYTOSINE.
. Tx of primary syphilis:
------------------------. Single I.M. BENZATHINE PENICILLIN.
. If the pt is allergic to penicillin.
. Give either single dose of AZITHROMYCIN or 2 weeks course of DOXYCYCLINE.
. Lyme dis. pt. if pregnant , lactating or child < 8ys:
------------------------------------------------------. Don't give DOXYCYCLINE but give AMOXICILLIN.
. Malignant Otitis externa:
--------------------------. D.M. pt. with ear pain & granulation tissue at the auditory canal.
. caused by pseudomonas Aeruginosa.

. Recall of a tick bite is not the main stay of the diagnosis of LYME disease caused by
BORRELIA BURGDORFERI !
. UTI INFECTION:
---------------. Acidic urine = E-Coli.
. Alkaline urine = Proteus.
. Rash of measles & Rubella r za same ... BUT:
---------------------------------------------. Measles is accompanied by KOPLIK's spots.
. Rubella is associated with ARTHRITIS.
. TRICHINELLOSIS:
----------------. GIT complaints.
. + Triad of ---> Peri-orbital edema + Myositis + Eosinophilia.
. other clues .. Splinter or sub-ungal hemorrhages.
. Actinomycosis:
---------------. Infection at the neck in a diabetic pt.
. Serosanguinous fluid draining from a defect in the center of the lesion.
. Culture : Gram +ve branching bacteria.
. Tx: I.V. Penicillin.
. Actinomycosis is a bacteria not a fungus so don't ttt it with Amphotericin !!
. Lesion: Slowly progressive non tender indurated mass
. evolving into multiple abscesses with draining sinus tracts.
. with sulfur yellowish granules !
. CMV Pneumonitis !!!!
--------------------. 45 days post Bone Marrow transplant recepients.
. CXR: Multi-focal diffuse patch infiltrates.
. Oral thrush.
. CMV colitis: Abdominal tenderness
. Toxic scock $yndrome:
----------------------. H/O of NASAL PACKING or MENSTRUATION TAMPOONS.
. Fever < 38.9 c.
. Hypotension < 90/60 mmHg.
. Rash & thrombocytopenia.
. Multisystem involvemet (Vomiting & Diarrhea & Myalgia).
. PCP = PNEUMOCYSTIS CARINII PNEUMONIA:
-------------------------------------. HIV pt. with CD4 < 200.
. Non prod. cough, dyspnea, fever, hypoxia.
. Bilateral interstitial infiltrates on CXR.
. Tx: TMP-SMX.
. Use steroids if: PaO2 < 70 mmHg or A-a gradient > 35 mmHg.
. NOCARDIOSIS:
--------------

. Crooked , branching , beaded , gram +ve partially acid fast filaments on microscopy.
. Tx: TMP-SMX.
. Symptomatic CAT scratch disease:
---------------------------------. Lymphadenopathy & Systemic symptoms.
. Tx: AZITHROMYCIN.
. PSEUDOMONAS AERUGINOSA:
------------------------. Gram -ve bacilli in the sputum of an intubated ICU pt. + fever + leukocytosis.
. Tx: CEFEPIME (4th g. cephalosporin) or PIPERACILLIN - TAZOBACTAM !
. Ceftriaxone is not effective against Pseudomonas.
. U should STOP it !
. Valvular diseases:
-------------------. MR is the most common valvular abnormality not related to IV drug abuse.
. If IV drug abuser .. TR is the the most common.
. D.M. pts with foot ulcer who developed osteo-myelitis:
-------------------------------------------------------. The route of infection is CONTAGIOUS SPREAD.
. A nail puncture wound resulting in Osteomyelitis in an adult:
--------------------------------------------------------------. is due to Pseudomonas Aeruginosa.
. Bacterial Meningitis with meningococcemia:
-------------------------------------------. Sudden onset fever + Neck stiffness + Nause + Headache + Myalgias.
. Hypotension + Tachycardia + Myalgia + Purpuric skin lesions.
. CSF findings of BACTERIAL cause:
=================================
** ++ WBCs .. 2000 (N: 0-5).
** Glucose .. 20 (N: 40-70).
** ++ Protein .. 175 (N: <40).
. Lyme disease:
---------------. is not associated with purpura.
. but associated with erythema migrans.
. with characteristic bull's eye appearance !
. INFLUENZA MANAGEMENT:
----------------------. Most pts with INFLUENZA r ttt with BED REST & SIMPLE ANALGESIA e.g. ACETAMINOPHEN.
. Anti-viral medications reduce the duration of influenza,
. but they r only effective if administered within 48 hours of the onset of illness. . Amantadine
& Rimantadine r only effective against type A.
. Zanamivir & Oseltamivir r only effective against both type A & B.
. INDINAVIR (Protease inhibitor):
--------------------------------. Anti-retroviral therapy.

. causing high creatinine & hematuria.


. Needle shaped crystals in sediment : causing crystal induced nephropathy!
. BACILLARY ANGIOMATOSIS:
-------------------------. caused by BARTONELLA HENSELAE.
. Manifest as several cutaneous & visceral angioma like blood vessels.
. EXOPHYTIC PURPLE SKIN LESIONS.
. I.V. drug abusers r more prone to developing tricuspid endocarditis:
---------------------------------------------------------------------. caused by STAPHYLOCOCCAL AUREUS.
. Fragments of the vgetation can embolize to the lungs,
. causing the characteristic nodular infiltrate with cavitation.
. PID PELVIC INFLAMMATORY DISEASE :
----------------------------------. Any PID .. Give .. Chlamydia (Azithromycin) + Gonorrhea (Ceftriazone).
. Any PID pt sh'd be routinely screened for $yphilis .. HIV .. HBV .. PAP smear.
. 2ry SYPHILIS:
--------------. Maculo-papular rash involving the palms & soles + Generalized lymphadenopathy. .
Spirochete infection.
. PNEUMONIAS:
------------. POST-INFLUENZA ---------------> STAPH. AUREUS.
. HIV---------------------------> PCP.
. D.M. & Alcoholics-------------> KLEBSIELLA.
. C.F. & Bronchiectasis---------> PSEUDOMONAS.
. Atypical $ dry cough----------> MYCOPLASMA.
. Aspiration--------------------> ANAEROBES.
. o"H"io----> "H"ISTOPLASMOSIS:
------------------------------. HIV pt with CD 4 cell count <100.
. T.B. like pulm. syms with FHMA & weight loss.
. HEPATOSPLENOMEGOLY + Palatal ulcers.
. CXR: Bilateral reticulonodular opacities.
. Dx: URINE ANTIGEN.
. Tx: ITRACONAZOLE.
Dr. Wael Tawfic Mohamed
-------------------------

NEUROLOGY TiKi TaKa


_____________________
. HEMORRHAGE -> HYPER-dense areas on CT. (WHITE).
. INFARCTIONS -> HYPO-dense areas on CT. (BLACK).
. GAITS IN NEUROLOGY:
_____________________
1. FESTINATING ----> PARKINSONISM:
___________________________________
(Mask face - bradykinesia - resting tremor - rigidity).
. N.B. PARKINSONISM's gait -> FESTINATING = HYPOKINETIC = SHUFFLING.
2. HIGH STEPPAGE --> TABES DORSALIS:
_____________________________________
(Neuro$ - Loss of proprioception - +ve Romberg sign).
3. SEMI-CIRCLE ----> STROKE HEMIPLEGIA:
_______________________________________
(Adducted affected arm & Extended affected leg).
4. WADDLING -------> MUSCULAR DYSTROPHY:
_________________________________________
(Weakness of gluteal muscles).
5. WIDE BASED & SHUFFLING -> NORMAL PRSSURE HYDROCEPHALUS:
___________________________________________________________
(Urine incontinence & dementia).
6. IPSILATERAL ATAXIA -> CEREBELLAR ATAXIA:
____________________________________________
The pt tends to fall towards the side of the lesion,
(Nystagmus-Hypotonia-Dysarthria-Loss of coordination-Dysdiadokokinesia).
7. SPASTIC ----> UMNL UPPER MOTOR NEURON LESION:
_________________________________________________
(Spinal cord injury or cerebral palsy).
8. STAGGERING -> VESTIBULAR ATAXIA:
____________________________________
(Ass. with nausea & vomiting).
. TREMORS IN NEUROLOGY:
_______________________
1. RESTING TREMORS (PARKINSON):
________________________________
* At rest - improves e' activity.
* High frequency tremors 5-7 Hz.
* Ass. e' rigidity & bradykinesia.
* Pill rolling quality.

* NOT ivolving the entire head.


2. ESSENTIAL TREMORS:
______________________
* Familial in up to 50 % of cases.
* Starts with fine movement in the upper extremity.
* Worst at the end of the goal directed activity (e.g. reaching a pen).
* Involving the entire head.
3. CEREBELLAR TREMORS:
_______________________
* Intension tremors.
* low fequency 3-4 Hz.
* Affect the extremity & the Whole head.
* Nystagmus & ataxia are present.
. TRIGEMINAL NEURALGIA:
_______________________
. Paroxysmal, LIGHTENING PAIN on the face.
. Severe intense burning or electric shock like.
. Tx: CARBAMAZEPINE.
. CEREBELLAR TUMORS:
____________________
. Ipsi-lateral ataxia (The pt. falls towards the side of the lesion).
. Ipsi-lateral muscular hypotonia.
. Titubation (Forward & backward movement of the trunk).
. Nystagmus.
. Intention tremors.
. Dysdiadokokinesia (Difficulty in performing rapid & alternating movements).
. INTRA-CRANIAL HEMORRHAGE:
___________________________
. HYPERTENSION is the most imp. risk factor.
. Focal neurological signs develop suddenly & gradually worsen over mins to hours.
. The degree of symptoms is not maximal at onset (# SAH or embolic stroke).
. Symptoms start during normal activity (may be ppt by sex).
. Site of INTRA-CRANIAL HEMORRHAGE ------> NEUROLOGICAL FINDINGS:
_________________________________________________________________
1- BASAL GANGLIA (PUTAMEN):
___________________________
* Hemi-plegia, hemi-sensory loss.
* Homonymous hemianopsia, gaze palsy.
* Stupor & coma.
2- "T"HALAMUS:
______________
* Hemi-paresis, hemi-sensory loss.
* Eyes deviate "T"owards hemiparesis.
* UP-GAZE palsy.
* (Non-reactive) miotic pupils.
3- CEREBELLUM:

______________
* NO hemiparesis.
* GAIT ATAXIA.
* OCCIPITAL HEADACHE (+nausea & vomiting).
* Gaze palsy (6th CN. paralysis)
* Facial weakness.
4- "P"ONS:
__________
* COMPLETE PARAPLEGIA.
* Followed by deep coma in a few mins.
* (REACTIVE) "P"IN POINT PUPILS.
5- CEREBRAL:
____________
* May be associated with seizures.
* Eyes deviate AWAY from the hemi-paresis.
. GUILLAIN BARRE' $YNDROME:
___________________________
. Acute idiopathic polyneuropathy.
. Ascending paralysis (i.e. affects LL 1st then involve the rest of the ms upwards!).
. Preceided by infection or vaccination.
. weakness in both legs then ascends to involve the arms, respiratory ms & face.
. Reflexes are diminished or symptoms.
. Distal paresthesia may occur.
. Dx: CSF ANALYSIS -> HIGH PROTEIN CONCENTRATION with NORMAL CELL COUNT.
. ++ PROTEIN & NORMAL (WBCs - RBCs - GLUCOSE) !
. i.e. CYTO-ALBUMINOUS DISSOCIATION.
. Tx: Supportive care, IVIG (Intravenous immunoglobulins) & plasmapharesis.
. GB$ may lead to respiratory failure.
. LUNG VITAL CAPACITY is the best way to monitor the respiratory function.
. N.B. TICK BORNE PARALYSIS:
____________________________
. Progressive ascending paralysis.
. Over hours - days.
. NO fever.
. Normal sensations.
. Normal CSF analysis.
. Meticulous search & removal of the tick results in improvement & complete recovery.
# PARA-NEOPLASTIC $YNDROMES !
_____________________________
.1. MYASTHENIA GRAVIS:
______________________
. Female 18 - 25 ys.
. NEURO-MUSCULAR JUNCTION DISEASE.
. Muscle weakness after a period of muscle use.
. Dysarthria - Dysphagia.
. Drooping eyelids (Ptosis) - Diplopia (Double vision). {Extraocular ms involvement}.
. Generalized weakness may develop (trunks - arms - legs). {Bulbar ms involvement}.
. RESOLUTION OF MUSCULAR WEAKNESS with REST is the HALLMARK of Myasthenia gravis.
. Dx: CT SCAN CHEST is MANDATORY to exclude THYMOMA.

.
.
.
.

Tx: Oral ANTI-CHOLINESTERASES e.g. PYRIDOSTIGMINE & NEOSTIGMINE.


Immunosuppressive agents & thymectomy may induce remission.
MYASTHENIC CRISIS may occur resulting in severe weakness of the respiratory muscles.
Tx with ENDOTRACHEAL INTUBATION & withdrawal of anti-cholinesterases.

.2. LAMBERT EATON $YNDROME:


___________________________
. H/O of cancer mostly LUNG CANCER (Heavy smoking - weight loss - malaise - lung mass).
. Small cell carcinoma.
. Proximal ms weakness.
. Auto-antibodies directed against the voltage gated calcium channels,
. leading to -- Acetylcholine release with proximal ms weakness.
. Dx: Electro-physiological studies.
. Tx: Plasmapharesis & immunosuppressive therapy.
. N.B. MYASTHENIA GRAVIS:
_________________________
. Auto-antibodies against the (post)-synaptic receptors.
. INTACT deep tendon reflexes.
. N.B. LAMBERT-EATON $:
________________________
. Auto-antibodies against the (pre) - synaptic receptors.
. LOSS of deep tendon reflexes.
.3. DERMATOMYOSITIS/POLYMYOSITIS:
___________________________________
. MUSCLE FIBER INJURY.
. Symmetric & more proximal ms weakness.
. Ass. ILD, esophageal dysmotility, Raynaud's phenomenon & polyarthritis.
. SKIN FINDINGS (Gottron's papules & Heliotrope rash).
. N.B. STEROID INDUCED MYOPATHY:
________________________________
. Due to ttt with HIGH doses of steroids over a prolonged period of time.
. ex: ttt of Temporal arteritis with high dose steroids.
. Proximal muscle weakness (LL before UL).
. No pain.
. Difficulty getting up from a chair - climbing stairs or brushing hair.
. Muscle power improves after discontinuation of the drug.
. SUB-ARACHNOID HEMORRHAGE:
___________________________
. Caused by rupture of arterial saccular "Berry" aneyrysm.
. Sudden severe headache (WORST HEADACHE EVER).
. Meningeal irritation may occur (Neck stiffness).
. Dx -> NON CONTRAST HEAD CT.
. Dx -> is imp. to rule out SAH.
. Dx -> Xanthochromia in CSF confirms the diagnosis.
. Dx -> CT cerebral Angiography is imp. to identify the bleeding source.
. Tx -> Coiling or restenting (Endovascular therapy).
. Tx -> Nimodipine (CCB) to -- the vasospasm.
. Complications:
. ______________
. 1 - Re-bleeding (1st 24 hours).

.
.
.
.

2
3
4
5

- Vasospasm (after 3 days).


- Hydrocephalus (++ ICT).
- Seizures.
- HYPO-NATREMIA (--Na due to SIADH).

. NEURO-FIBROMATOSIS TYPE 2:
____________________________
. YOUNG pt.
. S.C. neurofibromas + Cafe' au lait spots + Bilateral acoustic neuromas (Deafness).
. Family H/O.
. Autosomal dominant dis. caused by a mutation in chromosome 22.
. NON-SENSE or frame shift mutations are the cause.
. N.B. Silent (Same sense) mutations don't affect the structure of the protein.
. Dx: MRI with GADOLINIUM.
. PRONATOR DRIFT:
_________________
. It denotes UMNL.
. When the pt. closes his eyes & extends his arms with the palms up,
. The affected arm will tend to pronate.
. Bec. UMNL causes weakness in supination with dominance of the pronator muscles.
. ESSENTIAL TREMOR:
___________________
. ACTION tremor.
. Absence of other neurological signs.
. Suppressed at rest (# parkinsonism).
. Noticed when the pt. attempts a task that requires fine motor movement !
. Tx: BB (Propranolol) is the 1st line of ttt.
. Primidone may be used (Anti-convulsant which may ppt acute intermittent Porphyria,
. manifested as abdominal pain, neurologic & psychiatric abnormalities.
. MULTIPLE SCLEROSIS:
_____________________
. Affects women in child bearing peiod (15-50 ys).
. Multiple neurological deficits that can't be explained by single lesion.
. "PATCHY" neurological manifestations.
. Optic neuritis (painful loss of vision) & diplopia.
. Sensory symptoms -> Numbness & paresthesia.
. Motor symptoms -> Paraparesis & spasticity.
. Bowel/bladder dysfunction.
. "UHTHOFF phenomenon" Exacerbated by hot weather or exercise !
. "LHERMITTE's sign" Electric shock-like sensation down the spine on flexion of the neck.
. INTER-NUCLEAR OPHTHALMOPLEGIA (INO) is characteristic:
_______________________________________________________
* On attempted left gaze, the left eye abducts & exhibits horizontal jerk nystagmus,
* but the right eye remains stationary.
* On attempted right gaze, the right eye abducts & exhibits horizontal jerk nystagmus,
* but the left eye remains stationary.
* caused by demyelination of the MEDIAL LONGITUDINAL FASCICULUS.
. Dx: BRAIN MRI with & without GADOLINIUM.
. MRI:Multiple bilatreal asymmetric hyperintense lesions in periventricular white matter.
. CSF analysis: OLIGOCLONAL IgG bands - Normal pressure.
. Tx of acute exacerbation -----> HIGH DOSE IV GLUCOCORTICOIDS.
. Tx to prevent future attacks -> B-interferon or Glatiramer acetate.

. N.B. YOUNG FEMALE with BILATERAL TRIGEMINAL NEURALGIA = MS.


. AMYOTROPHIC LATERAL SCLEROSIS:
________________________________
. UPPER + LOWER motor neuron lesions.
. UMNL (Spasticity - bulbar symptoms - exagerrated deep tendon reflexes).
. LMNL (Fasciculations, wasting).
. Tx: RILUZOLE (Glutamate inhibitor) - Steroids are WRONGGGGGGGGGGGGGGG !
. CARPAL TUNNEL $YNDROME = MEDIAN NERVE ENTRAPPMENT $YNDROME:
_____________________________________________________________
. At WRIST !
. Numbness & pain in the palm.
. Thenar eminence atrophy.
. Paresthesia of the 1st three & a half digits
. ULNAR NERVE ENTRAPMENT $YNDROME:
___________________________________
. -- sensation over the 4th & 5th fingers with weak grip.
. due to involvement of the interosseus muscles of the hand.
. entrapment at the medial epicondylar groove.
. Leaning on the elbows while working at a desk or table is the typical scenario.
. VESTIBULO-TOXICITY by AMINO-GLYCOSIDES:
_________________________________________
. Gentamycin & Amikacin.
. Vertigo & gait imbalance.
. due to damage of the motion sensitive hair cells in the inner ear.
. TORTICOLLIS:
______________
. Example of FOCAL DYSTONIA.
. Dystonia -> Sustained ms contraction.
. Focal -> Affecting one muscle.
. Involuntary head turning & fixation to one side.
. Hypertrophy of the opposite side sterno-cleido-mastoid ms.
. It is a common side effect of Anti-psychotic drugs.
. UN-PROVOKED FIRST SEIZURE -> HEAD CT WITHOUT CONTRAST is the 1st initial step done:
_____________________________________________________________________________________
. To exclude intracranial or subarachnoid bleeding requiring urgent intervention.
. MRI is the best diagnostic modality in elective situations for seizures cases.
. LIMB ISCHEMIA:
________________
. Mostly due to migration of arterial emboli from the heart.
. The emboli source may be Af or recent MI.
. 5 Ps (Pain - Pallor - Paresthesia - Pulselessness & Paralysis).
. Tx: IV HEPARIN BOLUS followed by continous heparin infusion.
. Referral for emergency vascular surgery.
. METOCLOPRAMIDE:
_________________
. It is a pro-kinetic agent used to treat nausea , vomiting & gastro-paresis.
. Pts sh'd be monitored closely for the development of drug induced extra-pyramidal syms.

. Ex: Tardive dyskinesia - Dystonic reactions & prkinsonism.


. Manifested by stiff painful neck.
_________________________________________________________________________________________
_________________________________________________________________________________________
# MAIN CAUSES OF STROKE:
_________________________
. 1 . ISCHEMIC THROMBOTIC:
__________________________
-> H/O of previous TIAs (Transient ischemic attacks).
-> Atherosclerotic risk factors (Uncotrolled HTN & DM).
-> Local in-situ obstruction of an artery.
-> Symptoms may progress or regress with time.
. 2 . ISCHEMIC EMBOLIC:
_______________________
-> H/O of cardiac disease (Af, endocarditis or carotid atherosclerosis "Bruit").
-> Onset of symptoms is ABRUPT & usually MAXIMAL at the start.
-> Multiple infarcts within different territiories.
-> NO headache or impaired consciousness.
. 3 . HEMORRHAGIC:
__________________
-> H/O of uncontrolled HTN, co-agulopathy, illicit drug use e.g amphetamines & cocaine.
-> Sudden development of focal neurological signs.
-> Followed by ++ ICT symptoms (vomiting & headache).
-> Worsens gradually over mins to hours.
-> Symptoms may start with normal activity.
-> Hypertension is the most imp. risk factor.
. 4 . SPONTANEOUS SUB-ARCHNOID HEMORRHAGE:
__________________________________________
-> Rupture of an arterial saccular berry aneurysm or AV malformation.
-> Sudden dramatic onset of severe headache (WORST HEADACHE EVER).
-> Meningeal irritation e.g. neck stiffness.
-> Focal deficits are uncommon.
# The 1st step in STROKE management is NON CONTRAST HEAD CT.
_____________________________________________________________
# TOPOGRAPHY of the lesions in stroke pts (ACCORDING TO THE AFFECTED ARTERY):
______________________________________________________________________________
* POSTERIOR LIMB OF INTERNAL CAPSULE (LACUNAR INFARCT):
________________________________________________________
. Motor impairment without any higher cortical dysfunction.
. No visual field abnormalities.
* MIDDLE CEREBRAL ARTERY OCCLUSION:
____________________________________
. Contralateral hemiplegia.
. Contralateral hemianesthesia.
. Conjugate eye deviation toward the side of stroke.

. Homonymous hemianopia.
. Aphasia (dominant hemisphere).
. Hemi-neglect (Non dominant hemisphere).
* ANTRIOR CEREBRAL ARTERY OCCLUSION:
_____________________________________
. Contralateral weakness that predominantly affects the LLs.
. Abulia (loss of willing).
. Akinetic mutism.
. Emotional disturbances.
. Deviation of head & eyes towards the side of the lesion.
. Sphincter incontinence.
* VERTEBRO-BASILAR SYSTEM LESION (BRAIN STEM):
_______________________________________________
. Alternate $ with contralateral hemiplegia & ipsilateral CN involvement.
# Presentations according to the AFFECTED LOBE:
________________________________________________
. DOMINANT FRONTAL LOBE STROKE:
_______________________________
. Expressive (BROCA's) aphasia.
. Contralateral hemiparesis (due to involvement of the primary motor cortex).
. Contralateral apraxia (due to involvement of the supplementary motor cortex).
. DOMINANT PARIETAL LOBE STROKE:
________________________________
. Contralateral sensory loss (pain, vibration, agraphesthesia & astereognosis).
. Contralateral inferior homonymous quadrantanopsia (Superior optic radiation lesion).
. DOMINANT TEMPORAL LOBE STROKE:
________________________________
. APHASIAS due to ARCUATE FASCICULUS involvement.
. Reception aphasia (Affect comprehension).
. Anomic aphasia (inability to speak nouns).
. Conductive aphasia (Repitition)
. Contralateral inferior homonymous quadrantanopsia (Superior optic radiation lesion).
# PRESENTATION ACCORDING TO TEH AFFECTED PART OF THE BRAIN:
____________________________________________________________
.1. BRAIN STEM LESIONS:
________________________
. involve the cranial nerves.
. sensory loss of one half of the face & contralateral half of the body.
.2. THALAMUS LESIONS:
______________________
. Hemi-sensory loss with severe dysesthesia (THALAMIC PAIN PHENOMENON).
.3. CORTICAL LESIONS:
______________________
. sensory loss of one half of the face & ipsilateral half of the body.
. Aphasia - Neglect - Abnormal graphesthesia or stereognosis.

.4. MEDIAL MEDULLARY $YNDROME:


_______________________________
. Due to occlusion of the vertebral artery or one of its branches.
. Contralateral paralysis of the arm & leg.
. Contralateral loss of tactile, vibratory & position sensation.
. Ipsilateral Tongue deviation.
# THE MOST COMMON RISK FACTOR OF STROKE IS HYPERTENSION.
_________________________________________________________
# LACUNAR STROKES:
___________________
. Most common site is in the POSTERIOR LIMB OF THE INTERNAL CAPSULE.
. Most common cause is HYPERTENSION & DM.
. Lipo-hyalinotic thickening of the small vessels.
. Micro-atheromas.
. LIMITED neurological deficit.
. Pure motor or sensory stroke - Ataxic hemiparesis - Dysarthria with clumsy hand $.
. May be missed on CT due to their small size.
1. PURE MOTOR HEMIPARESIS:
__________________________
. Lacunar infarct in the POSTERIOR LIMB OF THE INTERNAL CAPSULE.
. Unilateral motor deficit (face, arm & to a lesser extent leg).
. Mild dysarthria (Slurred speech).
. NO sensory, visual or higher cortical dysfunction.
2. PURE SENSORY STROKE:
_______________________
. Lacunar infarct in the ventro-postero-lateral of the THALAMUS.
. Unilateral numbness, paresthesia.
. Hemisensory deficit in the face, arm, trunk & leg.
3. ATAXIC HEMIPARESIS:
______________________
. Lacunar infarct in the ANTERIOR LIMB OF THE INTERNAL CAPSULE.
. Weakness more prominent in LL extremity.
. Ipsi-lateral arm & leg incoordination.
4. DYSARTHRIA CLUMSY HAND $YNDROME:
___________________________________
. Lacunar stroke at the basis pontis.
. Hand weakness, mild motor aphasia.
. NO sensory deficits.
# STROKE MANAGEMENT:
_____________________
1- NON contrast head CT to rule out hemorrhagic stroke.
2- Ischemic stroke -> Give fibrinolytic therapy (if the pt comes within 3-4 hs of onset).
3- Make sure that the pt. has no contraindications to the fibrinolysins.
4- If there is contraindication -> Give Antiplatelets (ASPIRIN).
. Clinical presentation "ischemic stroke case" -> Anti-platelet/Anti-thrombotic therapy:
________________________________________________________________________________________

.
.
.
.
.

Presenting within 3 - 4.5 hs of symptoms onset with no cont'ds -> I.V. Alteplase.
Stroke with no prior anti-platelet therapy -> Aspirin.
Stroke on Aspirin therapy ->(Aspirin + dipyridamole) OR (Clopidogrel).
Stroke on Aspirin therapy + intracranial large art. sclerosis -> Aspirin + Clopidogrel.
Stroke with evidence of atrial fibrillation -> LONG TERM ANTICOAGULATION e.g. WARFARIN.

# THROMBOLYTICS = TISSUE PLASMINOGEN ACTIVATOR (t-PA) = ALTEPLASE:


___________________________________________________________________
# THROMBOLYTICS INDICATIONS & CONTRAINDICATIONS:
_________________________________________________
# THROMBOLYTICS INDICATIONS:
_____________________________
.1. Non hemorrhagic ischemic stroke.
.2. Symptoms onset < 3 - 4.5 hours before treatment initiation.
# THROMBOLYTICS CONTRA-INDICATIONS:
____________________________________
.1. Stroke or head trauma in the past 3 months.
.2. H/O of intracranial hemorrhage.
.3. Major surgery in the past 2 weeks.
.4. GI,GU or active bleeding in the past 3 weeks.
.5. Seizure at the onset of stroke.
.6. SBP > 185 mmHg or DBP > 110 mmHg.
.7. Platelets < 100000/mm3 , Glucose < 50 mg/dl , INR > 1.7.
__________________________________________________________________________________________
__________________________________________________________________________________________
. HEMI-NEGLECT $YNDROME = LESION in the (RIGHT PARIETAL LOBE CORTEX):
_____________________________________________________________________
. Lesion of the RIGHT (NON)-dominant hemi-sphere.
. which is responsible for spatial organization.
. So, In this disease, The pt ignores the left side of a space.
. Responds only to the stimuli coming from the RIGHT side.
. Pt may shave only the Right side of their face.
. Comb the Right side of his hair.
. Ignore the subject located in the left side of a space.
. Dx: Ask the pt to fill in the numbers o a clock !
. EXERTIONAL HEAT STROKE:
_________________________
. Severe exertion under direct sun light.
. Acute confusion, hyperthermia, tachycardia & persistent epistaxis.
. Due to FAILURE OF THERMO-REGULATORY CENTER to maintain a euthermic state.
. Core temperature > 40 with altered mental status.
. Factory workers, Military recruits exposed to hot humid environment.
. Complications: Rhabdomyolysis - RF - ARDS - Coagulopathic bleeding.
. Tx -> EVAPORATION COOLING (NOT immersion in cold water xxx).
. MALIGNANT HYPERTHERMIA:
_________________________
. Genetically susceptible pt during anesthesia.

. Ass. with halothane & succinyl choline.


. Uncotrolled efflux of calcium from the sarcoplasmic reticulum.
. CAVERNOUS SINUS THROMBOSIS:
_____________________________
. Un-controlled infection of the skin, sinuses & orbit may spread to he cavernous sinus.
. Bec. the facial / ophthalmic venous system is valveless !
. Cav. sinus inflammation may lead to cav. sinus thrombosis & intracranial hypertension.
. HEADACHE (INTOLERABLE) is the most common symptom.
. Vomiting is common due to ++ ICT.
. Fundoscopy will reveal papilledema.
. Binocular palsies, periorbital edema with hypo/hperesthesia.
. Dx: MRI.
. Tx: Broad spectrum Antibiotics.
. RESTLESS LEG $YNDROME:
________________________
. Uncomfortable "Crawling" sensation or urge to move the legs.
. Discomfort which worsens in the evening or during sleep.
. Discomfort which worsens at rest.
. Discomfort alleviated by movement of the affected limb.
. Tx : Dopaminergic agonists e.g. L-dopa.
. WERNICKE's ENCEPHALOPATHY:
____________________________
. Alcoholic pt.
. Altered mental status + Gait instability + Nystagmus + Conjugate gaze palsy.
. Due to Vit. B "1" defeciency 2ry to long term alcohol use.
. Triad of ecephalopathy, oculomotor dysfunction & gait ataxia is diagnostic.
. DECUBITUS ULCER:
__________________
. H/O of old pt in a care giver facility.
. Continued pressure on a bony prominence for a long period.
. Ischemic necrosis of the overlying ms, S.C. tissue & skin.
. Preveted by repositioning of the pt every 2-4 hours.
. LEVO-DOPA / CARBI-DOPA Side effects:
_____________________________________
. Dopamine precursors.
. Most common side effect is HALLUCINATIONS.
. Others: Dizziness, Headaches & agitation.
. Involuntary movements may occur.
. TRI-HEXY-PHENIDYL side effects ( VERY IMPORTANT .. ASKED TWICE in UW):
________________________________________________________________________
. It is an Anti-cholinergic drugs used for ttt of Parkinsonism.
. Red as beet, dry as bone, hot as hare, blind as bat, mad as hatter & full as a flask.
. Red as beet
: Flushing.
. Dry as bone
: Anhydrosis - dry mouth.
. Hot as hare
: Hyperthermia.
. Blind as bat : Mydriasis - vision changes.
. Mad as hatter : Delirium - cofusion.
. Full as a flask : Urine retention - constipation.

# DEMENTIAS:
_____________
.1. FRONTO-TEMPORAL DEMENTIA (Pick's disease):
______________________________________________
. Personality changes (euphoria - disinhibition - apathy).
. Compulsive behavior (peculiar eating habits - hyperorality).
. Impaired memory.
. Family H/O of the disease is common.
.2. LEWY BODIES DEMENTIA:
_______________________
. Fluctuating cognitive impairment.
. Bizarre visual hallucinations.
.3. ALZHEIMER's DISEASE:
______________________
. Progressive dementia.
. Age, female gender, +ve family H/O, head trauma are common risk factors.
. Subtle memory loss, language difficulties & apraxia.
. Impaired judgement & personality changes.
. N.B. HYPOTHYROIDISM is an imp. cause of reversible changes in memory & mentation:
___________________________________________________________________________________
. Accompanied by systemic changes e.g. weight gain, fatigue, hoarseness & constipation.
.4. HUNTINGTON's DISEASE:
_________________________
. Triad of mood disturbances + Choreiform movements + Dementia.
. Due to ATROPHY of the CAUDATE NUCLEUS.
. Autosomal dominant (Gene defect on chromosome 4).
. Affects both sexes equally.
. Family H/O of the disease is present.
. Age 30 - 50 ys.
. Mood disturbances (Depression & apathy).
. Choreiform movements (facial grimacing, ataxia, dystonia, tongue protrusion).
. Writhing movements of the extremeties.
.5. CREUTZFELDT - JAKOB DISEASE:
______________________________
. Age 50 - 70 ys.
. It is a spongiform encephalopathy caused by a prion.
. Rapidly progressive dementia & myoclonus.
. EEG -> SHARP TRI-PHASIC SYNCHRONOUS DISCHARGES.
. Pts die within one year of onset.
.6. NORMAL PRESSURE HYDROCEPHALUS:
________________________________
. Triad of Urine incontinence + Abnormal gait + Dementia.
. Gait -> Broad based & shuffling
. ++ in ventricular size without persistent ++ in ICT.
. Symptoms due to distortion of the periventricular brain matter.
. The cause is -- CSF ABSORPTION.
. Dx: CT or MRI -> ENLARGED VENTRICLES.
. Dx: LP -> NORMAL OPENING PRESSURE.

. Tx: VENTRICULO-PERITONEAL SHUNT.


.7. PSEUDO-DEMENTIA:
____________________
. Major depressive episode may present as pseudo-dementia.
. Elderly pts who r severely depressed may present with memory loss.
. H/O of emotional situation with the pt. (e.g. Pt's son moving out !).
. Symptoms coincides with the emotional situation.
. Tx -> Anti-depressants e.g. SSRIs (SLECTIVE SEROTONIN RE-UPTAK INHIBITORS).
.N.B. NORMAL AGING:
___________________
. Tiredness.
. occasional forgetfulness.
. occasional word finding difficulty
. Trouble falling asleep.
. Absence of functional impairments.
. Normal performance on mental status examination.
. BRAIN DEATH:
_______________
. Irreversible cessation of the brain activities.
. Absent cortical & brain stem functions.
. Absent corneal reflex.
. Absent gag reflex.
. Absent oculovestibular rflex.
. FIXED DILATED PUPILS.
. No spontaneous breathing when the ventillator is off for 10 mins.
. Spinal cord may be still functioning, so, DEEP TENDON REFLEXES may be STILL PRESENT.
. MUST BE CONFIRMED BY TWO PHYSICIANS.
. SHY DRAGER $YNDROME:
______________________
. MULTIPLE SYSTEM ATROPHY.
. PARKINSONISM pt.
+ Autonomic dysfunction (postural hypotension - bowel&bladder loss of control-impotence).
+ Widespread neurological signs (cerebellar, pyramidal or lower motor neuron).
. Chronic alcohol abuse -> Cerebellar damage:
_____________________________________________
. loss of co-ordiated movement.
. Ataxia.
. Broad based gait.
. Dysmetria.
. Intention tremors.
. Dysdiadokokinesia.
. Nystagmus.
. Ms hypotonia (pendular knee reflex).
. BELL's PALSY:
_______________
. Facial 7th cranial nerve peripheral neuropathy.
. Sudden onset of unilateral facial paralysis.
. Inability to close the eye on the affected side.
. Inability to raise the eye brow on the affected side.

.
.
.
.
.
.
.

Drooping of the mouth corner with disappearnce of the nasolabial fold,


so, the mouth is drawn to the spared side.
Diminished tearing.
Hyperacusis.
Loss of taste sensation over the anterior 2/3s of the tongue.
If the lesion in the CNS occuring above the facial nucleus,
it will typically CONTRALATERAL LOWER FACIAL WEAKNESS SPARING THE FOREHEAD.

. AMAUROSIS FUGAX:
__________________
. Painless loss of vision.
. Cholesterol particles may be seen in the eye.
. It is a warning sign of impending stroke.
. An underlying embolic disease is most always present.
. Emboli occur at the carotid bifurcation.
. Dx: NECK DUPLEX ULTRA$OUND.
. SUB-DURAL HEMATOMA:
_____________________
. Due to BLUNT or shearing trauma tearing the BRIDGING VEINS.
. causing them to slowly bleed into the subdural space.
. Headache & gradual loss of consciousness occur gradually.
. More common in older pts & alcoholics due to brain atrophy & vessel fragility.
. NON contrast head CT -> WHITE CRESCENT..
. Mass effect with mid line shift may be seen.
. Emergent neurosurgical consultation for hematoma evacuation is necessary.
. EPI-DURAL HEMATOMA:
_____________________
. Trauma to the TEPORAL bone.
. Injury to the MIDDLE MENINGEAL ARTERY.
. Non contrast head CT -> BICONVEX HEMATOMA.
. SYRINGOMYELIA = CORD CAVITATION:
__________________________________
. Idiopathic Cavitary expansion of the spinal cord.
. Affets the upper limbs in a CAPE like distribution.
. Areflexic weakness in the upper extremeties.
. Dissociated anesthesia (Loss of pain & temperature with intact position & vibration).
. A cord cavity is present !
. Lower cervical or upper thoracic are the most common affected sites.
. DIABETIC NEUROPATHY:
______________________
. Symmetric peripheral polyneuropathy, mononeuropathy or autonomic neuropathy.
. Mononeuropathies either cranial or somatic.
. CN 3 (Oculomotor) is the most common affected.
. The cause of neuropathy is ISCHEMIC.
. Somatic & parasympathetic fibers in CN 3 have separate blood supplies.
. So .. Only somatic fibers are affected while the parasympathetic fibers are intact.
. Manifested by PTOSIS & DOWN & OUT GAZE.
. Accomodation & light reflex are intact.
. SPINAL CORD COMPRESSION:
__________________________

.
.
.
.
.
.

isolated, symmetric, lower extremity symptoms.


Loss of sensations & signs of upper motor neuron lesion.
Weakness without fasciculations, hyperreflexia & +ve Babinski sign.
Possible etiologies: (Disk herniation - Epidural absess & malignancy).
It is a medical emergency !
Dx: MRI Spine .. NOT CT !!

. ALZHEIMER's DISEASE:
______________________
* It is the most common cause of dementia.
* NO disturbance in consciousness.
* Age group > 60.
* EARLY FINDINGS:
__________________
. Anterograde memory loss (immediate recall affected, distant memory preserved).
. Visuospatial deficits (lost in own neighborhood).
. Language difficulties (difficulty finding words).
. Cognitive impairment with progressive decline.
* LATE FINDINGS:
_________________
. Neuropsychiatric (hallucinations & wandering).
. Dyspraxia (difficulty performing learned motor tasks).
. Lack of insight regarding deficits.
. Non-cognitive neurological deficits (pyramidal & extra-pyramidal motor, myoclonus).
. Urinary incontinence.
* Dx: CT -> Diffuse cortical & subcortical atrophy,
which is disappropriately greater in the temporal & parietal lobes.
. GLIOBLASTOMA MULTIFORME (GBM) = HIGH GRADE ASTROCYTOMA:
_________________________________________________________
. Symptoms of ++ ICT (Nausea-vomiting-headache worsening with change in position).
. ++ ICT = Space occupying lesion.
. Personality changes & strange behavior (Due to involvement of the frontal lobe).
. Dx: CT or MRI -> BUTTERFLY appearance with central necrosis,
. with HETEROGENOUS SERPIGINOUS CONTRAST ENHANCEMENT.
. CRANIOPHARYNGIOMA = Hypopituitarism signs + Headaches + Bitemporal blindness:
_______________________________________________________________________________
. Benign tumors arising from Rathke's pouch.
. Bimodal age distribution i.e. children & 55-65 ys age group.
. It is located above sella turcica.
. Consists of multiple cysts filled with oily fluid.
. Presents with symptoms of hypothyroidism.
. In children (Retarded growth due to -- GH & TSH).
. In adults (Sexual dysfunction).
. Women may present with amenorrhea.
. It compresses the optic chiasma -> BITEMPORAL BLINDNESS.
. Headaches occur due to ++ ICT.
. Dx: MRI or CT.
. Tx: Surgery or radiotherapy.
. CAUDA EQUINA $YNDROME:

________________________
. Compression of the spinal nerve roots.
. Causes (Tumor - Herniated disk - Abscess - Trauma).
. Low bk pain.
. Bowel & bladder dysfunction.
. Saddle anesthesia.
. Sciatica.
. Lower extremity sensory & motor loss.
. Dx: Emergent MRI.
# HIV associated lesions on MRI:
________________________________
.1. PRIMARY CNS LYMPHOMA:
_________________________
. Solitary.
. {WEAKLY} ring enhancing peri-ventricular mass.
. Altered mental status.
. Associated EBV DNA in the CSF.
.2. TOXOPLASMOSIS:
__________________
. Multiple.
. {Ring - enhancing} spherical lesions in the basal ganglia.
. +ve serology is not specific !
. TMP-SMX is preventive.
.3. PROGRESSIVE MULTIFOCAL LEUKOENCEPHALOPATHY:
_______________________________________________
. {Non - enhancing}.
. No mass effects.
. Opportunistic viral infection (JC virus).
.4. AIDS DEMENTIA COMPLEX:
__________________________
. Cortical & sub-cortical atrophy.
. 2ry ventricular enlargement.
.5. ABSCESS:
____________
. Solitary.
. {Ring enhancing}.
. Isolated, round with smooth borders.
. H/O of known extra-cranial infections.
. Fluid collection in the maxillary sinus.
. The most common causative organisms are AEROBIC & ANAEROBIC STREPTOCOCCI &
BACTEROIDES.
# CAROTID ARTERY STENOSIS:
__________________________
. May progress to embolic stroke or TIAs.
. May be silent with no symptoms.
. Symptomatic -> sudden onset of focal neuro. syms ipsilateral to the blocked artery.
. Dx: NECK DUPLEX U/$.
. Tx: CEA or CAS.

. CAROTID END ARTERECTOMY (CEA) is recommended if:


_________________________________________________
. * Symptomatic pts with carotid stenosis 70 - 99 %.
. * Low surgical risk.
. * Good 5 year predicted survival.
. * Surgically accessible carotid lesion.
. CAROTID ANGIOPLASTY WITH STENTING (CAS) is recommended if:
___________________________________________________________
. * High surgical risk.
. * Poor 5 year predicted survival.
. * Lesion not amenable to surgery.
# IMPORTANT CRANIAL NERVES & THEIR FUNCTIONS:
_____________________________________________
* OPTIC NERVE (CN 2):
_____________________
. VISION.
* OCULOMOTOR (CN 3):
____________________
. Eye movement (Most).
. Adduction with medial rectus.
. Elevation with superior rectus.
. Depression with inferior rectus.
. Eye lid opening.
. Pupil constriction.
* TRIGEMINAL (CN 5):
____________________
. Three branches with both motor & sensory fibers.
. The 1st branch is called the Ophthalmic nerve,
. It carries sensory fibers to the scalp, forehead, upper eyelid, cornea & nose.
* FACIAL (CN 7):
________________
. Facial movement.
. Taste in the anterior 2/3s of the tongue.
. Lacrimation.
. Salivation.
. Eye closing.
* VAGUS (CN 10):
________________
. Swallowing.
. Palate elevation.
. Monitoring baro & chemo receptors of the aortic arch.
. IMPAIRED DAILY FUNCTIONING:
_____________________________
. is essential to distinguish between dementia & normal aging.
. Pts with dementia have functional impairment.
. Normal aging is not ass. with impairment.
. ACUTE GLAUCOMA:

_________________
. Occurs when a pre-existing narrow anterior chamber angle is closed,
. in response to pupillary dilatation from medications or another stimiulus.
. PPt by direct bright light e.g. watching TV.
. ++ IOP may lead to nausea & vomiting & tearing pain.
. Complain of seeing halos around light.
. Damage of the optic nerve is common & may lead to visual loss.
. Sudden onset of photophobia, eye pain, headache & nausea.
. Palpation -> very hard eye.
. NON REACTIVE MID DILATED PUPIL.
. Dx: TONOMETRY.
# HEADACHES:
____________
.1. MIGRAINE HEADACHE:
_______________________
. Unilateral.
. Pulsating quality.
. Attacks last from 4 - 72 hs.
. Photophobia.
. Common in younger females.
. AURA of neurological syms preceiding headache.
. Tx: TRIPTANS (Efficient only before the start of the attack) + NSAIDs.
. I.V. Anti-emetics e.g. (PRO-CHLOR-PERAZINE) or Metoclopramide {SEVERE VOMITING CASES}.
.2. CLUSTER HEADACHE:
______________________
. Intense unilateral retro-orbital pain.
. Starts suddenly (usually at night).
. More common in men.
. Redness of the ipsilateral eye.
. Tearing eye pain.
. Stuffed or runny nose.
. Ipsilateral Horner's $.
. Attacks occur in clusers.
. Prophylaxis is the key to management (Verapamil - Lithium - Ergotamine).
. Tx of acute attack -> 100 % OXYGEN & S.C. Sumatriptan.
.3. BENIGN IDIOPATHIC INTACRANIAL HYPERTENSION = PSEUDOTUMOR CEREBRI:
______________________________________________________________________
. Over-weight female in the child bearing period.
. H/O of OCPs intake or hypervitaminosis A.
. Headache - transient loss of vision - pulastaile tinnitus - diplopia.
. Ex: papilledema - peripheral visual defects.
. Dx: MRI & LP (CSF opening pressure > 250 mmHg with NORMAL analysis).
. Tx: Stop the offending medications, weight loss & Acetazolamide.
. ACETAZOLAMIDE +/- FUROSEMIDE is the 1st line therapy.
. Acetazolamide -> inhibits choroid plexus carbonic anhydrase -> -- CSF production.
. Most common complication is BLINDNESS !
. Shunting or optic nerve sheath fenestration is done to prevent blindness.
.4. SUB-ARACHNOID HEMORRHAGE:
______________________________
. WORST HEADACHE EVER !!

.
. BROWN SEQUARD $YNDROME:
_________________________
. Damage to the lateral spinothalamic tract.
. Causing contra-lateral loss of pain & temperature sensation,
. beginning two levels below the level of the lesion.
. N.B. The spino-thalamic tract crosses on very early in the spinal cord !
. so .. A lesion of the Rt-sided spino-thalamic tract at T10,
. will result in a Lt-sided loss of pain & temperature sensation beginning at T12.
. L5 RADICULOPATHY -> Foot drop -> Compensated by HIGH STEPPAGE GAIT:
_____________________________________________________________________
. Foot drop due to failure of the foot dorsiflexion.
. caused by trauma to the common peroneal nerve
. or one of the spinal roots contributing to it (L4 - S2).
. To compensate, HIGH STEPPAGE GAIT is done.
. Pts have to overly flex the hip & knee to bring the foot forward.
. The toes of the affected foot may drag on the ground.
. caused by peripheral neuropathy.
. Foot drop may be congenital (Charcot - Marie - Tooth disease).
. HERPES ENCEPHALITIS:
______________________
. Caused by HSV-1.
. Mainly affects the TEMPORAL lobe of the brain.
. Acute onset < 1 week duration.
. Altered mentation - focal neuro. deficits - hemiparesis - dysphasia - aphasia - ataxia.
. May present with seizures !
. FEVER is present in 90 % of cases.
. CSF analysis -> LYMPHOCYTIC PLEOCYTOSIS.
. .............-> ++ RBCs (Hemorrhagic destruction of the temporal lobes).
. .............-> ++ Ptn level.
. .............-> -- Glucose level
. Dx : HSV POLYMERASE CHAIN REACTION IS THE GOLD STANDARD.
. Tx : IV ACYCLOVIR.
. ETHICAL PROBLEM:
__________________
. REGARDLESS OF H/O OF DRUG ABUSE,,
. Pts with acute severe pain sh'd receive the same standard of pain management !!
. IV MORPHINE is the best ttt for acute severe pain.
. Physicians sh'd NEVER undertreat pain even if there is a risk for abuse.
. SITE OF THE LESION ----> DEFICIT:
___________________________________
. UPPER THORACIC SPINAL CORD ---> Paraplegia - Bladder & fecal incontinence,
. ............................... + Absent sensation from the (NIPPLE) downwards.
. LOWER THORACIC SPINAL CORD ---> Absent sensation from the (UMBILICUS) downwards.
. PARKINSON DISEASE = TREMORS + RIGIDITY + BRADYKINESIA:
________________________________________________________
. Neurodegenerative disorder.
. Caused by accumulation of alpha synuclein within the neurons of SUBSTANCIA NEGRA.
. The most common presenting symptom is asymmetric resting tremor in the upper extremity.

.1 * TREMOR:
___________
. A resting 4 to 6 Hz tremor with a pill-rolling quality.
. Frequently first manifests in one hand.
. May slowly generalize to involve the other side of the body & the lower extremity.
.2 * RIGIDITY:
______________
. Baseline ++ resistance to passive movement (Lead pipe or cog wheel).
.3 * BRADYKINESIA:
__________________
. Difficulty initiating movements as when starting to walk or rising from a chair.
. Narrow based, shuffling gait with short strides without arm swing (FESTINATING).
. Micrographia (Small hand writing).
. Hypomimia (-- facial expression).
. Hypophonia (soft speech).
.4 * POSTURAL INSTABILITY:
__________________________
. Flexed axial posture.
. Loss of balance during turning or stopping.
. Loss of balance when pushed slightly.
. Frequent falls.
. NO SPECIAL TEST FOR EXACT DIAGNOSIS.
. ONLY PHYSICAL EXAMINATION CAN LEAD TO THE Dx.
. STATUS EPILEPTICUS:
_____________________
. Single seizure lasting > 30 mins.
. H/O of seizure disorder with no compliance to anti-convulsant therapy.
. A brain seizing > 5 mins is at ++ risk of permanent injury : CORTICAL LAMINAR NECROSIS.
. Tx -> BENZODIAZEPINE -> IV DIAZEPAM.
. Failed -> ADD FOSPHENOTOIN.
. Failed -> ADD PHENOBARBITAL.
. Failed -> ADD SUCCINYL CHOLINE.
. DELIRIUM:
___________
. Acute confusion state.
. Reduced oe fluctuating level of consciousness.
. Inability to sustain attention.
. Anxiety, agitation & hallucinations.
. Common ppt factors (infections: UTI).
. Polypharmacy, medication side effects, volume depletion & electrolyte imbalance.
. SERUM ELECTROLYTES & URINALYSIS sh'd be done to detect the cause.
. Tx-> Typical & atypical anti-psychotics (HALOPERIDOL).
. Benzodiazepines (Lorazepam) are not recommended in old age.
. ONCE MORE: DIFFERENT CEREBRAL ARTERY OCCLUSIONS & THEIR EFFECTS:
__________________________________________________________________
* MIDDLE cerebral artery occlusion:

____________________________________
. Contralateral motor & sensory deficits.
. More pronounced in the {upper limb} than the lower limb.
. Homonymous hemianopia.
. If the dominant lobe (LEFT) is involved ------> APHASIA.
. If the NON dominant lobe (RIGHT) is involved -> HEMI NEGLECT $.
* ANTERIOR cerebral artery occlusion:
______________________________________
. Contralateral motor & sensory deficits.
. More pronounced in the {lower limb} than the upper limb.
. Urinary incontinence.
. Gait apraxia.
* POSTERIOR cerebral artery occlusion:
_______________________________________
. Homonymous hemianopia.
. Alexia without agraphia (dominant hemisphere).
. Visual hallucinations (cortex).
. Sensory symptoms (Thalamus).
. INTRACRANIAL HYPERTENSION:
____________________________
. ++ intra-cranial pressure > 20 mmHg.
. Causes: Trauma - space occupying lesion - hydrocephalus - impaired CSF outflow.
. Symptoms: Diffuse headache worse in the morning - Nausea & vomiting.
. Vision changes - papilledema - cranial nerve deficis.
. Somnolence - cofusion - Unsteadiness.
. Cushing's reflex: Hypertension & bradycardia.
. Dx: CT or MRI.
. CEREBRAL HEMORRHAGE due to EXCESS ANTI-COAGULATION:
_____________________________________________________
. Anti-coagulation therapy is the most common bleeding disorded causing brain hemorrhage.
. So .. Pts on anti-coagulants (e.g. Warfarin) sh'd be monitored regularly with INR.
. Risk of bleeding ++ with INR ++ !
. Correction of excess anti-coagulation is dependent upon the INR value:
. INR < 5 , NO significant bleeding -> Omit next Warfarin dose.
. INR 5-9 , NO significant bleeding -> Stop Warfarin temporarily.
. INR > 9 ---------------------------> Stop Warfarin, Give oral Vit. K.
. SERIOUS INTRA-CRANIAL BLEEDING -> FRESH FROZEN PLASMA (FFP).
. FFP reverses the actio of warfarin , works immediately & lasts for few hours.
. HYPOKALEMIA -> WEAKNESS, FATIGUE & MUSCLE CRAMPS:
___________________________________________________
. Electrolyte disturbance with K < 2.5 mEq/L.
. Flaccid paralysis, hyporeflexia, tetany, rhabdomyolysis & arrhythmia may occur.
. ECG -> BROAD FLAT T-waves, U waves & pre-mature ventricular beats.
. Af, Torsades de points & VF may occur.
. H/O of K wasting diuretic is common (Hydrochlorothiazide).
. Other causes -> Diarrhea - vomiting - anorexia - hyperaldosterinism.
. Symptoms resolve with K supplementation.
. TAKE CARE: Differentiate bet IIH & NPH:
_________________________________________

. NORMAL PRESSURE HYDROCEPHALUS = Triad of Urine incontinence + Abnormal gait +


Dementia.
_________________________________________________________________________________________
. BENIGN IDIOPATHIC INTACRANIAL HYPERTENSION = PSEUDOTUMOR CEREBRI:
___________________________________________________________________
. Over-weight female in the child bearing period.
. H/O of OCPs intake or hypervitaminosis A.
. Headache - transient loss of vision - pulastaile tinnitus - diplopia.
. Dx: MRI & LP (CSF opening pressure > 250 mmHg with NORMAL analysis).
=========================
. KEY-WORDS to RE-MEMBER:
=========================
. OLIGO-CLONAL BANDS -> MS.
. CYTO-ALBUMINOUS DISSOCIATION -> GB$.
. HEMORRHAGE -> HYPER-dense areas on CT. (WHITE).
. INFARCTIONS -> HYPO-dense areas on CT. (BLACK).
.
.
.
.
.
.
.
.
.

GAITS
FESTINATING = HYPOKINETIC = SHUFFLING -> PARKINSONISM.
HIGH STEPPAGE -------------------------> TABES DORSALIS or L5 Radiculopathy.
SEMI-CIRCLE ---------------------------> STROKE HEMIPLEGIA.
WADDLING ------------------------------> MUSCULAR DYSTROPHY.
WIDE BASED & SHUFFLING ----------------> NORMAL PRSSURE HYDROCEPHALUS.
IPSILATERAL ATAXIA --------------------> CEREBELLAR ATAXIA.
SPASTIC -------------------------------> UMNL UPPER MOTOR NEURON LESION:
STAGGERING ----------------------------> VESTIBULAR ATAXIA.

.
.
.
.

TREMORS
RESTING TREMORS (PARKINSON)-> At rest,imp. e' activity,High frequency tremors 5-7 Hz.
ESSENTIAL TREMORS-> Worst at the end of the goal directed activity (reaching a pen).
CEREBELLAR TREMORS -> Intension tremors - low fequency 3-4 Hz - Nystagmus & ataxia.

. TRIGEMINAL NEURALGIA: LIGHTENING PAIN on the face - electric shock - Tx:


CARBAMAZEPINE.
. GUILLAIN BARRE'$: Ascending paralysis - Pre.by infection - CYTOALBUMINOUS
DISSOCIATION.
. TICK BORNE PARALYSIS: Ascending paralysi - NO fever - Normal CSF - Tx: Tick removal.
. MYASTHENIA GRAVIS: Ptosis,Diplopia RESOLUTION OF MUSCULAR WEAKNESS WITH REST.
. LAMBERT EATON $YNDROME: LUNG CANCER H/O, Auto-Abs against voltage gated Ca
channels.
. N.B. MYASTHENIA GRAVIS: AutoAbs against (post)synaptic recs - INTACT DTRs.
. N.B. LAMBERT-EATON $: AutoAbs against (pre)synaptic receptors - LOST DTRs.
. DERMATOMYOSITIS: Sym. prox. ms weakness - SKIN (Gottron's papules & Heliotrope rash).

. STEROID INDUCED MYOPATHY: Prox. ms weakness (LL before UL) - No pain.


. SUB-ARACHNOID HEMORRHAGE: Rupture of "Berry" aneyrysm - WORST HEADACHE EVER.
. NEURO-FIBROMATOSIS "2": S.C. neurofibromas + Cafe' au lait spots + acoustic neuromas.
. PRONATOR DRIFT = UMNL.
. ESSENTIAL TREMOR: Tx: BB "Propranolol".
.
.
.
.
.
.
.

MULTIPLE
MULTIPLE
MULTIPLE
MULTIPLE
MULTIPLE
MULTIPLE
MULTIPLE

SCLEROSIS
SCLEROSIS
SCLEROSIS
SCLEROSIS
SCLEROSIS
SCLEROSIS
SCLEROSIS

=
=
=
=
=
=
=

Optic neuritis (painful loss of vision) & diplopia.


INTER-NUCLEAR OPHTHALMOPLEGIA (INO).
Dx: BRAIN MRI with & without GADOLINIUM.
CSF analysis: OLIGOCLONAL IgG bands - Normal pressure.
Tx of acute exacerbation -----> HIGH DOSE IV GLUCOCORTICOIDS.
prevent future attacks -> B-interferon or Glatiramer acetate.
YOUNG FEMALE with BILATERAL TRIGEMINAL NEURALGIA.

. AMYOTROPHIC LATERAL SCLEROSIS: UPPER + LOWER motor neuron lesions.


. CT$: MEDIAN N. - At WRIST - Thenar eminence atrophy - Paresthesia 1st 3.5 fingers
. ULNAR N. ENT. $: paresthesia of 4th & 5th fingers - ent.at medial epicondylar groove.
. VESTIBULO-TOXICITY by AMINOGLYCOSIDES: Gentamycin-Amikacin - Vertigo & gait
imbalance.
. TORTICOLLIS: Example of FOCAL DYSTONIA.
. UN-PROVOKED FIRST SEIZURE -> HEAD CT WITHOUT CONTRAST is the 1st initial step done.
. LIMB ISCHEMIA: 5 Ps (Pain - Pallor - Paresthesia - Pulselessness & Paralysis).
. METOCLOPRAMIDE: Side effect Dystonia - Manifested by stiff painful neck.
. HEMI-NEGLECT $YNDROME: RIGHT PARIETAL LOBE CORTEX lesion - (NON)-dominant hemisphere.
. EXERTIONAL HEAT STROKE: Tx -> EVAPORATION COOLING (NOT immersion in cold water
xxx).
. CAVERNOUS SINUS THROMBOSIS: Dx: MRI - Tx: Broad spectrum Antibiotics.
. RESTLESS LEG $YNDROME: "Crawling" sensation - Tx : Dopaminergic agonists e.g. L-dopa.
. WERNICKE's ENCEPHALOPATHY: ecephalopathy,oculomotor dysf. & gait ataxia.(--Vit B1).
. DECUBITUS ULCER: Preveted by repositioning of the pt every 2-4 hours.
. LEVO-DOPA / CARBI-DOPA Side effects: Most common side effect is HALLUCINATIONS.
. TRI-HEXY-PHENIDYL:
Red as beet, dry as bone, hot as hare, blind as bat, mad as hatter & full as a flask.
. DEMENTIAS

. FRONTO-TEMPORAL : Personality changes (euphoria - disinhibition - apathy).


. LEWY BODIES DEMENTIA: Bizarre visual hallucinations.
. ALZHEIMER's DISEASE: Progressive dementia - Impaired judgement & personality changes.
. HUNTINGTON's DISEASE: Triad of mood disturbances + Choreiform movements + Dementia.
. CREUTZFELDT - JAKOB DISEASE: EEG -> SHARP TRI-PHASIC SYNCHRONOUS DISCHARGES.
. NORMAL PRESSURE HYDROCEPHALUS: Triad of Urine incontinence + Abnormal gait +
Dementia.
. PSEUDO-DEMENTIA: Tx -> SSRIs.
. NORMAL AGING: Absence of functional impairments.
. BRAIN DEATH: DTRs may be STILL PRESENT - MUST BE CONFIRMED BY TWO PHYSICIANS.
. SHY DRAGER $YNDROME: MULTIPLE SYSTEM ATROPHY - PARKINSON pt. + bladder loss of
control.
. Chronic alcohol abuse -> Cerebellar damage.
. BELL's PALSY: Facial 7th cranial nerve peripheral neuropathy.
. AMAUROSIS FUGAX: BLACK CURTAIN FALLING - impending stroke - NECK DUPLEX
ULTRA$OUND.
. SUB-DURAL HEMATOMA: Tearing the BRIDGING VEINS .. CT -> WHITE CRESCENT.
. EPI-DURAL HEMATOMA: Injury to the MIDDLE MENINGEAL ARTERY .. CT -> BICONVEX
HEMATOMA.
. SYRINGOMYELIA: CAPE like - Loss of pain & temperature with intact position & vibration.
. SPINAL CORD COMPRESSION: It is a medical emergency - Dx: MRI Spine.
. ALZHEIMER's DISEASE: CT -> Diffuse cortical & subcortical atrophy.
. GLIOBLASTOMA MULTIFORM: BUTTERFLY appearance with central necrosis on VT or MRI.
. CRANIOPHARYNGIOMA = Hypopituitarism signs + Headaches + Bitemporal blindness.
. CAUDA EQUINA $YNDROME: LBP - bladder dys. - Saddle anesthesia - Dx: Emergent MRI.
.
.
.
.
.

HIV associated lesions on MRI:


PRIMARY CNS LYMPHOMA: Solitary - {WEAKLY} ring enhancing - Ass. EBV DNA.
TOXOPLASMOSIS: Multiple - {Ring - enhancing} spherical - TMP-SMX is preventive.
PROGRESSIVE MULTIFOCAL LEUKOENCEPHALOPATHY: {Non - enhancing} - No mass effects.
ABSCESS: Solitary - {Ring enhancing} - caused by ANEROBES

. CAROTID ARTERY STENOSIS: NECK DUPLEX U/$.- CAROTID END ARTERECTOMY if stenosis
70-99 %.
. ACUTE GLAUCOMA: Palpation -> very hard eye - NON REACTIVE MID DILATED PUPIL.
. HEADACHES
. MIGRAINE HEADACHE: female Unilateral Pulsating, AURA of neuro syms preceiding headache.
. CLUSTER HEADACHE: unilateral 5tearing retro-orbital pain .. Tx -> 100 % OXYGEN.
. PSEUDOTUMOR CEREBRI: Obese female- Vit A - OCPs - Most common complication is
BLINDNESS

. SUB-ARACHNOID HEMORRHAGE: WORST HEADACHE EVER !!


. BROWN SEQUARD $YNDROME: contra-lateral loss of pain & temperature sensation,
. BROWN SEQUARD $YNDROME: beginning two levels below the level of the lesion.
. L5 RADICULOPATHY -> Foot drop -> Compensated by HIGH STEPPAGE GAIT:
. HERPES ENCEPHALITIS: CSF analysis -> LYMPHOCYTIC PLEOCYTOSIS - Tx : IV ACYCLOVIR.
. PARKINSON DISEASE = TREMORS + RIGIDITY + BRADYKINESIA.
. DELIRIUM: Tx -> Typical & atypical anti-psychotics (HALOPERIDOL).
. ABSENCE seizures -> ETHOSUXIMIDE.
Dr. Wael Tawfic Mohamed
_________________________

OPHTHALMOLOGY TiKi TaKa


------------------------. PAPILLAEDEMA:
--------------. Transient loss of vision lasting few seconds with change in head psition.
. caused by ++ ICT manifested by morning headaches or change in headache intensity with
head position.
. Optic neuritis:
----------------. Associated with multiple sclerosis.
. Unilateral eye pain & visual loss.
. Exrenal hordeolum = Syte :
---------------------------. Staphylococclal abscess of the eyelid.
. Tx: Warm compresses.
. If not resolved within 48 hours ---> incision & drainage.
. Allergic conjunctivitis:
-------------------------. Intense itching - hyperemia - tearing - conjunctival oedema & eye lid edema.
. CATARACT:
----------. Progressive thickening of the lens.
. Blurred vision & glare.
. Tx: Lens extraction.
. Acute angle closure glaucoma:
------------------------------. Old pts 55 - 70 ys.
. Acute severe eye pain.
. Blurred vision , nausea & vomiting.
. Fixed dilated pupil non reactive to light.
. Open angle closure glaucoma:
-----------------------------. Loss of peripheral vision.
. preserved central tunnel vision.
. More common in AFRICAN AMERICANs.
. Macular degeneration:
----------------------. Loss of central vision.
. More common in OLD AGE.
. Post-operative ENDOPHTHALMITIS:
--------------------------------. H/O of recent ocular operation.
. symptoms manifest within 6 weeks of surgery.
. pain & -- visual acuity.

. swollen eyelids, corneal edema & infection.


. Herpes Zoster Ophthalmicus:
----------------------------. Dendriform ulcers.
. Vesicular rash in the trigeminal distribution.
. CMV Retinitis:
---------------. HIV pt with CD4 < 50.
. Fundoscopy: Yellow - white patches of retinal opacification & hemorrhages are
. Optic neuritis:
----------------. Central scotoma.
. Afferent pupillary defect.
. Change in colour perception.
. -- visual acuity.
. Ass. e' Multiple sclerosis.
. Vitreous hemorrhage :
----------------------. Sudden loss of vision.
. Floaters in the visual field.
. Diabetic retinopathy is the most common cause.
. Fundus is hard to be visualized with obscured details.
. HSV ---> dendriTIC ulcers.
. HZV ---> dendriFORM ulcers.
. Amaurosis Fugax:
-----------------. Curtain falling down.
. Whitened edematous retina following the distribution of the retinal arterioles.
. caused by retinal emboli from the ipsi-lateral carotid artery.
. CRAO = CENTRAL RETINAL ARTERY OCCLUSION:
----------------------------------------. Sudden unilateral painless loss of vision.
. Pallor of the optic disc + CHERRY RED FOVEA + Boxcar segmentation.
. CRVO = CENTRAL RETINAL VEIN OCCLUSION:
--------------------------------------. Sudden painless unilateral loss of vision.
. THUNDER & BLOOD appearance.
. Vitreous hemorrhage:
--------------------. Black curtain coming down infront of eyes = Retinal detachment.
. Photopsia (Flashes of light).
. Floaters (Spots in the visual field).
. Central retinal vein occlusion:
--------------------------------. Sudden monocular loss of vision.

diagnostic.

.
.
.
.
.

Blood & thunder appearance.


Optic disc swelling.
Retinal hemorrhage.
Dilated veins.
Cotton wool spots.

. Central Retinal Artery Occlusion:


----------------------------------. Sudden painless loss of vision.
. Pallor of the optic disc.
. Cherry red fovea.
. Boxcar segmentation of blood in the retinal veins.
. Diabetic retinopathy:
----------------------. H/O of D.M.
. -- visual acuity in both eyes.
. Micro-aneurysms.
. Dot & blot hemorrhages.
. Hard exudates.
. Macular edema.
. Tx ARGON laser photocoagulation to prevent complications.
. Presbyopia:
-----------. Difficulty in near vision.
. Prespyobia is due to -- in lens elasticity not due to macular degeneration !!
. A history of middle aged individual who has to hold books at an arms length to read is
classic.
. On Tx of Acute glaucome , ATROPINE is contraindicated. . 1st line Tx: I.V. MANNITOL.
. Multiple Sclerosis :
---------------------. FEMALE with multiple neurological presentations.
. Associated optic neuritis can lead to blurring of vision & retro-bulbar pain.
. Sub-conjunctival hemorrhage ---> No ttt .. Just Observation.
. CRAO Tx: ocular massage + High flow Oxygen.
. Sympathetic Ophthalmia:
------------------------. Damage of one eye (sympathetic eye) after a penetrating injury to the other eye.
. Due to UN-COVERING OF HIDDEN ANTIGENS !
. In HIV pts:
------------.. HSV & HZV :
-------------... Pain-ful.
... Ass. with karatitis & conjunctivitis.
... Fundus: Peripheral pain lesions & central necrosis.
.. CMV :

-------... Pain-less.
... Not ass. e'keratitis or. conjunctivitis.
... Fundus: Hemorrhages & fluffy or granular lesions around retinal vessels.
. Neuro-fibramatosis type 1 = Axillary freckling + Cafe' au lait patches + Optic glioma.
--------------------------. Diabetic pt with blurry vision:
--------------------------------. Type 2 D.M. with HYPEROSMOLAR HYPERKETOTIC state without ketoacidosis.
. Glucose in urine with NO KETONES.
. NKHS = Non Ketotic Hyperosmolar $
so, the cause of blurring of vision is HYPEROSMOLARITY !
. Macular degeneration:
----------------------. Grid test: distortion of the straight lines that appear wavy !
------------------------Dr. Wael Tawfic Mohamed
-------------------------

POISONING TiKi TaKa


_____________________

. OPIOID INTOXICATION:
______________________
. -- RESPIRATORY RATE is the most reliable & predictive sign.
. -- Bowel sounds.
. -- BP.
. -- Temp.
. H/O of heroin injection (Needle marks on extremities by P/E).
. Tx: NALOXONE.
. The goal of ttt is ++ RR from 6 to 12/min with improving Oxygen saturation.
. N.B. PUPIL EXAMINATION is NOT RELIABLE !!
. Opioid intoxication doesn't always present with miosis.
. Co-ingestions can lead to normal pupillary size or even mydriasis !!!
. OPIOID WITHDRAWAL:
____________________
. Symptoms develop within 6-12 hs after the last dose of short acting opioid.
. H/O of heroin injection (Needle marks on extremities by P/E).
. Nausea - vomiting - Abdominal pain - Diarrhea - Restlessness - Arthralgia & myalgia.
. Rhinorrhea - Lacrimation.
. Mydriasis - Piloerection & hyperactive bowel sounds.
. Tx -> METHADONE.
. METHANOL INTOXICATION:
________________________
. "ALCOHOL's SUBSTITUTE".
. H/O of homeless man.
. 1st 24 hs -> Headache, nausea, vomiting & epigastric pain.
. Later -----> Vision loss & coma.
. Optic disc hyperemia.
. ++ ANION GAP METABOLIC ACIDOSIS { (Na) - (Cl + HCO3) }. (Normal AG 8-12).
. ++ OSMOLAR GAP.
. Very low HCO3.
. ETHYLENE GLYCOL INTOXICATION:
_______________________________
. "ALCOHOL's SUBSTITUTE".
. H/O of homeless man.
. 1st 24 hs -> Headache, nausea, vomiting & epigastric pain.
. Later -> Flank pain, hematuria, oliguria, Acute renal failure.
. Glycolic acid (Metabolite) injuries the renal tubules.
. Oxalic acid binds calcium & deposits in the kidneys.
. Tx: FOMEPIZOLE or ETHANOL to prevent ethylene glycol to its harmful metabolites.
. N.B. METHANOL & ETHYLENE GLYCOL MAY HAVE SIMILAR PRESENTATIONS ! BUT !!
_________________________________________________________________________
. METHANOL DAMAGES THE EYES.
. ETHYLENE GLYCOL DAMAGES THE KIDNEYS.

. BETA-BLOCKERS BB INTOXICATION:
________________________________
. -- HR.
. -- BP.
. AV BLOCK.
. BRONCHOSPASM -> WHEEZES (MOST SPECIFIC).
. Cardiogenic shock may occur (Cold & clammy extremities).
. Neurological effects (Delirium & seizures).
. 1st line TTT -> ATROPINE + IV FLUIDS.
. FAILED -------> GLUCAGON ++ c-AMP -> ++ Ca -> ++ cardiac contractility.
. CANNABIS = MARIJUANA INTOXICATION:
____________________________________
. +++++++ APPETITE.
. DRY MOUTH.
. CONJUNCTIVAL INJECTION.
. ++ HR.
. ++ BP.
. ++ RR.
. -- concentration & short time memory.
. -- reaction time & impairs attention.
. ++ the risk of vehicle accidents.
. ALCOHOL INTOXICATION:
_______________________
. Slurred speech.
. Unsteady gait.
. Incoordination.
. Disinhibited behavior.
. Memory impairment.
. Nystagmus.
. ALCOHOL WITHDRAWAL:
_____________________
. Due to reflex hyperactivity of certain parts of the brain.
. Anxiety, insomnia, tremors & diaphoresis in the 1st 6-24 hs after alcohol cessation.
. Hallucinations & withdrawal seizures may occur.
. DELIRIUM TREMENS may occur in 5% of pts after 48 - 72 hs.
. Hypertension, agitation, tachycardia, hallucinations & fever.
. Tx -> CNS depressant -> Benzodiazepenes -> Chlordiazepoxide.
. BENZODIAZEPINE OVERDOSE:
__________________________
. Slurred speech.
. Unsteady gait.
. Incoordination.
. Respiratory compromise, stupor & coma.
. Distinguished from opioid overdose by lack of severe respiratory depression & miosis.
. Distinguished from alcohol & phenytoin toxicity by lack of nystagmus.
. BARBITURATE (SEDATIVES) INTOXICATION:
_______________________________________
. Slurred speech.
. Unsteady gait.
. Incoordination.

. COCAINE INTOXICATION:
_______________________
. SYMPATHETIC STIMULATION (++HR, ++BP, MYSRIASIS).
. Euphoria.
. Sense of self confidence.
. ++ arousal.
. improved performance.
. CAUSTICS "LYE" (SODIUM HYDROXIDE) INGESTION:
______________________________________________
. A strong alkaline solution.
. Mostly due to suicidal attempt.
. SEVERE ESOPHAGEAL DAMAGE due to LIQUEFACTIVE NECROSIS.
. May lead to perforation & mediastinitis.
. Retrosternal or epigastric pain - hypersalivation - Odynophagia & dysphagia.
. Tx -> HOSPITALIZATION + IV HYDRATION.
. ENDOSCOPY IS MANDATORY to determine the extent of esophageal damage !
. If perforation is suspected -> A gastrograffin study sh'd be performed.
. NEVER to neutralize the alkali with ana acid !!
. NEVER to induce vomiting !!
. Charcoal isn't effective !!
. N.B. NO ALTERATION IN CONSCIOUSNESS.
. CARDIOLOGICAL PROBLEM RELATED TO TOXICOLOGY !
_______________________________________________
. TORSADES DE POINTS -> Tx: MgSO4 !
____________________________________
. Polymorphic ventricular tachycardia.
. Occurs in the setting of a prolonged QT interval.
. Seen in pts with familial long QT $yndrome.
. Malnourished pts predisposed to hypomagnesemia (Alcoholics).
. Pts on TCAs (Tricyclic anti-depressants).
. Pts on anti-arrhythmics (Amiodarone - Sotalol).
. Anti-infective agents (Moxifloxacin - Fluconazole).
. Tx -> Cessation of the offending drug & MgSO4.
. PHENCYCLIDINE INTOXICATION:
_____________________________
. HALLUCINOGENIC STREET DRUG.
. VERTICAL NYSTAGMUS.
. VIOLENT BEHAVIOR.
. Agitation - confusion - Pupillary dilatation - tachycardia.
. Severe hypertension & hyperthermia may occur.
. Tx -> Benzodiazepines.
. CHEMICALS IN THE EYE:
_______________________
. FLUSH THE EYE WITH WATER (EYE UNDER A FAUCET OF RUNNING WATER FOR AT LEAST 15
mins) !
. DIPHENHYDRAMINE POISONING:
____________________________
. Anti-histaminic used in ttt of allergic rhinitis - insect bites & motion sickness.
. Mixture of anti-cholinergic symptoms.

. Drowsiness & confusion.


. Dry mouth - dilated pupils - blurred vision - Reduced bowel sounds & urine retention.
. Tx -> PHYSOSTIGMINE (Cholinesterase inhibitor).
. SALICYLATE INTOXICATION:
__________________________
. TINNITUS.
. Nausea & vomiting.
. Fever.
. Altered mental status & acid-base abnormalities.
. Tx -> ALKALINIZATION OF URINE with SODIUM BICARBONATE (Na HCO3).
. CARBON MONOXIDE (CO) POISONING:
_________________________________
. HISTORY is the most ipmortant clue to diagnosis.
. SMOKE INHALATION - BARBEQUE PARTY !!
. CO is a colorless - odorless gas.
. Emitted by automobiles, furnaces & charcoal grills.
. When inhaled, it prevents the body from utilizing Oxygen.
. Headache - Nausea - Vomiting - Abd. discomfort - confusion - coma.
. PINKISH RED SKIN HUE.
. Dx -> Obtain CARBOXYHEMOGLOBIN levels.
. Tx -> HYBERBARIC OXYGEN.
. ACETAMINOPHEN POISONING:
__________________________
. Rumack - Matthew Nomogram is a curve used to assess its hepatotoxic effects.
. It also provides the need for N-acetylcysteine as an antidote.
. The 1st data point on the curve is at 4 hours !
. The decision of whether or not to take the antidote can be made after 4 hours.
. Studies proved zat their is no correlation bet. z amount ingested & z toxicity level!
. So .. If a pt. came to u with H/O of ingestion of 14 pills 2 hours ago ,,
. You should wait 2 hours then obtain the acetaminophen level.
. ORGANO-PHOSPHATE POISONING:
_____________________________
. Organophosphates antagonizes acetylcholinesterase -> Cholinergic excess.
. -- HR - miosis - muscle fasciculations.
. Bronchorrhea - salivation - lacrimation - diarrhea - urination.
. Tx -> ATROPINE (Compete with acetylcholine at the muscarinic receptors).
. IMMEDIATE REMOVAL OF THE PT's CLOTHING (SOAKED with secretions),
. thus .. preventing continued absorption of organophosphates through the skin.
. TRI-CYCLIC ANTI-DEPRESSANTS (TCA) INTOXICATION:
_________________________________________________
. Hyperthermia - seizures - Hypotension.
. Anticholinergic effects (mydriasis - flushed dry skin - intestinal ileus).
. TCAs -> -- conductivity -> QRS prolongation -> Ventricular arrhythmia.
. THE BEST INDICATOR OF THE EXTENT OF THE OVERDOSE is the QRS COMPLEX DURATION.
. Tx -> NaHCO3 -> Narrows the QRS complex & -- the incidence of VT.
. NaHCO3 Mechanism -> Sodium load will alleviate the depressant action on Na channels.
. NEUROLEPTIC MALIGNANT $YNDROME:
_________________________________
. DRUG INDUCED IDIOSYNCRATIC REACTION.

.
.
.
.

Sudden onset of confusion, fever, muscle rigidity & diaphoresis.


Mostly caused by dopaminergic antagonists (HALOPERIDOL) for hallucinations ttt.
Rigidity & hyperthermia may lead to ms necrosis & ++ CPK > 50000.
Tx -> DANTROLENE (Muscle relaxant).

. FLUPHENAZINE OVERDOSE -> HYPOTHERMIA:


_______________________________________
. High potency "typical" anti-psychotic drug.
. It disrupts thermoregulation & body's shivering mechanism.
. Pts sh'd be advised to avoid prolonged exposure to extreme temperatures.

Dr. Wael Tawfic Mohamed


_________________________

PREVENTIVE MEDICINE TiKi TaKa


------------------------------. All women > 65 ys should be screened by DEXA scan to exclude Osteoporosis
. Needle stick injury with HBV:
------------------------------. Previusly immunized with HBsAb +ve ----> REASSURANCE.
. Unknown H/O or not immunized ----> HBIG + HBV.
. Adult 23 polyvalent pnemococcal vaccine:
-----------------------------------------. contains capsular polysaccharides.
. Mechanism : Humoral immunity.
. i.e. T cell IN-dependent, B cell response.
. i.e. B-cell NOT T-cell response !
. BUPROPION "not BUSPIRONE" is recommended to aid in smoking cessation.
. PROSTATE CANCER SCREENING:
---------------------------. There is no specific test for PROSTATE cancer screening.
. PSA measurment is controversial !
. Discuss checking the PSA with the patient !!
. Avoid decubitus ulcers by pt repositioning every 2 hours.
. LDL LEVEL CONTROL:
-------------------. A pt with known coronary heart disease "CHD" or a risk equivalent e.g. DM,
. should be ttt with life style modification + STATINS "NOT NIACIN",
. to reach a goal LDL < 100 mg/dl.
. CHD risk equivalents = DM. - CAD - PAD - AAA.
. Risk factors:
--------------. Cigarette smoking.
. BP < 140/90 mmHg.
. Premature CHD in family members (<55ys in men & < 65ys in women).
. Low HDL <40 mg/dl.
. Current age >45ys in men & >55ys in women.
. N.B. HDL > 60 mg/dl is a NEGTIVE risk factor !
. The best management for high LDL is Life style modification + SATATINS,
. to reach LDL < 100 mg/dl.
. Women beyond the age of 65 who have had 3 consecutive NEGATIVE PAP smears no longer
require regular screening.
. MAMMOGRAMS should be performed every 2 ys starting at the age of 50.
. Although LIVE vaccines are contraindicated in HIV pts, MMR vaccine is an exception.
------------------------------------------------------------------------------------

* It can be given to HIV pts with CD4 cells > 200 with no evidence or H/O of AIDS defining
illness.
* Live vaccines e.g. BCG, Oral polio, Meningococcal vaccines are CONTRAINDICATED in HIV pts.
* TETANUS & DIPHTHERIA TOXOID booster doses every 10 ys can be given safely.
. INFLUENZA vaccine is recommended ANNUALLY !
-------------------------------------------. Pneumococcal vaccine:
----------------------. Pt < 65 ys --> Once followed by a booster every 5 years.
. Pt > 65 ys --> Once.
. Travelers to developing countries e.g EGYPT should be vaccinated against HEPATITIS A.
. CHLAMYDIA TRACHOMATIS infection:
---------------------------------. should be screened in all sexually active women aged 24 ys & younger.
. If a pt has DM with no vaccines except flu vaccine since childhood series,
--------------------------------------------------------------------------. he should receive TdaP "Tetanus + Diphtheria + Pertussis" ,
. annual IM influenza & pneumococcal vaccines.
. CHRONIC LIVER disease VACCINATIONS:
------------------------------------. HEPATITIS A & B.
. INFLUENZA. "ANNUALLY".
. PNEUMOCOCCAL. "Immunocompromized".
. TdaP "Usual Boosters".
. All adults:
-----------. should have TETANUS & DIPHTHERIA boosters every 10 ys,
. with a one-time tetanus, diphtheria & pertussis booster.
. require INFLUENZA vaccine ANNUALLY.
. Cervical cancer screening:
--------------------------. should be started at age 21 & repeated every 2 - 3 ys.
. Women aged 30-65 ys:
--------------------.. Either PAP smear every 3 ys.
.. Or COMBINATION of PAP smear & HPV testing every 5 ys.
. Vaccinations in pts with HIV:
-----------------------------. Pts with HIV shouldn't receive live vaccines,
. such as BCG,anthrax,oral typhoid,intranasal influenza & oral polio.
. The exceptions r the MMR & VARICELLA vaccines,
. which can be used in pts without evidence of immunity if their CD4 counts > 200.
. Screening for BLADDER CANCER:

-----------------------------. is NOT RECOMMENDED even in pts with high risk factors !!!!!
. CANCER COLON SCREENING:
-----------------------. COLONOSCOPY is the best tool.
. All pts aged 50 ys & older.
. 1st degree relative cancer colon ---> 10 ys earlier screening i.e. at 40 ys.
. NO screening tests for PANCREATIC cancers !
-------------------------------------------. CEA -----------> x.
. CA 19-9 -------> x.
. CT or U$ ------> x.
. Abdominal Aortic Aneurysm screening:
------------------------------------. All male smokers >65 ys old should be screened with ABDOMINAL ULTRASOUND to exclude
AAA
Dr. Wael Tawfic Mohamed
-------------------------

PSYCHIATRY & ETHICS TiKi TaKa


_______________________________
. PSYCHOTIC DISORDERS:
_______________________
_______________________
. Combination of positive &/or negative symptoms.
. POSITIVE SYMPTOMS (Associated e' DOPAMINE receptors):
________________________________________________________
-> Delusions (Mostly bizarre).
-> Disorganized speech/behavior.
-> Hallucinations.
-> Agitation.
-> Impairment of baseline functions.
. NEGATIVE SYMPTOMS (Associated e' MUSCARINIC receptors):
__________________________________________________________
-> Flattened affect.
-> Social withdrawal.
-> Anhedonia.
-> Apathy.
-> Poverty of thought.
. N.B. Atypical anti-psychotics are the most effective ttt for -ve symptoms.
. N.B. The key differentiating feature is the DURATION of symptoms:
____________________________________________________________________
. SCHIZO-PHRENIA: > 6 MONTHS.
. SCHIZO-PHRENI-FORM DISORDER: < 6 MONTHS BUT > 1 MONTH.
. BRIEF PSYCOTIC DISORDER: < 1 MONTH (Look for a stressful life event precipitating it).
. SCIZO-AFFECTIVE DISORDER:
-> Distinguished from schizophrenia by the presence of mood symptoms
-> (mania or depression) during the course of the disease.
. N.B.
. Disorganized speech & CIRCUMSTANTIALITY is common in pts with schizophrenia;
. They deviate from the original subject but eventually returns to it !
. N.B.
. Schizophrenic pts have ++ ventricular size on CT !!
. N.B.
. When there is H/O of syms for many years e' NO IMPROVEMENT OF BASELINE FUNCTIONING,
. think of:
* DELUSIONAL DISORDER:

-> Key is that delusions are NON-bizarre.


-> Delusions may occur normally in daily life.
-> Delusions are false beliefs in high functioning person.
* PERSONALITY DISORDERS (Especially SCHIZOTYPAL PERSONALITY DISORDER):
-> Prsents with peculiar thinking & social isolation.
-> No psychosis.
. The previous two types of pts (DELUSIONS & PERSONALTY DISORDERS):
-> Don't respond to anti-psychotics.
-> PSYCHOTHERAPY is the preferred therapy.
.
.
.
.

N.B.
A DELUSION is a fixed, false belief not consistent with cultural norms.
Individuals with GRANDIOSE DELUSIONS typically believe they have
special powers extraordinary accomplishments or a special relationship e' god.

. RULE OUT other forms of psychosis that are NOT schizophrenia:


-> Get a DRUG SCREEN to rule our SUBSTANCE ABUSE.
-> Look for signs & symptoms of SEIZURE !
-> TEMPORAL LOBE EPILEPSY can present with hallucinations (Auditory & Olfactory).
.
.
.
.

N.B.
Watch out for SUICIDAL IDEATION in schizophrenia pts & schizophreniform pts.
50 % of them attempt suicide & 10 % are successful.
1st stepin management is always to HOSPITALIZE if there is risk of suicide.

.
.
.
.

The 1st step in management of any acute psychiatric condition is:


to determine if the pt needs hospitalization !
Hospitalize if the pt at risk of harm to self or to others.
Hospitalize against the pt's will if the pt has suicidal or homicidal ideation.

. The greatest risk factor for progression to schizophrenia is SCHIZOFRENIFORM DISORDER.


. Prognosis:
-> Females have a better prognosis & respond better to ttt than males.
-> Pts e' paranoid schizophrenia are more responsive to ttt.
. The prognosis is poor if there is:
-> Early age of onset.
-> Negative symptoms.
-> Poor premorbid functioning.
-> Family H/O of schizophrenia.
-> Disorganized or deficit subtype.
. Treatment:
-> If there is bizarre or paranoid syms -> HOSPITALIZE the pt.
-> Give BENZODIAZEPINES for agitation & start ANTI-PSYCHOTICS.
-> Anti-psychotic medications are given for 6 months.
-> They are most effective to prevent further episodes.
-> Long term anti-psychotics are ONLy given if there is H/O of REPEATED episodes.
-> Initiate log-term psychotherapy.
. ANTI-PSYCHOTICS:
___________________

___________________
. Have an IMMEDIATE QUIETING EFFECT in acute atacks.
. Delay relapse.
. Used for sedation when benzodiazepines are cont'd or as an adjunct during anesthesia.
. Used for ttt of movement disorders (Huntington's disease & Tourette $),
. to suppress tics & vocalization
. N.B. In ttt of Tourette $ -> We use TYPICAL antipsychotics (Haloperidol & PIMOZIDE).
. N.B. Antipsychotics are chosen based on side effect profile, NOT efficacy !
. A . CONVENTIONAL (TYPICAL) ANTIPSYCHOTICS:
_____________________________________________
(1) HIGH POTENCY {FLUPHENAZINE DECANOATE - HALOPERIDOL}:
__________________________________________________________
-> Less sedating - Fewer anticholinergic effects - Less hypotension.
-> Useful as depot injections (Haloperidol decanoate) for non-compliant pts.
-> Give IM route for acute psychosis when pt is unable or unwilling to take PO.
-> GREATEST ASSOCIATION WITH EXTRAPYRAMIDAL SYSTEMS (EPS).
-> ++ PROLACTIN.
(2) LOW POTENCY {THIORIDAZINE - CHLORPROMAZINE):
_________________________________________________
-> Less likely to cause EPS.
-> Greater anticholinergic effects - More sedation - More postural hypotension.
. B . ATYPICAL ANTIPSYCHOTICS (RISPERIDONE - OLANZAPINE - QUETIAPINE - CLOZAPINE):
___________________________________________________________________________________
-> OLANZAPINE is the best.
-> Drug of choice in initial therapy.
-> Greater effect on negative symptoms.
-> Little or no risk of EPS.
. N.B. SIDE EFFECTS of ATYPICAL ANTIPSYCHOTICS:
________________________________________________
. Clozapine -> Agranuloctosis (Order CBC before initiatin ttt & one week after).
. Quetiapine -> Cataracts.
. Olanzapine -> Weight gain - Hyperglycemia - Dyslipidemia.
. Respiredone -> Hyperprolactinemia.
. N.B. Anti-psychotic medications -> Dopamine receptor blockage -> Hyperprolactinemia.
. ++ PRL > 200 ng/ml -> Gynecomastia - Galactorrhea - Menstrual dysfunction & -- libido.
. N.B. Antipsychotics -- dopamine activity in the TUBERO-INFUNDIBULAR pathway.
. NOOOOOOOOTTTTTTTTT the mesolimbic pathway xxxxxxxx !
. LOW potency antipsychotics have the highest risk of causing ORTHOSTATIC HYPOTENSION
. Due to (ALPHA BLOCKAGE).
. LOW potency antipsychotics have the highest risk of causing ANTICHOLINERGIC SYMPTOMS
. Acute urine retention - Dry mouth - Blurry vision - Delirium.
. Thioridazine is associated with prolonged QT & arrhythmias.
. Order an EKG if there is chest pain - SOB - plapitations.

. Thioridazine is associated with abnormal retinal pigmentation.


. Routine eye exam is important.
. NONcompliance in males -> IMPOTENCE & INHIBITION OF EJACULATION.
. NONcompliance in females -> WEIGHT GAIN - HYPERPROLACTINEMIA - GALACTORRHEA
&AMENORRHEA
. Olanzapine - Quetiapine are 1st choice medications when INSOMNIA is a problem !
. RISPERIDONE is the 1st choice medication when SEDATION is a problem !
__________________________________________________________________________________________
. MOVEMENT DISORDERS:
______________________
______________________
. Extra-pyramidal symptoms (EPS) are the most common reason for failure to comply e' ttt.
. Acute dystonia - Bradykinesia - Tardive dyskinesia - Neuroleptic malignant $.
. Most common culprit atypical antipsychotic is RISPERIDONE -> Switch to CLOZAPINE.
{1} ACUTE DYSTONIA:
____________________
. Occurs in the 1st week.
. Muscle spasms (Torticollis) & difficulty swallowing.
. Young men are at higher risk.
. Tx -> Reduce the dose.
. Tx -> ANTICHOLINERGICS (Benztropine - Diphenhydramine - Trihexiphenidyl).
{2} BRADYKINESIA (PARKINSONISM):
_________________________________
. Within weeks.
. Bradykinesia - tremors - rigidity & other signs of parkinsonism.
. Elderly are at higher risk.
. Tx -> Reduce the dose.
. Tx -> ANTICHOLINERGICS (Benztropine - Diphenhydramine - Trihexiphenidyl).
. Tx -> Amantadine (Dopamine agonist).
{3} AKATHISIA:
_______________
. Weeks to chronic use.
. Motor restlessness (Do NOt nistake for anxiety or agitation).
. Tx -> Reduce the dose.
. Tx -> Add benzodiazepines or BBs (Propranolol).
{4} TARDIVE DYSKINESIA:
________________________
. Months to years.
. Choreoathtosis & other involuntary movements after chronic use.
. Often irreversible.
. Circumoral movements.
. Tx -> Stop older antipsychotics.
. Tx -> Switch to newer antipsychotics (Clozapine).

. It can be ttt with BENZTROPINE.


. Symptoms commonly worsen after medication discontinuation.
. N.B.
. Chronic use of dopamine antagonists eg. antiemetics (Metoclopramide & Prochlorperazine)
. can result in tardive dyskinesia.
{5} NEUROLEPTIC MALIGNANT $YNDROME:
____________________________________
. Any time !
. Muscle rigidity - Hyperthermia - Volatile vital signs - Altered consciousness.
. ++ WBCs & ++ Creatinine kinase level.
. Tx -> Stop antipsychotics.
. Tx -> DANTROLENE (Muscle relaxant).
. Transfer to ICU for monitoring.
. Mortality rate 20 %.
.
.
.
.
.

N.B.
CLOZAPINE is the most effective anti-psychotic for schizophrenia.
CLOZAPINE has NO incidence of movement disorders.
CLOZAPINE is a 2nd line therapy bec. of the risk of seizures & agranulocytosis.
Remember to monitor CBC to watch for bone marrow suppression.

. N.B.
. BENZTROPINE (Anticholinergic)
. is the 1st line ttt in management of acute dystonia & bradykinesia (parkinsonism).
. N.B.
. BBs (Propranlol) is the 1st line ttt of akathisia.
__________________________________________________________________________________________
. ANXIETY DISORDERS ( ):
_______________________________
_______________________________
. Anxiety that interferes e' daytime functioning not due to any other identifiable cause.
. Medical causes:
. Hyperthyroidism - Pheochromocytoma - Excess cortisol - Heart failure.
. Arrhythmia - Asthma - COPD.
. Drugs:
. Corticosterids - Cocaine - Amphetamines - Caffeine.
. Withdrawal from alcohol & sedatives.
{1} ADJUSTEMENT DISORDER ( ):
____________________________________
. Normal psychological reaction (anxiety - depression - irritability).
. occurs soon after profound changes in a person's life.
. such as divorce - migration - birth of handicapped child.
. Symptoms are not severe enough to be classified in another category.
. It is NOT a true anxiety disorder.
. Tx -> Psychotherapy & counselling to help with the pt adjust to the life stressor.
. NO medications.

{2} PANIC DISORDER ( ):


______________________________
. Brief attacks of intense anxiety with autonomic symptoms;
. tachycardia - hyperventillation - dizziness - sweating.
. Episodes occur REGULARLY without an obvious precipitant.
. Absence of any other psychiatric ilness.
. Tx -> COGNITIVE BEHAVIORAL THERAPY.
. Tx -> Relaxation training & desensitization.
. Tx -> Acute panic attack -> Benzodiazepines (Alprazolam - Clonazepam).
. Tx -> Long term symptomatic relief -> SSRIs (Fluoxetine).
. Tx -> Imipramine & MAOIs (Phenelzine) may be used.
. N.B. Panic disorder pts have ++ risk of depression !
{3} PHOBIC DISORDERS ( ):
_________________________________
. Persistent, unreasonable intense fear of situations, circumstances or objects.
. No known eliciting events in phobic disorders associated with the onset of symptoms.
. D.D. -> Post-traumatic stress diorder (PTSD) & Acute stress diorder (AST);
. which have a HISTORY OF TRAUMATIC EVENT (Threat to life).
(a) AGORAPHOBIA ( ):
_________________________
. Fear of avoidance of places due to anxiety about non being able to escape;
. public spaces - being outside alone - public transportation - crowds.
. It is more common in women.
(b) SOCIAL PHOBIA ( ):
____________________________
. Fear of humiliation or embarrasement in either general or specific social situations;
. public speaking - stage fright - Urinating in public restrooms.
. The pt knows that the response is excessive & unreasonable.
. D.D. AVOIDANT PERSONALITY DISORDER:
. The person does NOT believe the avoidance is excessive or unreasonable.
. Tx of generalized social anxiety disorder:
-> SSRIs (Paroxetine) & cognitive behavioral therapy.
(c) SPECIFIC PHOBIAS:
______________________
. Most common type of phobias.
. Animals (Carnivores or spiders) - Natural environments (Storms).
. Injury (Injection - Blood) - Situations (Heights - Darkness).
. Tx -> Exposure therapy -> ++ exposure to stimulus to induce habituation & -- anxiety.
. Benzodiazepenes & Beta blockers are helpful when given prior to exposure.
. BBs are useful in performance related anxiety.
{4} OBSESSIVE COMPULSIVE DISORDER (OCD) ( ):
________________________________________________________
. Recurrent obsessions or compulsions.
. The pt recognizes that the behavior is unreasonable & excessive (there is insight).
. Obsessions are anxiety provoking; thoughts are intrusive.
. Related to contamination, doubt, guilt, aggression & sex.

.
.
.
.
.
.
.

Compulsions are peculiar behaviors that reduce the anxiety.


Most commonly habitual hand washing, organizing, checking, counting & praying.
Pts with Tourette $ often also have OCD !!
Depression & substance abuse are common.
Tx -> 1st line of ttt is SSRIs (Paroxetine & Sertaline).
SSRIs alter neurotransmitter serotonin level.
Behavioral therapy is useful.

.
.
.
.

N.B.
Obsessive symptoms in psychotic disorders may be misdiagnosed as OCD.
You can differentiate psychosis from OCD by looking for:
a lack insight & loss of contact to reality.

. N.B.
. Pts with Tourette $ have a high risk of developing ADHD or OCD !
{5} ACUTE STRESS DISORDER (ASD) & POST-TRAUMATIC STRESS DISORDER (PTSD):
_________________________________________________________________________
. ACUTE STRESS DISORDER (ASD) ->
. POST-TRAUMATIC STRESS DISORDER (PTSD) ->
. Anxiety symptoms that follow a life threatening event.
. ASD -> Symptoms last LESS THAN ONE MONTH & occur within 1 month of stressor.
. PTSD -> Symptoms last MORE THAN ONE MONTH.
. Re-experiencing of the traumatic event: Dreams, flashbacks or intrusive recollections.
. Avoidance of stimuli associated e' trauma or numbing of general responsiveness.
. Increased arousal: Anxiety, sleep disturbances, hypervigilance & impulsiveness.
. Tx -> Benzodiazepines acutely for anxiety symptoms.
. SSRIs & anti-depressants can be helpful for long term therapy.
. N.B.
. GROUP COUNSELING is the most effective to prevent PTSD following a traumatic event.
{6} GENERALIZED ANXIETY DISORDER (GAD):
________________________________________
. Excessive, poorly controlled anxiety that occurs daily for more than 6 months.
. No single event or focus is related to anxiety.
. It often coexists e' major depression, specific phobi, social phobia & panic disorder.
. Tx -> SUPPORTIVE PSYCHOTHERAPY.
. Tx -> SSRIs, Venlafaxine, buspirone & benzodiazepenes may be used.
.
.
.
.
.

N.B.
Distinguish GAD from panic attack or social phobiaby what is causing the anxiety.
If the question describes persistent worry of a panic attack or social encounter,
then GAD is NOT the answer.
In GAD, multiple life circumstances, not just one, are causing the anxiety.

. ANXIOLYTIC MEDICATIONS:
__________________________
__________________________
. Adjustment disorder with anxious mood:

. Tx -> Benzodiazepines with brief psychotherapy.


. Rapid onset to therapy.
. Panic disorder:
. Tx -> SSRIs, Alprazolam & Clonazepam.
. They -- intensity & frequency of panic attacks.
. GAD: Tx -> Venlafaxine (-- overall anxiety).
. OCD: Tx -> SSRI (-- obsessional thinking).
. Social phobia -> SSRIs (-- fear ass. e' social situations).
. Benzodiazepines:
-> Don't change dosages abruptly.
-> Use the lowest dose in the elderly.
-> Advise against using machinery or driving.
-> Half life -> ALPRAZOLAM < LORAZEPAM < DIAZEPAM.
.
.
.
.

N.B.
Abrupt cessation of Alprazolam (used in sleeping difficuties),
which is a short acting benzodiazepine lead to withdrawal symptoms;
in the form of generalized tonic clonic seizures.

. BUSPIRONE:
-> Therapeutic effect can take up to 1 week.
-> No sedation or cognitive impairment.
-> Best option for people with occupations where driving or machinery is involved.
-> No withdrawal syndrome.
__________________________________________________________________________________________
. MOOD DISORDERS ( ):
_____________________________
_____________________________
{1} MAJOR DEPRESSIVE DISORDER ( ):
__________________________________________
. Depressed mood or anhedonia & depressive symptoms lasting at least 2 weeks.
. Major depressive disorder = Depressed mood + SIGECAPS.
.
.
.
.
.
.
.
.

S -> changes in (S)leep.


I -> loss of (I)nterest.
G -> thoughts of worthlessness or (G)uilt.
E -> loss of (E)nergy.
C -> trouble (C)oncentrating.
A -> changes in (A)ppetite or weight.
P -> changes in (P)sychomotor activity.
S -> thoughts about death & (S)uicide.

. All depressed pts sh'd be asked about death or suicideal thoughts.


. Look for other causes of depression where the 1st step in management is different:
-> Hypothyroidism (Check TSH).
-> Parkinson's disease.

-> Medications (Corticosteroids, BBs, antipsychotics).


-> Substance abuse (Alcohol - Amphetamines).
.
.
.
.

Tx -> Admit the pt if there is suicidal/homicidal ideation or paranoia.


Begin antidepressant medications (SSRI is the 1st drug of choice).
Give benzodiazepines if agitated.
Electroconvulsive therapy (ECT) is the best choice if the pt is acutely suicidal.

. N.B.
. The antidepressant of choice for depressed pts who don't respond to 1st line ttt
. with an SSRI (e.g. Paroxetine) is another medication of the same class (Citalopram).
. N.B.
. In management of single episode of major depression,
. the antidepressant sh'd be continued for a period of 6 months.
{2} DYSTHYMIC DISORDER ( ):
_______________________________________
. DYSTHYMIA = PERSISTENT DEPRESSIVE DISORDER.
. The pt is depressed over entire life.
. Low level depression symptoms on most days for at least 2 years.
. Superimposed acute major depressions may occur.
. Don't hospitalize the pt unless there's suicidal ideation.
. Tx -> Long term individual, insight oriented psychotherapy.
. Tx -> If failed, a trial of SSRIs may be done.
{3} SEASONAL AFFECTIVE DISORDER ( ):
__________________________________________________
. Depressive symptoms in the winter months (Shorter daylight hours).
. Absence of depressive symptoms during summer months (Longer daylight hours).
. Tx -> Psychotherapy or sleep deprivation.
{4} BIPOLAR DISORDER ( : ):
______________________________________________
. Episodes of depression, mania or mixed symptoms for at least 1 week.
. H/O of both manic syms & depressive syms as well as periods of normal mood.
. RAPID CYCLIC BIPOLAR is indicated by > 4 episodes of mania per year.
. Risk of bipolar disorder in general population is 1 %.
. It is 10 % risk in those with 1st degree relative H/O.
. MANIA SYMPTOMS:
-> Grandiosity - Less need for sleep - Excessive talking - Pressured speech.
-> Racing thoughts - Flight of ideas - Distractability - Sexual promiscuity.
-> Goal focused activity at home or at work.
. MAJOR DEPRESSIVE SYMOTOMS:
-> Depressed mood - Loss of pressure or interest.
. BIPOLAR TYPE (1) DISORDER: MANIC episodes; pts may or may not 've depressive episodes.
. BIPOLAR TYPE (2) DISORDER: Major depression + Hypomania.
. MANAGAEMENT:

*1* HOSPITALIZE (in case of severe manic symptoms despite mood stabilizer therapy).
*2* Mood stabilizers are used to induce remission.
. Lithium is the drug of choice (takes 1 week for effect).
*3* Antipsychotics are used until acute mania is controlled.
. Risperidone is the drug of choice.
*4* Give IM depot phenothiazine in non-compliant severely manic patients.
*5* Give antidepressants only when there's a H/O of recurrent episodes of depression,
. Given ONLY TOGETHER with mood stabilizers (to prevent including manic episode).
.
.
.
.

N.B.
The long term therapy of bipolar disorder is mood stabilizer (Lithium).
Lithium is NEPHROTOXIC.
If the pt has renal problems (++ urea & creat) -> Give VALPROIC ACID.

.
.
.
.
.

N.B.
Lithium in the 1st trimester of pregnancy is very dangerous.
It causes cardiac malformations.
Septal defects & Ebstein's anomaly (Atrialization of right ventricle).
In 2nd & 3rd trimesters, it causes goiter & neuromuscular dysfunction.

. N.B.
. Choose electro-convulsive therapy (ECT) for 1st trimester pts with manic episodes.
. LAMOTRIGENE may be used in 2nd or 3rd trimester.
. N.B.
. Pts who are extremely agitated, psychotic or manic, sh'd be initially managed with
. an antipsychotic medication such as "Haloperidol".
{5} CYCLOTHYMIA ( ):
__________________________
. H/O of recurrent episodes of depressed mood & hypomanic mood for at least 2 years.
. It is a mild form of bipolar affective disorder.
. Tx -> Psychotherapy is the 1st line of ttt.
. DIVALPROEX is used when functioning is impaired (More effective than Lithium).
{6} GRIEF & DEPRESSION ( ):
__________________________________
.

< GRIEF >

< DEPRESSION >

. Sadness - Tearfulness - -- Sleep - -- Appetite - -- interest in the world.


.
.
.
.
.
.

Symptoms wax & wane.


. Symptoms are pervasive & unremitting.
Shame & guilt are less common.
. More common.
Suicidal ideation is less common.
. More common.
Symptoms can last up to 1 year.
. Symptoms continue for more than 1 year.
Pt returns to normal functioning in 2 months . No return to base line functioning.
Tx -> SUPPORTIVE therapy.
. Tx -> ANTI-DEPREESANTs.

. N.B.
. BEREAVEMENT is a normal reaction o the loss of beloved one !

. PERSISTENT COMPLEX BEREAVEMENT DISORDER -> Severe impairment >12 months after the
loss!
.
.
.
.
.

N.B.
COMPLICATED GRIEF / EXTENDED BEREAVEMENT can present e' syms of major depression.
Bereaved pts who have at least 2 weeks of syms of depression,
6-8 weeks after a major loss, sh'd be considered for ttt with:
BOTH PSYCHOTHERAPY & TRIAL OF ANTIDEPRESSANTs.

. N.B.
. Pts e' both mood & psychotic symptoms respond to both antidepressants & antipsychotics.
. However, you must treat the worst symptoms first.
. N.B.
. Auditory hallucinations e'out other psychotic symptoms are normal in grief reaction.
{7} POST-PARTUM DEPRESSION:
____________________________
. A . POSTPARTUM BLUES = BABY BLUES:
_____________________________________
. After any birth.
. Mother cares about the baby.
. Mild depressive symptoms.
. Self limited, no ttt necessary.
. B . POSTPARTUM DEPRESSION:
_____________________________
. Usually after 2nd birth.
. Many have thoughts about hurting the baby.
. Severe depressive symptoms.
. Tx -> Antidepressants.
. C . POSTPARTUM PSYCHOSIS:
____________________________
. Usually after 1st birth.
. Mothers have thoughts about hurting the baby.
. Psychotic symptoms along with severe depressive symptoms.
. Tx -> Mood stabilizers or antipsychotics & antidepressants.
. Avoid medications if the pt is breastfeeding; use ECT instead !
{8} SUICIDE & SUICIDAL IDEATION:
_________________________________
* RISK FACTORS:
________________
. History of suicide threats & attempts is the most important predictor of suicide.
. Family H/O of suicide.
. Perceived hopelessness (Demoralization).
. Scizophrenia, borderline or antisocial personality.
. Drug use, especially alcohol.
. Males.
. Age > 65 ys.
. Socially isolated, recently divorced or widowed.
. Chronic physical illness.

. Low job satisfaction or unemployment.


* EMERGENCY ASSESSMENT:
________________________
. Take all suicide threats seriously.
. Detain & hospitalize (Usually 2 weeks).
. Never transport patient to emergency depratment without medically trained personnel.
. Don't identify with the pt.
. Tx of choice -> PSYCHOTHERAPY + ANTIDEPRESSANTs (SSRIs are the 1st choice).
. For acute severe risk of self-harm -> Tx of choice is ECT.
. N.B.
. Minors with suicidal attempts must be admitted to hospital ,
. even against their parents will (Their consent is NOT mandatory).
* INDICATIONS FOR ELECTROCONVULSIVE THERAPY (ECT):
___________________________________________________
. Major depressive episodes that are unresponsive to medications.
. High risk of immediate suicide.
. Contraindications to using antidepressants.
. Good response to ECT in the past.
. The biggest complication of ECT is TRANSIENT MEMORY LOSS,
. which worsens with prolonged therapy & resolves after several weeks.
. Use of ECT is cautioned in pts with space occupying intracranial lesions.
. ECT ++ ICT.
* ANTIDEPRESSANTs & MOOD STABILIZERs:
______________________________________
. SSRIs are the 1st line of therapy.
. TCAs are avoided bec. of risk of toxicity (U sh'd monitor BP).
. MAOIs are more helpful in atypical depressive disorders.
. Switch to another antidepressant if there is no response after 8 weeks.
. Treat the pt for 6 months then taper the dose gradually.
. SSRIs are the 1st line of therapy in the following disorders:
-> Major depressive disorder.
-> Bipolar disorder.
-> Anxiety disorders (Panic disorder - OCD - Social phobia - GAD).
-> Bulimia nervosa.
. When the Q. describes a pt concerned about weight gain or sexual side effects,
. Give Bupropion (causes modest weight loss).
. Bupropion is associated with SEIZURES !!
. When the Q. describes a pt who has poor appetite, loos of weight or insomnia,
. Give MIRTAZAPINE (Ass. e' weight gain).
. AMITRIPTYLINE is used to treat chronic pain (Neuropathic pain).
. Amitriptyline is ass. e' antichlinergic effects (If severe switch to SSIs).
. IMIPRAMINE is useful in noctunal enuresis (DESMOPRESSIN is the 1st choice).
. TRAZADONE is strongly sedating used in ttt of pts with insomnia.

. Trazadone causes PROLONGED ERECTION !


. SSRIs & TCAs are SAFE in pregnancy except for PAROXETINE !
. Seizures are common with TCAs & Bupropion.
. These medications sh'd be avoided in pts with seizures disorders.
. The best 1st line of ttt in pts with seizures is SSRIs.
. TCAs have anticholinergic effects & are an Alpha blocker,
. causing peripheral vasodilatation & hypotension.
. TCAs affect the sodium channels in the cardiac tissue.
. EKG is the single most important test to guide TCAs therapy.
. Watch out for prolonged QRS, QT & PR intervals.
. Most serious complication is ventricular tachycardia & fibrillation.
. SODIUM BICARBONATE Na HCO3 attenuates TCA cardiotoxicity by alkalinization of blood,
. which uncouples TCA from myocardial sodium channels & ++ extracellular Na concentration
. Lithium is the 1st line of ttt for BIPOLAR & SCHIZOAFFECTIVE disorders.
. Side effects:
-> Acne & weight gain are the most common problems.
-> Tremors, GI distress & headaches.
-> Hypothyroidism (Order TSH level).
-> Polyurea 2ry to lithium induced DI (Order Creinine level).
-> Fetal cardiac defects & Ebstein's anomaly if used in 1st trimester.
. DIVALPROEX is the 1st line of choice for rapid cyclic bipolar disorder.
. CARBAMAZEPINE is the SECOND line of ttt for bipolar disorder.
. Used when lithium is ineffective or contraindicated.
. Not used due to severe agranulocytosis & sedation side effects.
.
.
.
.

Lithium for life time !!


Pts who've experienced 2 episodes of acute mania sh'd be considered for long time
if previous manic episodes were severe or there is family H/O.
Pts with 3 or more relapses are recomended to have life time lithium therapy.

. Q. What is the 1st assessment prior to prescribing antidepressants ?


. A. Suicidal ideation.
__________________________________________________________________________________________
. MEDICATION OVERDOSES:
________________________
________________________
{1} LITHIUM TOXICITY:
______________________
. Elderly p who takes lithium with renal failure or hyponatremia.
. May be induced by diuretics, vomiting or dehydration.
. Nausea - vomiting - acute disorientation - tremors - ++ DTRs - seizures.
. Tx -> DIALYSIS.
{2} NEUROLEPTIC MALIGNANT $YNDROME:

____________________________________
. H/O of recent start with antipsychotics (Specially HALOPERIDOL).
. H/O of Parkinson's pt who has recently stopped Levo dopa.
. High fever - Tachycardia - Ms rigidity - Altered consciousness - Autonomic dysfunction.
. It is unrelated to dosage or previous drug exposure.
. 20 % mortality rate.
. Tx -> Transfer to ICU.
. Tx -> Discontinue antipsychotic.
. Tx -> Bromocriptine to overcome dopamine receptor blockage.
. Tx -> Ms relaxants (DANTROLENE or DIAZEPAM) to reduce ms rigidity.
{3} SEROTONIN $YNDROME:
________________________
. H/O of SSRIs use or migraine medication (triptans) or MAOIs.
. Agitation - Hyperreflexia - Hyperthermia - Muscle rigidity.
. Volume contraction 2ry to sweating & insensible fluid loss.
. Tx -> IV fluids.
. Tx -> Cryptoheptadine to -- serotonin production.
. Tx -> Benzodiazepine to -- muscle rigidity.
{4} MAOIs INDUCED HYPERTENSIVE CRISIS:
_______________________________________
. H/O of MAOI use with acute hypertension.
. H/O of antihistaminics or nasal decongestants may be a cause.
. H/O of consumption of tyramine rich foods (Cheeses - Pickled foods).
. May also be seen in pts who take a MAOI (Phenelzine) & a TCA concurrently.
. Tx -> As hypertensive crisis.
__________________________________________________________________________________________
. SOMATOFORM DISORDERS = :
______________________________________
. Physical symptoms without medical explanation.
. Severe enough to interfere with the pt's ability to function.
{1} SOMATIZATION DISORDER = :
_____________________________________
. 4 pain symptoms + 2 GIT symptoms + 1 Sexual symptom + 1 psudoneurologic symptom
. Tx -> Maintain a single physician as the primary care giver.
. Tx -> Schedule brief monthly visits.
. Tx -> Avoid diagnosting tests or therapies.
. Tx -> Schedule individual psychotherapy.
. Tx -> Do NOT hospitalize the pt.
{2} CONVERSION DISORDER = :
___________________________________
. One or more neurological symptoms that,
. can't be explained by any medical or neurological disorder.
. Most common syms (Blindness - Mutism - Paralysis - Anesthesia - Paresthesia).
. Look for psychologic factors associated with the onset of syms.
. THE PATIENTS ARE UNCONCERNED ABOUT THEIR IMPAIRMENT (LA BELLE INDIFFERENCE).
. You must first rule out other medical conditions.
. Tx -> Supportive physician-patient relationship.
. Tx -> Psychotherapy.

. N.B.
. Somatization disorder or conversion disorder are NEVER the correct diagnosis if:
. symptoms are produced intentionally or feigned.
{3} HYPOCHONDRIASIS = :
___________________________
. The pt has false belief that he has a specific disease,
. despite repeated negative medical tests & work up.
. Symptoms must have been present for at least 6 months.
. Physician's reassurance has failed to relief concerns.
. Tx -> Identify one primary care giver.
. Tx -> Schedule regular routine visits.
. Psychotherapy ( Initiate a discussion about current emotional stressors).
{4} FACTITIOUS & MALINGERING DISORDERS = :
__________________________________________________________
. INTENTIONALLY FEIGNED SYMPTOMS !
. A pt that has seen many doctors & visited many hospitals.
. A pt that has large amount of medical knowledge (e.g. Health care workers).
. A pt who demands a treatment.
. Always agitated & threatens litigation if tests return -ve !!
. No secondary gain (Unlike malingering).
. Factitious disorder by proxy:
-> If the signs & syms are faked by another person.
-> As in a mother making up symptoms in her child.
-> The motivation is to assume the caretaker role.
. Malingering:
-> When obvious gain results from feigned symptoms.
-> Ex: Shelter - medications - disability insurance.
-> Pts are more occupied with rewards or gain than alleviation of presenting symptoms.
. N.B.
. Factitious disorder -> The pt wants sick role.
. Malingering disorder -> The pt wants secondary gain.
. Tx -> Supportive psychotherapy.
. Do NOT confront or accuse the pt (The pt will become angry, more guarded & suspicious).
. Only provide the minimum amount of treatment & work up needed.
__________________________________________________________________________________________
. EATING DISORDERS = :
______________________________
______________________________
{1} ANOREXIA NERVOSA = :
_________________________________
. YOUNG FEMALE - UNDERWEIGHT.
. Food restriction & excessive exercise.
. No menstrual period for 3 months or more.
. H/O of purging ( ).
. N.B.

. REFEEDING $YNDROME:
-> Fluids & electrolytes shift -> Electrolyte depletion, arrhythmias & heart failure.
. N.B.
. ANOREXIA COMPLICATIONS:
-> Osteoporosis.
-> ++ Cholesterol & carotene levels.
-> Cardiac arrhythmias (Prolonged QT interval).
-> Euthyroid sick $.
-> Hypothalamic - pituitary axis dysfunction -> Anovulation.
-> Hyponatremia secondary to excess water intake.
-> Pregnants (Miscarriage - Hyperemesis gravidarum - postpartum depression - C.S.).
-> Fetus (IUGR - Prematurity).
{2} BULIMIA NERVOSA = :
______________________________
. YOUNG FEMALE - NORMAL WEIGHT RANGE.
. Frequent episodes of binge eating follwed by guilt, anxiety.
. Self induced vomiting, laxative, diuretics or enema use.
. Food restriction is NOT a feature of bulimia nervosa.
. Painless parotid gland enlargement.
. Dental enamel erosions.
. Metabolic alkalosis with hypochloremia & hypokalemia caused by emesis.
. Metabolic acidosis caused by laxative abuse.
. Risk of cardiomyopathy with excessice syrup of epicac use.
. MANAGEMENT of ANOREXIA NERVOSA & BULIMIA NERVOSA:
____________________________________________________
. HOSPITALIZE for IV hydration if electrolyte disturbance are present.
. OLANZAPINE in anorexia nervosa helps with weight gain.
. SSRI antidepressants (esp. FLUOXETINE) prevent relapses.
. Behavioral therapy.
{3} BODY DYSMORPHIC DISORDER = :
__________________________________________
. YOUNG FEMALE - Preoccupied with an imagined or slight defect in appearance.
. causing an impaired ability to function in a social or occupational life.
. Distress is most commonly related to facial features.
. The pt is often isolated & housebound.
. Tx -> High doses of SSRIs are the 1st line of ttt.
. N.B.
. If the only concern is body shape & weight -> ANOREXIA NERVOSA is more accurate Dx.
. If the only concern is sex characteristics -> GENDER IDENTITY DISORDER is more acc.
__________________________________________________________________________________________
. IMPULSE CONTROL DISORDERS = :
______________________________________________
______________________________________________
. People who are unable to resist impulses.
. Anxiety prior to the impulse that is relieved after the pt acts on it.
. Pts do NOT believe their actions or out of proportion.
. Pts lack insight (Unlike OCD).

{1} INTERMITTENT EXPLOSIVE DISORDER = :


_________________________________________________
. Episodes of aggression out of proportion to the stressor.
. H/O of head trauma.
. If there is a H/O of drug intake -> Intermittent explosive diorder is NOT the Dx !
. Tx -> SSRIs & mood stabilizers.
{2} KLEPTOMANIA = :
_________________________
. Individual who repeatedly steals items to relieve anxiety.
. The person doesn't steal because he needs the object.
. The person often secretely replaces the object after stealing it.
. Tx -> COGNITIVE BEHAVIORAL THERAPY.
{3} PYROMANIA = :
__________________________
. Individual who repeatedly lights fires.
. Pyromania is NOT the diagnosis if the motive is personal gain (insurance money),
. or when the motive is to show anger (Differenting it from CONDUCT DISORDER).
{4} PATHOLOGIC GAMBLING = :
___________________________________
. Obsession with gambling despite the consequences.
. Tx -> Group psychotherapy (GAMBLING ANONYMOUS).
{5} TRICHOTILLOMANIA:
______________________
. Uncontrollable urge to pull out the hair -> Alopecic areas.
. These areas still contain hair of varying lenghts.
__________________________________________________________________________________________
. TYPES OF ABUSE:
__________________
__________________
{1} CHILD ABUSE:
_________________
. PHYSICAL is the most common.
. Look for bruises - burns - lacerations - broken bones.
. Shaken baby $ -> Do eye exam !!
. Neglect.
. Sexual exploitation (STDs).
. Mental cruelty.
. MANDATORY REPORTING UP TO AGE 18.
. You must report ALL suspected cases.
. Protect the child (Separate from parents) & consider admission to hospital.
. Those who are younger than 1 year are at risk.
. Step children, premature, very active & defective children are at risk.
. Be careful not to mistake BENIGN CULTURAL PRACTICES (Coining - Moxibustion) for abuse.
. Treat FEMALE CIRCUMCISION as ABUSE !!

{2} ADULT MALTREATMENT = ELDER ABUSE:


______________________________________
. NEGLECT is the most common (50 % of all reported cases).
. Physical, psychological & financial are another forms.
. You must report ALL suspected cases.
. Protect pt from abuser & consider admission to hospital.
. Caretaker is the most likely source of abuse; spouses are often caretakers.
{3} SPOUSAL ABUSE:
___________________
. PHYSICAL is the most common.
. It is the number 1 cause of injury to American women.
. Psychological & financial are another forms.
. Reporting is NOOOOOTTTTT indicated !
. Provide information about local shelters & counseling.
. High risk categories:
-> More frequent in families with drug abuse, esp. Alcoholism.
-> Victim often grew up in a violent home (about 50 %).
-> Married at a young age.
-> Dependent personalities.
-> Pregnant, last trimester (Highest risk).
. MANAGEMENT OF ABUSE:
_______________________
1 - Complete physical examination.
2 - Radiographic skeletal survey.
3 - Coagulation profile (If multiple bruises).
4 - Report to child protective services.
5 - Admission of the child to hospital.
6 - Consultation with a psychiatrist & evaluation of family dynamics.
__________________________________________________________________________________________
. PERSONALITY DISORDERS (PD):
______________________________
______________________________
{A} CLUSTER A -> Peculiar thought processes & Inappropriate affect !
_____________________________________________________________________
(1) PARANOID PD:
_________________
. Mistrustful & suspiciousness of the motivations & actions of others.
. Secretive & isolated.
. Emotionally cold & odd.
. Often take legal action against other people.
. Often confused with paranoid schizophrenia.
. Main defense mechanism is projection.

. Ex. 62 ys old man lives in an apartment,


. constantly acuses his neighbors of stealing his mail & prying into his apartment.
. He believes that all his neighbors are conspiring to have him removed from the building
(2) SCHIZOID PD:
_________________
. Emotionally distant & fear intimacy with others.
. Absorbed in their own thoughts & feelings.
. Always disinterested.
. Main defense mechanism is projection.
. Ex. 68 ys old man lives in the country-side manning a lighthouse near a remote village.
. He is seen in town 2-3 times a year to purchase supplies.
. He has no known friends or family.
(3) SCHIZOTYPAL PD:
____________________
. Like schizoid PD except they also have MAGICAL THINKING.
. They have clairvoyance, ideas of reference & paranoid ideation.
.
.
.
.

Ex. 28 ys old man lives in a small coastal town,


attempting to start his own internet herbal business.
He believes that the herbs have magical power of healing.
He believes that spirits are guiding him to wealth.

{B} CLUSTER B -> Mood lability, dissociative symptoms & preoccupation with rejection:
______________________________________________________________________________________
(1) HISTRIONIC PD:
___________________
. Colorful exagerrated behavior & excitable.
. Shallow expression of emotions.
. Use of physical appearance to draw attention to self.
. Sexually seductive.
. Discomfort in situatios where not the center of attention.
.
.
.
.

Ex. 30 ys old woman presents to the doctor's office,


dressed in a sexually seductive manner,
insisting taht the doctor comment on her appearance.
When the doctor refuses to do so, she becomes upset.

(2) BORDERLINE PD:


___________________
. Unstable affect, mood swings, marked impulsivity, unstable relationships.
. Recurrent suicidal behaviors, chronic feelings of emptiness & identity disturbance.
. Inappropriate anger (Become intensily angered if they feel abondened).
. Main defense mechanism is SPLITTING.
.
.
.
.
.
.

Ex. 30 ys old woman reports that she has been to many doctors,
They were all wonderful until they started ignoring her or cutting her visits short,
then she realized what terrible doctors they were.
She startes the visit saying that the assistant at the front desk is the worst ever,
because she didn't smile at her.
The other assistant was just wonderful according to her !

(3) ANTISOCIAL PD:


___________________
. Continous anti-social or criminal acts.
. Inability to conform to social rules, impulsivity & aggressiveness.
. Disregard for the rights of others.
. Lack of remorse & deceitfullness.
.
.
.
.
.

Ex. 26 ys old man is caught lighting forest fires during a recent spate.
H/O of legal problems since childhood.
He reports that his mother is to blame.
He denies feeling regret.
He has no friends & is found to be hostile to everyone at the police station.

(4) NARCISSISTIC PD:


_____________________
. Sense of self-importance, grandiosity & pre-occupation with fantasies of success.
. Belief of being special, requires excessive admiration.
. Reacts with rage when criticized.
. Lacks empathy, is envious of others & is interpersonally exploitative.
.
.
.
.

Ex. Pt in hospital for chest pain & becomes very agitated,


because he feels he is not getting enough attention.
He reports that he is an important CEO.
Demands a special VIP room, more consideration & a dedicated nurse to attend his needs.

{C} CLUSTER C -> Anxiety, preoccupation with criticism or rigidity:


____________________________________________________________________
(1) AVOIDANT PD:
_________________
. Social inhibition, feelings of inadequacy & hypersensitivity to criticism.
. They shy away from starting anything new or attending social gatherings.
. Always fear of failure or rejection.
. They desire affection & acceptance.
. They are open about their isolation & inability to interact with others.
. Ex. 45 ys old singl man fears an upcoming social party being hosted by his parents.
. He dreads having to meet other people & doesn't feel comfortable speaking e' others.
. He is planning on staying at home to avoid speaking to others.
(2) DEPENDENT PD:
__________________
. Submissive & clinging behavior related to a need to be taken care of.
. They are always worry about abandonment.
. They feel inadquate & helpless & avoid disagreements with others.
. They usually focus dependency on a family member or a spouse.
.
.
.
.
.
.

Ex. 28 ys old woman seeks counseling bec. of a recent relationship breakup.


They were dating for 6 months.
She continues to call her ex 15 - 20 times a day eventhough he doesn't pick up.
She says she can't understand why they broke up bec. she never disagreed with him.
She never left the house without him & she always asked his opinion.
She can't imagine life without him.

(3) OBSESSIVE COMPULSIVE PD:


_____________________________
. They are preoccupied with orderliness, perfectionism & control.
. They are always consumed by details of everything & lose their sense of overall goals.
. They are strict & perfectionistic, overconscientious & inflexible.
. Associated with difficult interpersonal relationships.
. Ex. 38 ys old man presents with his wife for marital counseling.
. The wife reorts that he is inflexible & has unrealistic demands of orderliness.
. Both partners agree that his demands are causing marital problems.
__________________________________________________________________________________________
. SUBSTANCE USE DISORDERS:
___________________________
___________________________
(1) ALCOHOL DEPENDENCE = ALCOHOLISM:
_____________________________________
. Frequent use of alcohol -> Tolerance & physical & psycholical dependence.
. Alcohol abuse -> Failure to fulfill obligations, legal troubles.
. Tolerance is NOT included in the diagnosis of alcohol abuse.
. Dx -> CAGE QUESTIONNAIRE (Lab tests are never included in the diagnosis).
. CAGE -> An answer of YES to any 2 of the following Qs is suggestive of abuse:
-> Have you ever felt that you should CUT down your drinking ?
-> Have you ever felt ANNOYED by others who have criticized your drinking ?
-> Have you ever felt GUILTY about your drinking ?
-> Have you ever had an EYE-OPENER to steady your nerves or alleviate a hangover ?
. Order toxicology to look for another drugs: breath, blood & urine drug screens.
. Look for 2ry effects of alcohol use: ++ GGTP, AST, ALT & LDH.
. If there is suggestion of IV drug use (treack marks) -> Order HIV, HBV, HCV & PPD.
. Management of abuse or prevention of relapse -> ALCOHICS ANONYMOUS (AA).
. ACUTE OUT-PATIENT MANAGEMENT OF ALCOHOL DEPENDENCE:
-> Prevent further ETOH intake.
-> Prevent individual from driving a car, operating machinery.
-> Sedate pt if he or she becomes agitated.
-> Transfer to inpatient.
. ACUTE IN-PATIENT MANAGEMENT PAERLS:
-> Look for withdrawal symptoms.
-> Prevent Wernicke-Korsakoff (ataxia - nystagmus - ophthalmoplegia - amnesia).
-> Give IV or IM thiamine & Mg ASAP plus vit. B12 & folate.
-> Benzodiazepine of choice is CHLORDIAZEPOXIDE or DIAZEPAM.
-> Choose short-acting benzodiazepam if there is H/O of severe liver disease: LORAZEPAM
-> Do NOT give seizure prophylaxis; repaeted seizures sh'd be ttt e' diazepam.
-> Haldol is NEVER the answer (It reduces seizure threshold).
. CHRONIC MAINTENANCE MANAGEMENT:
-> Refer to inpatient rehabilitation or outpatient group therapy (AA).
-> Never give drug therapy without group psychotherapy.
-> Naloxone & acamprosate -- relapse rate only when given with psychotherapy.

-> Disulfiram has poor compliance and hasn't been shown to be effective.
. WITHDRAWAL $ MANIFESTATIONS:
_______________________________
(1) MINOR WITHDRAWAL SYMPTOMS:
_______________________________
-> Onset after last drink -> 6 hours.
-> Syms -> insomnia, tremulousness, mild anxiety, headache, diaphoresis & palpitations.
-> Exam tips -> Give thiamine, folate, multivitamin & glucose.
(2) ALCOHOLIC HALLUCINOSIS:
____________________________
-> Onset after last drink -> 12 - 24 hours.
-> Symptoms -> Visual, auditory & tactile hallucinations.
-> Exam tips -> If there are hallucinations with disorientation & altered mental status
-> then alcoholic hallucinosis is NOT the answer.
(3) WITHDRAWAL SEIZURES:
_________________________
-> Onset after last drink -> 48 hours.
-> Symptoms -> Tonic clonic seizures.
-> Exam tips -> Perform CT scan if repeated seizures to rule out structural causes.
(4) DELIRIUM TREMENS:
______________________
-> Onset after last drink -> 48 - 96 hours.
-> Symptoms -> Hallucinations, disorientation, tachycardia, hypertension.
-> Symptoms -> Low grade fever, agitation & diaphoresis.
-> Exam tips -> Time of onset is important.
-> This is the diagnosis if the case describes symptoms 2 DAYS after last drink.
__________________________________________________________________________________________
. SUBSTANCE ABUSE:
___________________
___________________
{1} ALCOHOL:
_____________
. Intoxication syms -> Talkative, sullen, gregarious & moody.
. Intoxication ttt -> Mechanical ventillation if severe.
. Withdrawal syms -> Tremors, hallucinations, seizures & delirium.
. Withdrawal ttt -> Long acting benzodiazepeines (Chlordiazepoxide).
. No seizure prophylaxis.
. Disulfiram or naloxone for adjunct to supervised therapy after acute withdrawal.
{2} AMPHETAMINES & COCAINE:
____________________________
. Intoxication syms -> Euphoria, hypervigilance, autonomic hyperactivity & weight loss.
. Intoxication syms -> Pupil dilatation, disturbed perception, stroke & MI !
. Intoxication syms -> -- appetite (Picky eater).
. Intoxication syms -> Erythema of turbinates & nasal septum.

. Intoxication ttt -> Antipsychotics, benzodiazepines, inderal & vit C to ++ excretion.


. Withdrawal syms -> Anxiety, tremors, headache, ++ appetite, depression & suicide risk.
. Withdrawal ttt -> Antidepressants.
{3} CANNABIS:
______________
. Intoxication syms -> Impaired motor coordination, impaired time perception.
. Intoxication syms -> Social withdrawal, ++ appetite, dry mouth, tachycardia.
. Intoxication syms -> Conjunctival redness.
. Intoxication ttt -> NONE.
. Withdrawal syms -> NONE.
. Withdrawal ttt -> NONE.
{4} HALLUCINOGENS (LSD = LYSERGIC ACID):
_________________________________________
. Intoxication syms -> Visual hallucinations & intensified perception.
. Intoxication syms -> Ideas of reference, impaired judgment & dissociative syms.
. Intoxication syms -> Pupillary dilatation, panic, tremors & incoordination.
. Intoxication ttt -> Supportive counseling (talking down), antipsychotics & benzos.
. Withdrawal syms -> NONE.
. Withdrawal ttt -> NONE.
{5} INHALANTS:
_______________
. Intoxication syms -> Belligerence, apathy, assaultiveness & impaired judgement.
. Intoxication syms -> Blurred vision, stupor & coma.
. Intoxication ttt -> Antipsychotics if delirious or agitated.
. Withdrawal syms -> NONE.
. Withdrawal ttt -> NONE.
{6} OPIATES (HEROIN):
______________________
. Intoxication syms -> Respiratory depression, pin point pupils & CNS depression (Coma).
. Intoxication syms -> Apathy, dysphoria, drowsiness, slurred speech.
. Intoxication syms -> Impaired memory, coma & death.
. Intoxication ttt -> NALOXONE.
. Withdrawal syms -> Fever, chills, lacrimation, runny nose, abdominal cramps.
. Withdrawal syms -> Muscle spasms, insomnia, yawning & secretions from all openings !
. Withdrawal ttt -> METHADONE & clonidine.
{7} PHENYLCYCLIDINE (PCP):
___________________________
. Intoxication syms -> Panic reactions, assaultiveness & agitations.
. Intoxication syms -> Nystagmus, HTN, seizures, coma & hyperacusis.
. Intoxication ttt -> Talking down, benzodiazepines, antipsychotics & resp. support.
. Withdrawal syms -> NONE.
. Withdrawal ttt -> NONE.
{8} BARBITURATES & BENZODIAZEPINES:

____________________________________
. Intoxication syms -> Inappropriate sexual or aggressive behavior.
. Intoxication syms -> Impaired memory & concentrations.
. Intoxication ttt -> FLUMAZENIL.
. Withdrawal syms -> Autonomic hyperactivity, tremors, insomnia, seizures & anxiety.
. Withdrawal ttt -> Substitute short with long acting barbiturates (chlordiazepoxide).
__________________________________________________________________________________________
. HUMAN SEXUALITY:
___________________
___________________
{1} HOMOSEXUALITY:
___________________
. It is NOT a mental illness.
{2} GENDER IDENTITY DISORDER & TRANS-SEXUALISM:
________________________________________________
. An individual who insists that he/she is the opposite gender.
. Intense discomfort about his or her sex.
. It is NOT the Dx when the Q. describes an individual who desires to be another gender
. because of the perceived advantages of the other sex.
. e.g. a boy who wants to be a girl so that he will receive the same special ttt as his younger
sister.
{3} PARAPHILIAS:
_________________
. Recurrent, sexually arousing preoccupations which are usually focused on humiliation,
. The use of non-living objects & non-consenting partners.
. Occurs for more than 6 months & causes impairment in pt's level of functioning.
. Tx -> Individual psychotherapy & averse conditioning.
. If severe impairment -> Give antiandrogens or SSRIs to help reduce pt's sexual drive.
. TYPES OF PARAPHILIAS:
________________________
________________________
.1. VOYEURISM:
_______________
. Recurrent urges to observe an unsuspecting person who is engaging in sexual activity or
disrobing.
. This is the earliest paraphilia to develop.
.2. PEDOPHILIA:
________________
. Recurrent urges or arousal toward prepubescent children.
. This is the most common paraphilia.
.3. EXHIBITIONISM:
___________________
. Recurrent urge to expose oneself to strangers.

.4. FETISHISM:
_______________
. Use of non-living objects usually associated with the human body.
.5. FROTTEURISM:
_________________
. Recurrent urge involving touching or rubbing against a non-consenting partner.
.6. MASOCHISM:
_______________
. Recurrent urge or behavior involving the act of humiliation.
.7. SADISM:
____________
. Recurrent urge or behavior involving acts in which ..
. physical or psychological suffering of a victim is exciting to the patient.
. PHARMACOLOGICAL AGENTS THAT CAUSE SEXUAL DYSFUNCTION:
________________________________________________________
________________________________________________________
.
.
.
.
.
.

Alpha 1 blockers -> Impaired ejaculation.


Beta blockers -> Erectile dysfunction.
Neuroleptics -> Erectile dysfunction.
SSRIs -> Inhibited orgasm.
Trazodone -> Priapism.
Dopamine agonists -> ++ Erection & libido.

. N.B. POOR SLEEP HYGIENE:


___________________________
-> can be associated with insomnia.
-> Due to performance of bad daily living activities.
-> that are inconsistent with maintainance of sleep.
-> Poor sleep scheduling with variable wake & sleep times.
-> Frequent day time naping.
-> Routine use of caffeine, Alcohol, Nicotine in period preceiding sleep.
-> Engaging in mentally or physically stimulating activities too close to bed time.
-> Frequent use of the bed for activities other than sleep.
. N.B. DELAYED SLEEP PHASE $YNDROME:
_____________________________________
-> Circadian rhythm disorder.
-> Inability to fall asleep at normal bed times such as 10 p.m. to midnight.
-> These pts often can't fall asleep until 4-5 a.m. BUTTTTTT ..
-> their sleep is normal if they are allowed to sleep until late morning.
============================================================================
==============
============================================================================
==============
ETHICS TiKi TaKa
__________________

. AUTONOMY:
____________
. An adult e' capacity to understand his/her medical problems can refuse any ttt or test
. It doesn't matter if the ttt or test is simple, safe & risk free.
. It doesn't matter if the person will die without the ttt or the test.
. Respecting autonomy is MORE IMPORTANT to do the right thing for a pt !
.
.
.
.

Even though an adult pt e' capacity can refuse anything,


USMLE wants u to discuss things 1st.
Eventhough u may eventually honor his wishes,
if an answer says "meet", "confer", or "discuss", the do that first !

. CAPACITY:
____________
. Capacity is determined by physicians.
. Competence is a legal term & is determined by courts & judges.
. An adult who is alert & not mentally handicapped is deemed to have capacity.
. PSYCHIATRY CONSULTATION:
___________________________
. is the answer when a pt's capacity to understand is NOT clear.
. It is NOT necessary if the pt is clearly competent or clearly in coma !
__________________________________________________________________________________________
. MINORS:
__________
. Minors aren't determined to have the capacity to understand their medical problems,
. until the age of 18.
. Emancipation means that although the pt is under 18, he can make his own decisions.
. Emancipated minors are living independently & self supporting, married or in military.
. Partial emancipation is considered for (Sex - Reproductive health - Substance abuse).
.
.
.
.

If the pt is a minor & seeks ttt for contraception, STDs, HIV or prenatal care,
she is partially emancipated.
i.e. she can make these decisions on her own,
her privacy sh'd be resprected like that of an adult.

. An exception is ABORTION: 36 states have parental notification laws for abortion.


.

MINOR STATUS
|
______________________________________________
|
|
UN-emancipated
Emancipated
|
(Can consent for care)
|
|
. Age < 17 ys & must have consent
. Married
from parent or legal guardian
. In the military
. Lives separately from parents
& manages own financies.

. N.B. Parents can NOT refuse lifesaving therapy for minors.

. Ex. If a blood transfusion w'd be lifesaving, the parents can't refuse.


. Doing so w'd be considered child abuse.
. JEHOVAH's WITNESSES may refuse therapy for themselves but not for a child.
. In an emergency, family members or friends of a Jehovah's witness who suggest that a pt
. would not accept blood transfusion should be asked to provide documentary evidence,
. such as an advance directive.
. Without this documentation or when uncertainity remains,
. it is advisable NOT to withhold blood in life threatening conditions.
__________________________________________________________________________________________
. INFORMED CONSENT:
____________________
. It is based on autonomy.
. Only a fully informed pt e' the capacity to understand the issues can grant it.
. The pt must be informed of the benifits & the risks of the procedure.
. Alternatives of the procedure must be given.
. The information is in a language the pt can understand.
. The informed consent must be given for each procedure.
.
.
.
.
.
.

For emergency procedures, consent is implied in an emergency,


when there isn't suffecient time to determine capacity or prior wishes.
If prior wishes are fully known, then this information takes precedence.
Consent obtained via telephone is considered valid.
If the pt's proxy isn't present at the time of the procedure, then,
consent via telephone counts.

. Pregnant women can refuse therapy, even if the life of the fetus is at risk !!
. Until the fetus comes out of the body, it is considered part of the woman's body.
. Ex. A woman can refuse a blood transfusion while pregnant.
. She can refuse antiretroviral ttt during prgnancy, even if the fetal's life is at risk.
. Once the baby comes out, she can't refuse ttt for the baby.
__________________________________________________________________________________________
. CONFIDENTIALITY:
___________________
. The pt has an absolute right to privacy concerning his own medical information.
. The following persons do NOT have a right to any of the medical information of the pt:
-> Relatives, employers, friends & spouses.
-> Other physicians -> U can't release it without the express consent of the pt.
-> Members of law enforcement: U can't release medical information to courts or
police
without a court order or subpoena.
. BREAKING CONFIDENTIALITY TO PREVENT HARM TO OTHERS:
______________________________________________________
. If a pt has a TRANSMISSIBLE disease, such as T.B. or HIV,
. the physician can violate the pt's confidentiality to protect innocent 3rd parties.
. If u have T.B., your doctr can contact your close associates with OUT your consent.
. If u have $, HIV or gonorrhea, your doctor can safely inform others e'OUT your consent.
. The classic ex. is of a pt e' a psychiatric illness who may be planning to harm others.
. The physician has the right to alert the person at risk to prevent harm.
__________________________________________________________________________________________
. END OF LIDE ISSUES:

______________________
______________________
. An adult with capacity can withhold or withdraw any form of therapy.
. If the pt begins ttt, he or she has the right to withdraw any form of ttt.
. The reasons for the withdrawal or withholding of care are not important.
. An advice directive is a set of instructions from an adult pt.
. with capacity directing the care of himself or herslf prior to losing capacity.
. HEALTH CARE PROXY:
_____________________
. The strongest advance directive is a health care proxy.
. The proxy is both a document describing the care the person desire,
. as well as the appointment of an agent to be the decision maker.
. The agent as a decision maker doesn't take hold until the pt loses the capacity.
. If I appoint a proxy but I'm still here, alert & communicative,
. you can't ask the agent for consent for my procedures.
. LIVING WILL:
_______________
. It is a writen document outlining the care desired by the patient.
. If a pt doesn't have a health care proxy, the living will can be very useful.
. If the pt writes out "I never want to be intubated", this is valid.
. If he writes "No heroic measures", this is not valid.
. To be useful, a living will must be clear & precise.
.
.
.
.

If a pt's family members disagree with a living will,


and demand care that contradicts the pt's written wishes,
the best initial step is -> DISCUSS THE MATTER WITH THEM.
If discussion fails o resolve the condition -> consult the hospital's ETHICS COMMITTEE.

. DO NOT RESUSCITATE (DNR) ORDERS:


___________________________________
. It means the refusal of endotracheal intubation & cardiopulmonary resuscitation,
. in the event of the loss of the ability to breathe or the heart stopping.
. A DNR doesn't mean the elimination of testing or medical therapy.
. PATIENT WITH NO CAPACITY & NO ADVANCE DIRECTIVE (PROXY OR LIVING WILL):
__________________________________________________________________________
. This is the most complex & the most common circumstance.
. The care is based on the best understanding of the pt's wishes for himself.
. Family & friends attempt to outline what they heard the pt say he wanted.
. This is not the same as saying "This is what is best for the pt".
. Decisions are based on the best possible understanding of clearly expressed wishes.
. If there is no clear expression of wishes,
. the the weakest basis on which to act is the "best interests of the pt".
. ETHICS COMMITTEE:
____________________
. The ethics committee is used for cases in which the following are true:
-> The pt is not an adult with capacity.
-> There are no clearly stated wishes on the part of the pt.
. Also, the ethics committee is the answer if:
-> the caregivers, such as family, are split or in disagreement about the nature of care.

-> If some family members say "He never wanted to be on a ventilator, ever".
-> and some family members say "He might have wanted a ventilator sometime",
-> then this a case for an ethics committee.
. COURT ORDER:
_______________
. It is the option when all the other options haven't given clarity.
. If their is disagreement after all the other steps, including an ethics committee.
. You don't need a court order if the proxy clearly states wishes or family in agreement.
. If parents refuse to consent to ttt of their child for a non-emergent but ..
. potentially fatal medical case, the physician sh'd seek a court order mandating ttt.
. FLUID & NUTRITION ISSUES:
____________________________
. An adult pt with capacity may refuse all forms of nutrition.
. There is no ethical basis for forcing fluids or nutrition upon a pt.
. If the pt is not an adult with the capacity to understand,
. the proxy or living will can direct the removal of fluid & nutrition,
. provided the pt's clearly expressed wishes while competent stated that:
. "No artificial nutrition be started".
. In absence of clearly stated wishes on the issue fluids & nutrition, they sh'd be given
. PHYSICIAN-ASSISTED SUICIDE & EUTHANASIA:
___________________________________________
. It means providing the pt with the means to end his own life.
. THIS IS ALWAYS WRONG !
. Euthanasia means the physician directly administers the means of ending the pt's life.
. THIS IS ALWAYS WRONG !
. These are not the same as providing pain medications that may end the pt's life.
. It is ethical to give pain medication, even if the only way to relieve pain,
. may result in shortenening of life !
. The primary difference is clear:
-> In physician assisted suicide, the 1ry intent is to end life.
-> With a life shortened by pain medication, the 1ry intent is to relieve suffering.
. FUTILE CARE:
_______________
. There is NO obligation on the part of the physician to provide care that won't work !
. There is NO obligation to provide treatment without possible benifit.
. Ex. A pt with widely metastatic cervical cancer develops renal failure,
. the family members insist that dialysis be started. What do u tell them ??!
-> You don't have to provide dialysis to a person who will certainly die !!
. BRAIN DEATH:
_______________
. You are NOT obliged to provide care for a brain-dead patient.
. Brain death = Dead.
__________________________________________________________________________________________
. REPRODUCTIVE ISSUES:

_______________________
_______________________
. 1 . ABORTION:
________________
. A woman's right to an abortion varies by trimester of pregnancy:
-> 1st trimester -> A woman has UNRESTRICTED right to an abortion.
-> 2nd trimester -> A woman has access, but her rights are LESS CLEAR.
-> 3rd trimester -> NO CLEAR ACCESS to abortion (The fetus is potentially viable).
. N.B. YOU DO NOT NEED THE CONSENT OF THE FATHER FOR THE ABORTION.
. 2 . DONATION OF GAMETES:
___________________________
. Pts have UNRESTRICTED RIGHT to donate sperm & eggs.
. There is no ethical problem with being a PAID DONOR for sperm & eggs.
. Note that one can't be a paid donor for organs, such as the kidneys or the cornea.
__________________________________________________________________________________________
. HIV ISSUES:
______________
. A pt has a right to confidentiality of his HIV status.
. However, this confidentiality can be broken to prtect the uninfected,
. such as sexual & needle-sharing partners.
. No obligation for HIV +ve health care workers to disclose their HIV status.
. This include surgeons.
. A surgeon doesn't have to disclose her HIV status to patient.
. Physicians have the legal right to refuse to treat any patient.
. It is not legal to refuse to take care of HIV +ve persons.
. It is unethical to refuse care to HIV +ve pts simply as they are HIV +ve, BUT .....
. It is legal to do so !
__________________________________________________________________________________________
. DOCTOR PATIENT RELATIONSHIP:
_______________________________
_______________________________
. ACCEPTING A PATIENT:
_______________________
. A physician doesn't have an obligation to accept a patient.
. The need of a person doesn't compel the physician to accept that person as a pt.
. Ex. if there is only 1 neurosurgeon at a hospital & a pt needs neurosurgery,
. this situation does NOT compel the physician to accept the pt.
. Once having accepted a pt, however, the physician can NOT simply abandon the pt.
. The physician has an obligation to inform the pt that he must find another physician,
. and the physician must render care until a substitute caregiver can be identified.
. GIFTS:
_________
. Ethically acceptable -> Small gifts not tied to specific ttt or tests.
. Ethically UNacceptable -> Gifts given e' intention of getting a specific prescription.

. SEXUAL CONTACT:
__________________
. Psychiatrists -> NEVER acceptable.
. Other physicians -> They must end the doctor-patient relationship FIRST !
. ELDER ABUSE:
_______________
. Can be reported even against the will of the patient.
. Doesn't imply a specific age; it has to do e' the FRAGILITY of the pt.
. If the pt is frail & vulnerable, the abuse can be reported even against the pt's will.
. IMPAIRED DRIVERS:
____________________
. Such as pts suffering from a seizure disorder,
. can NOT have their license taken away by a physician.
. Only the department of motor vehicles can remove or restrict a license.
. TORTURE:
___________
. Physician participation in torture, on any level, is always WRONG.
. You can't even agree to certify the patient dead !
__________________________________________________________________________________________
. IMPAIRED PHYSICIANS:
_______________________
. Must be reported to an authority figure.
-> Physicians in training -> Reported to program director ar department chair.
-> Faculty -> Reported to the department chair or the dean of the medical school.
-> Those in practice -> Reported to the state medical board.
. The impairment must involve potential danger to medical care.
. If u c a physician stealing a car, behavior is NOT reportable to the department chair.
. If u c a physician at a bar dancing naked on the table top,
. but her medical performance is not impaired, this is NOT reportable.
__________________________________________________________________________________________
. TIPS & TRICKS:
_________________
_________________
. Tx of Alzheimer's dementia -> Acetylcholinesterase inhibitors (Donepezil, Rivostigmine)
. Pts with homicidal thoughts sh'd be admitted at the psychiatric ward.
. Pt with meningitis -> Admit him against his will & start ttt.
. PASSIVE AGGRESSIVE BEHAVIOR:
. Individual expresses his aggression toward another person,
. with repeated passive failures to meet the other person's needs.
. When dealing with an angry pt, the most appropriate response is to:
. encourage a discussion about the source of feelings.
. ex. You seem to be angry about something, May I ask what is bothering u so I can help?!

. If a pt is interested in alternative therapy, the physician sh'd 1st inquire as to why?


. If a pt refuses ttt after being informed about cancer, he sh'd be asked why u refuse ?
. DISPLACEMENT DEFENSE MECHANISM:
-> Shifting of emotions associated with an upsetting person to a safer alternate object,
-> that represent the original.
-> Ex. Husband angry with his wife, breaks the car she gave to him !
. INTELLECTUALIZATION DEFENSE MECHANISM:
-> Transformation of an unpleasant event into a purely intellectual problem - No emotions ->
Ex. A doctor received the results of his investigations & discovered that has cancer,
-> He went home & surfed the net for the most recent ways of ttt of cancer.
. REACTION FORMATION DEFENSE MECHANISM:
-> Transformation of unwanted thought or feeling into its opposite.
-> REACTION FORMATION is NEITHER a splitting, NOR a dissociation.
. Genito-pelvic pain (Vaginismus) -> Pain with intercourse or attempted penetration.
. Treatment response
-> When a pt demonstrates significant improvement with or without remission.
-> Generally defined as a 50 % reduction in base line of severity.

Dr. Wael Tawfic Mohamed


_________________________

PULMONOLOGY TIKI TAKA


_______________________
. BRONCHIAL ASTHMA:
___________________
___________________
. SHORTNESS OF BREATH (SOB) + EXPIRATORY WHEEZES.
. Severe asthma -> Use of accessory muscles & inability of speaking complete sentence.
. SEVERE ASTHMA EXACERBATION manifestations:
_____________________________________________
-> ++ RR = Hyperventillation.
-> -- in peak flow.
-> -- O2 = Hypoxia.
-> -- pH = Respiratory acidosis.
-> Possible absence of wheezes (To wheeze, one must have air flow!).
. Dx -> Pt with SOB & unclear if the cause is BA:
__________________________________________________
-> Do "PULMONARY FUNCTION TESTS" (PFTs) before & after INHALED BRONCHODILATORS:
-> ++ in FEV1 > 12 % -> Confirmed BA.
. Dx -> Asymptomatic pt now i.e. H/O of intermittent SOB episodes but now he is normal:
________________________________________________________________________________________
-> Do "METACHOLINE STIMULATION TEST":
-> -- in FEV1 in response to synthetic acetylcholine (if the pt has BA).
. Tx -> ACUTE ASTHMA:
______________________
-> INHALED BRONCHODILATORS (SABA) -> ALBUTEROL.
-> BOLUS "Not inhaled" of steroids (Methyl prednisone).
-> INHALED IPRATROPIUM.
-> OXYGEN.
-> Magnesium.
. N.B. Any BA pt. with RESPIRATORY ACIDOSIS & CO2 RETENTION sh'd be placed in the ICU.
-> Persistent resp. acidosis is an indication of INTUBATION & MECHANICAL VENTILLATION.
. The following therapies have "NO BENIFIT" in acute asthma exacerbation:
-> Theophylline - Cromolyn - Montelukast - INHALED steroids - LABA "Salmeterol".
. NON-ACUTE BA:
________________
-> Best initial -> INHALED BRONCHODILATORs (ALBUTEROL).
-> Not controlled -> ADD + INHALED STEROIDs.
-> Not controlled -> ADD + INHALED LABA (SALMETEROL).
.
.
.
.

Extrinsic allergies (HAY FEVER) -> Cromolyn or nedocromil.


High Ig E levels not controlled with Cromolyn -> Omalizumab.
Atopic disease -> Montelukast.
COPD -> Ipratropium.

. N.B. VVVVVVVVVVVVVV. imp. GERD can exacerbate airflow obstruction in asthmatics:


___________________________________________________________________________________
. Due to ++ vagal tone & micro-aspiration of gastric contents into the upper airway.
. Risk factors: Obesity, supine position after meals, laryngitis.
. Manifestations: Change in voice & NOCTURNAL COUGH. (ACE Is lead to day & night cough!).
. Anti-GERD life style modification.
. Give a trial of a proton pump inhibitor (Esomeprazole).
.
.
.
.

GERD is present in 75% of asthma pts & may be the trigger of many cases.
Adult onset asthma with GERD (Worsening syms after meals or with lying down).
Obesity, hoarsness, pharyngitis & laryngitis tend towards GERD.
A trial of proton pump inhibitors (Omeprazole) can be both diagnostic & therapeutic.

. N.B. Efficacy of BETA blockers for mortality in cases of MI & CHF is more important than its
adverse effects e.g. Asthma & COPD.
. N.B. Exercise induced asthma -> Tx with INHALED BRONCHODILARORS prior to exercise.
. N.B. All pts with SOB sh'd 've -> O2 - pulse oximeter - CXR & ABG.
. TREATMENT OF BRONCHIAL ASTHMA DEPENDS ON ITS SEVERITY:
_________________________________________________________
* INTERMITTENT -> CONTINUE CURRENT REGIMEN SABA (B-agonists: ALBUTEROL):
_________________________________________________________________________
. Day time syms < 2 /week.
. Night time awakenings < 2 / month.
. B-agnists < 2 / week.
. Normal PFTs.
. No limitations on daily activities.
* MILD PERSISTENT -> ADD INHALED CORTICOSTEROIDS:
__________________________________________________
. Day time syms > 2 /week.
. Night time awakenings 3-4 / month.
. Normal PFTs.
. MINOR limitations on daily activities.
* MODERATE PERSISTENT -> ADD INHALED LABA (SALMETEROL):
________________________________________________________
. Daily symptoms.
. Weekly Night time awakenings.
. FEV1 <60 - 80 % of predicted.
. Moderate limitations on daily activities.
* SEVERE PERSISTENT -> ADD ORAL PREDNISONE:
____________________________________________
. Symptoms through out the day.
. Frequent night time awakenings.
. FEV1 < 60 % of predicted.
. Severe limitation on daily activity.

. IMPORTANT DRUG SIDE EFFECTS:


______________________________
______________________________
. N.B. The most common adverse effect of INHALED CORTICOSTEROIDS is OROPHARYNGEAL
THRUSH.
.
.
.
.

N.B. The most common adverse effect of "IV" CORTICOSTEROIDS is -- WBCs "NEUTROPHILIA".
Glucocorticoids ++ bone marrow release of of neutrophils.
Glucocorticoids mobilize the marginated neutrophilic pool.
Eosinophils & lymphocytes are decreased.

. N.B. High doses of B2 agonists may develop HYPOKALEMIA !


. Hypokalemia may present as ms weakness, arrhythmia & EKG abnormalities.
.
.
.
.
.

N.B. Theophylline toxicity:


CNS stimulation (Headache, insomnia & seizures).
GIT disturbances (Nausea & vomiting).
Cardiac toxicity (Arrhythmia - Multifocal atrial tachycardia & premature beat).
Dx -> Measure serum theophylline levels.

. INDICATORS OF SEVERE ASTHMATIC ATTACK:


________________________________________
. NORMAL or INCREASED CO2 is the worst sign indicating acute severe attack.
. CO2 retention is due to severe airway obstruction (air trapping) & respirat. ms fatigue
. Speech difficulties.
. Diaphoresis.
. Altered sensorium.
. Cyanosis.
. SILENT lungs.
. ACUTE EPISODES of SOB MANAGEMENT:
___________________________________
-> Oxygen & ABG.
-> CXR.
-> SABA "ALBUTERL" INHALED.
-> IPRATROPIUM INHALED.
-> BOLUS of steroids (Methyl prednisone).-------> VVVVVVVVVVV. imp.
-> Chest, heart, extremity & nerological exam.
-> If fever, sputum & or new infiltrate is present on CXR:
ADD CEFTRIAXONE & AZITHROMYCIN for community acquired pneumonia.
.
.
.
.

N.B. In pts with acute asthma exacerbation, an ELEVATED or even NORMAL PCO2 = RF.
Respiratory failure due to -- respiratory drive due to respiratory muscle fatigue.
ENDO-TRACEAL INTUBATION & MECHANICAL VENTILLATION is MANDATORY.
Add inhaled SABA (Albeterol) & inhaled ipratropium & systemic corticosteroids.

. CHRONIC OBSTRUCTIVE PULMONARY DISEASE (COPD):


_______________________________________________
_______________________________________________
. H/O of long term smoker with ++ SOB & -- exercise tolerance.
. Barrel shaped chest.
. Clubbing of fingers.

. ++ A-P diameter of the chest.


. Loud P2 heart sound (Sign of pulmonary hypertension).
. Edema (Sign of -- Rt ventricular out put).
.
.
.
.
.
.

EKG -> Rt. axis deviation - Rt atrial & ventricular hypertrophy.


CXR -> Elongated heart - Flattenning of the diaphragm due to hyperinflated lungs.
N.B. FLATTENING OF THE DIAPHRAGM ++ The WORK OF BREATHING.
CBC -> ++ Hematocrit & reactive microcytic eryhthrocytosis due to chronic hypoxia.
ABG -> ++ pCO2 & -- pO2 & -- pH (Respiratory acidosis).
Chemistry -> ++ serum bicarbonate as metabolic compensation for respiratory acidosis.

. N.B. (1):
. ABG is critical in acute SOB due to COPD (No other way to assess for CO2 retention !).
. N.B. (2):
. ABG is important to assess for CO2 retention.
. ABG is important to assess for the need for chronic home oxygen based on pO2.
.
.
.
.

N.B. (3):
In moderate & severe cases of COPD, pts may become members of the 50/50 club !!
Both pO2 & pCO2 are around 50s !
Ex -> pH. 7.35 - pCO2 49 - pO2 52 - HCO3 32.

. PULMONARY FUNCTION TESTS in COPD -> OBSTRUCTIVE PATTERN:


___________________________________________________________
-> -- FEV1.
-> -- FVC (Loss of elastic recoil of the lung).
-> -- FEV1/FVC ratio.
-> ++ Total Lung Capacity (++ TLC due to air trapping .. VVVVVVVVVVVV.imp.).
-> ++ Residual Volume.
-> -- Diffusion capacity lung CO (-- DLCO due to destruction of lung interstitium).
-> INCOMPLETE IMPROVEMENT WITH ALBUTEROL (# Asthma).
-> LITTLE OR NO IMPROVEMENT WITH METACHOLINE (# Asthma).
. N.B. A bronchodilator response test to differentiate COPD from BA:
____________________________________________________________________
. Measuring FEV1 before & after adminstration of bronchodilator (B2 agonist).
. Significant improvement in FEV1 (> 15%) after bronchodilator -> Reversibility = Asthma.
. Little or no improvement in FEV1 after bronchodilator -> Irreversibility = COPD.
. N.B. Chronic hypercapneic respiratory failure due to COPD:
____________________________________________________________
. Marked acidosis should be the result of respiratory failure in COPD.
. But .. RENAL TUBULAR COMPENSATION occurs.
. Kidneys ++ HCO3 retention to compensate for ++ CO2 !
. Pts with chronic hypoventillation have gradual ++ in pCO2 -> Respiratory acidosis.
. To compensate, kidneys ++ HCO3 retention & -- Chloride reabsorption instead !
. BOTTOM LINE -> The body compensates for chronic hypercapnea by ++ bicarbonate retention.
. CHRONIC MEDICAL THERAPY of COPD:
___________________________________
. IPRATROPIUM or TIOTROPIUM INHALED (Most effective therapy to reduce syms in COPD).
. SABA ALBUTEROL INHALED.

. Pneumococcal vaccine -> Hepatavalent vaccine (Pneumovax).


. Influenza vaccine yearly.
. Long term home oxygen therapy (If pO2 < 55 or SO2 < 88%).
. N.B. Long term O2 therapy in a pulmonary hypertension pt or HCT > 55% -> PaO2 < 60 mmHg.
. N.B. Both smoking cessation & home oxygen therapy & vaccines lower mortality in COPD.
. N.B. SABA (Albuterol), Anticholinergic (Anti-muscarinic ipratropium),LABA & STEROIDS:
improve symptoms only without -- mortality rate.
. N.B. INHALED ANTI-CHOLINERGICS = INHALED MUSCARINIC ANTAGONISTS - INHALED
IPRATROPIUM
are the most effective in COPD.
. N.B. Cromolyn & Montelukast have no benefit in COPD.
. ACUTE EXACERBATION OF COPD TTT:
_________________________________
. Acute worsening of symptoms in a pt. with COPD.
. Caused by upper respiratory tract infection.
. May be preceided by cough & fever.
. Exam -> Bilateral wheezes.
. ABG -> Respiratory acidosis & hypoxia.
.
.
.
.
.
.

Inhaled bronchodilators (B2 agonists = Albuterol).


Inhaled anti-cholinergics (Ipratropium).
Broad spectrum antibiotics.
INHALED CORTICOSTEROIDS for 2 weeks then tapered gradually.
Smoking cessation.
Oxygen (If pO2 < 55 mmHg or SO2 < 88%).

.
.
.
.
.

N.B. Pts with acute on chronic respiratory failure ttt with high flow supplemental O2,
are at risk for developing worsening HYPERCAPNIA & CO2 NARCOSIS,
due to a combination of reduced alveolar ventillation & ++ dead space ventillation,
causing ventillation perfusion mis-match & -- Hb affinity for CO2.
The goal oxy-hemoglobin saturation in these pts is 90 - 94 % (Not > 95%)!

. NON INVASIVE POSITIVE PRESSURE VENTILLATION (NIPPV):


_______________________________________________________
. Used in acute exacerbations of COPD REFRACTORY to ttt with B-agonist & inhaed steroids.
. Used before intubation to avoid its side effects e.g. infection.
. Recommended in pt e' respiratory distress with a pH<7.35 or pCO2>45mmHg or RR>25/min.
. It is contraindicated in septic, hypotensive or dysrhythmic pts.
. NIPPV will provide more O2 & wash out excess CO2.
. If the pt. is refractory to NIPPV -> Intubate with mechanical ventillation !
. SPONTANEOUS PNEUMOTHORAX (A complication of COPD):
____________________________________________________
. COPD pt presenting with catastrophic worsening of respiratory symptoms.
. Cigarette smoking markedly ++ risk of pneumothorax.
. It leads to chronic airway inflammation & respiratory bronchiolitis.
. The chronic destruction of the alveolar sacs -> Formation of large alveolar blebs.
. which can rupture & leak air into the pleural space.
. presents with acute onset of chest pain & shortness of breath.

. Breath sounds are markedly reduced & hyperresonance to percussion on affected side.
. VVVVVVVVV. IMP. TWO PRIMARY SUB-TYPES OF COPD: CHRONIC BRONCHITIS & EMPHYSEMA:
_________________________________________________________________________________
{A} . COPD with EMPHYSEMA pre-dominance -> (-- DLCO):
______________________________________________________
. Thin pts with severe dyspnea, hyperinflated chest.
. DECREASED vascular markings.
. SEVERE flattening of diaphragm.
. DECREASED DLCO -> due to alveolar destruction.
{B} . COPD with CHRONIC BRONCHITIS pre-dominance -> (NORMAL DLCO):
___________________________________________________________________
. Chronic productive cough for > 3months over 2 consecutive years.
. Due to hypersecretion of mucus & structural changes in the tracheo-bronchial tree.
. PROMINENT vascular markings.
. MILD flattening of diaphragm.
. NORMAL DLCO.
. EXACERBATION OF CONGESTIVE HEART FAILURE:
___________________________________________
. H/O of coronary artery disease -> Lt ventricular dysfunction -> Heart failure.
. Un-controlled hypertension & smoking H/O are risk factors for coronary vascular disease
. LVF -> Tachypnea -> fluid pooling in the lungs -> pleural effusion -> Hypoventillation.
. Hypoventillation -> Hypoxemia.
. Tachypnea -> Hypocapnia & respiratory alkalosis.
. Signs of fluid overload - S3 & S4 gallops & cardiomegaly.
. Lung exam -> Bi-basilar crackles.
. Lung exam -> -- breath sounds at lung bases due to pleural effusion from CHF.
. Wheezing can occasionally be present (Cardiac asthma).
. ABG -> HYPOXIA - HYPOCAPNIA - RESPIRATORY ALKALOSIS (COPD -> Respiratoy ACIDOSIS).
. Dx -> BNP & PCWP.
. ALPHA 1 ANTI-TRYPSIN DEFECIENCY:
__________________________________
__________________________________
. Genetic disorder.
. Liver cirrhosis + COPD.
. NON-smoker.
. Early age < 40 ys NON-smoker having BULLAE at the base of the lungs.
. Dx -> CXR -> Findings of COPD (Bullae - Barrel chest - Flat diaphragm).
. Dx -> Blood test -> -- ALBUMIN & ++ PT (Cirrhosis).
. Dx -> -- Alpha-1 antitrypsin level.
. Tx -> Alpha-1 antitrypsin infusion !
. BRONCHIECTASIS:
_________________
_________________
. Cough - mucopurulent sputum - hemoptysis.
. Profound dilatation of the bronchi.
. due to anatomic defect in the lungs mostly due to infection in childhood.
. Episodes of lung infection with high volume of sputum.
. Hemoptysis & fever may occur.

.
.
.
.

Dx -> CXR -> Dilated bronchi (TRMA TRACKING).


Dx -> CT Chest -> Most accurate test.
Tx -> No curative therapy.
Just ttt the infectious episodes with rotating antibiotics to avoid resistance.

. CYSTIC FIBROSIS:
__________________
__________________
. Young pt.
. Mutation in the Chloride transporter protein CFTR.
. Abnormally thick secretions.
. Affect the respiratory tract - sinuses - pancreas - intestines & reproductive systems.
. Respiratory tract -> Chronic cough e' frequent exacerbations & superimposed infections.
. Most pts develop BRONCHIECTASIS leading to HEMOPTYSIS.
. Pancreas -> Fat malabsorption with bloating & greasy, floating stools.
. Dx -> CT -> Atrophic pancreas with calcifications.
. INTERSTITISAL LUNG DISEASES (ILD):
____________________________________
____________________________________
. Pulmonary fibrosis 2ry to environmental or occupational exposure (Pneumoconiosis).
. Also caused by medications (NITROFURANTOIN & TMP-SMX "BACTRIM").
. If the etiology is unknown (IDIOPATHIC PULMONARY FIBROSIS).
.
.
.
.
.
.

ASBESTOSIS -> Shipyard - Mining - Construction workers - Pipe fitters).


SILICOSIS -> Glass workers - Mining - Sandblasting & Brickyards.
COAL WORKER's PNEUMONIA -> Coal worker !
BYSSINOSIS -> COTTON.
BERYLLIOSIS -> Electronics - Ceramics - Fluorescent & Light bulbs.
PULMONARY FIBROSIS -> Mercury.

.
.
.
.
.
.

Shortness of breath.
"DRY" = NON productive cough & chronic hypoxia.
Dry rales - Bi-basilar end-inspiratory crackles.
Loud P2 (Sign of pulmonary hypertension).
Digital clubbing.
NOOOO FEVER - NOOOO systemic findings.

.
.
.
.
.

Dx
Dx
Dx
Dx
Dx

->
->
->
->
->

CXR -> Interstitial fibrosis & Honeycombing.


CXR -> Pulmonary vascular congestion at the hilum.
CT -> PLEURAL PLAQES ARE PATHOGNOMONIC (Pneumoconiosis)!
Lung biopsy.
PFTs -> ALL MEASURES ARE DECREASED but PROPORTIONATELY.

. PULMONARY FUNCTION TESTS in ILD -> RESTRICTIVE PATTERN:


__________________________________________________________
-> -- FEV1.
-> -- FVC.
-> NORMAL FEV1/FVC ratio.
-> -- TLC.
-> -- RV.
-> -- DLCO (VVVVVVVVVV. imp.).
-> VVVVVVVVVV. imp. -> ILF -> +++ A-a gradient !

. Tx -> No specific therapy to reverse any of ILD forms.


. If the lung biopsy shows an inflammatory infiltrate, a trial of steroids is used.
. The only form of ILD that responds to steroids is BERYLLIOSIS (Granulomatous disease).
. N.B. The most common type of cancer in ASBESTOSIS is LUNG CANCER not mesothelioma.
. N.B. ILD may be complicated by COR PULMONALE:
-> peripheral edema - Hepatojugular reflex - Jugular venous distension - Rt ventr. heave.
. COMPARISON BETWEEN PFTs in COPD & ILD:
________________________________________
. COPD -> OBSTRUCTIVE PATTERN & ILD -> RESTRICTIVE PATTERN:
___________________________________________________________
. PFTs ___________ COPD __________ ILD
.
.
.
.
.
.

FEV1 ___________ ---- __________ -FVC ___________ -- __________ -FEV1/FVC _______ -- __________ NORMAL
TLC ____________ ++ __________ -RV _____________ ++ __________ -DLCO ___________ -- __________ --

. BRONCHILOTIS OBLITERANS ORGANIZING PNEUMONIA BOOP / CRYPTOGENIC ORGANZING


PNEUMONIA COP:
__________________________________________________________________________________________
__________________________________________________________________________________________
. Inflammation of the small airways with a chronic alveolitis of an unkown origin !
. Associated with Rheumatoid arthritis.
. Resembles ILD but more acute presentation (Over weeks to months).
. (SOB - Cough - rales) + FEVER + MALAISE + MYALGIA.
. No occupational exposure in history !
. CXR -> Bilateral PATCHY infiltrates.
. CT -> Inerstitial disease & alveolitis.
. Most accurate -> OPEN LUNG BIOPSY !
. Tx -> Steroids (No response to antibiotics).
. COMPARISON BETWEEN BOOP/COP & ILD:
____________________________________
. BOOP/COP _______________________________ . ILD
.
.
.
.

Fever- myalgia - malaise _______________ . NO.


Presents over days to weeks ____________ . 6 months or more of symptoms.
PATCHY infiltrates _____________________ . INTERSTITIAL infiltrates.
STEROIDs EFFECTIVE _____________________ . Only BERYLLIOSIS may respond to steroids.

. SARCOIDOSIS:
______________
. AFRICAN AMERICAN WOMEN.
. Age < 40s.

.
.
.
.
.
.
.
.

SOB - Cough & fatigue over a few weeks to months.


Lung - > Rales.
Eye -> ANTERIOR UVEITIS (Sight threatening).
Neural -> Facial palsy (7th cranial nerve).
Skin -> ERYTHEMA NODOSUM.
Joint -> Polyarthralgia.
Heart -> RESTRICTIVE CARDIOMYOPATHY.
HYPERCALCEMIA (2ry to Vit.D production by the granulomas).

. Dx -> Best initial test -> CXR.


. CXR -> BILATERAL HILAR LYMPHADENOPATHY & diffuse interstitial infiltrates.
. Dx -> Most accurate test -> LUNG or LN biopsy -> NON-CASEATING GRANULOMA.
. Dx -> ++ Ca & ++ ACE levels
. Dx -> BAL -> ++ helper cells.
. Tx -> STEROIDs.
. SYSTEMIC SCLEROSIS:
_____________________
. Pulmonary symptoms (Due to interstitial fibrosis).
. Dysphagia.
. Raynaud's phenomenon.
. Hypertension.
. Telangiectasia.
. PULMONARY HYPERTENSION:
_________________________
. Mean pulmonary arterial blood pressure > 25 mmHg.
. Overgrowth & obliteration of pulmonary vasculature -> -- outflow of the Rt ventricle.
. SOB more often in young women.
. May be 2ry to (MS - COPD - PCV - ILD & chronic pulmonary emboli).
. Physical findings (Loud P2 - TR - RV heave).
. Dx -> TRANS-THORACIC ECHOCARDIOGRAM (TTE) -> Rt atrial & ventricular hypertrophy.
. Dx -> EKG -> Rt axis deviation.
. Dx -> CXR -> Pulmonary arteries enlarg. & RVE & tapering of distal vessels (Pruning).
. Most accurate -> RIGHT HEART SWAN GANZ CATHETERIZATION -> ++ PULMONARY ARTERY
pressure.
. Tx -> BOSENTAN -> Endothelial inhibitor.
. May be complicated by RVF (Rt ventricular heave - JVD - Tender hepatomegaly - Ascites).
. COR PULMONALE:
________________
. Rt sided heart failure due to pulmonary disease.
. Jugular venous distension.
. Right sided S3 gallop.
. Right ventricular heave.
. Hepatomegaly.
. Ascites.
. Dependent LL edema.
. Most commonly caused by COPD (Flattened diaphragm - prominent pulmonary vessels on
CXR)
. CXR -> Prominent right ventricle & pulmonary artery.
. PULMONARY EMBOLISM:

_____________________
_____________________
. PERFUSION DEFECT & NO VENTILLATION DEFECT.
. ++++++++++++++++++++++++++++ A-a gradient.
.
.
.
.

SUDDEN onset SOB + CLEAR LUNGs.


Risk factors of DVT (Immobility - Malignancy - Trauma - Surgery - Thrombophilia).
H/O of recent orthopedic surgery followed by bed rest.
No specific physical finding for PE.

. MODIFIED WELL'S CRITERIA for PRE-TEST PROPABILITY of PE:


___________________________________________________________
-> Score + 3 points (Clinical signs of DVT).
-> Score + 1.5 points (Prev PE/DVT - HR>100 - Recent surgery <4wks - Immobilization>3ds)
-> Score + 1 point (Hemoptysis - cancer).
-> Total score for clinical propability (< 4 -> PE UN-likely .. > 4 -> PE likely).
.

. Clinical assessment for pulmonary embolism


.____________________________________________
.< Modified Well's criteria>
.____________________________
|
.________________________________
.|
.|
. PE UN-likely
. PE likely
.______________
.___________
.|
.|
. D-dimer assay
.|
._______________
.|
.|
.|
.___________________
.|
.|
.|
.(< 500 ng/ml)
.(> 500 ng/ml)-->. CT PULMONARY ANGIOGRAPHY
.|
.____________________________
. PE EXCLUDED
.|
.(-ve = PE EXCLUDED BUT +ve = PE CONFIRMED)

. INITIAL DIAGNOSTIC TESTS -> CXR - EKG - ABG.


. CONFIRMATORY TESTS -> Spiral CT - V/Q scan - LL Doppler - D-Dimer.
. MOST ACCURATE TEST -> PULMONARY ANGIOGRAPHY = CHEST CT ANGIOGRAPHY with IV
CONTRAST.
. 1 . CXR:
___________
. Most common result -> NORMAL.
. Most common abnormailty -> Atelectasis.
. Wedge shaped infarction & pleural humps are rare.
. 2 . EKG:
___________
. Most common showing -> SINUS TACHYCARDIA.
. Most common abnormality -> NON-SPECIFIC ST-T WAVE CHANGES.

. Right axis deviation & Rt BBB are rare.


. 3 . ABG:
___________
. HYPOXIA -> ++ A-a gradient.
. Mild respiratory alkalosis.
. 4 . SPIRAL CT -> TEST OF CHOICE if the CXR is ABNORMAL:
__________________________________________________________
. Standard to confirm the presence of a pulmonary embolus.
. Excellent if +ve being specific.
. Not specific as it can miss some emboli if they are small & in the periphery.
. Chest CT showing a WEDGE SHAPED infarction is PATHOGNOMONIC for pulmonary
embolism.
. 5 . VENTILLATION PERFUSION V/Q SCAN -> TEST OF CHOICE if the CXR is NORMAL:
______________________________________________________________________________
. PERFUSION DEFECT with NO VENTILLATION DEFECT.
. NORMAL V/Q scan excludes pulmonary embolism.
. 6 . LOWER EXTREMITY DOPPLER:
_______________________________
. If +ve -> No further tests are needed to confirm PE.
. The problem is that 30 % of PEs originate in pelvic veins, so the LL Doppler is NORMAL.
. So it has low sensitivity i.e. can't exclude PE.
. 7 . D-DIMER TESTING = FIBRIN SPLIT PRODUCTS TESTING:
_______________________________________________________
. SINGLE TEST TO EXCLUDE PE.
. Very sensitive test with poor specificity.
. D-DIMER -> NEGATIVE -> NO PULMONARY EMBOLISM.
. D-DIMER -> Not specific -> May be other causes.
. The best use of D-DIMER test is in a pt with LOW propability of PE,
. & u want a single test to exclude PE !!
. 8 . ANGIOGRAPHY -> SINGLE MOST ACCURATE TEST FOR PE:
______________________________________________________
. ANGIOGRAPHY = CHEST CT ANGIOGRAPHY WITH INTRAVENOUS CONTRAST (VVVVVV. imp.).
. INVASIVE with risk of death (0.5%).
. MANAGEMENT of PULMONARY EMBOLISM:
___________________________________
{1} HEPARIN & OXYGEN -> Standard of care.
{2} Warfarin -> Sh'd be used at least for 6 months after Heparin.
{3} IVC filter -> in case of contraindication to Anticoagulants (e.g. hematoma).
{4} Thrombolytics -> used in pts who r hemodynamically UN-stable (e.g. hypotension).
{5} Embolectomy is rarely done (High risk of death).
.
.
.
.

N.B. When the case so clearly suggests a pulmonary embolism,


i.e. Pt presenting with sudden onset of SOB & clear lungs with H/O of major surgery,
the 1st thing to do is CXR & ABG followed by HEPARIN.
Don't wait the results of V/Q scan or spiral CT to start heparin !!

. When there is a contraindication to anticoagulation e.g. hematoma,


. Don't use heparin ! Place an IVC filter.

. For anticoagulation, "Un-fractionated" heparin is preferred in pts with -- GFR !


. LMW heparin (Enoxaparin) can't be given as it causes severe renal insuffeciency.
.
.
.
.

Warfarin can be thrombogenic without heparin as a bridge !


It sh'd be given after initiating heparin with PTT goal 1.5-2 times of normal.
Warfarin takes up to 5-6 days to reach its therapeutic level.
After reaching therapeutic INR level (2-3), heparin can be stopped.

.
.
.
.
.

VVVVVV. imp. N.B. A PROGRESSING CLOT in a pt with sub-therapeutic INR (ex. 1.2),
requires BRIDGING HEPARIN until the INR is therapeutic (2-3),
Example .. A pt recently hospitalized for LL DVT then discharged,
After 5 days, U$ reveals popliteal vein thrombosis extending into the deep femoral vein
So .. U sh'd START INTRAVENOUS UNFRACTIONATED HEPARIN & CONTINUE WARFARIN.

. The proximal deep leg veins are the most common source of symptomatic pulmonary
embolism
. Less common sources of emboli include calf, pelvic & upper evtremity veins & Rt heart.
. "Factor V Leiden" is the most common genetic disorder causing hypercoagulability & DVT.
.
.
.
.

N.B. Acute massive pulmonary embolism can present initially with syncope & shock.
e.g. sudden loss of consciousness at work, BP:80/40 & HR:120/min with cold clammy skin.
Rt heart catheterization -> ++ Right atrial & pulmonary artery pressures.
Normal PCWP Pulmonary artery capillary wedge pressure.

.
.
.
.

N.B. Massive pulmonary embolism usually presents with signs of low arterial perfusion,
Hypotension, acute dyspnea, pleuritic chest pain, tachycardia & syncope.
The thrombus ++ pulmonary vascular resistance & Rt ventricular pressure,
causing Rt ventricular hypokinesis -> Rt ventricular dilatation.

. APPROACH TO MANAGEMENT OF PATIENT WITH SUSPECTED PULMONARY EMBOLISM:


______________________________________________________________________
______________________________________________________________________
. Stabilize the pt with Oxygen & IV fluids
._________________________________________
.|
. CONTRAINDICATIONS to Anticoagulate ?
._____________________________________
.|
._______________________________________________
.|
.|
. YES = Diagnostic tests to evaluate for PE
. NO = MODIFIED WELL's CRITERIA
.__________________________________________
._______________________________
.|
.|
._____________________
._______________
.|
.|
.|
.|
. +ve PE
. -ve PE
. PE Un-likely . PE likely
.________
.________
.______________ .___________
.|
.|
.|
.|
. IVC FILTER
. No further tests
.| . START anticoagulation
.| .______________________
.|
.|

. D-DIMER TESTING for PE


.________________________
.|
._____________________________________________________________________
.|
.|
. +ve
. -ve
. Start or continue anticoagulation,
. STOP anticoagulation
. consider surgery or thrombolysis if indicated.

. PLEURAL EFFUSION:
___________________
___________________
. Best initial test -> CXR.
. Decubitus films (Pt lying on one side) sh'd be done next to assess the fluid mobility.
.
.
.
.
.
.

Most accurate test -> THORAC-CENTESIS.


Un-diagnosed pleural effusion is best evaluated with THORACOCENTESIS,
To detect whether it is a transudate or an exudate.
Except in pts with clear-cut evidence of congestive heart failure,
Associated fluid overgain, pedal edema & bilateral lung base crackles.
Diuretics & echo sh'd be done not thoracocentesis.

. COMPARISON BETWEEN EXUDATE & TRANSUDATE (VVVVVVVVVVV. imp.):


_______________________________________________________________
. EXUDATE PLEURAL EFFUSION ______________________ . TRANSUDATE PLEURAL EFFUSION
. Cancer & infection & Pulmonary embolism _______ . Congestive heart failure & cirrhosis.
. High ptn level > 50 % of serum level __________ . Low ptn level < 50 % of serum level.
. High LDH level > 60 % of serum level __________ . Low LDH level < 60 % of serum level.
. LDH > 2/3 upper limit of normal serum LDH (250) . < 2/3 !
. pH > 7.3 (Normal 7.6) ______________________ . pH < 7.3 (++ acid prod. by bacteria).
. NO CHANGE IN GLUCOSE OR AMYLASE LEVELS IN BOTH TYPES !
.
.
.
.
.

Tx -> Small pleural effusions don't need therapy !


Diuretics can be used for those caused by congestive heart failure.
Larger effusions esp. those caused by empyema -> Drain by CHEST TUBE.
Large recurrent effusion from an un-correctable cause -> PLEURODESIS.
If pleurodesis failed -> Decortication.

. N.B. 1 -> EXUDATE -> MALIGNANCY OR INFECTION -> ++ Capillary permeability.


. N.B. 2 -> TRANSUDATE -> CONGESTIVE HEART FAILURE -> ++ HYDROSTATIC PRESSURE.
. N.B. 3 -> TRANSUDATE -> CIRRHOTIC LIVER FAILURE -> -- PLASMA ONCOTIC PRESSURE.
. COMPLICATED PARA-PNEUMONIC EFFUSION CRITERIA:
________________________________________________
. Exudative pleural effusion.
. Pleural fluid acidosis.
. Low pleural fluid glucose < 60 mg/dl(High metabolic activity of leukocytes or bacteria)

. INDICATIONS OF TUBE THORACOTOMY in PARA-PNEUMONIC FLUID ACCUMULATION:


________________________________________________________________________
1- pH of the pleural fluid < 7.2.
2- Glucose < 60 mg/dl.
. EMPYEMA = INFECTION OF THE PLEURAL SPACE:
___________________________________________
. Due to untreated pneumonia cased by bacterial invasion of a pleural effusion.
. or contamination of the pleural space by rupture of a lung abscess.
. Others: Bronchopleural fistula - penetrating trauma - thoracotomy or ruptured viscus.
. May complicate hemothorax, the residual blood is an excellent medium for bacteria.
. A mixed aerobic & anaerobic bacterial infection (Strept. - Staph. - Klebsilella).
. Low grade fever.
. Dx -> CT scan.
. Tx -> Drainage & antibiotics.
. Tx -> SURGERY (If localized - complex or having thick rim).
. SLEEP APNEA:
______________
. Obese pt complaining of daytime somnolence.
. The pt's partener will report severe snoring.
. Hypertension - Headache - Erectile dysfunction & fat neck.
. Obstructive sleep apnea from fatty tissues of the neck blocking breathing.
. Central sleep apnea due to -- respiratory drive from the CNS.
. Dx -> NOCTURNAL POLYSOMNOGRAPHY (GOLD STANDARD OF DIAGNOSIS).
. Mild sleep apnea -> 5 - 20 apneic periods per hour.
. Severe sleep apnea -> > 30 apneic periods per hour.
. Tx of obstructive sleep apnea -> Weight loss & CPAP:Continous positive airway pressure
. If not effective -> Uvulo-palato-pharyngo-plasty.
. Tx of central sleep apnea -> Avoid alcohol & sedatives.
. Medroxyprogesterone -> Central respiratory stimulant.
. OBESITY HYPOVENTILLATION $YNDROME (OH$) = PICKWICKIAN $YNDROME:
_________________________________________________________________
. Severe obesity (Greater then 150% of ideal body weight -> BMI = 55!).
. Thin neck & hypersomnolence.
. Obesity -> Distant heart sounds & Low voltage QRS complexes on EKG.
. Alveolar hypoventillation during WAKEFULLNESS !
. Polycythemia secondary to alveolar hypoventillation.
. ABG -> Hypoxemia & Hypercapnia & Respiratory acidosis.
. Due to DECREASED LUNG & CHEST WALL COMPLIANCE ! (Not resp. ms weakness xxx).
. Tx -> Weight loss - Ventilator support - Oxygen - Avoid supine posture during sleep.
. COMPLICATIONS of long-standing OSA or OH$:
____________________________________________
. Pulmonary hypertension with cor pulmonale.
. Secondary erythrocytosis.
. Hypoxia, chronic hypercapnea & respiratory acidosis (Due to chronic hypoventillation).

. N.B. Chronic hypercapneic respiratory failure due to OH$:


___________________________________________________________
. Marked acidosis should be the result of respiratory failure in OH$.
. But .. RENAL TUBULAR COMPENSATION occurs.
. Kidneys ++ HCO3 retention to compensate for ++ CO2 !
. Pts with chronic hypoventillation have gradual ++ in pCO2 -> Respiratory acidosis.
. To compensate, kidneys ++ HCO3 retention & -- Chloride reabsorption instead !
. HOW TO DIFFERENTIATE BETWEEN OBSTRUCTIVE SLEEP APNEA & OBESITY
HYPOVENTILLATION $:
____________________________________________________________________________________
____________________________________________________________________________________
.{1}. OBSTRUCTIVE SLEEP APNEA:
_______________________________
. Air flow is impeded by AIRWAY OBSTRUCTION,
. due to POOR ORO-PHARYNGEAL TONE.
. NORMAL ABG !
.{2}. OBESITY HYPOVENTILLATION $:
__________________________________
. Air flow is impeded by diminished expansion of chest & abdominal wall due to obesity.
. ABG -> HYPO-ventillation -> Chronic hyoxia & hypercapnia.
. ALLERGIC BRONCHO-PUMONARY ASPERGILLOSIS (ABPA):
_________________________________________________
. Asthmatic pt with worsening asthma symptoms.
. Coughing of brownish mucous plugs with recurrent infiltrates.
. Peripheral eosinophilia.
. ++ Ig E levels.
. Central bronchiectasis may be seen.
. Tx -> ORAL (Not inhaled) corticosteroids.
. PULMONARY EDEMA:
__________________
. Hypoxia - SOB - Tachypnea.
. CXR -> Diffuse alveolar infiltrates.
. May be cardiogenic (LVF) or non cardiogenic (ARD$).
. Differentiate bet. the two types using pulmonary capillary wedge pressue (PCWP).
. PCWP > 18 -> Cardiogenic pulmonary edema.
. PCWP < 18 -> Non cardiogenic = ARD$.
. ACUTE RESPIRATORY DISTRESS $YNDROME (ARD$) = NON-CARDIOGENIC PULMONARY
EDEMA:
_______________________________________________________________________________
. Sudden severe respiratory failure resulting from diffuse lung injury,
. secondary to a number of overwheming systemic injuries e.g.
. Sepsis - Aspiration of gastric contents - shock - severe infections,
. Lung contusion - trauma - toxic inhalation - drowning - pancrestitis - burns.
. CXR -> Diffuse patchy infiltrates.
. NORMAL wedge pressure -> i.e. < 18.
. pO2/FiO2 ratio < 200.
. Tx -> Ventilatory support with low tidal volume of 6 ml/kg.

.
.
.
.
.
.

PEEP to keep the alveoli open. (Sh'd reach 15 cm H2O).


++ FiO2 (Never exceed 60 %).
Prone positioning of the pt's body.
Possible use of diuretics & +ve inotropes such as dobutamine.
Transfer the pt to the ICU if not already there !
STEROIDS ARE NOTTTTTTT EFFECIVE !

. ARD$ pts on MECHANICAL VENTILLATION:


_______________________________________
. Mechanical ventillation includes two components FiO2 & PEEP.
. FiO2 = Fraction of inspired oxygen.
. PEEP = Positive end expiratory pressure.
. ++ FiO2 -> Improves oxygenation.
. PEEP -> Prevent alveolar collapse.
. Arterial pO2 is influenced by FiO2 & PEEP.
. Arterial pCO2 is influenced by RR & TV.
. When you find a given ABF with pO2 55 mmHg = Low oxygenation. & FiO2 = 70%
. So .. You should add PEEP 1st to improve oxygenation.
. Don't decrease the FiO2 before adding PEEP or you will worsen the condition !
. When you find a given ABG with pO2 105 mmHg = TOXIC OXYGEN LEVEL.
. You should decrease the fractionated oxygen level FiO2 to non toxic value < 60% !
. PEEP may be ++ as needed to maintain adequate oxygenation but avoid tension pneumothx.
.
.
.
.
.
.
.

When you are given an ABG with respiratory alkalosis (pH > 7.4) & hypocapnia (--CO2),
With appropriate tidal volume < 6 ml/kg (pt. 70 kg -> 420 ml).
With appropriate FiO2 (Ex. 40 %),
With appropriate PEEP (Ex. 5 cm H2O),
Look at the respiratory rate (If it is high e.g. 18),
This respiratory alkalosis will be due to HYPER-ventillation.
So .. Decreasing the respiratory rate is the most appropriate step.

.
.
.
.

Ventillation = RR x TV.
Respiratory alkalosis results from hyperventillation.
The RR sh'd be lowered.
-- in TV can trigger ++ in RR -> worsening the condition.

. POSITIVE END-EXPIRATORY PRESSURE (PEEP):


__________________________________________
. Used in cases of hypoxemic respiratory failue e.g. ARD$ & cardiogenic edema.
. Helps to maintain air way pressure above atmospheric pressure at the end of expiration.
. Complications -> Alveolar damage - tension pneumothorax & hypotension.
. Sudden SOB - --BP & ++ HR - tracheal deviation & unilateral absence of breath sounds.
. SWAN-GANZ (PULMONARY ARTERY) CATHETERIZATION:
_______________________________________________
-> Hypovolemic shock -> -- COP & -- CPWP & ++ TPR.
-> Cardiogenic shock -> -- COP & ++ CPWP & ++ TPR.
-> SEPTIC SHOCK ------> ++ COP & -- CPWP & -- TPR.
. COP -> LOW except in septic shock (High).
. PCWP -> LOW except in cardiogenic shock (High).
. TPR -> HIGH except in septic shock (Low).

. PCWP is NORMAL in ARD$. (VVVVVVVVVV. imp.).


. PCWP is NORMAL in PE. (VVVVVVVVVVVV. imp.).
. PNEUMONIA:
____________
. Fever, cough & sputum.
. Severe illness -> SOB.
. COMMUNITY ACQUIRED PNEUMONIA (CAP) -> PNEUMOCOCCUS.
. HOSPITAL ACQUIRED PNEUMONIA (HAP) -> Gram -ve bacilli.
. PPI ++ the risk of hospital acquired pneumonia.
. Pts > 65ys with chronic dis. of lungs or liver are more prone to respiratory failure.
. DM - HIV - Steroid use - Asplenia -> Worse prognosis.
. ELDERLY HYPOXIC PT WITH OR WITHOUT FEVER SHOUL BE ADMITTED !
. Dx -> Best initial test -> CXR.
. Dx -> Most accurate test -> Sputum gram stain & culture.
. N.B. All pts with suspected pneumonia sh'd have a CXR done as the 1st step.
. Antibiotics sh'd be adminstered ASAP without waiting for sputum gram stain or culture.
. Tx -> OUT-PATIENT PNEUMONIA:
_______________________________
-> Macrolide (Azithromycin - Doxycycline - Clarithromycin).
-> Respiratory fluoroquinolone (Levofloxacin - Moxifloxacin).
. Tx -> IN-PATIENT PNEUMONIA:
______________________________
-> Ceftriaxone & Azithromycin.
-> Fluoroquinolone as a single agent.
. REASONS TO HOSPITALIZE pts with pneumonia:
_____________________________________________
. Hypotension -> SBP < 90 mmHg.
. Tachycardia -> HR > 125/min.
. Temperature -> T -> 104 F.
. Respiratory rate -> RR > 30/min.
. PO2 < 60 mmHg.
. pH < 7.35
. BUN > 30 mg/dl.
. Na < 130.
. Glucose > 250.
. Confusion.
. Age > 65 ys or older.
. Co-morbidities eg. cancer, COPD, CHF & RF or liver disease.
. HYPOXIA & HYPOTENSION as single factors are a reason to hospitalize !
. Tx -> VENTILLATOR ASSOCIATED PNEUMONIA (VAP):
________________________________________________
. VAP -> Fever - Hypoxia - New infiltrate & ++ secretions.
-> Imipenim - Cefepime or Piperacillin/Tazobactam.

-> Gentamycin & Vancomycin.


. INDICATIONS OF TUBE THORACOTOMY in PARA-PNEUMONIC FLUID ACCUMULATION:
________________________________________________________________________
1- pH of the pleural fluid < 7.2.
2- Glucose < 60 mg/dl.
. SPECIFIC ASSOCIATIONS:
_________________________
* Recent viral infection -> Staphylococcus.
* Alcoholics -> Klebsiella.
* GIT syms & confusion -> Legionella.
* Young healthy pts -> Mycoplasma.
* Animal contact -> Coxiella Burnetii.
* Arizona construction workers -> Coccidioidmycosis.
* HIV with < 200 CD4 cells -> Pneumocystis carinii PCP.
. MYCOPLASMA PNEUMONIAE:
________________________
. Most common cause of atypical pneumonia.
. Non productive i.e. dry cough.
. Many extra-pulmonary symptoms (Headache - sore throat - skin rash).
. ERYTHEMA MULTIFORME -> Dusky red TARGET shaped skin lesions on extremities.
. CXR -> Lower lobe interstitial infiltrates.
. No cell wall (Only polymorphnuclear cells will appear on gram stain).
. MYCOBACTERIAL PNEUMONIA:
__________________________
. HIV pts have a higher risk of reactivation of tuberculosis.
. Non specific symptoms (Cough - Weight loss - Fatigue - Low grade fever & Night sweats).
. CXR -> UPPER LOBE INFILTRATES WITH CAVITATION.
. ASPIRATION PNEUMONIA = ANAEROBIC PNEUMONIA:
_____________________________________________
. Impaired swallowing due to IMPAIRED EPIGLOTTIC REFLEX is the most imp. predisp. factor.
. Aspiration of oro-pharyngeal secretions.
. May be a complication of upper GI endoscopy.
. Usually caused by ANAEROBES & Streptococcal viridans.
. Advanced age, poor dentition, dementia, alcohol addiction are predisposing factors.
. Pt presents with systemic syms e.g. fever & malaise & FOUL SMELLING SPUTUM.
. Tx -> CLINDAMYCIN.
. KLEBSIELLA PNEUMONIA = FRIEDLANDER's PNEUMONIA:
_________________________________________________
. Gram -ve bacilli.
. More associated with ALCOHOLICS & immunocomprized pts with neutropenia.
. Mechanism -> Colonization in the oropharynx followed by microaspiration of secretions.
. Mostly affect the UPPER lobes.
. produce CURRANT JELLY sputum.
. Sputum culture -> Mucoid colonies.
. PNEUMOCYSTIS CARINII PNEUMONIA (PCP):
_______________________________________
. Almost exclusively in AIDS pts with CD4 count < 200.
. The HIV pt is usually not on prophylaxis for PCP!

.
.
.
.
.
.
.
.
.
.
.
.
.
.

Immunocompromized pt due to chemotherapy.


Dyspnea on exertion, dry cough & fever.
Dx -> Best initial test -> CXR -> Bilateral interstitial infiltrates (CHARACTERISTIC).
Dx -> ABG -> Hypoxia & ++ A-a gradient. (VVVVVVV imp.).
Dx -> ++ LDH level (Normal LDH level excludes PCP).
Dx -> Most accurate test -> BRONCHO-ALVEOLAR LAVAGE. (VVVVVVVVV. imp.).
Dx -> Sputum stain -> if +ve -> Confirm PCP & if -ve -> Bronchoscopy.
Tx -> Best initial therapy for treatment & prophylaxis -> TMP-SMX.
If PCP is severe (pO2 < 70 or A-a gradient > 35) -> Add STEROIDS to -- mortality.
If there is toxicity from TMP-SMX (Rash - BM depression) -> PENTAMIDINE or Primaquine.
If the pt is African American with G6PD (Bite cells on smear) -> Don't give Primaquine.
For PCP prophylaxis -> TMP-SMX .. if there is a rash or neutropenia -> Atovaquone.
If CD4 count is ++ & maintained above 200 for several months -> Stop prophylaxis.
But, NEVER to stop the anti-retroviral medications against HIV !

. LEGIONNAIRE's DISEASE:
________________________
. H/O of recent TRAVEL or trip (BAHAMAS).
. Linked to cruise ship & hotel water supplies.
. HIGH GRADE FEVER > 39 c.
. GIT symptoms (Nausea & vomiting & loose stools).
. Mild ++ LFTs.
. HYPONATREMIA (PATHOGNOMONIC for LEGIONELLA).
. CXR -> Focal lobular consolidation.
. Gram -ve stain rod & stains poorly (Intracellular organism).
. So.. Gram stain will show many neutrophils but no organisms is chracteristic.
. Most accurate test -> Urine antigen test.
. Tx -> AZITHROMYCIN or Levofloxacin.
. N.B. ACUTE PNEUMONIA WITH CONSOLIDATION & PHYSILOGIC SHUNT:
______________________________________________________________
. -- Breath sounds, ++ Tactile vocal fremitus.
. Alveoli of the affected lung become filled with exudative fluid & cellular debris.
. These alveoli may have persistent blood flow to areas with impaired ventillation.
. Leading to a physiologic intra-pulmonary shunt & arterial hypoxemia.
. Positioning of the pt. with the affected lung in dependent position can worsen the case
. i.e. his SO2 will drop for example from 94% when lying on one side to 84% on other side
. RECURRENT PNEUMONIA:
______________________
. {A} INVOLVING SAME REGION OF THE LUNG:
_________________________________________
.1. Local anatomic obstruction:
________________________________
.. Bronchial compression (Neoplasm).
.. Bronchial obstruction (Bronchiectasis - Retained FB).
.2. Recurrent aspiration:
__________________________
.. Seizures.
.. Ethanol or drug use.
.. GERD.

. {B} INVOLVING DIFFERENT REGION OF THE LUNG:


______________________________________________
. Sino-pulmonary disease (Cystic fibrosis).
. Non-infectious (BOOP).
. Immunodefeciency (HIV - Leukemia - --immunoglobulins).
. BRONCHOGENIC CARCINOMA is the most common cause of recurrent pneumonia in same
region.
. Associated H/O of old age & prolonged smoking H/O
. Dx -> CT chest. (If CT is -ve -> Bronchoscopy).
. HYPERSENSITIVITY PNEUMONITIS (HP):
____________________________________
. Inflammation of the lung parenchyma caused by antigen exposure.
. Ex: Fancier's lung -> Inhalation of aerosolized bird droppings.
. Ex: Farmer's lung -> Inhalation of molds associated with farming.
. Acute episodes of cough, breathlessness, fever & malaise within 4-6 hs of Ag exposure.
. Chronic exposue may lead to weight loss, clubbing & honey-combing of the lung.
. The cornerstone of HP management is AVOIDANCE OF THE RESPONSIBLE ANTIGEN !
. TUBERCULOSIS (T.B):
_____________________
. Immigrants - HIV - Homeless - Prisoners & Alcoholics.
. Most important epidemiologic factor is FOREIGN BORN INDIVIDUAL (Not US born: MEXICO!).
. Fever - cough - sputum - weight loss & night sweats.
. Dx -> CXR & Sputum acid fast stain & culture to confirm TB.
. If culture is +ve -> Start 6 months course of ANTI-TUBERCULOUS THERAPY.
. ISONIAZID 6 m - RIFAMPIN 6m - PYRAZINAMIDE & ETHAMBUTOL stop after 2 months.
. All of them can lead to liver toxicity.
. TB medications sh'd be stopped if the transaminases raised up to 5 times of normal.
.
.
.
.

Isoniazid -> Peripheral neuropathy (Give Vit.B6).


Rifampin -> Red colored bodily secretions.
Pyrazinamide -> Hyperuricemia.
Ethambutol -> Optic neuritis.

. Conditions need ttt > 6ms: Osteomyelitis, Meningitis, Miliary - cavitary TB & pregnancy
. LATENT T.B.
_____________
. PPD -> PURIFIED PROTEIN DERIVATIVE TEST:
___________________________________________
. PPD is a screening test for high risk groups.
. POSITIVE TEST IF:
-> 5 mm -> Close contacts, steroid users, HIV +ve.
-> 10 mm -> Homeless - Immigrants - Alcoholics - Health care workers & prisoners.
-> 15 mm -> Those without any risks.
. If PPD is +ve -> Proceed as follows:
______________________________________
. CXR -> to make sure that occult active disease hasn't been detected.
. If CXR is abnormal -> Sputum staining for TB is done.

. If sputum staining is +ve -> Give full dose 4 drug therapy.


. ISONIAZID alone is used for 9 months to treat a +ve PPD.
. It -- the risk of developing TB from 99% to 1%.
. Once a PPD is +ve, the test sh'd never be repeated.
. RHINITIS:
___________
{A} ALLERGIC RHINITIS:
_______________________
. Watery rhinorrhea & sneezing with more prominent eye symptoms.
. Early age of onset.
. Identifiable trigger (animals - environmental exposure).
. Usually seasonal symptoms but can be persistent throughout year.
. Nasal mucosa can be normal, pale blue or pale on exam.
. Associated with allergic disorders e.g. eczema & asthma.
. Tx -> Allergen avoidance.
. Tx -> Topical intra-nasal glucocorticoids.
{B} NON-ALLERGIC RHINITIS = VASOMOTOR RHINITIS:
________________________________________________
. Nasal congestion - Rhinorrhea - Postnasal discharge (postnasal drip = dry cough).
. Late age of onset > 20 ys.
. Can't identify clear trigger !
. Symptoms throughout the year but sometimes worse with seasons change.
. Nasal mucosa may be normal or erythematous.
. Less commonly associated with allergic disorders e.g. asthma or eczema.
. Routine allergy testing isn't necessary prior to initiating empiric ttt.
. May respond to 1st generation oral H1 antihistaminics (Chloramphenicol),
. Never ever responds to antihistaminics without anticholinergic properties (Loratidine)!
. Tx -> TOPICAL INTRANASAL GLUCOCORTICOIDS.
. The 3 most common causes of CHRONIC COUGH (> 8 weeks):
________________________________________________________
. UPPER AIRWAY COUGH $YNDROME (Post-nasal drip).
. BRONCHIAL ASTHMA.
. GERD.
. UPPER AIRWAY COUGH $YNDROME = POST-NASAL DRIP:
_________________________________________________
. NON-smoker.
. Caused by rhino-sinusitis conditions.
. Dry cough is most likely due to post-nasal drip associated with allergic rhinitis.
. Dx -> Confirmed by improvement of the nasal discharge & cough with H1 Anti-histaminics.
. Chlorpheniramine is an H1 receptor blocker that decreases the allergic response.
. Decrease in NASAL SECRETIONS is most likely to significally improve symptoms.
. ANAPHYLAXIS = ANAPHYLACTIC SHOCK:
___________________________________
. Type 1 hypersensitivity reaction.
. Pts usually have prior exposure to the offending substance.
. Pts have preformed Ig E -> Histamine mediated peripheral vasodilatation.
. Bee stings - food & medications are the most common allergens.
. Acute onset of hypotension & tachycardia.
. Dangerous allergic reaction may progress to respiratory failure & circulatory collapse.

.
.
.
.
.
.
.

Allergen exposure -> Sudden onset of symptoms in more than one system,
Cutaneous (hives - flushing - pruritis).
GIT ( Lip / tongue swelling - vomiting).
Respiratory (Dyspnea - wheezing - stridor - hypoxia).
Cardiovascular (Hypotension).
It is a medical emergency.
Tx -> INTRA-MUSCULAR EPINEPHRINE into the THIGH.

. ASPIRIN SENSITIVITY $YNDROME:


_______________________________
. Aspirin ingestion - persistent nasal blockage - Episodes of bronchoconstriction.
. Pathogenesis -> Psudo-allergic reaction.
. Aspirin -> PGs/LKs imbalance.
. Tx -> Avoid NSAIDs & Leukotriene recptor antagonists (Drug of choice).
. MEDIASTINAL TUMORS:
_____________________
_____________________
. Dx -> Helical CT CHEST.
. ANTERIOR mediastinum --> THYMOMA & GERM CELL TUMORS.
. MIDDLE mediastinum ----> BRONCHOGENIC CYST.
. POSTERIOR mediastinum -> Neurogenic tumors e.g. Meningocele.
. GERM CELL TUMORS:
___________________
. Affect young adults.
. Present as large ANTERIOR mediastinal mass.
. Two types of germ cell tumors (Seminomatous & Non-seminomatous).
. Both types produce B-HCG (B-Human chorionic gonadotropin).
. ONLY "NON"-seminomatous type produces Alpha-feto protein (AFP).
. CHORIOCARCINOMA:
__________________
. Metastatic form of gestational trophoblastic disease.
. It may occur after molar pregnancy or normal gestation.
. The lungs are the most frequent site of metastatic spread.
. Any postpartum woman e' pulmonary sympotms & multiple nodules on CXR =
CHORIOCARCINOMA.
. Dx -> ++++++ B-HCG levels.
. INCIDENTALLY DISCOVERED SOLITARY PULMONARY NODULE:
____________________________________________________
. May be BENIGN -> Infectious granuloma or hamartoma.
. May be MALIGNANT -> Bronchogenic carcinoma & metastasis.
. BIOPSY is the only way to definitively detect whether a nodule is benign or malignant.
. Clinical characteristics favoring malignancy:
. Age > 50 - H/O of smoking - Weight loss - Previous malignancy.
. Radiographic characteristics of malignancy:
. Large size - Low density - Spiculated borders - Absence of calcifications.
. Rate of lesion growth is an important parameter:

. Malignant nodules tend to double in size bet. one month & one year.
. OBTAINING PREVIOUS X-RAY if possible is the FIRST BEST STEP in management.
. If a previous x-ray demonstrates that the lesion has been stable in size > 2 ys,
. Malignancy is effectively ruled out & no further testing is necessary.
. LOW propability nodules are followed by serial high resolution CT CHEST.
. INTERMEDIATE propability nodules are followed by PET SCAN or BIOPSY.
. HIGH propability nodules are removed surgically.
. PULMONARY - RENAL ASSOCIATIONS:
_________________________________
.1. WEGENER's GRANULOMATOSIS WITH POLYANGIITIS:
________________________________________________
. SYSTEMIC VASCULITIS + UPPER & LOWER RESPIRATORY TRACT INFECTION +
GLOMERULONEPHRITIS.
. Age around 40s.
. URT symptoms (Bloody or purulent nasal discharge - oral ulcers - sinusitis).
. LRT symptoms (Dyspnea - cough - Hemoptysis).
. Renal symptoms (Microscopic hematuria - RBC casts).
. Granulomatous inflammation of nasopharynx (Epistaxis - Rhinorrhea - Otitis - sinusitis)
. Saddle nose deformity due to destruction of the nasal cartilage.
. Cutaneous manifestations (Painful SC nodules - palpable purpura - pyoderma gangrenosum)
. BEST INITIAL TEST -> +ve C-ANCA = serum anti-neutrophilic cytoplasmic antibody.
. CXR -> Bilateral multiple nodular opacities.
. Urinalysis -> RBCs casts - proteinuria & sterile pyuria.
. Tx -> CYCLOPHOSPHAMIDE & High dose corticosteroids.
.2. GOODPASTURE's DISEASE:
___________________________
. Due to renal basement membrane antibodies !
. Young male.
. Lungs (cough - dyspnes - hemoptysis).
. Kidneys (Nephritic proteinuria - ARF - Dysmorphic RBCs & red cell casts on urinalysis).
. Systemic symptoms are un common.
. Dx -> Renal biopsy -> LINEAR IgG antibodies along the glomerular basement membrane.
. EFFECTS OF ARTERIAL OXYGENATION & VENTILATION IN VARIOUS ENVIRONMENTS:
________________________________________________________________________
_____________________________ Example ________ A-a gradient ____ Pa CO2 ___ Corrects e' O2
. -- inspired O2 tension = HIGH ALTITUDE:
_________________________________________
. A-a gradient -> Normal.
. Pa CO2 -> Normal.
. Corrects with supplemental O2 -> YES.
. Hypoventillation = CNS DEPRESSION:
____________________________________
. A-a gradient -> Normal.
. Pa CO2 -> +++++.
. Corrects with supplemental O2 -> YES.
. Diffusion limitation = INTERSTITIAL LUNG DISEASES:

______________________________________________________
. A-a gradient -> +++++.
. Pa CO2 -> Normal.
. Corrects with supplemental O2 -> YES.
. Shunt = Intracardiac shunt or extensive ARD$:
_______________________________________________
. A-a gradient -> +++++.
. Pa CO2 -> Normal.
. Corrects with supplemental O2 -> NOOOOOO.
. V/Q mis-match = Obstructive diseases, atelectasis, pulmonary edema & pneumonia:
_________________________________________________________________________________
. A-a gradient -> ++++++.
. Pa CO2 -> Normal.
. Corrects with supplemental O2 -> YES.
. Low lung compliance.
. UPPER AIRWAY OBSTRUCTION WITH LARYNGEAL EDEMA:
________________________________________________
. ACUTE ONSET dyspnea & difficulty swallowing.
. Agitation & gasping of breath.
. Excessive accessory respiratory muscle use.
. Retraction of the subclavicular fossae during inspiration.
. H/O of previous food allergy.
. Identifiable precipitating event e.g. peanut ingestion.
. Physical exam. may reveal stridor & harsh respiratory sounds from trachea.
. Wheezing is generally absent on lung auscultation.
. A fixed upper airway obstruction will -- air flowrate in all inspiration & expiration.
* NORMAL LUNG EXAMINATION:
__________________________
. Percussion -> Resonant.
. Auscultation -> Vesicular breathing.
* LUNG CONSOLIDATION EXAM:
__________________________
. Percussion -> Dullness.
. Auscultation -> LOUDER vesicular breathing if airways are patent (Faint if blocked).
. Bronchial breathing with full expiratory phase.
. ++ TVF.
. Bronchophony.
. Egophony (Ask the pt to say "E", it will sounds like "A").
. Widespread pectoriloquy.
* PLEURAL EFFUSION EXAM:
________________________
. Inspection -> -- movements of ipsilateral chest.
. Percussion -> Dullness.
. Auscultation -> Decreased breath sounds.
. -- TVF.
* PNEUMOTHORAX EXAM:
____________________

.
.
.
.

Percussion -> Hyper-resonance.


Auscultation -> Decreased breath sounds (Will be absent entirely if large pneumothorax)
-- TVF.
JVD, Hypotension & Tracheal deviation to the opposite side.

* EMPHYSEMA EXAM:
_________________
. Percussion -> bilateral resonance.
. Auscultation -> Vesicuar breathing with fine crackles at inspiration.
.
.
.
.

N.B. Recurrent bacterial infections in an adult may indicate a HUMORAL IMMUNITY defect.
Recurrent sino-pulmonary & gastro-intestinal infections.
Dx -> Quantitative measurment of serum immunoglobulin "G" levels -> DECREASED.
Cystic fibrosis may have similar presentation BUT (Earlier in life & e'out GIT infects).

. ACE INHIBITORS & DRY COUGH:


_____________________________
. Always consider ACE Is as a potential cause of chronic cough.
. Pathogenesis -> Accumulation of bradykinins & prostaglandins.
. Simple discontinuation of the drug sh'd precede any diagnostic tests !
. SINGLE PULMONARY NODULE APPROACH:
___________________________________
. SOLITARY PULMONARY NODULE = Lesion < 3 cm completely surrounded by pulmonary
parenchyma
._________________________________________________________________________________________
.|
._______________________________________________
.|
.|
.|
. HIGH MALIGNANCY RISK . INTERMEDIATE RISK
. LOW MALIGNANCY RISK
._____________________ .___________________
._____________________
.|
.|
.|
. Surgical excision.
. NODULE SIZE ?
. SERIAL CT SCANS
. < 1cm: Serial CTs.
. > 1cm: PET scan.
* FUNGAL INFECTIONS OF THE LUNG:
________________________________
.1. HISTOPLASMOSIS:
___________________
. Asymptomatic pulmonary nodule.
. Residence in suburban Mississippi or o"H"io river valleys !
. Absence of any complaints.
. Absence of significant past H/O.
. Absence of any cavitary lesions.
. Calcified nodes in the lung may be seen.
. It is a dimorphic fungus found in soil with high concentration of bird or bat droppings
. Infection through inhalation of the spores of Histoplasma capsulatum fungus.
.2. BLASTOMYCOSIS -> ULCERATED SKIN LESIONS & LYTIC BONE LESIONS:

_________________________________________________________________
. Fungal infection of the lung..
. Residence in great lakes, Mississippi, Ohio river & Wisconsin.
. Pulmonary symptoms resembling T.B. & Histoplasmosis.
. ULCERATED SKIN LESIONS & LYTIC BONE LESIONS (Characteristic!).
. Skin lesions -> Multiple well circuscribed verrucus crusted lesions.
. Bone lesions -> Lytic lesions in the anterior ribs.
. Dx -> Sputum culture -> BROAD BASED BUDDING YEAST.
. Tx -> ITRACONAZOLE or Amphotericin B.
.3. COCCIDIOIDOMYCOSIS:
_______________________
. Fungal infection of the lung.
. Residence in Southwestern US.
. Fever, cough & night sweats.
. Extra-pulmonary -> skin, meninges & skeleton.
.4. ASPERGILLOSIS = A MOBILE LUNG CAVITARY MASS + INTERMITTENT HEMOPTYSIS:
__________________________________________________________________________
. Fungal infection of the lung.
. Coarse fragmented septae.
. Hyphae are typically seen.
. CXR -> Radio-lucency next to a rounded mass.
. Cavitary lesion may form due to destruction of the underlying pulmonary parenchyma.
. Debris & hyphae may coalese forming a FUNGUS BALL.
. The ball lies freely in the cavity & moves around with position change.
. A MOBILE CAVITARY MASS + INTERMITTENT HEMOPTYSIS = ASPERGILLOMA.
. SUPERIOR SULCUS TUMOR:
________________________
. Apical lung tumor causing compression effects.
. Superior vena cava -> SVC $yndrome.
. Sympathetic trunk -> Horner $yndrome.
. Brachial plexus -> Pancoast $yndrome (Pain - paresthesia - weakness of arm).
. Rt recurrent laryngeal nerve -> Hoarsness of voice.
. PANCOAST $YNDROME:
____________________
. Apical lung tumor at the thoracic inlet.
. Compress the inferior portion of the brachial plexus.
. Shoulder pain radiating in an ulnar distribution.
. SUPERIOR VENA CAVA $YNDROME (SVC):
____________________________________
. Obstruction of SVC impedes venous return from the head, neck, face & arms to the heart.
. Dyspnea - Venous congestion & swelling of the head, neck & arms.
. Malignancy is the most common cause of obstruction (Lung cancer - Hodgkin's lymphoma).
. H/O of chronic heavy smoker with recent un-intentional weight loss -> Lung cancer.
. Best initial test -> CXR -> If abnormal -> Follow up with Ct chest.
. HYPERTROPHIC OSTEOARTHROPATHY:
________________________________
. Development of clubbing & sudden onset joint arthropathy in a chronic smoker.
. Bilateral wrist tendrness, thickening of distal fingers & convex nail beds.
. Associated with lung cancer.

. CXR is mandatory to rule out malignancy.


. FINGER CLUBBING:
__________________
. Thickening of the nail bed that causes a devrease in the angle bet the nail bed & fold.
. In severe cluccing, the terminal parts of the fingers appear swollen like drumsticks.
. It is NOT a feature of simple COPD.
. NEW CLUBBING in COPD pts indicates the development of lung cancer or occult malignancy.
. GOLDEN SCHEME:
________________
________________
.

. SPIROMETRY
.____________
.|
.____________________________________________________
.|
.|
. LOW FEV1/FVC
. NORMAL OR HIGH FEV1/FVC
.______________
._________________________
.|
.|
. OBSTRUCTIVE DISEASE
. RESTRICTIVE DISEASE
._____________________
._____________________
.|
.|
. BRONCHO-DILATOR CHALLENGE
. DLCO
____________________________
.______
.|
.|
._________
.________________
.|
.|
.|
.|
. ++ FEV1
. No ++ in FEV1
. NORMAL
. -- DLCO
._________ ._______________
._______
._________
. ASTHMA.
. COPD.
. CHEST WALL WEAKNESS . ILD.
.|
. DLCO
._____________________
.|
.|
. (--) -> Emphysema . (++) -> Chronic bronchitis.

. N.B. RIGHT MAIN STEM BRONCHUS INTUBATION:


___________________________________________
. Relative complication of endotracheal intubation.
. It causes asymmetric chest expansion during inspiration.
. Markedly decreased or absent breath sounds on the left side on auscultation.
. Solve the problem by repositioning of the tube,
. Tx -> Pull it back slightly, this will move its tip between the carina & vocal cords.
. N.B. 2ry MALIGNANCY AFTER CHEMOTHERAPY !
__________________________________________
. Up to 4% of pts with HODGKIN's disease wil develop a 2ry malignancy (Lung - breast)
. After being treated with chemotherapy & radiation !
. N.B. POST-ICTAL STATE ABG:
____________________________

.
.
.
.
.

Repiratory ACIDOSIS.
Acisosis (-- pH).
Hypercarbia (++ CO2).
Normal or ++ HCO3 !
HYPO-ventillation is a major cause of respiratory acidosis.

. N.B. MOST COMMON CAUSE OF HEMOPTYSIS is -> CHRONIC BRONCHITIS:


________________________________________________________________
. Chronic productive cough for 3 months in 2 successive years with ciagarette smoking.
. Other important causes -> BRONCHOGENIC CARCINOMA & BRONCHIECTASIS.
. CXR is mandatory to exclude malignancy.
.
.
.
.

N.B. Acute bronchitis is a common cause of blood-tinged sputum.


It is usually viral in etiology.
In an "A"FEBRILE pt with NEW-ONSET BLOOD TINGED SPUTUM e'OUT significant serious signs,
OBSERVATION & CLOSE CLINICAL FOLLOW UP is the best ttt strategy.

. MITRAL STENOSIS:
__________________
. Most common cause is rheumatic fever.
. Pt. 40 - 50ys.
. presents with gradual & progressively worsening dyspnea on exertion.
. Orthopnea & hemoptysis due to pulmonary edema.
. Auscultation -> Loud S1 & Opening snap after S2 at apex.
. Low pitched diastolic rumble at apex (When pt lies on left side with breath holding).
. Atrial fibrillation is a common complication.
. Af causes rapid decompensation in a previously asymptomatic pt.
. Long-standing MS can cause Left atrial enlargement -> Elevation of left main bronchus.
. ACE inhibitors side effect -> Dry cough:
__________________________________________
. Pathophysiology -> Accumulation of KININs due to activation of arachidonic acid pathway
. N.B. ACID-BASE BALANCE in two different situations:
_____________________________________________________
_____________________________________________________
. 1 . Chronic hypercapneic respiratory failure due to COPD:
___________________________________________________________
. Marked acidosis should be the result of respiratory failure in COPD.
. But .. RENAL TUBULAR COMPENSATION occurs.
. Kidneys ++ HCO3 retention to compensate for ++ CO2 !
. Pts with chronic hypoventillation have gradual ++ in pCO2 -> Respiratory acidosis.
. To compensate, kidneys ++ HCO3 retention & -- Chloride reabsorption instead !
. BOTTOM LINE -> The body compensates for chronic hypercapnea by ++ bicarbonate retention.
. 2 . Mechanically vetillated pt following head trauma:
_______________________________________________________
. Hyper-ventillation (Due to ++ TV or RR) -> Excessive CO2 loss & Respiratory Alkalosis.
. Hypo-ventillation (Due to -- TV or RR) -> Excess CO2 Retention & Respiratory Acidosis.
. Respiratory alkalosis:

->
->
->
->
->
->

++ pH (N = 7.4).
-- PCO2 (N = 40 mmHg).
-- HCO3 (N= 24) -> DECREASED due to attempted renal compensation for resp. alkalosis.
The kidneys retain increased amounts of Hydrogen H (protons)
& excrete ++ amounts of bicarbonate (HCO3) in attempt to normalize serum pH.
The ++ amount of HCO3 in urine ALKALIZES the urine.

Dr. Wael Tawfic Mohamed


__________________________

RHEUMATOLOGY TiKi TaKa


-----------------------. Systemic lupus erythematosus "SLE":
-----------------------------------. Female 20-30 ys.
. Malar "Butter fly" rash.
. Arthritis.
. Painless oral ulcers.
. Renal disease.
. +ve Ds DNA.
. Newly diagnosed "Lupus Nephritis" ----> RENAL BIOPSY to detect the type of pathology.
. Grading from class 1 "Minimal mesangial" up to class 4 "Advanced Sclerosis".
. Then the immunosuppressive therapy is given accordingly.
. BLUE TOE $YNDROME:
------------------. Due to CHOLESTEROL EMBOLISM.
. Cardiac catheterization may cause atheroembolism.
. BLUE TOES " But intact paulse".
. ++ CREATININE.
. Abdominal tendrness.
. Accompanied e' Livedo reticularis.
. SLE:
----. Young, African American woman.
. Aged 20 - 40 ys.
. Fatigue "Anemia".
. Painful oral ulcers.
. Non deforming arthritis.
. Hematologic abnormalities "pancytopenia".
. Low grade fever.
. Weight loss.
. Malar or discoid rash.
. Lupus Arthritis as RA involves MCP & PIP BUT "NO DEFORMITIES".
. REMEMBER ---> LUMBAR SPINAL STENOSIS Dx ----> Spinal MRI.
----------------------------------------------------------. Ankle brachial index is used to diagnose peripheral artery disease.
. ERYTHEMA NODOSUM:
------------------. Painful S.C. pre-tibial nodules.
. Associated with SARCOIDOSIS.
. Ask for a CXR to detect sarcoidosis.
. CXR: Bilateral hilar adenopathy.
. AFRICAN AMERICAN FEMALE !
. Cough, Arthritis & uveitis.
. Disease --------------------------> Best initial Tx:

-----------------------------------------------------. Rheumatoid arthritis --------------> Methotrxate.


. Osteoarthritis --------------------> Weight loss & Acetaminophen.
. Gout acute attack -----------------> NSAIDs, Indomethacin.
. Gout prevent. of new attack -------> Colchicine.
. CPPD ------------------------------> NSAIDs.
. Disk herniation -------------------> NSAIDs.
. Epidural abscess ------------------> Abs "Vancomycin".
. Cord compression ------------------> Steroids.
. Spinal stenosis -------------------> Weight loss & Steroid injection.
. Fibromyalgia ----------------------> Amitriptyline.
. Carpal Tunnel. $ ------------------> Wrist splint & NSAIDs.
. Polymyositis ----------------------> HIGH dose steroids.
. Rotator cuff injury ---------------> NSAIDs.
. SLE -------------------------------> High dose steroids.
. Sjogren $ -------------------------> Water the mouth & atrificial tears.
. Polymyalgia Rheumatica ------------> LOW dose steroids.
. Temporal "Giant cell" arteritis ---> HIGH dose steroids.
. Ankylosing Spondylitis ------------> NSAIDs.
. Psoriatic arthritis ---------------> NSAIDs.
. Reactive arthritis "Reiter's $" ---> NSAIDs.
. Septic arthritis ------------------> CEFTRIAXONE & VANCOMYCIN.
. Gonococcal arthritis --------------> Ceftriaxone or cefotaxime.
. D.D. of CALF SWELLINGS:
------------------------.. Cellulitis:
-------------... Infection of skin & S.C. tissue.
... Risk factors: Obesity & Tinea pedis !
... Red, edematous skin that is hot to touch.
... Regional lymphadenopathy.
... Caused by STAPH & STREPT Group A.
.. DVT = Deep Venous Thrombosis:
-------------------------------... Same presentation as cellulitis .
... BUT .. FEVER never exceeds 38.5 c.
... No regional lymphadenopathy.
... Ruptured BAKER's CYST.
. OSTEO-ARTHRITIS (OA):
----------------------. Old age.
. Affects hands & weight bearing joints.
. Mild morning stiffness < 30 mins (RA > 1 hour).
. Pain ++ with exercise & -- by rest.
. Bony crepitus, bony enlargement.
. Painful & - range of motion.
. Synovial fluid analysis: 200-2000 WBCs,
. (Normal 0-200 & Inflammatory 2000-50000 & Septic arthritis >50000).
. X-ray: -> NARROWED JOINT SPACE.
. X-ray: -> OSTEPHYTE FORMATION.

. X-ray: -> SUBCHONDRAL CYSTS.


. GOUTY ARTHRITIS:
-----------------. Middle aged male.
. Acute joint pain (1st Metatarsophalangeal joint is the most common).
. Swelling & -- range of motion.
. Low grade fever.
. Synovial fluid analysis is cirtical for diagnosis,
. WBCs 2000-50000,
. NEEDLE shaped, NEGATIVELY bireferingent crystals under polarized light.
. NEGATIVE gram stain & culture.
. ++ serum Uric acid is neither sensitive nor specific !!
. Tx of acute attack --> INDOMETHACIN (Cot'd in RF or GIT bleeding) & COLCHICINE.
. TREATMENT ----> NSAIDs, Colchicine & steroids.
. PREVENTION ---> Allopurinol & probenicid.
. PSEUDO-GOUT:
-------------. Calcium pyrophosphate dihydrate (CPPD) deposition.
. Acute onset, pinful , monoarthropathy affecting the knee.
. Synovial fluid ---> RHOMBOID shaped with POSITIVE +ve birefringence.
. Ass. with HYPERPARATHYROIDISM:
.. ++ Ca & -- PO4 --> constipation & excess urination.
.. Disease of GROANS (Abd. pain), STONES (urinary) & Psychic MOANS.
. GONOCOCCAL SEPTIC ARTHRITIS:
-----------------------------. YOUNG, SEXUALLY ACTIVE FEMALE.
. Fever > 38.5 c.
. Redness, hotness, swelling, pain, limitation of movement.
. Synovial fluid analysis: ++ WBCs > 50000.
. Asymmetric polyarthritis.
. May be associated with tenosynovitis or rash.
. OSTEO-ARTHRITIS:
-----------------. Narrowed joint space.
. Osteophytes.
. Suchondral sclerosis or cysts.
. Obesity is the most common risk factor.
. Weight loss is the best initial ttt.
. LUMBAR SPINAL STENOSIS:
------------------------. Old pt.
. Combined low back & leg pain.
. Posture dependent --> Flexion of the back causes widening of the spinal canal,
. while extension causes narrowing of the spinal canal.
. So, the leg pain is exacerbated by extension of the spine (Standing & walking),
. but improved by flexion (Sitting & lying down).
. Called "Neurological Claudication".
. Differentiated from claudication of peripheral vascular disease by normal pulses!
. Normal Ankle / Brachial index.
. Normal neurological examination.

. -ve Straight leg test.


. Dx: MRI.
. Tx: Conservative or Laminectomy.
. TEMPORAL = GIANT CELL ARTERITIS:
---------------------------------. Age > 50 ys.
. New headache.
. Jaw claudication.
. Scalp pain.
. Visual loss.
. ++ ESR.
. Dx: TEMPORAL ARTERY BIOPSY.
. Tx: HIGH dose steroids.
. ANKYLOSING SPONDYLITIS TIPS & TRICKS:
--------------------------------------. Not only in young males, may affect females !
. BILATERAL SACRO-ILIITIS is DIAGNOSTIC.
. Most imp. extra-articular manifestation is ANTERIOT UVEITIS !!
. May be associated with Aortic insuffeciency with AV Block.
. (Not ass. with Aortic Aneurysm xxxxx).
. N.B. Apophyseal joint arthritis = Ankylosing spodylitis:
--------------------------------------------------------. "Sero-negative spondylo-arthropathies".
. HERNIATED DISC:
----------------. Pain worsens with sitting.
. Low bk pain & sciatica.
. +ve stress leg test.
. VERTEBRAL METASTASIS:
----------------------. Low bk pain.
. H/O of malignancy.
. Weight loss.
. CONSTANT DULL PAIN.
. Failure to improve with conservative therapy.
. Osteomyelitis:
--------------. Caused by STAPH. AUREUS.
. Tx-> Ox, Clox, Dicloxacillin.
. ROTATOR CUFF TENDONITIS:
-------------------------. Due to repititive activity above shoulder height e.g. Painter.
. Passive motion of the arm above the head cause pain & guarding confirming impingement.
. Lidocaine injection cause pain relief,
. in contrast to persistence of pain with NO relief in case of ROTATOR CUFF TEAR,
. Rotator cuff tear is due to fall on out stretched hand.
. CYCLOPHOSPHAMIDE side effect ---> Hemorrhagic cystitis & Bladder carcinoma.

. ROTATOR CUFF TEAR:


-------------------. results from chronic tendonitis & shoulder trauma.
. Shoulder pain & weakness when lifting the arm above the head.
. Lidocaine injection relieves the pain in case of R.C. Tendonitis,
. while it persists in case of R.C. Tear.
. Dx----> MRI Shoulder.
. VIRAL ARTHRITIS:
-----------------. Secondary to PARVO-virus 19 infection.
. Similar presentation as Rheumatic arthritis !
. Arthritis --> PCP & PIP & wrists.
. Resolves within just 2 months !!
. H/O of frequent contact with children e.g. day care workers.
. so the diff. bet viral & RA are:
---------------------------------. . ACUTE ONSET.
. . Lack of inflammatory markers "Anti-CCP & RF".
. . Resolution within 2 months !
. SE of HYDROXYCHLOROQUINE ---> RETINOPATHY.
. SE of CYCLOPHOSPHAMIDE ---> BLADDER CARCINOMA.
. SARCOIDOSIS:
-------------. AFRICAN AMERICAN FEMALE.
. Lung involvement --> Cough & dyspnea.
. Erythema nodosum.
. Anterior uveitis.
. Acute polyarthritis.
. Parahilar adenopathy.
. ++ ACE enzymes (Give ACE Is)!
. Biopsy: Non caseating granuloma.
. Tx: SYSTEMIC GLUCOCORTICOIDs.
. CARPAL TUNNEL $YNDROME:
------------------------. Compression of the median nerve within the carpal tunnel in the wrist.
. Pain & paresthesia in the median nerve distribution.
. Worse with wrist flexion (+ve PHALEN test).
. May be due to HYPOTHYROIDISM (Fatigue, constipation, menorrhagia & dry skin).
. PATHOLOGY ----> Deposition of MUCOPOLYSACCHARIDE PROTEIN COMPLEXES (MATRIX
SUBSTANCE).
. HYPOTHYROIDISM ---> BILATERAL CT$.
. FIBRO-MYALGIA:
---------------. WOMEN 20-50 ys.
. Point tenderness in at least 11 - 18 points !!
. H/O of generalized musculoskeletal pain not related to another illness.
. Disturbed sleep, easy fatiguability.
. Normal lab values.

. Tx: TCAs e.g AMITRIPTYLINE.


. DERMATOMYOSITIS:
-----------------. Proximal extensor ms inflammatory myopathy.
. Violaceous poikiloderma.
. Periorbital edema with rash "Heliotrope sign".
. Rash on chest & lateral neck "Shawl sign".
. Rash on the knuckles, elbows & knees "Gottron's sign".
. Lichenoid papules "Gottron's papules".
. Anti-Mi-2 Abs.
. Ass. with internal malignancies "Most common is OVARIAN CANCER" !
. POLYMYOSITIS:
--------------. Slowly progressive proximal muscle weakness of the lower limbs.
. Difficulty with stair climbing.
. Difficulty with rising from a seat.
. Muscle tendrness.
. Best diagnostic test ---> MUSCLE BIOPSY.
. Metastatic disease of the vertebrae:
-------------------------------------. H/O of Lung cancer (Non small cell lung carcinoma).
. Most common causing cancers: Lung, breast, prostate & thyroid.
. CONSTANT pain, worse at night.
. LUMBAR STRAIN:
--------------. Follow twisting of the bk while lifting heavy objects.
. ++ by activity & -- by rest.
. No point tendrness.
. SPINAL STENOSIS:
----------------. Low bk pain at lumbar spine,
. ++ with activity.
. DISK HERNIATION:
----------------. LBP radiating down the buttock,
. +ve straight leg raise test.
. ANSERINE BURSITIS:
-------------------. Anserine bursa is located antero-medially over the tibial plateau,
. just below the joint line of the knee.
. Inflammation may be due to overuse or trauma.
. LOCALIZED pain over the ANTEROMEDIAL tibia.
. Valgus stress test -->-ve. "Ruling out Medial collateral ligament injury".
. NORMAL X-ray.
. Tx: Cortico-steroids injection into the bursa.
. Pre-patellar bursitis:
------------------------

. Pain & swelling directly over the patella.


. Disease ---> Specific Antibodies:
----------------------------------. Rhematoid Arthritis "RA" ------------> Anti-Cyclic Citrulinated Peptide "CCP".
. Systemic Lupus Erythematosus "SLE" --> Anti-Double Stranded DNA "DS DNA".
. Scleroderma -------------------------> Anti-topoisomerase "Scl 70".
. CREST $ -----------------------------> Anti-centromere.
. Sjogren $ ---------------------------> SS-A "Ro" & SS-B "La".
. Wegener's granulomatosis ------------> Anti-neutrophil cytoplasmic Antibody "C-ANCA".
. Chrug-Strauss -----------------------> Anti-myeloperoxidase antibody "P-ANCA".
. WHIPPLE's disease:
-------------------. H/O of malabsortion diarrhea (Steatorrhea, flatulence, abd. distension).
. Weight loss.
. Migratory arthritis.
. caused by Tropheryma Whippelii.
. Dx: Small intestinal biopsy ---> PAS +ve macrophages in the lamina propria.
. Celiac disease = Celiac sprue = GLUTEN SENSITIVE ENTEROPATHY :
--------------------------------------------------------------. Malabsorptive diarrhea.
. Anti-endomysial & Anti-transglutaminase Antibodies.
. Biopsy --> Effacement of SI villi.
. ANKYLOSING SPONDYLITIS & IBD are associated !
---------------------------------------------. Both are associated with HLA B27.
. Both are associated with P-ANCA.
. IBD (UC): Bloody diarrhea, anemia & -ve stool culture, erythema nodosum.
. AS: SACROILIITIS.
. REACTIVE ARTHRITIS = 3 Can't ( see, pee, climb a tree):
-------------------------------------------------------. Following infectious diarrhea,
. caused by shigella, salmonella, Yersinia, Campylobacter or C.dificile.
. Conjunctivitis "Can't see".
. Urethritis "Can't Pee".
. Arthritis "Can't climb a tree"
. POLY-MYALGIA RHEUMATICA (PMR):
-------------------------------. Age > 50 ys.
. Aching pain in neck, shoulders & pelvic girdle lasting at least 1 month.
. Morning stiffness > 1 hour !
. ESR < 40 mm/hr.
. No pain with active or passive range of movement.
. Tx: LOW DOSE PREDNISONE.
. When do u give HIGH dose prednisone ?
. -> If associated with GIANT CELL TEMPORAL ARTERITISto prevent visual loss.
. It is diagnosed by Temporal artery biopsy.
. RHEUMATOID ARTHRITIS MANAGEMENT:
----------------------------------

.
.
.
.
.
.
.

All pts should be started on DMARDs ASAP as joint damge begins early !
METHOTREXATE is the best initial therapy.
NSAIDs are adjunctive therapy for symptomatic relief,
NSAIDs don't -- disease progression.
Gucocorticoids may reveal symptoms temporarily but they don't prevent future worsening.
Pts sh'd be tested for HEPATITIS B & C & T.B. bef. starting MTx.
MTx sh'dn't be used in pregnants !

. Disseminated Gonococcemia:
---------------------------. Migratory polyarthritis.
. Skin lesions (Pustules) on the extremeties.
. Tenosynovitis.
. High fever & chills.
. Blood & pustule culture --> NEGATIVE (Need specific growth requirements).
. SJOGREN $YNDROME:
------------------. Women 50 - 60 ys.
. Kerato-conjunctivitis sicca (Xerophthalmia & dry eyes).
. XEROSTOMIA (dry mouth).
. Lack of normal amount of saliva -> Dental carries & dysphagia.
. Enlargement & firmness of the salivary glands.
. Histology -> Lymphocytic infiltration of the salivary glands.
. +ve Anti-SSA(Ro) & or Anti-SSB(La).
. SYSTEMIC SCLEROSIS:
--------------------. AFRICAN AMERICAN FEMALE.
. Widespread organ involvement.
. Esophagus -->GERD.
. Heart ------> Rt Heart failure.
. Kidney -----> hypertension.
. Most common cause of death is PULMONARY ARTERIAL HYPERTENSION.
. +ve Anti-topo-isomerase-I Abs = +ve Anti-Scl70.
. CREST $ = LIMITED scleroderma:
------------------------------. Calcinosis cutis.
. Raynaud's phenomenon.
. Esophageal dysmotility.
. Sclerodactyly.
. Telangiectasia.
. +ve Anti-Centromere Abs.
. PAGET DISEASE OF BONE = OSTEITIS DEFORMANS:
-------------------------------------------. ++ Osteoclastic activity ---> Bone RESORPTION.
. Distorted bone formation.
. Common sites: Femur & skull & vertebra.
. Hypertrophy of the skull -----> Vestibulocochlear nerve compression,
. 8th cranial n. compression ---> Deafness "Sensorineural hearing loss".
. NORMAL CALCIUM & PHOSPHATE LEVELS.
. HIGH ALKALINE PHOSPHATASE LEVEL.
. HIGH HYDROXYPROLINE LEVEL "Bone marker".

. SUB-ACROMIAL BURSITIS:
----------------------. Subacromial bursa lies between the acromion & the tendon of the supraspinatous ms.
. caused by chronic microtrauma to the supraspinatous tendon.
. e.g. overhead work or tennis playing.
. Tendrness hen the arm is internally rotated & forward flexed at the shoulder.
. No signs of deltoid atrophy.
. LATERAL EPICONDYLITIS = TENNIS ELBOW:
-------------------------------------. Due to repeated forceful wrist extension & supination.
. ex: Backhand in tennis or use of a screw driver.
. Point tendrness near the lateral epicondyle.
. Due to degeneration of extensor carpi radialis brevis.
. ROTATOR CUFF INJURY:
---------------------. Shoulder pain, weakness & -- range of motion.
. Due to impingement of the supraspinatous tendon.
. De QUERVAIN TENO-SYNOVITIS:
----------------------------. NEW MOTHERS who hold their babies with out-stretched thumb (ABDUCTED & EXTENDED).
. Affects tendons of abductor pollicis longus & extensor pollicis brevis.
. Passive stretch of these tendons elicits pain.
. Chronic Tophaceous Gout:
------------------------. Metatarsophalangeal joint is the most affected.
. Severe swelling & pain in the big toe.
. Due to deposition of the monosodium urate crystals "PODAGRA".
. U.A. crystals ---> Nephrolithiasis.
. Urate crystals may deposit in the soft tissues forming tumors "TOPHI".
. Tophi may ulcerate & drain a chalky material.
. H/O of water-pills intake for hypertension "THIAZIDEs".
. Hydrochlorothiazide ---> Hypovolemia ---> ++ U.acid reabsorption.
. BEHCET's $YNDROME:
------------------. Recurrent oral ulcers.
. Recurrent genital ulcers.
. Eye lesions: Anterior uveitis.
. Skin lesions: Erythema nodosum.
. Tx: Corticosteroids.
. More common in TURKISH, ASIAN & MIDDLE EASTERN population.
. Main mechanism of kidney damage in SLE is IMMUNE COMPLEX MEDIATED.
. Best initial ttt in cases of DISK HERNIATION is NSAIDs & EARLY MOBILIZATION.
. SLE ARTHRITIS:
--------------. Cortico-steroid induced "AVASCULAR NECROSIS" of the femoral head.
. Progressive hip or groin pain.

. without restriction of motion range.


. Normal radiograph on early stages.
. Dx: MRI is the gold standard.
. LUMBOSACRAL STRAIN:
-------------------. Most common cause of pain.
. Pain starts acutely after physical exertion.
. Pain concentrated in the lumbar area.
. No radiation to thighs.
. Paraspinal tendrness.
. Normal neurological exam.
. -ve straight leg raising test.
. Tx: NSAIDs & early mobilization.
. HERNIATED DISK:
---------------. Pain radiation to thighs.
. +ve straight leg raising test.
. COMPRESSION # of the VERTEBRA:
------------------------------. OLD age.
. Acute onset of pain without an obvious trauma.
. H/O of osteoporosis.
. H/O of steroids intake.
. Normal neurological exam (Absent Babinski reflex is considered NORMAL in elderly).
. -ve Straight leg raise test (+ve with DISK HERNIATION).
. VERTEBRAL OSTEOMYELITIS:
------------------------. Lumbar spine.
. Back pain.
. Low grade fever.
. ++ ESR.
. Local tendrness on percussion.
. Paravertebral muscular spasm.
. Dx: MRI.
. CERVICAL SPONDYLOSIS:
---------------------. Due to BONY SPUR.
. Age > 50ys.
. H/O of CHRONIC NECK PAIN is TYPICAL.
. Limited neck rotation & Lateral bending.
. Sensory deficits due to osteophyte induced radiculopathy.
. X-ray --> ** BONY SPURS & sclerotic facet joints.
. X-ray --> ** Narrowing of disk spaces.
. X-ray --> ** Hypertrophic vertebral bodies.
. ACUTE GOUTY ATTACK:
-------------------. Tx: NSAIDs "INDOMETHACIN" ,COLCHICINE or steroids.
. ALCOHOL CESSATION & LOW PURINE DIET are imp. to prevent future attacks.

. LUMBAR STRAIN:
--------------. Related to lifting a heavy object.
. No radicular signs.
. Good response to conservative therapy.
. Pt education -----> KEEP THE BACK STRAIGHT WHILE LIFTING AN OBJECT !
. DMARDs Disease Modyfying Anti-Rheumatic Drugs:
----------------------------------------------.METHOTREXATE:
-------------. inhibits dihydrofolate reductase.
. SE: Macrocytic anemia (MCV > 100 & -- Hb).
. Other SEs: Nausea, stomatitis, rash, hepatotoxicity, Alopecia.
. HYDROXYCHLOROQUINE:
--------------------. GI distress.
. Visual disturbances.
. Hemolysis in G6PD defeciency.
. CYCLOPHOSPHAMIDE:
-----------------. Nephrotoxicity & Bladder carcinoma.
. RED FLAGS of LOW BACK PAIN:
----------------------------. = SYSTEMIC DISORDER or HERNIATED DISC,
. or BONY ABNORMALITIES such as LYTIC LESIONS or Compression #s.
. Age > 50ys.
. H/O of previous cancer.
. Unexplained weight loss.
. Pain > 1 month duration.
. Nighttime pain causing difficulty with sleep.
. No response to previous therapy.
. Neurological symptoms.
. Pain to palpation of the vertebra = spinal infection or lytic lesions in the spine.
. 1st step is X-RAY PLAIN FILM then MRI.
. SPINAL STENOSIS:
----------------. Bk pain radiating to the buttocks & thighs.
. Numbness & paresthesia may occur.
. Symptoms r worse during walking & lumbar extension,
. while lumbar flexion alleviates the pain.
. Dx: MRI.
. ILIAC ARTERY ATHEROSCLEROSIS:
------------------------------. Claudication in the buttocks & thighs.
. Pain ++ with activity & -- by rest.
. Not affected by lumbar flexion or extension.
. LUMBAR DISK HERNIATION:

------------------------. Acute onset bk pain.


. e' or e' out radiation to one leg.
. Pt recalls an incinting event e.g. lifting heavy objects.
. +ve straight leg raise test.
. ANKYLOSING SPONDYLITIS:
-----------------------. Young men < 40 ys.
. Low bk pain & stiffness.
. Worse in the morning & improves as the day progress.
. Ass. e' anterior Uveitis (Monocular pain,blurring,photophobia).
. X-ray pelvis --> SACRO-ILIITIS.
. -ve RF & +ve ESR.
. +ve HLA B 27.
. Tx: Pain relief & TNF Alpha antagonists.
. VERTEBRAL OSTEOMYELITIS:
------------------------. Injection drug user.
. Pts with sickle cell disease.
. immunocompromized pts.
. STAPHYLOCOCCUS AUREUS.
. TENDERNESS to GENTLE PERCUSSION.
. Pain not relieved by rest.
. Fever & ++WBCs --> UN-RELIABLE !
. ++ Platelet count.
. ++ ESR > 100 mm/hr.
. Dx: MRI.
. Tx: Long term IV Antibiotics.
. EPIDURAL ABSCESS:
-----------------. Enclosed infection in the epidural space.
. Bk pain, fever, chills & leukocytosis.
. More common in injection drug users.
. It may cause SPINAL CORD COMPRESSION.
. LL weakness & Urinary incontinence.
. Acute epidural abscess requires immediate surgical debridement.
. COMPRESSION #:
--------------. due to VERTEBRAL BODY DEMYELINIZATION.
. Intense focal pain.
. Without neurological symptoms.
. Occur in cases of osteomalacia or osteoporosis.
. ACUTE GOUTY ATTACK = PODAGRA:
-----------------------------. May be due to MYELO-PROLIFERATIVE disorder,
. e.g. POLYTHYCEMIA (SPLENOMEGALY & PRURITIS AFTER HOT BATHS).
. MPD "polythycemia" -> ++ Catabolism & turn over of proteins -> ++ uric acid production.
. GIANT CELL "TEMPORAL" ARTERITIS:
---------------------------------

.
.
.
.
.
.
.
.
.

Headache.
Jaw claudication.
Muscle fatigue.
Visual disturbance.
Scalp tenderness.
-- Temporal artery pulse.
++ ESR > 50 mm/hr.
May involve the branches of the AORTA ----> AORTIC ANEURYSM.
Serial CXRs are imp. to exclude Aortic aneurysm.

. REMEMBER AGAIN OA "Matet5ene2sh :)


----------------------------------. Degenerative Joint disease.
. Wear & Tear Arthritis.
. ++ by walking & -- by rest.
. Morning stiffness < 30 mins.
. limited range of movement.
. Tenderness on passive movement.
. Due to destruction of the articular cartilage --> Bone on bone friction.
. Most common risk factor is OBESITY !
. AVASCULAR NECROSIS = ASEPTIC NECROSIS = ISCHEMIC NECROSIS = OSTEOCHONDRITIS
DESSICANS:
--------------------------------------------------------------------------------------. Disruption of bone vasculature.
. Corticosteroid related osteonecrosis of the right femoral head.
. H/O Excessive Alcohol ingestion.
. Slowly progressive anterior hip pain with limitation of range of motion.
. Dx: MRI.
. D.D. of INFLAMMATORY MONOARTHRITIS: (RED HOT SWOLLEN PAINFUL LIMITED
MOVEMENT):
-------------------------------------------------------------------------------. Septic arthritis.
. Crystal induced arthritis.
. Trauma.
. Pts with RA are at high risk of developing septic arthritis esp. with STAPH. AUREUS.
. RA cause osteopenia & osteoporosis "NOT osteomalacia caused by Ca & Vit. D defeciency".
. MTx side effects:
-----------------. Stomatitis.
. Nausea.
. Anemia.
. Hepatotoxicity.
. Tx: Give FOLIC ACID !
. RA:
---. MORNING STIFFNESS > 1 hour.
. Small joints (MCP & PIP).
. Spares DIP "Unlike OA".
. Tenosynovitis (Trigger finger).
. Rheumatoid nodules (Elbow).

.
.
.
.
.

Cervical joint involvement ---> Spine sublaxation ---> Spinal cord compression.
+ve Anti-CCP Abs.
+ve RF.
++ CRP & ESR.
X-ray: Soft tissue swelling , joint sapce narrowing & bone erosions.

. Both Obstructive & Restrictive lung disease cause -- in FEV & FEV 1 !
---------------------------------------------------------------------. But .. RESTRICTIVE lung disease cause much more -- in FEV 1 than Obstructive type.
. So .. In RESTRICTIVE lung dis. FEV 1 / FEV is > 80 %.
. Examples of RESTRICTIVE causes:
-------------------------------.. Interstitial lung disease.
.. Neuromuscular diseases.
.. Chest wall abnormalities.
. Ankylosing spondylitis -> costovertebral joint fusion -> chest wall motion restriction.
. ENTHESITIS:
-----------. Inflammation & pain at ligaments & tendons attached to bone.
. Associted with Negative spondylo-arthropathies,
. e.g. ANKYLOSING SPONDYLITIS, psoriatic arthritis & reactive arthritis.
. Associated with HLA B 27.
. Most common sites are shoulder & hip.
. REACTIVE ARTHRITIS = Seronegative spondyloarthropathy:
------------------------------------------------------. TRIAD of: CAN'T SEE, CAN'T PEE, CAN'T CLIMB A TREE !!
. 1- CONJUNCTIVITIS "CAN'T SEE".
. 2- NON GONOCOCCAL URETHRITIS "CAN'T PEE".
. 3- ASYMMETRIC OLIGOARTHRITIS "CAN'T CLIMB A TREE".
. Mucocutaneous lesions.
. Enthesitis "Achilles tendon pain".
. STERILE Synovial fluid analysis.
. Tx: NSAIDs.
. Pts with prolonged H/O of ANKYLOSING SPONDYLITIS:
-------------------------------------------------. are at ++ risk of VERTEBRAL #,
. due to -- bone mineral density & may occur with minimal trauma !!
. FIBROMYALGIA:
-------------. Women 20 - 50 ys.
. Generalized musculoskeletal pain in absence of joint swelling or lab abnormalities.
. Excessive tendrness on palpation of at least 11 of 18 soft tissue locations.
. The sites include the upper quadrants of the buttocks & medial aspect of the knees.
. As well as Sternocleidomastoid & Trapezius muscles.
. Absent of joint swelling or ms weakness.
. PSORIATIC ARTHRITIS:
--------------------. DIP.
. Dactylitis --> SAUSAGE shaped digits = diffusely swollen fingers.
. Nail involvement: pitting & oncholysis "separation of nail bed".

. Well demarcated red palques with silvery scaling.


. Tx: NSAIDs & MTx.
. Steroids are contraindicated.
. N.B. RA ----> MCP & PIP.
. N.B. OA ----> DIP.
. PSEUDO-GOUT = CHONDRO-CALCINOSIS:
---------------------------------. Acute arthritis.
. Due to CPPD Calcium pyrophosphate dehydrate crystals deposition.
. H/O of recent surgery or medical illness.
. Synovial fluid analysis --> RHOMBOID shaped, POSITIVELY birefringent crystals.
. GOUT synovial fluid analysis --> NEEDLE shaped, NEGATIVELY birefringent crystals.
. BAKER CYST:
-----------. Due to excessive fluid production by an inflammed synovium.
. Occurs in cases of Rheumatoid Arthritis.
. Excess fluid accumulates in the popliteal bursa which expands,
. creating a tender mass in the popliteal fossa.
. May burst & release their contents into the calf,
. resulting in an appearance similar to DVT.
. AMYLOIDOSIS:
------------. Ass. with Nephrotic $ (facial swelling, LL edema, massive proteinuria).
. Palpable kidneys.
. Hepatomegaly.
. Cardiomegaly (Audible S4).
. H/O of chronic infections e.g. Bronchiectasis & recurrent pulm. infections.
. Tx: COLCHICINE.
. OSTEO-ARTHRITIS:
----------------. Age > 50 ys.
. Morning stiffness < 30 mins.
. Bony tendrness.
. Bony enlargement.
. CREPITUS on active motion.
. No warmth i.e. COOL joint !
. SLE Arthritis:
--------------. Like RA but --------> NO PERMANENT DEFORMITIES.
. SEPTIC ARTHRITIS:
-----------------. H/O of PROSTHETIC joint.
. Red, hot, swollen, painful joint with limited range of motion.
. ++ WBCs > 50000.
. STAPH. AUREUS is the most common causative organism.
. Disseminated Gonococcemia:
---------------------------

. H/O of recent unprotected sex with a new partener.


. A triad of Polyarthralgia + Tenosyvovitis + Vesiculo-pustular skin lesions.
. PAGET's disease of bone:
------------------------. Age > 40 ys.
. ++ bone turn over.
. ++ OSTEOCLAST ACTIVITY.
. Mosaic pattern of lamellar bone.
. Enlarged cranial bone --> ++ hat size.
. Entrapment of 8th cranial nerve --> Deafness.
. X-ray --> Femoral bowing.
. NORMAL CALCIUM & PHOSPHOROUS LEVELS.
. ++ ALKALINE PHOSPHATASE LEVEL.
. NEUROGENIC ARTHROPATHY = CHARCOT's JOINT:
-----------------------------------------. Due to DIABETIC NEUROPATHY.
. H/O of DM is the key word.
. -- pain, proprioception & temperature.
. caused by D.M., peripheral nerve damage, syringomyelia & B12 defeciency.
. X-ray ---> Loss of cartilage, osteophytes formation & loose bodies.
. Tx: ttt the cause & special shoes !
. GOUT X-ray ---> PUNCHED OUT EROSIONS + Overhanging rim of cortical bone.
Dr. Wael Tawfic Mohamed
-------------------------

SURGERY TIKI TAKA


___________________
. TRAUMA:
__________
__________
(1) AIRWAY:
____________
. Establishing & securing the airway is always the 1st step in management.
. Altered mental status is the most common indication for intubation in a trauma pt.
. As an unconscious pt can't maintain his airway.
. The preferred method of securing an airway -> OROTRACHEAL INTUBATION.
. Trauma with cervical spine injury -> FLEXIBLE BRONCHOSCPE.
. Extensive facial trauma & bleeding into airway -> CRICOTHYROIDOTOMY or TRACHEOSTOMY.
.
.
.
.

N.B.
Pts with cervical spine injury should 1st have stabilization of the cervical spine.
Oro-tracheal intubation with rapid sequence intubation is the preferred way,
to secure an airway in an apnein pt with a cervical spine injury.

.
.
.
.
.
.
.
.

N.B.
In burn victims, clinical indicators of thermal inhalation injury to the upper airway,
or smoke inhalation injury to the lungs include burns on face, singing of eye brows,
oropharyngeal inflammation & blistering, oropharyngeal carbon deposits,
carbonaceous sputum, stridor, carboxyhemoglobin level > 10 %.
H/O of confinement in a burnung buiding.
The presence of one or more of these indicators warrants early intubation,
to prevent upper airway obstruction by edema.

(2) BREATHING:
_______________
. Check oxygen saturation, if SpO2 < 90 %:
-> ++ oxygen concentration & flow rate.
-> Obtain an ABG.
-> Determine the likely cause of hypoxia from H/O.
(3) CIRCULATION:
_________________
* CHEST TRAUMA (Hypovolemic shock - Pericardial tamponade - Tension pneumothorax):
___________________________________________________________________________________
___________________________________________________________________________________
-> HYPOVOLEMIC SHOCK:
______________________
. The most common type of shock.
. Pale, cold , shivering pt with diaphoresis, hypotension & tachycardia.
. Look for a source of bleeding.
. The pt may lose a large volume of blood in the abdomen or thigh following femur #.

.
.
.
.

N.B.
When hemorrhage occurs, tachycardia & peripheral vasoconstriction are the 1st changes.
These responses act to maintain the blood pressure within normal limits.
PULSE CHANGE IS THE FIRST INDICATOR FOR HYPOVOLEMIA.

-> Pericardial tamponade:


__________________________
. Cause distended neck veins & high central venous pressure.
. Enlarged heart on CXR (May be normal cardiac silhouette).
. Electrical alternans on EKG.
. Pulsus paradoxus on vital signs.
. Tx -> immediate pericardiocentesis tap or pericardial window.
.
.
.
.
.
.

N.B.
Acute cardiac tamponade:
occurs due to a sudden rise in intra-pericardial pressure.
Should be suspected in all adult pts with blunt chest trauma.
Jugular venous distension, Tachycardia & Hypotension despite aggressive fluid resusc.
CXR findings typically reveal a normal cardiac silhouette without tension pneumothorax.

-> Tension pneumothorax:


_________________________
. Cause distended neck veins & high central venous pressure.
. Respiratory distress, tracheal deviation, absent breath sounds.
. Hyperresonance to percussion.
. Tx -> immediate placing of a large-bore needle or IV catheter into the pleural space.
. Chest tube placement.
. Never wait for a CXR for diagnosis.
.
.
.
.

N.B.
Don't be distracted by head trauma or dilated pupils in a hypotensive trauma pt.
Intracranial bleeds are never the cause of hypotensive shock.
The 1st step in management is to identify & control the site of bleeding.

.
.
.
.
.
.

N.B.
Most causes of shock in the setting of trauma are 2ry to hypovolemia from blood loss.
However, ++ CVP/PCWP or failure of hypotension to resolve after a bolus of IV fluids,
should suggest an alternative diagnosis.
Myocardial contusion sh'd be suspected in pts with evidence of injury to anterior chest
MI can be confirmed with +ve cardiac markers & EKG changes.

. Tension pneumothorax is excluded if there is no tracheal deviation.


. Hypovolemia is excluded if there is failure to respond to an IV fluid bolus.
.
.
.
.
.
.

N.B.
High energy blunt trauma to the chest commonly causes aortic injury.
In most cases of aortic rupture, death is the immediate result.
Widened mediatinum, large left sided hemothorax & mediastinal deviation to right side.
Disruption of the normal aortic contour..
Bilateral COLLAPSED neck veins.

. Cardiac contusion & rupture cause pericardial tamponade only.


. Muffled heart sounds - Hypotension - DISTENDED NECK VEINS.

. N.B. PULMONARY CONTUSION:


____________________________
. Represents pulmonary bruising of the lung.
. Common after high-speed car accidents.
. Clinical manifestations develop in the 1st 24 hours (Often within few minutes).
. Tachypnea - Tachycardia - Hypoxia.
. P/E -> Chest wall bruising & -- breath sounds on the side of pulmonary contusion.
. CXR -> Patchy irregular alveolar infiltrate.
. ABG -> Hypoxemia.
. It is very important to differentiate pulmonary contusuion from ARD$.
. ARD$ manifests 24 - 48 hours from the trauma & BILATERAL involvement.
. Pulmonary contusion manifests in the 1st 24 hours.
.
.
.
.
.

N.B. PNEUMOTHORAX:
Primary spontaneous pneumothorax -> No preceiding event & No H/O of lung disease.
Secondary spontaneous pneumothorax -> Complication of underlying COPD.
Tx -> Small ( < 2cm between lung & chest wall on CXR) -> Observation & oxygen.
Tx -> Large (Stable) -> Needle aspiration or chest tube.

. Tension pneumothorax:
________________________
. Life threatening; trapped air with mediastinal shift.
. Compromised cardiopulmonary function.
. Chest pain or dyspnea.
. -- Breath sounds / -- TVF / -- chest movement.
. Hyperresonance to percussion on the affected side.
. Tachycardia, hypotension.
. Tracheal deviation away from the affected side.
. Imaging -> Notable visceral pleural line.
. Imaging -> Air in hemithorax -> Contralateral mediastinal shift.
. Imaging -> Radiolucent costophrenic sulcus.
. Tx -> Urgent needle decompression then chest tube placement (Tube thoracostomy).
. Tx -> IV lines & fluid resuscitation follow urgent needle decompression.
. N.B.
. ONLY TWO CAUSES OF DISTENDED NECK VEINS -> TENSION PNEUMOTHORAX & CARDIAC
TAMPONADE.
. N.B.
. In HEMOTHORAX -> Neck veins are COLLAPSED !
. N.B. FLAIL CHEST:
____________________
. Follows major thoracic trauma.
. Multiple contigious ribs are fracutred in two or more locations.
. Causing a segment of rins losing its continuity with the rest of thoracic wall.
. The pt takes shallow breaths due to severe pain.
. The pt compensates for the hypoxemia by hyperventillation.
. Tachypnea & PARADOXICAL THORACIC WALL MOVEMENTS.
. Paradox is corrected with positive pressure mechanical ventillation.
. The isolated thoracic wall segment exhibits paradoxical motion,
. inward motion on inspiration & outward motion during expiration !
. Tx -> Pain control & supplemental oxygen are the most important steps.

. Positive pressure mechanical ventillation replaces the negative intrapleural pressure.


. so the flail chest movement will be normal with the rest of the rib cage on inspiration
. N.B. HEMO-THORAX:
____________________
. After blunt chest trauma, hemorrhagic shock associated e'
. decreased breath sounds & dullness to percussion over one hemithorax.
. & contralateral tracheal deviation.
. COLLAPSED NECK VEINS.
. Most common cause is damage to intercostal or internal mmamary artery.
* ABDOMINAL TRAUMA:
____________________
. The 1st step in management is always to control the site of bleeding if known.
-> Apply direct pressure when the site is visible (e.g. extremity).
-> Blind clamping & the use of tourniquet is NEVER the answer.
. The next priority is FLUID RESUSCITATION.
. Do several things at once in preparation for immediate exploratory laparotomy:
-> Set up 2 large gauge IV lines.
-> Give fluids & blood.
-> Insert Foley catheter.
-> Administer IV antibiotics.
. If surgery isn't needed (blunt trauma),
. fluid resuscitation is the 1st step in management (Also diagnostic).
. If the pt responds promptly, then he's propably no longer bleeding.
. N.B.
. Intraosseous cannulation in the proximal tibia is used in children (generally < 6ys).
. Give an initial bolus of Ringer's lactate at 20 ml/kg of body weight.
. N.B. BLUNT ABDOMINAL TRAUMA (BAT):
_____________________________________
. After a car accident of a restrained driver.
. Usually occurs when a lap belt (without shoulder attachment) compresses the abdomen,
. and lacerates solid organs most commonly the spleen & liver.
. Hypotension, tachycardia, facial lacerations & abdominal wall ecchymosis.
. Most reliable symptoms -> Abdominal pain, tendrness & peritoneal signs.
. Intraabdominal injury sh'd be suspected in pts with:
. abdominal wall ecchymosis,abdominal distension & hyperactive bowel sounds.
.
.
.
.
.
.
.

1st step after fluid resuscitation to determine if the pt needs exploratory laparotomy.
All pts with BAT sh'd 1st be assessed for intraperitoneal free fluid or hemorrhage.
Best test is -> BEDSIDE ULTRASONOGRAPHY to detect free intraperitoneal fluid,
in hepatorenal space, splenorenal recess & inferior portion of intraperitoneal cavity.
When combined with pericardial evaluation -> known as FAST.
FAST exam -> (Focused assessment with sonography for trauma).
It is the best to detect hemoperitoneum, pericardial effusion or intraperitoneal fluid.

. If FAST exam is limited or equivocal -> A diagnostic peritoneal lavage (DPL) is done.
. DPL is done to evaluate for hemoperitoneum.
. Pts with +ve findings on either FAST or DPL -> should undergo exploratory laparotomy.

. Hemodynamically stable pts with -ve findings on FAST may undergo abdominal CT,
. to determine need for laparotomy.
. Hemodynamically un-stable -> FAST or DPL.
.
.
.
.
.

N.B.
Blunt abdominal trauma to the upper abdomen can cause pancreatic contusion,
crush injury, laceration or transection to the pancreas.
Pancreatic injuries may be MISSED by CT scan during the 1st 6 hours following trauma.
Untreated pancreatic injury can be complicated by retroperitoneal abscess or pseudocyst

.
.
.
.
.
.
.
.
.
.

N.B.
The spleen is the most commonly injured organ following blunt abdominal trauma.
Left upper quadrant abdominal pain.
Abdominal wall contusion, Lt lower chest wall tendrness.
Lt shoulder pain referred from splenic hemorrhage irritating phrenic nerve & diaphragm.
It is called "KEHR" sign.
Splenic rupture causes acute left upper quadrant abdominal pain.
Delayed hypotension may result due to blood loss.
No signs of sepsis will be present.
Dx -> Abdominal CT with IV contrast.

.
.
.
.

N.B.
Blunt deceleration trauma (Motor vehicle accident or fall from > 10 feet):
Blunt aortic trauma must be ruled out.
CXR is the initial screening test -> WIDENING of the mediastinum.

. N.B.
. Duodenal hematoma:
_____________________
. mostly follow abdominal blunt trauma in children.
. The hematoma may cause duodenal obstruction with nausea & vomiting.
. Epigastric pain & vomiting due to failure to pass gastric secretions past obstruction.
. Tx -> NASOGASTRIC SUCTION & PARENTERAL NUTRITION.
. Most hematomas will resolve spontaneously in 1-2 weeks.
. N.B.
. Any gun shot wound below the 4th intercostal space (level of the nipple) is:
. considered to involve the abdomen & requires an exploratory laparotomy in unstable pts.
.
.
.
.

N.B.
All hemodynamically UN-STABLE pts with penetrating abdominal trauma,
must undergo immediate exploratory laparotomy to diagnose & treat source of bleeding
as well as to diagnose & treat perforation of any abdominal viscus to prevent sepsis.

.
.
.
.

N.B.
Abdominal CT used to detect intra-abdominal injury in hemodynamically stable trauma pts
In hemodynamically un-stable pts, a FAST U/$ should be the initial test.
DPL Diagnostic peritoneal lavage is used in hemodynamically unstable pts if -ve FAST.

. N.B.
. DIAPHRAGMATIC TRAUMA:
________________________
. Blunt abdominal trauma -> Mild respiratory distress & Abnormal CXR.

.
.
.
.
.
.
.

Sudden ++ in intra-abdominal pressure -> Large radial tears in the diaphragm.


Rupture is more common on LEFT side bec. the right side is protected by the liver.
Dx -> CXR -> Hemi-diaphragmatic elevation.
Dx -> CXR -> Naso-gastric tube in the pulmonary cavity = Diaphragmatic hernia.
Dx -> CT is the next best step (to Confirm).
The small bowel may be present in the thoracic cavity.
Tx -> Surgical repair & exploration for other traumatic injuries.

. N.B.
. TRACHEO-BRONCHIAL RUPTURE:
_____________________________
. Due to rapid decceleration blunt chest trauma.
. 1st manage the ABCs.
. Dx -> CXR -> Persistent pneumothorax & pneumomediastinum despite chest tube placement !
. Subcutaneous emphysema (Palpable crepitus below the skin).
. The RIGHT MAIN BRONCHUS is the most commonly injured.
. Dx -> High resolution CT scan (Confirm).
. Tx -> Surgical repair.
. BLUNT ABDOMINAL TRAUMA MANAGEMENT:
_____________________________________
_____________________________________
. HEMODYNAMICALLY UN-STABLE PATIENT:
_____________________________________
-> Cervical spine immobilization.
-> Intravenous hydration.
-> FAST (Focused assessment with sonography for trauma).
-> If FAST is +ve for blood & pt is still UNSTABLE AFTER A TRIAL OF FLUID RESUSCITATION,
-> URGENT LAPAROTOMY with surgical repair is indicated.
. HEMODYNAMICALLY STABLE PATIENT:
__________________________________
-> CT scan abdomen with contrast (to detect the amount of bleeding & the site of injury).
-> The surgeon can then select either laparotomy or admission & observation.
. MANAGEMENT OF BLUNT ABDOMINAL TRAUMA
_______________________________________
. in HEMODYNAMICALLY UN-STABLE PT
__________________________________
|
FAST EXAMINATION
________________
|
____________________________________________
|
|
|
+ve
inconclusive
-ve
|
|
|
LAPAROTOMY <--POSITIVE--- DPL ----NEGATIVE---> Signs of
____________
_____
extra-abdominal
hemorrhage (Pelvic/long bone #)
|
___YES_____________NO_____

|
|
STABILIZE
STABILIZE
ANGIOGRAPHY & SPLINT
then CT ABDOMEN

* VASOMOTOR SHOCK:
___________________
. Hypotension & tachycardia in pts who are warm & flushed (Not pale & cold!).
. Look for a H/O of medication use (penicillin allergy).
. H/O of spinal anesthesia or exposure to allergen (bee stings).
* TRAUMA TO LOCALIZED SITES:
_____________________________
. All penetrating wounds with damage to internal organs will need to go to the OR.
. If the case describes an object embedded in the pt, NEVER to remove it.
. Never remove it in the ER or at the scene of the accident (Only in the operating room).
* HEAD TRAUMA:
_______________
. "No" surgical intervention is needed for ..
. an asymptomatic head injury with a closed skull # (No overlying wound) alone.
. The next step of management is to clean any lacerations.
. Surgery "Repair or craniotomy" is always done for ..
. COMMINUTED or DEPRESSED SKULL # even if the pt is asymptomatic !
. Send the pt to the OR.
.
.
.
.
.

For head trauma & loss of cosciousness


The 1st step of management is ordering a HEAD & NECK CT with "OUT" contrast.
If the head CT & neurological exam are normal,
he can go home if someone can closely observe him over the next 24 hours.
i.e. wake him up frequently & watch for changes in mentation.

. Give tetanus toxoid & prophylactic antibiotics to all pts with open skull #s.
* BASAL SKULL #:
_________________
. Ecchymosis around both eyes (Racoon eyes).
. Ecchymosis behind the ear (Battle's sign).
. Clear fluid drippling from the ear or nose (CSF leak).
. CT scan of head & neck -> Basal skull #. "X-ray is a wrong answer".
. A CSF leak will stop by itself & requires no specific management.
. Prophylactic antibiotics are NOT indicated !!
. Facial palsy may occur 2-3 days later due to neuroapraxia (Use Steroids).
* EPI-DURAL HEMATOMA:
______________________
. Side head trauma & rupture of middle meningeal artery in the foramen spinosum.
. H/O of head trauma & SUDDEN LOSS OF CONSCIOUSNESS.

.
.
.
.
.
.
.
.

Accumulation of blood in the potential space inbetween the cranium & dura matter.
Honeymoon period (The period when the pt immediately awakes & appears normal).
Pt typically has ipsilateral pupil dilatation due to oculomotor nerve compression.
Then the pt quickly deteriorates, so .. It is important to manage quickly.
Dx -> CT scan -> BICONVEX LENS shaped hematoma with or without midline deviation.
Tx -> EMERGENCY CRANIOTOMY.
If the pt is treated, the prognosis is good.
If not, the prognosis is fatal within hours.

.
.
.
.
.
.

Epidural hematoma results from rupture of middle meningeal artery,


higher arterial pressure can rapidly expand the hematoma -> Compress the temporal lobe.
Fluid resuscitation ++ the rate at which the epidural hematoma expanded.
Hypertension, bradycardia & respiratory depression (Cushing's reflex) = ++ ICP.
The uncus is the innermost part of the temporal lobe & herniated through the tentorium,
leading to the following pressure effects:

. TRANS-TENTORIAL (UNCAL) HERNIATION:


______________________________________
______________________________________
. Compression of the contralateral crus cerebri against the tentorial edge:
____________________________________________________________________________
. Ipsilateral hemiparesis.
. Compression of the ipsilateral oculomotor nerve (CN 3) by the herniated uncus:
_________________________________________________________________________________
. Loss of parasympathetic innervation causes mydriasis.
. Loss of motor innervation causes ptosis & down-outwards gaze of the ipsilateral pupil,
. due to un-opposed trochlear (CN 5) & abducent (CN 6).
. Compression of the ipsilateral posterior cerebral artery:
____________________________________________________________
. causes ischemia of the visual cortex -> Contralateral homonymous hemianopia.
. Compression of the reticular formation:
__________________________________________
. Altered level of consciousness; coma.
* "S"UB-DURAL HEMATOMA:
________________________
. Low pressure bleeding from the "VENOUS SYSTEM".
. Accumulation of blood in the subdural space between the dura & arachinoid membrane.
. Head trauma with FLUCTUATING CONSCIOUSNESS i.e.
. gradual headaches, memory loss, personality changes, dementia, cofusion & drowsiness.
. Dx -> CT scan -> "S"EMILUNAR, CRESCENT shaped hematoma e' or e'out midline deviation.
. Tx -> CONSERVATIVE management with STEROIDS.
. Emergency craniotomy is done if there are lateralizing signs & midline displacement.
* DIFFUSE AXONAL INJURY:
_________________________
. Results from ACCELERATION-DECELERATION injuries to the head.
. The pt will be deeply unconscious.
. Dx -> CT -> Normal or diffuse small bleeds at the junctions of the grey & white matter.

.
.
.
.

CT -> Numerous punctate hemorrhages.


Prognosis is terrible !
Surgery can't help.
Therapy is directed at preventing further injury from ++ ICT.

* ELEVATED INTRACRANIAL PRESSURE (++ ICP):


___________________________________________
. Briefly depressed consciousness after head trauma.
. Improvement.
. Progressive drowsiness.
. ++ ICT is a medical emergency.
. GRADUAL DILATATION OF ONE PUPIL & DECREASING RESPONSIVENESS TO LIGHT is an imp.
sign.
. It indicates clot expansion on the ipsilateral hemisphere.
. Dx -> Head CT -> Midline shift or dilated ventricles.
. Don't think about performing a lumbar tap in any pt before getting a head CT 1st !
. If you perform a lumbar puncture on a person with ++ ICT, you'' herniate the brain !
. Tx -> Head elevation - Hyperventillation - Avoid fluid overload.
. Tx -> Mannitol & furosemide (use very cautiously as they can reduce cerbral perfusion).
. Tx -> sedation & hypothermia may lower oxygen demand.
. N.B. Lowering ICP is not the ultimate goal; preserving brain perfusion is.
. Systemic hypotension or excessive cerebral vasoconstriction may be counterproductive.
. N.B. Steroids are good for cerebral edema 2ry to tumors & abscesses,
. But they have no role in head trauma pts !
. N.B. Pts with mild to moderate traumatic brain injury:
. can be discharged under the care of an adult if they have a NORMAL CT.
. The caretaker sh'd be given printed instructions detailing signs & symptoms that,
. warrant immediate return to the hospital.
__________________________________________________________________________________________
. ACUTE ABDOMEN:
_________________
_________________
. 4 main causes -> Perforation - Obstruction - Inflammation/Infection - Ischemia.
. When is "SURGEY" the answer ?
________________________________
1. Peritonitis (Exclude primary peritonitis).
2. Abdominal pain/tendrness + sepsis signs.
3. Acute intestinal ischemia.
4. Pneumoperitoneum.
. In all of the above cases, make sure pancreatitis is 1st ruled out !
. N.B. Primary peritonitis is spontaneous inflammation with nephrosis in children.
. or .. An adult with ascites & mild abdominal pain (even there is fever & leukocytosis).
. When is "MEDICAL ttt" the answer ?
_____________________________________

1. Primary peritonitis.
2. Pancreatitis.
3. Cholangitis.
4. Urinary stones (Look for stones on X-ray).
5. Things that can mimic an acute abdomen:
-> Lower lobe pneumonia (Look for infiltrate on CXR).
-> Myocardial ischemia (Look for EKG changes).
-> Pulmonary embolism (Look for immobilized pt).
6. Ruptured ovarian cyst.
. N.B.
. Cholangitis is a GIT medical emergency & intervention with ERCP is the ttt of choice.
. NON-surgical causes of an acute abdomen:
___________________________________________
1. Myocardial infarction - acute pericarditis.
2. Lower lobe pneumonia - pulmonary infarction.
3. Hepatitis - GERD.
4. DKA - Adrenal insuffeciency.
5. Pyelonephritis - Acute salpingitis.
6. Sickle cell crisis.
7. Acute porphyria.
. N.B.
. Be sure to differentiate GERD from peptic ulcer perforation (surgical emergency).
* 1 * PERFORATION:
___________________
___________________
(1) GASTRO-INTESTINAL PERFORATION:
__________________________________
. Acute abdominal pain that is sudden, severe, constant & generalized.
. Pain is excruciating with any movement (may be blunted in elderly pts).
. Most common causes of GIT perforations:
(a) Diverticulitis: Elderly pt with lower abdominal pain & fever.
(b) Perforated peptic ulcer: Epigastric pain waking up the pt at night.
(c) Chron's disease.
.
.
.
.
.

Dx -> Supine & erect abdominal x-ray (free air under diaphragm).
Tx -> Nothing by mouth (NPO) & IV fluid hydration.
Tx -> IV antibiotics such as flagyl & gentamycin.
Tx -> IV 2nd generation cephalosporins (Cefotetan or cefoxitin).
Tx -> Emergency surgery.

(2) ESOPHAGEAL PERFORATION:


___________________________
. Most common cause is IATROGENIC.
. Pain in chest or upper abdomen.
. Dysphagia or odynophagia.
. S.C. emphysema shortly after endoscopy.
. It is a surgical emergency.
. Dx -> GASTROGRAFFIN CONTRAST ESOPHAGOGRAM is the best (Do NOT use Barium xx).

* 2 * OBSTRUCTION:
___________________
___________________
. Severe colicky pain.
. Absence of flatus or feces.
. Nausea & vomiting.
. Constant movement as the pt tries to find a comfort position.
.
.
.
.
.

H/O
H/O
H/O
H/O
H/O

of prior surgery (Think adhesions).


of elderly pt with anemia, weight loss & melanotic stools (Think tumor).
of recuurent lower abdominal pain (Think diverticulitis).
of hernia (incarcerated hernia).
of sudden abdominal pain in elderly pt (Think volvulus).

. Dx -> CBC & ++ lactate level.


. Dx -> Supine & erect abdominal X-ray:
-> Dilated loops of bowel, absence of gas in rectum, bird's beak sign for volvulus.
. Tx -> NPO, (NG) suction & IV fluid hydration.
. Consider Gastrograffin contrast study (Until perforation has been ruled out).
. Volvuls -> Perform procto-sigmoidoscop with rigid instrument.
-> Leave the rectal tube in place.
-> Perform sigmoid resection for recurrent cases.
. Abdominal hernia -> Perform elective repair for all abdominal hernias.
-> except umbilical hernia in pts < 2 ys.
-> except esophageal sliding hernia.
. All other obstructions -> Perform emergency surgery.
. N.B.
. In a pt with a hernia, immediate surgery is the answer if the case describes:
. fever, leukocytosis, constant pain & signs of peritoneal irritation (Strangulation).
.
.
.
.

N.B.
Complete small bowel obstruction
Nausea - vomiting - Abdominal bloating - Dilated loops of bowel on abdominal x-ray.
Adhesions are the most common etiology.

. N.B.
. SMALL BOWEL OBSTRUCTION:
___________________________
. Colicky abdominal pain & vomiting.
. No bowel movement or passing gas (Obstipation), abd. distension & diffuse tendernesss.
. The contents of the vomitus are typically bilious in proximal SBO.
. The contents of the vomitus are feculent with more distal obstructions.
. Hyperactive bowel sounds due to peristaltic rush.
. Dx -> Abd. x-ray -> DILATED BOWEL LOOPS with MULTIPLE AIR FLUID LEVELS.
. Tx -> Complete bowel rest - Decompression e' nasogastric tube.
. Tx -> Pain control - Fluid resuscitation.
. Tx -> If no improvement -> Surgical intervention to avoid strangulation.
. Strangulation signs (fever - tachycardia - leukocytosis - Metabolic acidosis).
. N.B.
. Immediate surgical intervention is indicated for pts with intestinal obstruction who,

. develop clinical or hemodynamic instability, fail to improve after conservative ttt,


. or develop syms of strangulation (fever-tachycardia-leukocytosis-Metabolic acidosis).
* 3 * INFLAMMATION:
____________________
____________________
. Causes (Acute diverticulitis - Acute pancreatitis - Acute appendicitis).
. Gradual onset of constant abdominal pain that slowly builds up over several hours.
. Initially ill defined pain that becomes localized to the site of inflammation.
. Note that signs of peritoneal irritation are ABSENT in pancreatitis.
(1) ACUTE DIVERTICULITIS:
__________________________
. Acute abdominal pain in the LEFT LOWER QUADRANT (LLQ).
. Middle age or older pt with fever, leukocytosis & peritoneal irritation in the LLQ.
. Palpable tender mass in the LLQ.
. In women, think about fallopian tubes & ovaries as potential sources.
. Dx -> CT -> Abscess & free air.
. Never order contrast studies or endoscopy in acute phase.
. Tx -> If there is no peritoneal signs -> Manage as outpatient with antibiotics.
. Localized peritoneal signs & abscess -> Admit pt - NPO - IV fluids - IV antibiotics.
. Generalized peritonitis or perforation -> Emergency surgery.
. Recurrent attacks of diverticulitis -> Elective surgery.
. N.B. When diagnosing acute diverticulitis,
. don't forget to order a urine pregnancy test on all women of childbearing age.
.
.
.
.

Complicated diverticulitis: Associated e' abscess, perforation, obstruction or fistula.


Fluid collection < 3cm -> IV antibiotics & observation.
Fluid collection > 3cm -> CT guided percutaneous drainage.
If no response within 5 days -> Surgery for drainage & debridement.

(2) ACUTE PANCREATITIS:


________________________
. Alcoholic pt who develops an acute (over several hours) upper abdominal pain,
. radiating to the back, with nausea & vomiting.
. It may be edematous, hemorrhagic or suppurative (pancreatic abscess).
. Late complications include pancreatic pseudocyst & chronic pancreatitis.
. Dx -> Serum or urinary amylase or lipase (serum 12 - 48 hs, urinary 3rd - 6th day).
. Dx -> CT if diagnosis is uncertain.
. Tx -> NPO, NG suction & IV fluids.
. N.B. Look out for the risk factors for acute pancreatitis:
-> Alcoholism.
-> Gall stones.
-> Medications (Didanosine, pentamidine, Flagyl, Tetracycline, Thiazides & Furosemide).
-> Hypertriglyceridemia.
-> Trauma.
-> Post-ERCP.
. N.B. COMPLICATIONS:
-> Abscess:
. Often appears 10 days after onset with persistent fever & high WBC count.

. Surgical drainage is the ttt.


-> Pseudocyst:
. Appears 5 weeks after initial symptoms.
. when a collection of pancreatic juice causes anorexia, pain & a palpable mass.
. If < 6 cm & present < 6 weeks -> OBSERVATION.
. If > 6 cm or present > 6 weeks -> Percutaneous drainage or endoscopic drainage.
-> Chronic damage:
. causes diabetes & steatorrhea.
. Treat with insulin & pancreatic enzyme supplementation.
.
.
.
.

N.B. The most common causes of acute pancreatitis are gallstones & alcohol use.
Identifying the underlying cause can prevent recurrent pancreatitis.
ULTRASOUND is the preferred test to detect gall stones.
Stable pts sh'd undergo cholecystectomy for biliary pancreatitis prior to discharge.

(3) ACUTE APPENDICITIS:


________________________
. Begins with anorexia.
. Followed by vague peri-umbilical pain.
. several hours later, it becomes sharp, severe, constant & localized to RLQ of abdomen.
. RIGHT LOWER QUADRANT PAIN.
. Tenderness, guarding & rebound tendrness are found on the right & below the umbilicus.
. Dx -> Fever, leukocytosis 10000 - 15000 with neutrophilia & immature forms.
. Dx -> Reactive thrombocytosis.
. Dx -> Abdominal U/$ or CT scan if clinically unclear.
. Tx -> IV antibiotics before appendectomy.
. Tx -> If appendix is perforated -> Continue IV until fever & WBC count normalize.
. N.B. APEENDICEAL PERFORATION complicated by PSOAS ABSCESS:
_____________________________________________________________
. Localized Rt lower quadrant findings > 5days after onset of appendicitis.
. perforation occurs with abscees formation.
. Psoas abscess -> Flexion of the hip against resistance (Psoas sign).
. Tx -> IV hydration - Antibiotics - Bowel rest - Interval appendectomy after 6-8 weeks.
. N.B. APPENDICEAL PERFORATION complicated by PELVIC ABSCESS:
______________________________________________________________
. Rupture of appendix with pelvic abscess formation.
. Drainage of fluid into the dependent recto-vesical pouch.
. Tender, fluctuant mass palpable only e' the tip of finger = recto-vesical pouch abscess
. Fever, lukocytosis, painful defecation & diarrhea.
. Tx -> Abscess drainage.
. N.B. CHRONIC ULCERATIVE COLITIS (CUC):
_________________________________________
. CUC is managed medically.
. Elective surgery is done in the following conditions:
-> Disease is present > 20 ys "High incidence of malignant degeneration".
-> Multiple hospitalizations.
-> Pt needs chronic high dose steroids or immunosuppressants.
-> Toxic megacolon (Abd. pain - fever - leukocytosis - epigastric tendrness).
-> Massively distended transverse colon on X-rays with gas within the wall of the colon.

* 4 * ISCHEMIA:
________________
________________
. Acute mesenteric ischemia in older pts.
. H/O of arrhythmia (Af -> Absence of P waves with irregular rhythm).
. Coronary artery disease.
. Recent MI.
. Severe acute onset abdominal pain that is out of proportion to exam.
. Dx is clinical but look for acidosis & sepsis signs.
. If ischemia is suspected, don't w8 for lab findings (acidosis or ++ lactate),
. Go straight to surgery or order angiography.
. If diagnosis is during SURGERY -> Perform embolectomy & revascularization or resection.
. If diagnosis is during ANGIOGRAPHY -> Give vasodilators or thrombolysis.
. Acute embolic mesenteric ischemia may progress to bowel infarction.
. N.B. INTRA-ABDOMINAL ABSCESS:
________________________________
. H/O of previous operation, trauma or intra-abdominal infection/inflammation.
. Abscesses can occur anywhere in the abdomen or retroperitoneum.
. Dx -> CBC & contrast CT of abdomen or pelvis.
. Tx -> Drain an intraabdominal abscess (either surgically or percutaneously).
. Tx -> Give antibiotics to prevent spread of infection (Doesn't cure abscess).
.
.
.
.
.
.

Ex. PSOAS ABSCESS:


It is not an exclusive complication of appendicitis.
It may present alone (Absence of rebound tenderness of appendicitis !).
Presence of multiple furuncles on the inner thighs is a clue of septic focus !
Dx -> CT Abdomen.
Tx -> Surgical or percutaneous drainage.

. N.B. BOWEL ISCHEMIA:


_______________________
. One of the complications of abdominal aortic aneurysm repair surgery.
. Due to inadequate colonic collateral arterial perfusion to the left & sigmoid colon.
. Due to loss of the inferior mesenteric artery during aortic graft placement.
. Abdominal pain (Dull pain over ischemic bowel) & bloody diarrhea (Hematochezia).
. Fever & leukocytosis.
. Colonoscopy -> Discrete segment of cyanotic & ulcerated bowel.
. Prevention -> Checking sigmoid colon perfusion following graft placement.
. SURGICAL JAUNDICE:
____________________
____________________
* OBSTRUCTIVE JAUNDICE CAUSED BY STONES:
_________________________________________
. Obese, fecund woman in her 40s.
. Recurrent episodes of abdominal pain.
. High alkaline phosphatase.
. Dilated ducts on sonogram.
. Non-dilated gall bladder full of stones.
. Dx -> Abdominal U/$.

.
.
.
.

Dx -> Confirm e' endoscopic ultrasound (EU$).


Dx -> Confirm e' Magnetic resonance cholangiopancreatography (MRCP).
Tx -> Perform Endoscopic retrograde cholangiopancreatography (ERCP).
Tx -> Cholecystectomy sh'd follow ERCP.

. N.B. ERCP & EU$ are never the 1st step in diagnosis.
. N.B. ERCP is mostly a management step on exam.
* OBSTRUCTIVE JAUNDICE CAUSED BY TUMOR:
________________________________________
. Progressive symptoms in the preceeding weeks & weight loss.
. Adenocarcinoma at the head of pancreas.
. Adenocarcinoma at the ampulla of Vater.
. Cholangiocarcinoma arising in the common bile duct itself.
. Dx -> Abdominal U/$.
. Dx -> CT scan.
. Dx -> For lesions on CT -> Obtain a tissue diagnosis via EU$.
. Dx -> If no lesions on CT -> Order MRCP.
. MRCP -> will show the ampullary or common bile duct tumors not seen on CT scan.
. Obtain tissue diagnosis via ERCP.
. Tx -> Surgical resection.
. GALL STONES:
_______________
_______________
(1) BILIARY COLIC:
___________________
. TEMPORARY occlusion of the CYSTIC DUCT.
. Colicky pain in the upper right quadrant RUQ.
. Radiating to the right shoulder & back.
. Often triggered by fatty food.
. Episodes are brief (20 mins).
. No signs of peritoneal irritation or systemic signs.
. Dx -> U/$.
. Tx -> Elective cholecystectomy.
.
.
.
.
.
.

N.B. Ingestion of a fatty meal causes the gall bladder to contract.


so .. it presses the gall stones against the cystic duct opening,
increasing the intra gall bladder pressure causing VISCUS DISTENSION & colicky pain.
Subsequent gall bladder relaxation alows the stone to fall back from the duct,
causin the pain to resolve completely.
Biliary colic pain may be referred to right shoulder.

. N.B. Pain of biliary colic is distinguished from that of acute cholecystitis by:
. its intermittent nature & relation to meals as well as absence of fever.
(2) ACUTE CHOLECYSTITIS:
_________________________
. PERSISTENT occlusion of the CYSTIC DUCT.
. Caused by a stone.
. Constant pain.
. Fver, leukocytosis & peritoneal irritation in the RUQ.
. Dx -> U/$ (Gall stones - Thick walled gall bladder - Pericholecystic fluid).

.
.
.
.

Tx -> NG suction - NPO - IV fluids - IV antibiotics.


Tx -> Followed by elective cholecystectomy after 6 - 12 wks.
Tx -> Emergency cholecystectomy is needed if there is:
generalized peritonitis or ephysematous cholecystitis (i.e. perforation or gangrene).

(3) ACUTE ASCENDING CHOLANGITIS:


_________________________________
. Obstruction of the COMMON BILE DUCT causes obstruction & ASCENDING INFECTION.
. High fever & very high WBC count.
. High levels of alkaline phosphatase.
. High levels of total & DIRECT bilirubin.
. Mild elevation of transaminases.
. Tx -> IV antibiotics.
. Tx -> Emergency decompression of the common bile duct is life saving !
. Decompression by ERCP or PTC (Percutaneous transhepatic cholangiogram) or surgery.
. Tx -> Cholecystectomy must follow.
. N.B.
. A pregnancy test sh'd be performed in any woman of childbearing period age before,
. ordering diagnostic tests such as x-rays or computed tomography scans.
. PRE-OPERATIVE & POST-OPERATIVE CARE:
______________________________________
______________________________________
* PRE-OPERATIVE ASSESSMENT:
____________________________
{1} CARDIAC RISK:
__________________
. Ejection fraction < 35 % -> Prohibits non-cardiac surgery.
. JVD (sign of CHF) -> Give ACEIs, BB, Digitalis & Diuretics prior to surgery.
. Recent MI -> Defer surgery for 6 months post MI.
. Severe progressive angina -> Cardiac catheterization for coronary revascularization.
{2} PULMONARY RISK:
____________________
. Smoking (Compromised ventillation = High pCO2 & FEV1 < 1.5):
-> Order PFTs to evaluate for FEV1.
-> If FEV1 is abnormal -> Obtain ABG.
-> Cessation of smoking 8 weeks prior to surgery.
{3} HEPATIC RISK:
__________________
. Bilirubin > 2 mg/dl.
. Prothrombin time > 16.
. Serum albumin < 3.
. Encephalopathy.
-> 40 % mortality with any single risk factors.
-> 80 % mortality if 3 or more risk factors are present.
{4} NUTRITIONAL RISK:
______________________

.
.
.
.

Loss of 20 % of body weight over several months.


Serum albumin < 3.
Anergy to skin antignes.
Serum transferrin < 200 mg/dl.

-> Provide 5-10 days of nutritional supplements (preferrably via gut) before surgery.
. N.B. DIABETIC COMA is an ABSOLUTE contraindication to surgery.
. 1st stabilize diabetes.
. Rehydrate & normalize acidosis prior to surgery.
.
.
.
.
.
.

N.B.
If a pt presents with an acute abdomen due to perforation of hollow abdominal viscus,
(Rebound tendrness & subdiaphragmatic free intraperitoneal air on abdominal x-ray),
the pt will require IMMEDIATE LAPARATOMY !
Pre-operative naso-gastric tube decompression is a must.
Give IV fluids & IV antibiotics.

.
.
.
.

In a pt on warfarin due to Af, Warfarin induced anti-coagulation must be reversed !


Bec. if it isn't reversed, it will lead to intra & postoperative bleeding complications
The most rapid mean of normalizing PT: restoration of vit K dependent clotting factors.
through infusion of FRESH FROZEN PLASMA.

. POST-OPERATIVE COMPLICATIONS & MANAGEMENT:


____________________________________________
____________________________________________
{1} MALIGNANT HYPERTHERMIA (Exceeding 104 F):
______________________________________________
. Shortly after the onset of the anesthetic (Halothane or succinyl choline).
. Tx -> IV DANTROLENE, 100% oxygen, Acidosis correction & cooling blankets.
. Watch for development of myoglobinuria.
{2} BACTEREMIA (Exceeding 104 F):
__________________________________
. Within 30-45 mins of invasive procedures (UTI instrumentation).
. 3 successive blood cultures.
. Start empiric antibiotics.
{3} POST-OPERATIVE FEVER (101 - 103 F):
________________________________________
* ATELECTASIS (Day 1):
________________________
-> Lobar or segmental collapse of the lung -> -- lung volume.
-> Due to impaired cough & shallow breathing.
-> Due to accumulation of pharyngeal secretions.
-> Due to the tongue prolapsing posteriorly into the pharynx.
-> Due to airway tissue edema or residual anesthetic effects.
-> Causes significant ventillation - perfusion mis-match -> hypoxemia & ++ breathing work
-> Atelectasis is MOST SEVERE at the SECOND POSTOPERATIVE DAY NIGHT.
-> As a compensation for hypoxia -> Hyperventilation -> Respiratory alkalosis & -- pCO2.
-> Ex -> pH 7.49, pO2 70 mmHg, pCO2 50 mmHg.

-> Prevention: Breathing exercises - Incentive spirometry - Forced expiratory techniques.


. N.B.
. Moving from supine to sitting position ++ the functional residual capacity FRC by 25%.
. ++ FRC prevents post-operative atelectasis.
* "WIND" PNEUMONIA (Day 3):
____________________________
-> CXR -> Infiltrate.
-> Sputum culture.
-> Antibiotics (Hospital acquired pneumonia).
-> Prevention : Post-operative breathing exercises & incentive spirometry.
* "WATER" URINARY TRACT INFECTION (Day 3):
___________________________________________
-> Urinalysis & urinary culture.
-> Antibiotics.
* "WALKING" DEEP VENOUS THROMBOPHLEBITIS (Day 5):
__________________________________________________
-> Doppler U/$ of deep veins of legs & pelvis.
-> Anticoagulation.
* "WOUND" WOUND INFECTION (Day 7):
___________________________________
-> Antibiotics if only cellulitis.
-> Incision & drainage if abscess is present.
* DEEP ABSCESSES (SUBPHRENIC - PELVIC - SUBHEPATIC) (Day 10 - 15):
___________________________________________________________________
-> CT scan of the appropriate body cavity is diagnostic.
-> Percutaneous radiologically guided drainage is therapeutic.
{4} PERIOPERATIVE MYOCARDIAL INFARCTION:
_________________________________________
. Precipitated by hypotension when intraoperative.
. Postoperative MI seldom presents with chest pain.
. Thrombolytics are contraindicated even in postoperative setting !
. Mortality rate is higher than for non surgery related MI.
{5} PULMONARY EMBOLUS (Day 7):
_______________________________
. Tachycardia - SOB - Hypoxia & ++ A-a gradient.
. Dx -> CT angiogram.
. Tx -> Anticoagulate with heparin.
. IVC filter if recurrent PE.
{6} ASPIRATION:
________________
. SOB - Hypoxia - Infiltrates on CXR.
. Lavage & remove gastric contents.
. Bronchodilators & respiratory support.
. Steroids don't help.
{7} INTRA-OPERATIVE TENSION PNEUMOTHORAX:

__________________________________________
. Positive pressure breathing; pt becomes progressively more difficult to bag.
. BP steadily declines & CVP steadily rises.
. Insert needle to decompress & place chest tube later.
{8} POST-OPERATIVE CONFUSION:
______________________________
. Suspect hypoxia 1st ! (Check ABG).
. Consider sepsis then ! (Get blood cultures & CBC).
{9} ACUTE RESPIRATORY DISTRESS $YNDROME (ARD$):
________________________________________________
. Bilateral pulmonary infiltrates & hypoxia with no evidence of CHF.
. Tx -> PEEP = Positive end expiratory pressure.
{10} DELIRIUM TREMENS (Day 2-3):
_________________________________
. Tachycardia - Hyperthermia - Hypertension - Altered mental status.
. Give benzodiazepines (Barbiturates are 2nd line agents due to low therapeutic range).
. Watch for seizures & rhabdomyolysis.
. N.B.
. Post-operative oliguria & azotemia:
______________________________________
. Oliguria (< 400 cc) of urine output per day.
. Azotemia ( ++ BUN/Creatinine ratio > 20:1) = Acute Pre-renal failure from HYPOVOLEMIA !
. Urinary catheter obstruction should be ruled out 1st.
. Next step is an IV FLUID CHALLENGE.
. N.B.
. Post-operative ileus:
________________________
. An ileus is a functional defect in the bowel motility without physical obstruction.
. Following most abdominal surgeries.
. Nausea, vomiting, abdominal distension, failure to pass flatus or stools.
. Hypoactive or absent bowel sounds.
. In contrast (Mechanical obstruction e.g. adehsions cause "HYPERactive" bowel sounds).
. Causes of ileus:
-> ++ splanchnic nerve sympathetic tone following violation of the peritoneum.
-> Local release of inflammatory mediators.
-> Postoperative narcotic (opiate) analgesics e.g Morphine causes disordered peristalsis.
. N.B.
. Post-operative DVT:
______________________
. DVT occurs due to Virchow triad (Stasis - endothelial injury - Hypercoagulability).
. Major surgery is a significant risk factor.
. Pts sh'd be ttt with LMW HEPARIN acutely & warfarin for several months.
. Stable pts can be ttt with anticoagulation as early as 48 - 72 hours after surgery.
. N.B.
. Transfusion reactions:
_________________________
. occur acutely during or immediately following transfusion of blood products.

.
.
.
.
.

They are immune mediated;


preformed host antibody reacts with antigens on transfused blood products,
causing the release of inflammatory mediators & complement activation.
They may be HEMOLYTIC -> Severe reaction that may cause death.
or NON-HEMOLYTIC -> Dose dependent self limited reaction with fever & rigors.

. N.B.
. CATHETER (CENTRAL LINE) ASSOCIATED INFECTIONS:
_________________________________________________
. Intra-venous catheters are one of the most common causes of nosocomial infections.
. Femoral central venous catheters carry a higher risk of bacteremia than subclavian cath
. IV catheter infections are mostly caused by cutaneous organisms such as STAPHYLOCOCCI.
. Femoral catheters may also cause gram -ve bacteremia.
. N.B. Post-operative ACUTE ADRENAL INSUFFECIENCY:
___________________________________________________
. Acute onset of nausea, vomiting, abdominal pain, hypoglycemia & hypotension.
. follows a stressful event e.g. surgical procedure.
. Preoperative steroid use -> A steroid sependent pt is a common scenario (H/O of lupus)!
. Exogenous steroids depress the pituitary-adrenal axis.
. N.B.
. POST-OPERATIVE MEDIASTINITIS:
________________________________
. May follow a cardiac surgery due ti intra-operative wound contamination.
. Complicates 5 % of sternotomies.
. 14 days postoperative.
. Fever, tachycardia, chest pain, leukocytosis.
. Sternal wound drainage drainage of purulent discharge.
. CXR -> WIDENED MEDIASTINUM.
. Tx -> Drainage, surgical debridement with immediate closure & prolonged antibiotic ttt.
. High mortality rate.
__________________________________________________________________________________________
. PEDIATRIC SURGERY:
_____________________
_____________________
. CONDITIONS THAT NEED SURGERY AT BIRTH:
_________________________________________
_________________________________________
. Congenital anomalies constitute the conditions that need surgery at birth.
. The most imp. step is to rule out other associated congenital anomalies.
. VACTER -> Vertebral, Anal, Cardiac, Tracheal, Esophageal, Renal & Radial anomalies.
{1} ESOPHAGEAL ATRESIA:
________________________
. Excessive salivation is noted shortly after birth.
. Chocking spells are noticed when 1st feeding is attempted.
. Confirm the diagnosis with an NG tube -> Coiled in the upper chest on CXR.
. Tx -> Primary surgical repair.
. If surgery needs to be delayed for further workup,
. perform gastrotomy to protect the lungs from acid reflux.

{2} IMPERFORATED ANUS:


_______________________
. ABSENCE of flatus or stools.
. Look for a fistula near by (to vagina or perineum).
. If present, delay repair until further growth (but before toilet training time).
. If not present, a colostomy needs to be done for high rectal pouches.
{3} CONGENITAL DIAPHRAGMATIC HERNIA:
_____________________________________
. Dyspnea is noted at birth !
. Loops of bowel in left chest are seen on x-ray.
. The primary abnormality is the HYPOPLASTIC LUNG with FETAL TYPE CIRCULATION.
. Tx -> Endotracheal intubation, low pressure ventillation, sedation & NG suction.
. Delay repair 3-4 days to allow lung maturation.
{4} GASTROSCHISIS & OMPHALOCELE:
_________________________________
* GASTROSCHISIS:
_________________
. The umbilical cord is NORMAL.
. The defect is to the right of the cord where is no protective membrane.
. The bowel looks angry & matted.
* OMPHALOCELE:
_______________
. The umbilical cord goes to the defect.
. The defect has a thin membrane (one can see normal looking bowel & little liver slice).
.
.
.
.

Tx -> Small defects -> Close small defects primarily.


Tx -> large defects -> Silastic "silo" to protect the bowel.
Manual replacement of the bowel daily until complete closure (in about 1 week).
Give parenteral nutrition (The bowel will not work in gastrochisis).

{5} EXSTROPHY OF THE URINARY BLADDER:


______________________________________
. This is an abdominal wall defect over te pubis.
. Refer to a specialized center offering surgical repair in 1st 1-2 days of life.
. Do NOT delay surgery.
{6} INTESTINAL ATRESIA:
________________________
. Like annular pancreas, it presents with green vomiting.
. But, instead of double-bubble sign, there are multiple air-fluid levels in the abdomen.
. There is no need to suspect other congenital anomalies,
. because this condition results from a vascular accident in utero !
. SURGICAL CONDITIONS IN THE FIRST TWO MONTHS OF LIFE:
_______________________________________________________
_______________________________________________________
{1} NECROTIZING ENTEROCOLITIS:
_______________________________

.
.
.
.
.
.
.
.

This shows up as feeding intolerance in premature infants when they are 1st fed.
There is abdominal distension.
Rapid drop in platelet count (A sign of sepsis in babies).
Tx -> Stop all feeds.
Tx -> Broad spectrum IV antibiotics.
Tx -> IV fluids & nutrition.
Tx -> Surgery if there are signs of necrosis or perforation:
(Abdominal wall erythema - Portal vein gas - Bowel wall gas).

{2} MECONIUM ILEUS:


____________________
. Feeding intolerance & bilious vomiting.
. Family H/O of cystic fibrosis.
. Dx -> X-ray -> Multiple dilated loops of small bowel.
. Dx -> X-ray -> Ground glass appearance in lower abdomen.
. Gastrograffin enema is both diagnostic & therapeutic.
. Diagnostic -> Microcolon & inspissated pellets of meconium in the terminal ileum.
. Therapeutic -> Gastrograffin draws fluid in & dissolves the pellets.
{3} HYPERTROPHIC PYLORIC STENOSIS:
___________________________________
. Approximately at 3 weeks of age.
. NON-bilious projectile vomiting after each feeding.
. Look for gastric peristaltic waves.
. Palpable "olive-size" mass in the RUQ.
. Dx -> Abd. U/$.
. Tx -> Correct dehydration & associated hypochloremic hypokalemic metabloic alkalosis.
. Follow this with Ramstedt pyloromyotomy.
{4} BILIARY ATRESIA:
_____________________
. 6 - 8 weeks old babies.
. Persistent progressively increasing jaundice Conjugated bilirubin).
. Dx -> Conduct serologies & sweat chloride test to rule out other problems.
. Dx -> HIDA scan after 1 week of phenobarbital (A powerful choleretic).
. If no bile reaches duodenum even e' phenobarbital stimulation: Do surgical exploration.
{5} HIRSCHSPRUNG's DISEASE = AGANGLIONIC MEGACOLON:
____________________________________________________
. The most important clue is chronic constipation.
. A rectal exam may lead to explosive expulsion of stool & flatus,
. followed by relief of abdominal distension.
. Dx -> Full thickness biopsy of rectal mucosa.
. SURGICAL CONDITIONS LATER IN INFANCY:
________________________________________
________________________________________
{1} INTUSSUSCEPTION:
_____________________
. 6 - 12 months old chubby, healthy-looking kids.
. Brief episodes of colicky abdominal pain that makes them double up & squat !
. A vague mass on the right side of the abdomen.
. An empty right lower quadrant.
. CURRANT JELLY STOOLS.

. Dx -> Barium or air enema -> Both diagnostic & therapeutic.


. Tx -> Perform surgery if enema fails to achieve reduction.
{2} MECKEL's DIVERTICULUM:
___________________________
. Lower GI bleeding in a child of pediatric age.
. Dx -> Radioisotope scan -> to look for gastric mucosa in the lower abdomen.
__________________________________________________________________________________________
. ORTHOPEDICS:
_______________
_______________
. GENERAL RULES ABOUT #s:
__________________________
. When you suspect a #, order 2 views at 90 to one another.
. Always iclude the joints above & below the #.
. CLOSED REDUCTION -> for #s that are not badly displaced or angulated.
. OPEN REDUCTION & INTERNAL FIXATION -> for severely displaced or angulated #s.
. Open #s (The broken bone sticking out through a wound) require cleaning in the OR,
. & reduction within 6 hours from time of injury.
. Always worry about gas gangrene in any deep penetrating or dirty wounds.
. 3 days later, the pt will be septic with gas crepitus.
. Tx -> Large doses of IV penicillin & hyperbaric oxygen.
. Always perform cervical spine films in any pt with facial injury.
. MANAGEMENT OF COMMON ADULT ORTHOPEDIC #s:
____________________________________________
____________________________________________
{1} CLAVICULAR #:
__________________
. # of the MIDDLE 1/3 -> Brace (Figure 8 sling), rest & ice.
. # of the DISTAL 1/3 -> Open reduction & internal fixation to prevent malunion.
. All pts sh'd've a creful neurovascular examination to rule out injury to:
. the underlying brachial plexus & subclavian artery.
. Hearing a loud bruit warrants an angiogram to rule out subclavian artery injury.
{2} COLLE's #:
_______________
. Closed reduction & casting.
. Elderly woman falling on an out-stretched hand.
. Painful wrist.
. Dinner fork deformity.
{3} DIRECT BLOW TO ULNE (MONTEGGIA #) or RADIUS (GALEAZZI #):
______________________________________________________________
. Combination of diaphyseal # & displaced dislocation of the nearby joint.
. Open reduction & internal fixation is needed for the diaphyseal reduction.

. Closed reduction for the displaced joint.


{4} SCAPHOID #:
________________
. Young adult with fall on an out-stretched hand.
. Persistent pain in the anatomical snuff box.
. Takes > 3 weeks to be seen on x-ray.
. If the initial x-ray is -ve, subsequent x-ray is done in 7-10 days.
. Wrist x-ray -> Fine radiolucent lines in nondisplaced scaphoid #.
. Tx -> Wrist immobilization for 6 - 10 weeks.
. Place thumb spica cast to help to prevent non-union.
{5} HIP #:
___________
. Any elderly pt who sustains a fall.
. Look for externally rotated & shortened leg.
. Femoral neck # -> High risk of avascular necrosis - Tx: Femoral head replacement.
. Intertrochanteric # -> Open reduction & pinning.
. Femoral shaft # -> Intra-medullary rod fixation.
{6} TRIGGER FINGER:
____________________
. Woman who awakens at night with an acutely flexed finger,
. that snaps when forcibly extended.
{7} DE QUERVAIN TENOSYNOVITIS:
_______________________________
. Young mother carrying baby,
. with flexed wrist & extended thumb to stabilize the baby's head.
. Steroid injection is the best therapy.
{8} DUPUYTREN CONTRACTURE:
___________________________
. Contracture of the palm & palmar fascial nodules.
. Surgery is the only ttt.
{9} POSTERIOR HIP DISLOCATION:
_______________________________
. H/O of head-on car collision where the knees hit the dashboard (Orthopedic emergency).
. Differentiate it from hip # by an internally rotated leg (The leg is also shortened).
. Emergency ruduction is needed to avoid avascular necrosis.
{10} KNEE INJURIES:
____________________
(a) Medial & lateral collateral ligament injury:
__________________________________________________
. Caused by a direct blow to the opposite side of the joint.
. Casting if isolated ligament injury.
. Surgical repair if multiple ligaments injured.
. Medial -> Due to abduction injury to knee - Dx -> VALGUS stress test.
. Lateral -> Due to adduction injury to knee - Dx -> VARUS stress test.
(b) Anterior & posterior cruciate ligament injury:

____________________________________________________
. Swelling & pain.
. Anterior / posterior drawer sign.
. Young athletes need arthroscopic repair.
. Older pts need immobilization & rehabilitation.
. Anterior -> H/O of forceful hyperextension injury to knee.
. Effusion is seen rapidly following injury.
. Dx -> Lachman's test, ANTERIOR drawer test & pivot shift test.
. Posterior -> H/O of dashboard injury.
. Forceful posterior-directed force on the tibia with knee flexed at 90 dgrees.
. Dx -> POSTERIOR drawe test, REVERSE pivot test & posterior sag test.
(c) Meniscal injury:
______________________
. Twisting injuries with the foot flexed.
. Medial meniscus is more commonly injured than the lateral meniscus.
. POPPING SOUND followed by severe pain at time of injury.
. Prolonged pain & swelling.
. Localized tendrness at the side of the knee.
. Catching & locking of knee koint on extension (BUCKET HANDLE TEARS).
. Palpable or audible snap while extending the leg from full flexion (McMurray's sign).
. Tx -> Arthroscopic repair.
{11} TIBIAL STRESS INJURY:
___________________________
. H/O of military or cadet marches.
. X-ray may be -ve initially.
. Tx -> Cast.
. Order the pt not to bear weight.
. Repeat x-ray in 2 weeks.
{12} ACHILLES TENDON RUPTURE:
______________________________
. Middle-aged man overdoes it at tennis or basketball match.
. Pt with H/O of fluoroquinolone use.
. Complaining of sudden "POPPING" & limping.
. Tx -> Casting in equinous position or surgical repair.
{13} ANTERIOR SHOULDER DISLOCATION:
____________________________________
. Most common form of shoulder dislocation.
. Direct blow or fall on out-stretched arm.
. Adducted arm & (EXTERNALLY) rotated forearm.
. Numbness over deltoid (Axillary nerve is stretched).
. Easily seen on erect postero-anterior (PA) & lateral views.
{14} POSTERIOR SHOULDER DISLOCATION:
_____________________________________
. Pt with recent seizure or electrical burn.
. H/O of an eclampsic pt is common.
. Due to violent muscle contractions during a tonic-clonic seizure.
. Flattening of the anterior shoulder & prominent coracoid process.
. Axillary or scapular view x-ray.

. Adducted arm & (INTERNALLY) rotated forearm.


. Inability of external rotation.
. Tx -> Closed reduction.
{15} FEMORAL #:
________________
. Femoral shaft # is an orthopedic emergency.
. Can result in massive blood loss & high rate of infection.
. Immediate surgery & cleaning within 6 hours is needed.
.
.
.
.
.

N.B.
Hip #s are common in the elderly,
1st -> Stabilization & treatment for pain control & DVT prophylaxis.
Next -> Discover the etiology of the pt's fall with appropriate investigations.
Do EKG , CXR & cardiac enzymes !

. N.B.
. Acute shoulder pain after forceful abduction & external rotation at glenohumeral joint,
. suggests an anterior shoulder dislocation -> AXILLARY NERVE INJURY.
{16} NURSEMAID ELBOW:
______________________
. common injury in pre-school children.
. SUBLAXATION of HEAD of RADIUS at ELBOW joint.
. Due to swinging a young child by the arms or pulling a child arm while in a hurry.
. The child will be calm but will cry on an attempt to flex the elbow or supinate forearm
. Dx -> Clinically (Radiographs are often normal).
. Tx -> GENTLE PASSIVE ELBOW FLEXION & FOREARM SUPINATION.
. 1st -> Extend & distract the elbow.
. Next -> Supinate the forearm.
. Hyperflex the elbow with your thumb over he radial head to feel reduction.
. No post reduction films are needed.
. The child will resume the use of the previously unused extremity without crying.
* COMPARTMENT $YNDROME:
________________________
. Most frequent in the forearm or lower leg.
. H/O of prolonged ischemia followed by reperfusion, crushing injuries or other traumas.
. Pain & tightness & tenderness to palpation at the affected area.
. EXCRUCIATING PAIN with PASSIVE EXTENSION.
. Pulses may be normal !!
. Tx -> 1st step is emergent fasciotomy.
. N.B. When a pt complains of pain at the site of a cast,
. Always remove the cast & examine for compartment $.
.
.
.
.
.
.

N.B. ISCHEMIA REPERFUSION $YNDROME:


A form of compartment $.
Extremeties subjected to at least 4-6 hours of ischemia,
suffer from intracellular & interstitial edema upon reperfusion (SOFT TISSUE SWELLING).
When edema causes the pressure within a muscular fascial compartment to rise > 30 mmHg
compartment $ occurs leading to further ischemic injury to the confined tissue.

. N.B.
. Escharotomy is indicated for circumferential full thickness burns of an extremity,

. with an eschar causing significant edema & constriction of the vascular supply.
. Pts sh'd be evaluated for clinical signs of adequate perfusion after escharotomy.
. Fasciotomy sh'd be done if there is NO signs of relef.
* NEURO-VASCULAR INJURIES:
___________________________
___________________________
. OBLIQUE DISTAL HUMERUS #:
____________________________
. Radial nerve injury.
. Inability to dorsiflex (extend) the wrist.
. Function regained after reduction.
. Surgery is indicated if paralysis persists after reduction.
. POSTERIOR KNEE DISLOCATION:
______________________________
. Popliteal artery injury.
. Decreased distal pilses.
. Doppler studies or arteriogram.
. Prophylactic fasciotomy if reduction is delayed.
. BACK PAIN:
_____________
_____________
{1} DISC HERNIATION:
_____________________
. Sudden onset severe back pain after lifting heavy object.
. Electric shock like pain shooting down the leg.
. Straight leg raising test gives excruciating pain.
. Mostly lumbar in origin L4, L5 & S1.
. Peak age 43-46 ys.
. Tx -> Anti-inflammatories & brief bed rest.
. Immediate surgical compression is needed if the H/O suggests Cauda equina $.
. (Bowel/Bladder incontinence - flaccid anal sphincter - Saddle anesthesia).
. MRI -> Confirm both disc herniation & causa equina.
. Trial of anti-inflammatories is always the 1st step in management.
{2} ANKYLOSING SPONDYLITIS:
____________________________
. Man in his 30s or early 40s.
. Chronic back pain.
. Morning stiffness improving with activity.
. X-ray -> Bamboo spine.
. Associated with HLA B-27 antigen (Screen for uveitis & IBD).
. Tx -> Anti-inflammatory agents & physical therapy.
{3} METASTATIC MALIGNANCY:
___________________________
. Elderly pt with progressive & constant back pain.
. Worse at night & unrelieved by rest.
. H/O of weight loss.

.
.
.
.
.
.
.
.
.
.

X-ray -> Lytic lesions or blastic lesions.


Blastic metastatic lesions -> Prostate cancer & breast cancer.
Lytic metastatic lesions -> Lung, renal, breast, thyroid & multiple myeloma.
Hypercalcemia & ++ ALP.
1st -> Order plain radiographs (Especially important in multiple myeloma).
Bone scan is most sensitive in early disease.
MRI shows the greatest amount of details.
MRI -> test of choice if there are any neurologic syms to rule out cord compression.
Bone scan will not be helpful in purely lytic lesions (Multiple Myeloma).
Instead order plain radiographs or MRI.

* FOOT PAIN:
_____________
_____________
. PLANTAR FASCIITIS:
_____________________
. Older, overweight pts with sharp heel pain every time their foot strikes to the ground.
. Pain is worse with walking & in the mornings.
. X-ray -> Bony spur matching the location of the pain.
. Exquisite tenderness to palpation over the spur.
. Burning pain in nature.
. More common in runners with repeated microtrauma,
. who develop local point tendrness on plantar aspect of foot.
. However, surgical resection of the bony spur is not indicated !
. MORTON NEUROMA:
__________________
. Inflammation of the common digital nerve at the 3rd interspace.
. Between the 3rd & 4th toes.
. Mechanically induced neuropathic degeneration.
. Numbness & burning of the toes, aching & burning in the distal forefoot.
. Pain radiates forward from the metatarsal heads to the 3rd & 4th toes.
. PALPATION & SQUEEZING the metatarsal joints -> CLICKING SENSATION (MULDER SIGN).
. Caused by wearing pointy-toed shoes.
. The neuroma is palpable with very tender spot there.
. Management is analgesics & appropriate foot wear.
. STRESS # = HAIR LINE #:
__________________________
. Sudden ++ in repeated tension or compression without adequate rest.
. Sharp localized pain over a bony surface that is worse with palpation.
. The tibia is the most common bone in the body to be affected by stress #s.
. Occur in the anterior part of the middle 1/3 of the shin of tibia in jumping sport pts.
. Occur in the postero-medial part of the distal 1/3 of the tibia in runners.
. X-ray are frequently normal during initial evaluation.
. Stress # of the meta-tarsals are common in atheletes & military recruits.
. The 2nd metatarsal is the most commonly injured.
. Tx -> Rest, analgesia & a hrd soled shoe.
. TARSAL TUNNEL $YNDROME:
__________________________
. Compression of the tibial nerve as it passes through the ankle.
. Usually caused by a # of the bones around the ankle.

. Burning, numbness & aching of the distal plantar surface of foot or toes.
. Pain may radiate up to the calf.
__________________________________________________________________________________________
* UROLOGY:
___________
___________
. VARICOCELE:
______________
. Tortuous dilatation of pampiniform plexus of veins surrounding spermatic cord & testis.
. Results from incompetence of the valves of the testicular vein.
. Occurs most frequently on the left side, bec.
. Lt testicular vein enters Lt renal vein inferiorly at right angle -> impaired drainage.
. Dull or dragging discomfort scrotal pain that becomes worse on standing.
. Examination -> Bag of worms (Enlarge with Valsalva maneuver).
. NEGATIVE TRANSILLUMINATION.
. HYDROCELE:
_____________
. Due to fluid accumulation in tunica vaginalis.
. POSITIVE TRANSILLUMINATION.
. TESTICULAR NEOPLASIA:
________________________
. Painless testicular mass with negative transillumination.
. SPERMATOCELE:
________________
. Cystic dilatations of the efferent ductules.
. Painless fluif-filled cysts containing sperms.
. Located on superior pole of testis in relation to epididymis.
. +ve transillumination.
. TESTICULAR TORSION:
______________________
. Severe, sudden onset testicular pain.
. NO fever - NO pyuria.
. The testis is swollen & exquisitely tender.
. High riding testicle with transverse lie.
. Dx -> U/$.
. Tx -> Immediate surgical intervention with bilateral orchipexy.
. ACUTE EPIDIDYMITIS:
______________________
. Acute scrotal pain (may be referred to abdomen).
. FEVER & urinary symptoms.
. Dx -> Urinalysis & urine cultures & discharge culture if present.
. Tx -> Males < 35 ys -> Treat for gonorrhea & chlamydia -> Ciprofloxacin & Doxycycline.
. Tx -> Older males -> Treat as UTI (E-coli) with Levofloxacin.
. UROLOGIC OBSTRUCTIONS:
_________________________

.
.
.
.
.

Combination of obstruction & infection is a urologic emergency.


It can lead to destruction of the kidney in few hours.
Tx -> Immediate decompression of the urinary tract above the obstruction.
Tx -> IV antibiotics are given to prevent infection.
Tx -> A ureteral stent or percutaneous nephrostomy is the most important intervention.

.
.
.
.
.
.
.

N.B.
Urinary calculi present as flank or abdominal pain radiating to the groin.
Nausea & vomiting is common.
Unlikepts with an acute abdomen, pts with urinary stones are WRITHING in pain.
Unable to sit still in exam room (No peritoneal irritation so movements don't ++ pain).
Dx -> A NON-contrast spiral CT of the abdomen & pelvis is the most accurate test.
Dx -> X-ray can miss radio-lucent urinary stones (15 % of stones).

.
.
.
.
.
.
.
.
.
.
.
.

N.B.
Nephrolithiasis
Flank pain & hematuria accompanied by nausea & vomiting.
Pts with Chron's disease or small bowel dis -> Fat malabsorption.
Fat malabsorption -> predispose to hyperoxaluria.
Oxalate is obtained from diet & is a normal product of human metabolism.
Symptomatic hyperoxaluria is the result of ++ oxalate absorption in the gut.
Under normal circumstances: Calcium binds oxalate in the gut preventing its absorption.
In pts with fat malabsorption, Ca is bound by fat leaving oxalate free & unbound.
Failure to adequately absorb bile salts in cases of fat malabsorption,
leads to -- bile salt reabsorption in small intestine.
Excess bile salts may damage colonic mucosa -> ++ oxalate absorption.

. CONGENITAL UROLOGIC DISEASES:


________________________________
________________________________
{1} POSTERIOR URETHRAL VALVE:
______________________________
. The most common cause for a new born boy not to urinate during the 1st day of life.
. Dx -> Voiding cystourethrogram.
. Tx -> Catheterize to empty the bladder.
{2} HYDROCELE:
_______________
. Fluid collection within the processus vaginalis or tunica vaginalis.
. Peritoneal fluid accumulation -> hydrocele
. POSITIVE TRANSILLUMINATION.
. Tx -> REASSURANCE -> Will resolve spontaneously by the age of 12 months.
. Tx -> If not resloved by 12 months -> Surgical removal to avoid inguinal hernia.
{3} HYPOSPADIUS:
_________________
. Urethral opening at the ventral side of the penis.
. Never to perform circumcision on this child.
. The prepuce will be needed for the plastic reconstruction.
. N.B. A child who has HEMATURIA from TRIVIAL TRAUMA,
. has an undiagnosed congenital anomaly until proven otherwise.

.
.
.
.
.

N.B. A child who has URINARY TRACT INFECTION,


has an undiagnosed congenital anomaly until proven otherwise.
e.g. vesico-ureteral reflux.
Dx -> Voiding cystourethrogram.
Tx -> Long term antibiotics.

{4} LOW IMPLANTATION OF A URETER:


__________________________________
. A girl who void appropriately but also found to be constantly wet,
. due to urinating into vagina.
{5} URETERO-PELVIC JUNCTION (UPJ) OBSTRUCTION:
_______________________________________________
. Only symptomatic when diuresis occurs.
. A teenager who drinks large volumes of beer & develops colicky flank pain.
__________________________________________________________________________________________
. VASCULAR SURGERY:
____________________
____________________
{1} SUBCLAVIAN STEAL $YNDROME:
_______________________________
. Due to an arteriosclerotic stenotic plaque at the origin of subclavian artery.
. This allows enough blood to reach the arm for normal activity, but,
. Not enough to meet the ++ demands of an exercised arm,
. resulting in BLOOD BEING STOLEN FROM THE VERTEBRAL ARTERY.
. Posterior neurological signs (Visual symptoms - Equilibrium problems).
. Claudication in the arm during arm exercises.
. Don't confuse this condition with thoracic outlet $!
. Thoracic outlet $ causes vascular symptoms only with-OUT neurological signs.
. Dx -> Angiography.
. Tx -> Bypass surgery.
{2} AORTIC ANEURYSM:
_____________________
. Size & symptoms are key to management of "ABDOMINAL" aortic aneurysm:
-> Aneurysms < 5cm -> Observe with serial annual imaging.
-> Aneurysms > 5cm -> Elective surgical repair.
. More urgent surgery is needed if:
-> A TENDER AAA will rupture within a day or two requiring urgent repair.
-> Excruciating back pain in a pt e' large AAA means that,
. the aneurysm is already leaking, necessitating emergency surgery.
. N.B. The following contributes to the development of "THORACIC" aortic aneurysm:
-> Chronic hypertension.
-> Hyperlipidemia.
-> Smoking.
-> Marfan $.
-> Untreated tertiary $yphilis.

. N.B. The most imp. modifiable risk to prevent worsening of existing aneurysms is:
-> UNCONTROLLED HYPERTENSION.
. N.B. Asymptomatic lesions -> BLOOD PRESSURE MANAGEMENT is the most important.
. N.B. Symptomatic lesions (including active dissection) -> Surgical intervention.
. (Look for sudden onset tearing pain in the back).
{3} ARTERIOSCLEROTIC OCCLUSIVE DISEASE OF THE LOWER EXTREMETIES:
_________________________________________________________________
. Pain in the legs on exercise that is relieved by rest (intermittent claudication).
. If the claudication doesn't affect the pt's life style -> No intervention is needed.
. The only management indicated is CESSATION OF SMOKING & THE USE OF CILOSTAZOL.
. If the pain is more severe,
. Dx -> Doppler studies (Pressure gradient ABI < 0.9).
. Dx -> Arterigram to identify stenosis.
.
.
.
.

If there is DISABLING symptoms (Affect work or daily life activity),


or there is impending ischemia to the extremity,
Tx -> SURGERY (Angioplasty & stenting for stenotic segments).
More extensive disease requires bypass grafts or sequential stents.

. N.B. PAIN AT REST indicates END STAGE DISEASE (Pt complains of calf pain at night).
. N.B. VVVVVVVVVVVVVVVVV. imp.
. The 1st step in evaluating a pt with suspected peripheral artery disease (PAD) is:
. to obtain an ANKLE-BRACHIAL INDEX (ABI) to confirm the diagnosis.
. Aspirin & cilostazol are antiplatelet agents that can be given after confirming PAD.
. They are not given upon clinical suspicion !
. Pts with significant symptoms & NORMAL ABI may have MILD diesase at rest.
. They sh'd undergo EXERCISE TESTING with pre & post exercise ABI measurment to confirm.
. ABI (1.0 - 1.3) -> Normal.
. ABI < 0.9 -> > 50 % occlusion of a major vessel.
. ABI < 0.4 -> Limb ischemia.
.
.
.
.
.
.

N.B. ISCHEMIA REPERFUSION $YNDROME:


A form of compartment $.
Extremeties subjected to at least 4-6 hours of ischemia,
suffer from intracellular & interstitial edema upon reperfusion (SOFT TISSUE SWELLING).
When edema causes the pressure within a muscular fascial compartment to rise > 30 mmHg
compartment $ occurs leading to further ischemic injury to the confined tissue.

. N.B. COMPARTMENT $ 5 "P"s:


____________________________
. May be caused by direct trauma (Hemorrhage), prolonged compression of an extremity.
. May be caused after revascularization of an acutely ischemic limb.
. Muscles of extremity are encased in fascial compartments preventing tissue expansion.
. The ++ pressure interferes with perfusion leading to muscle necrosis.
. Pressure > 30 mmHg leads to cessation of blood flow through capillaries.

. Tx -> EMERGENT FASCIOTOMY.


-> Pain
. Earlest symptom.
. ++ by passive stretch of the muscles in the affected compartment.
-> Paresthesia
. Burning or tingling sensation.
. occurs in the distribution of the affected peripheral nerve.
-> Pallor
. of the overlying skin
. result from tense swelling & compromised perfusion.
-> Pulselessness
. Late finding.
. Presence of a pulse on exam does NOT rule out compartment $.
-> Paresis/Paralysis
. Late finding.
. resilt from nerve & muscle ischemia & necrosis.
.
.
.
.
.
.
.

N.B. ESCHAR !
Eschar is a firm necrotic tissue formed on on exposed tissue following burn wounds.
When eschar occurs circumferentially on an extremity,
it restricts the outward expansion of the compartment as edema follows burn.
Interstitial pressure increases -> compromise vascular flow to the limb.
Deep pain out of proportion to injury, pulselessness, paresthesia, cyanaosis & pallor.
Tx -> Escharotomy.

.
.
.
.
.

N.B. Compartment $ 2ry to SUPRA-CONDYLAR # of humerus:


2ry to fall on out-stretched hand.
Due to ++ pressure in a limited space.
Pain, pallor, pulselessness, paralysis & presthesia.
Tx -> Immediate fasciotomy.

. N.B. VOLKMANN's ISCHEMIC CONTRACTURE:


. is the final sequel of compartment $ (The dead muscle is replaced by fibrous tissue).
{4} ARTERIAL EMBOLIZATION OF THE EXTREMETIES:
______________________________________________
. H/O of Af or recent MI.
. Sudden onset painful, pale, cold, pulseless, paresthetic & paralytic lower extremity.
. Dx -> Doppler studies to locate the obstruction.
. Tx -> Thrombolytics (if early) & Embolectomy (if later) with fasciotomy.
. N.B. LERICHE $YNDROME = AORTO-ILIAC OCCLUSION:
_________________________________________________
. Arterial occlusion at the bifurcation of the aorta in the common iliac arteries.
. Triad of bilateral hip, thigh & buttock claudication, impotences &
. symmetric atrophy of the bilateral lower extremeties due to chronic ischemia.
. Absence of impotence excludes the condition.

. N.B. SPINAL CORD ISCHEMIA:


_____________________________
. Follows aortic vascular surgery due to anterior spinal artery $.
. The spinal cord derives its blood supply from the:
. anterior spinal artery & 2 posterior spinal arteries originating from vertebral artery.
. Presents with flaccid paralysis, bowel/bladder dysfunction, sexual dysfunction.
. Possible hypotension & loss of deep tendon reflexes.
. Spasticity & hyperreflexia develop over days to weeks.
. Vibratory & proprioceptive sensation is preserved as posterior circulation is preserved
. Dx -> Emergent MRI.
. Tx -> Supportive care & lumbar drains to reduce spinal pressure.
__________________________________________________________________________________________
. MISCELLANEOUS TOPICS:
________________________
________________________
. GASTRIC OUTLET OBSTRUCTION:
______________________________
. Can be caused by many diseases causing mechanical obstruction e.g.
. Gastric malignancy - Peptic ulcer disease - Chrons dis - Strictures e' pyloric stenosis
. Strictures 2ry to ingestion of caustics.
. Characterized by early satiety, nausea, non-bilious vomiting & weight loss.
.
.
.
.

P/E -> ABDOMINAL SUCCUSSION SPLASH, elicited by placing the stethoscope,


over the upper abdomen & rocking the pt back & forth at the hips,
Retained gastric material > 3 hours after a meal will generate a splash sound,
indicating the presence of a hollow viscus filled with both fluids & gas.

.
.
.
.
.

In a pt with a H/O of acid ingestion, pyloric stricture is the most likely cause.
H/O of a recent acid ingestion is a risk factor of developing pyloric stricture.
Acid ingestion causes fibrosis 6-12 weeks after the resolution of acute injury.
Dx -> Upper endoscopy.
Tx -> Surgery.

. TOTAL BODY BURN & SYSTEMIC INFLAMMATORY RESPONSE $:


______________________________________________________
. Systemic inflammation & tissue injury.
. Burn -> Dysregulated host response,
. Massive uncontrolled release of proinflammatory substances -> extensive tissue damage.
. This is known as systemic inflammatory response $yndrome:
-> Temperature -> > 38.5 c (101.3 F) or < 35 c (95 F) !!!!!
-> Pulse -> > 90/min.
-> Respirations -> > 20/min.
-> WBC > 12000 or < 4000 or > 10 % bands !!!!
. SIR$ can follow pancreatitis, autoimmiune dis, vasculitis & burns.
. Sepsis (SIRS e' a known infection) is considered severe when there is end organ failure
. Oliguria - Hypotension (SBP < 90mmHg) - Thrombocytopenia (PLT < 80000).

.
.
.
.
.

Metabolic acidosis - Hypoxemia.


Hyperglycemia occur due to insulin resistance.
Muscle wasting & protein loss & Hyperthermia.
Sepsis with septic shock may occur in the 1st week post-burn.
Main causes of sepsis are pneumonia & wound infections (Staph aureus & Pseudomonas).

. Criteria indicating sepsis -> Leukocytosis - Thrombocytopenia - Mild hypothermia < 36.
. Tachypnea & tachycardia due to associated pneumonia.
. Worsening hyperglycemia due to worsening insulin resistance.
. Bottom line:
. In pts with severe significant total body surface areas burns,
. The major cause of morbidity & mortality is HYPOVOLEMIC SHOCK.
. In case of adequate initial fluid resuscitation,
. Bacterial infection (Bronchopneumonia or burn wound infection) -> Sepsis & septic shock
. INTRA-PERITONEAL RUPTURE OF THE BLADDER:
___________________________________________
. Intra-abdominal pathology causing shoulder pain = Subdiaphragmatic peritonitis.
. Among the possible blunt traumatic bladder injuries,
. Only an INTRA-PERITONEAL RUPTURE OF THE (BLADDER DOME) -> CHEMICAL PERITONITIS.
. The dome of the bladder is the only region covered by peritoneum.
. Pain could be transferred to the ipsilateral shoulder because,
. Phrenic nerve originates from C3 to C5 spinal nerves mediating sensation for shoulders.
. INTRA-ABDOMINAL MALIGNANCY (CANCER PANCREAS):
________________________________________________
. Day time fatigue, anorexia, significant weight loss.
. Visceral type abdominal pain interfering with sleep.
. Constant eigastric pain radiating to the back, weight loss & jaundice.
. Migratory thrombophlebitis is a classic association.
. N.B.
. A peptic duodenal ulcer causes periodic epigastric pain relieved by meals.
. PILO-NIDAL SINUS:
____________________
. Acute pain & swelling of the midline sacro-coccygeal skin & subcutaneous tissues.
. Due to infection of a dermal sinus tract originating over the coccyx.
. RIB #:
_________
. Pain relief is the prime objective in management of rib #.
. As it allow proper ventillation & prevent atelectasis & pneumonia.
. TETANUS PROPHYLAXIS:
_______________________
_______________________

. Un-immunized, uncertain or < 3 tetanus toxoid doses:


_______________________________________________________
. Minor clean wound -> Tetanus toxoid only.
. Severe or dirty wound -> Tetanus toxoid & tetanus immunoglobulins.
. > 3 tetanus toxoid doses:
____________________________
. Minor & clean wound -> None.
. Severe or dirty wound -> Tetanus toxoid if latest boster given > 5 years ago.
. N.B. Tetanus-diphtheria toxoid sh'd be given to individuals with severe or dirty wounds
. who received a booster > 5ys ago
. & those with minor clean wounds who received a booster dose > 10 ys ago.
. N.B. Tetanus immune globulin sh'd be given to any individual with severe dirty wound
. & unclear or incomplete immunization history.
. CO CARBON MONOXIDE POISONING:
________________________________
. H/O of smoke inhalation.
. CO is a tasteless, colorless & odorless gas.
. It has affinity 200 times more than O2 for hemoglobin.
. Confusion, wheezes, headache, nausea, dyspnea, malaise, altered mentation, dizziness.
. If severe -> Seizure, coma, syncope, heart failure & arrhythmias.
. Bright cherry lips can be seen but not specific.
. Dx -> ++ Carboxyhemoglobin level > 3 % in non-smokers & > 15 % in smokers.
. Tx -> 100 % OXYGEN NON-BREATHER FACE MASK.
__________________________________________________________________________________________
. BREAST PROBLEMS:
__________________
__________________
. 1 . INTRA-DUCTAL PAPILLOMA:
______________________________
. Benign breast disease.
. Most common in peri-menopausal women.
. Intermittent BLOODY discharge from one nipple.
. Most intraductal papillomas are situated beneath the areola.
. Difficult to palpate on physical examination due to their small sizes (< 2 mm).
. Soft in consistency.
. U/$ will be normal because it can detect masses only greater than 1 cm in diameter.
. 2 . FIBRO-CYSTIC DISEASE:
____________________________
. Very common in pre-menopausal women.
. Bilateral breast pain.
. Associated with cystic changes of the breast.
. Benign condition.
. Symptoms vary cyclically with the menstrual cycle.
. P/E -> Lumpiness of the breast.

. 3 . FIBRO-ADENOMA:
_____________________
. Solitary breast lesion.
. Painless, firm, mobile breast lump.
. Average size about 2 cm.
. Women ages 15 - 25 ys.
. Benign condition.
. Do NOT change with menstrual cycle.
. 4 . DUCTAL CARCINOMA IN-SITU:
________________________________
. Post-menopausal women.
. Incidental finding on mammography.
. Nipple discharge & breast mass are the most common complaints.
. It is a HISTOLOGICAL diagnosis.
. 5 . INFLAMMATORY BREAST CARCINOMA:
_____________________________________
. Brawny edematous cutaneous plaque.
. "P'eau d'orange" orange peel appearance overlying a breast mass.
. It is an aggressive tumor.
. 1/4 of the pts have metastatic disease at the time of presentation.
. Most pts present with axillary lymphadenopathy.
. Spontaneous nipple discharge is a sign of breast cancer.
. Nipple discharge in a non-lactating woman sh'd always raise suspicion for cancer,
. spontaneous, unilateral, localized to single duct, bloody discharge in pt > 40 ys old.
. Mass association is an imp. sign of malignancy.
. Clinicalyy, you can't differentiate it from an inflammatory process (breast abscess).
. A BIOPSY FOR HISTOLOGY IS THE MAIN STAY OF DIAGNOSIS !
. PALPABLE BREAST MASS EVALUATION:
___________________________________
PALPABLE BREAST MASS
______________________
|
________________________________
|
|
< 30 ys
> 30 ys
|
|
ULTRASONOGRAM ONLY
MAMMOGRAM & ULTRASONOGRAM
|
|
____________________
|
|
|
|
SIMPLE CYST
SOLID MASS
SUSPICIOUS FOR MALIGNANCY
|
|
|
NEEDLE ASPIRATION
CORE BIOPSY
CORE BIOPSY
.
.
.
.
.

N.B.
BREAST FAT NECROSIS
shows clinical signs & radiographic findings similar to breast cancer !
Syms include (Skin or nipple retraction - Calcification on mammography).
Biopsy of the mass -> FAT GLOBULES & FOAMY HISTIOCYTES.

. No ttt is indicated (Self limiting condition).


__________________________________________________________________________________________
. SPINAL CORD INJURIES:
________________________
________________________
. ANTERIOR CORD $YNDROME:
__________________________
. Burst # of the vertebra -> Occlusion of vertebral artery.
. Total loss of the motor function below the level of the lesion.
. Loss of pain & temperature on both sides below the lesion.
. NORMAL proprioception.
. NEGATIVE Straight leg raising test.
. Dx -> MRI.
. CENTRAL CORD $YNDROME:
_________________________
. Hyperextension injury in elderly pts with degenerative diseases of the cervical spine.
. Selective damage to the central portion of the spinal cord.
. specially the corticospinal & decussating fibers of the lateral spinothalamic tracts.
. Burning pain & paralysis in the UPPER extremeties e' relative SPARING of lower limbs.
. POSTERIOR CORD $YNDROME:
___________________________
. Bilateral loss of vibratory & proprioceptive sensation.
. BROWN SEQUARD $YNDROME:
__________________________
. Acute hemisection of the spinal cord.
. Ipsilateral motor & proprioception loss below the level of the lesion.
. Contralateral pain loss below the level of the lesion.
. ACUTE DISK PROPLAPSE:
________________________
. Severe radicular pain.
. +ve Straight leg raising test.
. CAUDA EQUINA $YNDROME:
_________________________
. Paraplegia.
. Variable sensory loss.
. Urinary & fecal incontinence.
. SYRINGOMYELIA:
_________________
. May follow spine cord trauma.
. Whiplash is often the incinting injury.
. Symptoms develop months to years later.
. Enlargement of the central canal of the spinal cord due to CSF retention.
. Impaired strenght & pain/temperature sensation in upper extremeties.
. Preservation of dorsal column function (Light touch - vibration - position sense).
. CAPE LIKE DISTRIBUTION.
__________________________________________________________________________________________

. # Mid-shaft humerus -> Radial nerve injury -> Wrist drop.


. # Supracondylar humerus -> Brachial artery -> pain, pallor, pulselessness, paresthesia.
. # Humerus -> Ulnar nerve -> Claw hand.
. ROTATOR CUFF TEAR:
_____________________
. Rotator cuff is formed by tendons of:
. (supraspinatous, infraspinatous, teres minor & subscapularis muscles).
. The supraspinatous is most commonly injured,
. due to repeated bouts of ischemia near its insertion on the humerus,
. induced by its compression between the humerus & the acromion.
. Common cause of tear is fall on out-stretched hand.
. Severe shoulder pain & edema following the trauma.
. Inability to abduct the arm at 90 degrees.
. When the pt's arm is abducted passively to greater than 90 degrees,
. And the pt is asked to lower the arm slowly; the pt's arm drops suddenly !
. N.B.
. D.D. for rotator cuff tear is "RUPTURE OF TENDON OF LONG HEAD OF BICEPS";
. POSITIVE POPEYE SIGN (The biceps muscle belly becomes prominent in the mid upper arm.
. PAGET'S DISEASE OF BONE = OSTEITIS DEFORMANS:
________________________________________________
. Disordered bone remodelling.
. ++ Osteoclast activity -> ++ bone resorption.
. Accelereated osteoblastic activity to rebuild the degraded bone.
. WOVEN BONE formation (Various stages of bone throughout the body).
. The woven bone is larger than normal bone & more liable to bowing & #.
. Most common presenting symptom is secondary arthritis of hip or knee.
. ++ ALKALINE PHOSPHATASE.
. NORMAL serum phosphorous & calcium levels.
. Enlargement of skull bones -> Bossing, ++ head size (Old hats no longer fits!).
. Headaches & cranial nerve palsies.
. Hearing loss is due to damage of the cochlear nerve,
. due to enlargement of the temporal bone & entrapment at the internal auditory meatus.
. VARICES:
___________
. NON-BLEEDING VARICES are managed with BB "Prporanolol".
. After 1st episode of bleeding -> Sclerotherapy, endoscopic band ligation & surgery.
. If not responsive to medical or endoscopic intervention -> Porto-systemic shunt (TIPS).
. RESPIRATORY QUOTE (RQ):
__________________________
. RQ is the ratio bet. CO2 produced to O2 consumed.
. Used to make assessmentsof metabolism taking place in the body.
. In mechanically ventillated pt, the RQ is 1.05.
. The ratio depends upon the major fuel being oxidized for ATP production.
. An RQ close to 1 indicates that CARBOHYDRATE is the major nutrient being oxidized.
. The RQ for protein & lipid as sole energy sources are 0.8 & 0.7 respectively.

. Massive atelectasis could affect ABG, but once a new steady state is achieved,
. the RQ value w'd still depend only upon the nature & proportions of metabolics used.
. TROCHANTERIC BURSITIS:
_________________________
. Unilateral hip pain in a MIDDLE-AGED adult.
. Inflammation of the bursa around the insertion of gluteus medius greater trochanter.
. Excessive frictional forces 2ry to overuse or trauma are common causes.
. Hip pain when pressure is applied (When sleeping) & external rotation or abduction.
. FEMORAL HEAD AVASCULAR NECROSIS = LEGG CALVE' PERTHES DISEASE:
_________________________________________________________________
. Boys between 4 & 10 ys with peak incidence bet. 5 & 7 us.
. Hip, groin or knee pain + Antalgic pain.
. Dx -> X-ray -> Flattened & fragmented femoral head.
. Alternating regions of lucency & density = Reflects of necrotic tissue by new bone.
. Tx -> Conservatively with observation & bracing.
. Tx -> Surgery in cases where femoral head isn't well contained within the acetabulum.
. SLIPPED CAPITAL FEMORAL EPIPHYSIS (SCFE):
____________________________________________
. Obese male child with pain during LATE CHILDHOOD or EARLY ADOLESCENCE.
. Metaphysis & femur slip relative to the epiphysis at the epiphyseal plate.
. The capital femoral epiphysis remains structurally intact within the acetabulum.
. Loss of abduction & internal rotation of the hip.
. Loss of external rotation of the thigh while the hip is flexed.
. Dx -> FROG-LEG LATERAL X-RAY VIEW (Diagnostic).
. Tx -> SURGICAL PINNING to avoid avascular necrosis.
. TRENDELENBURG SIGN:
______________________
. Drooping of the contralateral pelvis when the pt stands on one foot.
. Associated with TRENDELENBURG gait (Waddling) caused by the trunk rocking,
. to compensate for the pelvic drooping !
. Caused by weakness or paralysis of the gletues medius & minimus muscles,
. due to superior gluteal nerve trauma or inflammation or entrappment.
. The pt presents with unilateral intermittent knee pain.
. Physical activity e.g. stair clumbing exacerbates the pain.
. Hip tenderness is common.
. SQUAMOUS CELL CARCINOMA:
___________________________
. Suspected in all non-healing wounds.
. SCC may arise within chronicallu wounded, scarred or inflammaed skin.
. SCC arising within burn wounds is known as MARJOLIN ULCER.
. SCC arise in skin overlying a focus of osteomyleitis, radiotherapy or venous ulcers.
. SCC arising within chronic wounds exhibit aggressive behavior.
. Early diagnosis is the key to prevent metastatic spread
. A biopsy sh'd be obtained in all chronic wounds failing to heal to rule out malignancy.
. BASAL CELL CARCINOMA:
________________________

. Presents on chronically sun-exposed skin.


. Lesions are PEARLY TELANGIECTATIC papules with a central RODENT ulceration.
. BILATERAL LOWER LIMB EDEMA & STASIS DERMATITIS:
__________________________________________________
. Both are due to lower extremity venous valvular incompetence.
. resulting in pooling of venous blood & ++ pressure in post-capillary venules.
. ++ pressure = VENOUS HYPERTENSION.
. ++ pressure -> Damages capillaries -> Loss of fluid, plasma ptns & RBCs into tissue.
. Erythrocytes extravasation -> Hemosiderin deposition & bluish discoloration.
. May be complicated by venous ulcers.
. It involves the medial leg below the knee & above the medial malleolus.
. Xerosis is the most common early finding.
. Lipodermatosclerosis & venous ulcerations are late findings.
. RETRO-PERITONEAL HEMORRHAGE:
_______________________________
. An iatrogenic complication after cardiac catheterization.
. After cannulation of the femoral artery to access the cardiac vessels.
. A hematoma is formed at the upper thigh -> Extends into the retro-peritoneal space.
. Significant belleding with hypotension & tachycardia.
. Ipsilateral flank/back pain.
. Dx -> CT scan of ABDOMEN & PELVIS with-OUT contrast.
. Tx -> Supportive -> Blood transfusion - IV fluids - Bed rest.
. Tx -> Immediate surgical decompression if there are neurological deficits.
. PNEUMO-PERITONEUM:
_____________________
. AIR UNDER DIAPHRAGM = Intra-peritoneal air.
. Best seen bet. the liver & the diaphragm.
. Caused by PERFORATED VISCUS e.g. PERFORATED PEPTIC ULCER.
. PERFORATED PEPTIC ULCER (H/O of epigastric pain & discomfort with eating).
. Tx -> SURGICAL CONSULATATION IMMEDIATELY for EXPLORATORY LAPAROTOMY.
. CHILD ABUSE:
_______________
. Patterned scalds & burns = forceful immersion of hot object e.g. cigarette or hot iron.
. Incoherent or impropable explanation of the injuries.
. Delay in seeking care after injury.
. #s of long bones or ribs, #s in various stages of healing.
. Suspicious bruises include those on thighs, abdomen, cheeks & genitalia.
. Subdural hematoma & retinal hemorrhages in very young infants.
. Inaapropriate affect of the care giver.
. Physician should perform a thorough physical exam. & full radiographic skeletal survey.
. Report the case to child protective services.
. Admit the pt to ensure their safety.
. The child should never be sent home.
. The caregiver should never be confronted.
. Physicians are mandatory reporters.

. AMPUTATION INJURY:
_____________________
. Amputated parts sh'd be wrapped in SALINE-MOISTENED GAUZE,
. SEALED IN A PLASTIC BAG,
. PLACED ON ICE,
. brought to the emergency department with the patient.
. CAUSES OF HEMOPTYSIS:
________________________
. Pulmonary -> Bronchitis - Pulmonary embolism - Bronchiectasis - Lung cancer.
. Cardiac -> Mitral stenosis - Acute pulmonary edema.
. Infectious -> Tuberculosis - Lung abscess.
. Hematologic -> Caogulopathy.
. Vascular -> Arteriovenous malformation.
. Systemic diseases -> Wegener's granulomatosis - Goodpasture's $ - SLE - Vasculitis.
. HEMOPTYSIS MANAGEMENT
________________________
|
. H/O & P/E to rule out other causes (Oropharynx & GIT)
________________________________________________________
|
_______________________________
|
|
. MILD/MODERATE
. MASSIVE (>600 ml/24hs)
________________
_________________________
|
|
. CXR, CBC, COAGULATION STUDIES . SECURE AIRWAY, BREATHING & CIRCULATION
. RENAL FUNCTIONS & URINALYSIS
|
. RHEUMATOLOGY WORK UP
. IF BLEEDING
|
|
<----------------STOPS---------------------CONTINUES
|
|
. CT SCAN + BRONCHOSCOPY
|
|
|
. treat the cause;persistent bleeding
|
treated via bronchoscopic interventions <------------------------embolization or resection.
.
.
.
.
.
.
.
.

N.B.
Massive hemoptysis = > 600 ml/24 hs.
Greatest danger is asphyxiation due to airway flooding with blood.
Establishing an adequate patent airway is the most imp. initial step.
The pt should be placed with the bleeding lung un the dependent lateral position,
to avoid blood collection in the airways of the opposite lung.
Bronchoscopy is the best to localize the bleeding site, provide suction.
Bronchoscopy is both diagnostic & therapeutic.

. Pt from endemic area - Night fever - weight loss - Upper lobe involvement = T.B.
. Respiratory isolation is mandatory to prevent spread of infection.

. FAT EMBOLISM:
________________
. Common in pt with polytrauma with multiple #s of long bones.
. Severe respiratory distress, petichial rash, subconjunctival hemorrhage.
. Tachycardia, tachypnea & fever.
. May occur after 12-72 hs after trauma.
. CNS dysfunction -> Confusion - Agitation - Stupor - Seizures - Coma.
. Dx -> Fat droplets in urine.
. Dx -> Intra-arterial fat globules on fundoscopy.
. Dx -> CXR -> Diffuse bilateral pulmonary infiltrates.
. Tx -> Respiratory support.
. NECROTIZING SURGICAL INFECTION:
__________________________________
. Intense pain in wound.
. Fever, hypotension & tachycardia.
. Decreased sensitivity at the edge of the wound.
. Cloudy gray discharge.
. Tense edema out-side the involved skin.
. Subcutaneous gas with crepitus.
. More common in diabetics.
. Caused by mixed gram +ve & gram -ve flora.
. Tx -> Early surgical exploration & debridement of the necrotic tissues.
. Adjunctive ttt -> Antibiotics, adequate hydration & tight glycemic control.
. MASTITIS ASSOCIATED WITH BREAST FEEDING:
___________________________________________
. Due to transmission of bacterial organism from the infant's nasopharynx,
. to a fissure on the mother's nipple or areola.
. Most commonly Staph. aureus.
. Tx -> Analgesics, antibiotics (Dicloxacillin-Cephalosporin) & CONTINUE BREAST FEEDING.
. Continued nursing from the affected breast -> -- the progression of mastitis to abscess
. Incision & drainage only if there is abscess formation !
. Mammogram is not useful in mastitis !
. Mammogram is not useful before age of 50 due to dense breast tissue.
. Suppression of breast milk is NOT recommended.
. HYPOVOLEMIC SHOCK & POSITIVE PRESSURE MECHANICAL VENTILLATION:
_________________________________________________________________
. +ve pressure mechanical ventillation -> ++ intrathoracic pressure -> -- VR to heart.
. -- VR -> -- Ventricular preload.
. In pts with hypovolemic shock, this effect may cause circulatory collapse !
. if the pt's intravascular volume isn't replaced before mechanical ventillation begins.
. URETHRAL INJURY & PELVIC #:
______________________________
. POSTERIOR urethral injury is associated with pelvic #s.
. Blood at urethral meatus.
. High riding prostate.

. Scrotal hematoma.
. Inability to void despite sensing an urge to void.
. Palpable distended bladder.
.
.
.
.
.
.

ANTERIOR urethral injury is due to blunt trauma to perineum (Straddle injury).


May be caused by instrumentation to urethra.
Perineal tenderness & hematoma.
NORMAL PROSTATE.
Bleeding from urethra.
NORMAL URINATION.

. DUMPING $YNDROME:
____________________
. Common post-gastrectomy complication.
. Due to rapid emptying of gastric contents into the duodenum & small intestine.
. Post-prandial abd. cramps - weakness - lightheadedness - diaphoresis.
. Symptoms diminish over time.
. Symptoms result from fluid shift from intravascular space to small intestine.
. Stimulation of intestinal vasoactive peptides -> Stimulation of autonomic reflexes.
. Dietary changes are helpful to control symptoms.
. In resistant cases, octreotide sh'd be tried.
. Reconstructive surgery is reserved for intractable cases.
. HEMATOCHEZIA:
________________
. Bright red blood in stool.
. Due to lower GI bleeding (distal to ligament of Treitz).
. May occur in very brisk upper GI bleeding.
. Most common causes of lower GI bleeding in pts >50 ys-> DIVERTICULOSIS ANGIODYSPLASIA
. Nasogastric tube placement with bile not blood = No active upper GI bleeding.
. Upper endoscopy sh'd be done next not to miss duodenal bleeding.
.
.
.
.
.
.

In cases of hematochezia due to diverticulosis,


The initial step is COLONOSCOPY.
If -ve -> Radio-nuclide (Technetium 99 Labelled eryhthrocyte scintigraphy tagged RBCs).
It is less invasive & more sensitive than angiography.
It localizes the source of bleeding so that,
the region can be further evaluated by colonoscopy or angiography.

. CENTRAL VENOUS CATHETERIZATION:


__________________________________
. A CXR sh'd be done to confirm proper placement & absence of complications.
. e.g. subclavian artery injury, pneumothorax, hemothorax, thrombosis & air embolism.
. To avoid myocardial perforation the catheter tip sh'd be located proximal to either:
. the cardiac silhouette or the angle between the trachea & right main stem bronchus.
. The catheter sh'd lie in the superior vena cava.
. DIABETIC FOOT ULCERS:
________________________
. Result from neuropathy, microvascular insuffeciency & immunosuppression.

. They occur on the plantar surface of the foot under points of greatest pressure,
. such as under the head of the 1st metatarsal bone.
. SOLITARY PULMONARY NODULE EVALUATION:
________________________________________
SOLITARY PULMONARY NODULE EVALUATION
______________________________________
|
CHEST CT WITH CONTRAST
________________________
|
__________________________
|
|
Benign features
Intermediate or suspicious for malignancy
_________________ ___________________________________________
|
|
SERIAL CT SCANS TO MONITOR FURTHER INVESTIGATION WITH BIOPSY or PET
scan

. VENOUS VALVULAR INCOMPETENCE:


________________________________
. is the most common cause of lower extremity edema.
. It classically worsens thoroughout the day & resolves overnight when pt is recumbent.
. NASAL SEPTAL PERFORATION:
____________________________
. Any pt develops a whistling noise during respiration following rhinoplasty.
. One suspect nasal septal perforation due to Septal hematoma.
. TRAUMATIC SPINAL CORD INJURIES:
__________________________________
. Should be 1st hemodynamically stabilized.
. Proper airway management.
. Urinary catheterization placement is imp. to assess for urine retention !
. It is important to prevent bladder distension & damage.
. A retrograde urethrogram sh'd be the 1st step in management of urethral injury,
. Foley catheterization is contraindicated as it will worsen the condition.
. ACUTE BACTERIAL PAROTITIS:
_____________________________
. Fever, leukocytosis & parotid inflammation.
. Dehydrated post-operative pts & elderly are most prone to develop infection.
. Painful swelling of the parotid gland aggravated by chewing.
. Tender, swollen & erythematous gland with purulent saliva expressed from parotid duct.
. Most common infectious agent is STAPHYLOCOCCUS AUREUS.
. Tx -> Adequate fluid hydration & oral hygiene can prevent this condition.

. N.B. Spirometry prevents post-operative respiratory complications not parotitis.


. DIVERTICULOSIS:
__________________
. is the most common cause of a lower gastro-intestinal hemorrhage in an elderly.
. Bright red bleeding from the rectum is usually caused by a lower GI hemorrhage.
. Colonic diverticula are formed due to high intra-luminal pressure,
. which causes the mucosa to herniate through the bowel wall penetrating its vasculature.
. They don't include all layers of the bowel (False divertuculae).
. Chronic constipation is due to low fiber diet.
. Most common predisposing factor to diverticulosis.
. Diverticulae erode the bowel vasculature leading to profuse bleeding per rectum.
. Most common site is sigmoid colon.
. Dx -> CT Abdomen.
.
.
.
.

N.B.
Diverticulosis -> Non-inflammed diverticula -> Painless bleeding.
Diverticulitis -> Abdominal pain & infectious syms 2ry to obstruction of diverticula.
It is uncommon to see bleeding with diverticulitis !

. GASTRO-ESOPHAGEAL MURAL INJURY CHARACTERISTICS:


_________________________________________________
_________________________________________________
(A) MALLORY WEISS $YNDROME:
____________________________
. Upper gastro-intestinal MUCOSAL TEAR.
. Caused by forceful retching (++ pressure).
. Submucosal arterial or venule plexus bleeding.
. Vomiting, retching, hematemesis & epigastric pain.
. Dx -> EGD confirms diagnosis.
. Most tears heal spontaneously.
. Endoscopic therapy for continous bleeding.
(B) BOERHAAVE $YNDROME:
________________________
. Esophageal TANS-MURAL tear.
. Caused by forceful retching (++ pressure).
. ESOPHAGEAL AIR/FLUID LEAKAGE into nearby areas e.g. pleura.
. Vomiting, retching, chest & upper abdominal pain.
. Odynophagia, fever, dyspnea & septic shock may occur.
. Subcutaneous emphysema may be seen.
. Dx -> CT or CONTRAST ESOPHAGOGRAPHY e' GASTROGRAFIN (Water soluble) confirms
diagnosis.
. CXR -> Pneumo-mediastinum & pleural effusion.
. Pleural fluid analysis -> EXUDATIVE, LOW pH, VERY HIGH AMYLASE > 2500 IU.
. Tx -> Surgery for thoracic perforations.
. Conservative measures e.g. antibiotics for cervical perforation.
.
.
.
.

IATROGENIC esophageal perforation:


CXR -> Pleural effusion, pneumomediastinum & pneumothorax.
Dx -> Water soluble contrast esophagogram.
Avoid endoscopy not to worsen the condition !

. MECHANISMS OF LOWERING INTRA-CRANIAL TENSION:


________________________________________________
. HEAD ELEVATION -> ++ venous outflow from the head.
. SEDATION -> -- metabolic demand & control of HTN.
. IV MANNITOL -> Extraction of free water out of brain tissue -> Osmotic diuresis.
. HYPERVENTILLATION -> CO2 washout -> Cerebral VASOCONSTRICTION.
. POST-SPLENECTOMY VACCINES:
_____________________________
. Following splenectomy, pts are at ++ risk for sepsis 2ry to encapsulated organisms
. Capsulated organisms (S. pneumoniae - N. meningitidis - H. influenzae).
. Vaccination against these organisms sh'd be administered.
. Pneumococcal vaccine boosters are required every 5 years.
. PAROTID NEOPLASM:
____________________
. The two lobes of the parotid gland are separated by the facial nerve.
. Parotid surgery involve the deep lobe of the parotid gland -> facial palsy.
. Facial palsy -> Facial droop.
. INJURY -> CULPRIT INJURED NERVE:
___________________________________
. HOARSENESS -> RECURRENT LARYNGEAL BRANCES OF VAGUS NERVE (Thyroid/Parathyroid
surgery).
. TIC DOULOUREUX (TRIGEMINAL NEURALGIA) -> TRIGEMINAL NERVE.
. TONGUE PALSY -> HYPOGLOSSAL NERVE (Submandibular gland surgery).
. WINGED SCAPULA -> LONG THORACIC NERVE (Axillary lymphadenectomy surgery).
. PERIPHERAL ARTERY ANEURYSM:
______________________________
. Pulsatile mass that can compress adjacent structures (nerves - veins).
. May lead to thrombosis & ischemia.
. Most common are popliteal & femoral aneurysms.
. Associated with peripheral artery aneurysm.
. PENILE #:
____________
. Crush injury of an erect penis.
. Common during intercourse with female on top of male.
. Dx -> Emergent urethrogram to assess for urethral injury.
. Tx -> Surgical exploration to evacuate hematoma & mend the torn tunica albuginea.
. URIC ACID STONES:
____________________
. Ureteral colic -> Vagal reaction -> ILEUS.
. Urinalysis -> Needle shaped crystals = uric acid stones.
. Dx -> CT abdomen or IV pyelography.

. Tx -> Stones < 0.6 cm -> Pass spontaneously with hydration & analgesia.
. Tx -> Stones > 0.6 cm -> Surgical removal.
. NASOPHARYNGEAL CARCINOMA (NPC):
__________________________________
. Undifferentiated carcinoma of squamous cell origin.
. Higher frequency in people of Mediterranean or far eastern descent.
. Most NPC are metastatic at the time of diagnosis.
. Recurrent otitis media (Due to eustachian tube obstruction by tumor).
. Recurrent epistaxis or nasal obstruction.
. Associated with positive serology for EPSTEIN BARR VIRUS (EBV).
. It is associated with smoking & chronic nitrosamine consumption (Salted fish diet).
. PARALYTIC ILEUS:
___________________
. Abdominal pain after a traumatic injury.
. Associated with vertebral # or retro-peritoneal hemorrhage.
. Ileus is caused by an exagerrated intestinal reaction after abdominal surgery.
. Due to disruption of normal neurologic & motor control of the gastrointestinal tract.
. Failure to pass stool or flatus, abdominal distension, nausea & vomiting.
. Distended abdomen with tympany.
. Decreased or absent bowel sounds.
. Abdominal x-ray -> Air-fluid levels & distended gas-filled loops of small & large int.
. Tx -> Conservative with bowel rest & supportive care.
. LUDWIG's ANGINA:
___________________
. Infection of the submandibular & sublingual glands.
. Source of infection -> Infected tooth (2nd or 3rd mandibular molar).
. Most common cause of death -> Asphyxia.
. TORUS PALATINUS:
___________________
. CONGENITAL !
. Young individual.
. Fleshy immobile mass on the midline hard palate.
. No medical or surgical ttt is required unless the growth becomes symptomatic.
. i.e. interfering with speech or eating.
. NEURO-ANATOMY:
_________________
_________________
. FEMORAL NERVE:
_________________
. Motor to anterior compartment of thigh (Quadriceps femoris - Sartorius - Pectineus).
. Responsible of knee extension & hip flexion.
. Sensory to the anterior thigh & medial leg via saphenous branch.
. TIBIAL NERVE:

________________
. Motor to posterior compartment of thigh, posterior compartment of leg & plantar foot ms
. Responsible of knee flexion & digits & plantar flexion of foot.
. Sensory to the leg (except the medial side) & plantar foot.
. OBTURATOR NERVE:
___________________
. Motor to medial compartment of thigh.
. Responsible of thigh adduction.
. Sensory to the medial thigh.
. COMMON PERONEAL NERVE = FIBULAR NERVE:
_________________________________________
. Give rise to superficial & deep peroneal nerves.
. Motor to anterior & lateral leg.
. Sensory to antero-lateral leg & dorsum of the foot.
. GLASGOW COMA SCALE:
______________________
______________________
. EYE OPENING:
_______________
4 -> Spontaneous.
3 -> To verbal command.
2 -> To pain.
1 -> None.
. VERBAL RESPONSE:
___________________
5 -> Oriented.
4 -> Disoriented/confused.
3 -> Inappropriate words.
2 -> Incomprehensible sounds.
1 -> None.
. MOTOR RESPONSE:
__________________
6 -> Obeys.
5 -> Localizes.
4 -> Withdraws.
3 -> Flexion posturing (Decorticate).
2 -> Extension posturing (Decerebrate).
1 -> None.

Dr. Wael Tawfic Mohamed


_________________________

You might also like